Labor Relations Case Digest

July 26, 2017 | Author: Danie Flynchy Lynch | Category: Strike Action, Trade Union, Employment, Collective Bargaining, Demonstration (Protest)
Share Embed Donate


Short Description

Law, Labor Relations, Labor Law, case, case digest...

Description

University of San Carlos College of Law

LABOR RELATIONS CASE DIGESTS (FINALS) Submitted to: Atty. Jefferson M. Marquez

Submitted by: Dana Flynch R. de Lira EH 406

1

TABLE OF CONTENTS TOPIC 12: RIGHT TO SELF-ORGANIZATION.....................................................................6 SAN MIGUEL CORP., VS. MANDAUE PACKING PRODUCTS, G.R. NO. 152356, AUGUST 16, 2005............6 GSIS VS. KAPISANAN NG MGA MANGGAGAWA SA GSIS, G.R. NO. 170132, DECEMBER 6, 2006............8 DIOKNO VS. CACDAC, G.R. NO. 168475, JULY 4, 2007.......................................................................... 9 SAN MIGUEL CORP. EMPLOYEES UNION-PTGWO VS. SAN MIGUEL PACKAGING PRODUCTS EMPLOYEES UNION, G.R. NO. 171153, SEPT. 12, 2007.......................................................................................... 11 DONG SEUNG INC., VS. BUREAU OF LABOR RELATIONS, G.R. NO. 162356, APRIL 14, 2008...............16 DEL PILAR ACADEMY ET AL., VS. DEL PILAR ACADEMY EMPLOYEES UNION, G.R. NO. 170112, APRIL 30, 2008.................................................................................................................................................. 18 S.S. VENTURES INTERNATIONAL INC., VS. SS VENTURES LABOR UNION, G.R. NO. 161690, JULY 23, 2008 .......................................................................................................................................................... 20 INGUILLO ET AL, VS. FIRST PHIL SCALES INC., ET AL., GR NO. 165407, JUNE 5, 2009.........................22 STA LUCIA EAST COMMERCIAL CORP., VS. SOLE ET AL., GR NO. 162355, AUGUST 14, 2009..............26 MARIWASA SIAM CERAMICS INC. VS. SECRETARY OF DOLE, ET AL., G.R. NO. 183317, DECEMBER 21, 2009 .......................................................................................................................................................... 28 GENERAL MILLING CORP VS. CASIO ET AL., GR NO. 149552, MARCH 10, 2010..................................30 THE HERITAGE HOTEL MANILA VS. NATL UNION OF WORKERS IN HOTEL ETC., GR NO. 178296, JANUARY 12, 2011............................................................................................................................................ 33 LEGEND INTERNATIONAL RESORTS LTD., VS. KILUSANG MANGGAGAWA NG LEGENDA, G.R. NO. 169754, FEB. 23, 2011.................................................................................................................................... 35 SAMAHANG MANGGAGAWA SA CHARTER CHEMICAL SOLIDARITY OF UNIONS IN THE PHILS FOR EMPOWERMENT AND REFORMS (SMCC-SUPER) ET AL., VS. CHARTER CHEMICAL AND COATING CORP., G.R. NO. 169717, MARCH 16, 2011........................................................................................................... 36 SAN MIGUEL FOODS VS. SAN MIGUEL CORP SUPERVISORS AND EXEMPT UNION G.R. NO. 146206 AUGUST 1, 2011.............................................................................................................................................. 38 BPI VS. BPI EMPLOYEES UNION-DAVAO CHAPTER, GR NO. 164301, OCTOBER 19, 2011 RESOLUTION ON THE MAIN DECISION OF AUG. 18, 2010.............................................................................................. 40 OCTAVIO vs. PHILIPPINE LONG DISTANCE TELEPHONE COMPANY.......................................................42 NATIONAL UNION OF BANK EMPLOYEES vs. PHILNABANK EMPLOYEES ASSOCIATION.........................44 TOPIC 13: RIGHTS OF LEGITIMATE LABOR ORGANIZATION............................................46 RIVERA VS. ESPIRITU, G.R. NO. 135547, JANUARY 23, 2002...............................................................46 SAN MIGUEL CORP., VS. NLRC, G.R. NO. 119293, JUNE 10, 2003........................................................49 SAMAHANG MANGGAGAWA SA SULPICIO LINES VS. SULPICIO LINES, G.R. NO. 140992, MARCH 25, 2004 .......................................................................................................................................................... 51 NOTRE DAME OF GREATER MANILA VS. LAGUESMA, G.R.NO. 149833, JUNE 29, 2004.......................53 MANILA DIAMOND HOTEL EMPLOYEES UNION VS. COURT OF APPEALS, G.R. NO. 140518, DECEMBER 16, 2004.................................................................................................................................................. 55 UNIVERSITY OF IMMACULATE CONCEPCION VS. SEC. OF LABOR, G.R. NO. 151379, JANUARY 14, 200556 CAPITOL MEDICAL CENTER VS. NLRC, G.R. NO. 147080, APRIL 26, 2005...........................................58 PLDT VS. MANGGAGAW NG KOMUNIKASYON SA PILIPINAS, G.R. NO. 162783, JULY 14, 2005..............60 LIGHT RAILWAY TRANSIT VS. VENUS, JR., G.R. NO. 163782, MARCH 24, 2006.....................................61

2

UNIVERSITY OF SAN AGUSTIN EMPLOYEES UNION-FFW VS. COURT OF APPEALS, G.R. NO. 169632, MARCH 28, 2006............................................................................................................................................ 63 SUKHOTAI CUISINE & RESTAURANT VS. COURT OF APPEALS, G.R. NO. 150437, JULY 17, 2006...........64 PHILCOM EMPLOYEES UNION VS. PHIL. GLOBAL COMMUNICATION, G.R. NO. 144315, JULY 17, 2006..67 FAR EASTERN UNIVERSITY-DR. NICANOR REYES MEDICAL FOUNDATION VS. FEU-NRMF EMPLOYEES ASSO., G.R. NO. 168362, OCTOBER 12, 2006................................................................................................ 68 GSIS VS. KAPISANAN NG MGA MANGGAGAWA SA GSIS, G.R. NO. 170132, DECEMBER 6, 2006..........71 BIFLEX PHILS., INC., LABOR UNION VS. FILFLEX INDUSTRIAL & MFG., CORP., G.R. NO. 155679, DECEMBER 19, 2006............................................................................................................................................ 72 MANILA HOTEL EMPLOYEES ASSOCIATION VS. MANILA HOTEL CORP., G.R. NO. 154591, MARCH 5, 2007, CITING GRAND BOULEVARD HOTEL VS. DACANAY, G.R. NO. 153665, JULY 18, 2003...........................74 FACULTY ASSOCIATION OF MAPUA INSTITUTE OF TECHNOLOGY VS. COURT OF APPEALS, G.R. NO. 164060, JUNE 15, 2007................................................................................................................................... 76 PILIPINO TELEPHONE CORP., VS. PILIPINO TELEPHONE EMPLOYEES ASSO. G.R. NO.160058, JUNE 22, 2007 .......................................................................................................................................................... 77 LANDTEX INDUSTRIES VS. CA, G.R. NO. 150278, AUGUST 9, 2007....................................................79 SAN MIGUEL FOODS INC., VS. SAN MIGUEL CORP EMPLOYEES UNION-PTGWO , G.R. NO. 168569, OCTOBER 5, 2007.............................................................................................................................................. 80 TOYOTA MOTOR PHILS WORKERS ASSO.VS. NLRC, G.R. NO. 158786, OCTOBER 19, 2007..................82 YOKOHAMA TIRE PHILS VS. YOKOHAMA EMPLOYEES UNION, G.R. NO. 159553, DECEMBER 10, 2007. 86 PHIL. AIRLINES INC. VS. PHIL AIRLINES EMPLOYEES ASSOCIATION, G.R. NO. 142399, MARCH 12, 200887 STEEL CORP. VS. SCP EMPLOYEES UNION-NFL G.R. NO. 169829-30, APRIL 16, 2008..........................88 STANDARD CHARTERED BANK EMPLOYEES UNION VS. STANDARD CHARTERED BANK ET AL., G.R. NO. 161933, APRIL 22, 2008.................................................................................................................... 91 SAMAHAN NG MGA MANGGAGAWA SA SAMMA-LAKAS SA INDUSTRIYA NG KAPATIRANG HALIGI NG ALYANSA (SAMMA-LIKHA) VS. SAMMA CORP., G.R. NO. 167141, MAR. 13, 2009................................................93 HOTEL ENTERPRISES OF THE PHILS., (HYATT REGENCY) VS. SAMAHAN NG MGA MANGGAGAWA SA HYATT(NUWHRAIN) G.R. NO. 165756, JUNE 5, 2009..................................................................................... 94 TEODORICO S. MIRANDA, JR. VS. ASIAN TERMINALS, INC. (ATI) AND COURT OF APPEALS GR NO. 174316 JUNE 23, 2009................................................................................................................................... 96 NATIONAL UNION OF WORKERS IN HOTELS RESTAURANTS AND ALLIED INDUSTRIES-MANILA PAVILION HOTEL CHAPTER VS. SOLE, ET AL. G.R. NO. 181531, JULY 31, 2009...................................................97 A. SORIANO AVIATION VS. EMPLOYEES ASSOCIATION OF A. SORIANO AVIATION ET AL. G.R. NO. 166879, AUG. 14, 2009................................................................................................................................. 100 YSS EMPLOYEES UNION-PHIL TRANSPORT AND GENERAL WORKERS ORGANIZATION VS. YSS LABORATORIES INC. G.R. NO. 155125, DECEMBER 4, 2009..............................................................103 GOVERNMENT SERVICE INSURANCE SYSTEM (GSIS) VS. DINNAH VILLAVIZA, ELIZABETH DUQUE, ADRONICO A. ECHAVEZ, RODEL RUBIO, ROWENA THERESE B. GRACIA, PILAR LAYCO, AND ANTONIO JOSE LEGARDA, G.R. NO. 180291, JULY 27, 2010...................................................................................... 105 PICOP RESOURCES INC. VS. TANECA ET. AL., G.R. NO. 160828, 09 AUGUST 2010............................107 INSULAR HOTEL EMPLOYEES UNION-NFL VS. WATERFRONT INSULAR HOTEL-DAVAO, G.R. NO. 174040-41, SEPTEMBER 22, 2010...................................................................................................................... 109 CITREK EMPLOYEES LABOY UNION-FFW VS. CITREK ELECTRONICS INC. G.R. NO. 190515, 15 NOVEMBER 2010................................................................................................................................................ 111

3

SOLIDBANK CORP. VS. GRAMIER, ET. AL, , G.R. NO. 159460-61, NOVEMBER 15, 2010......................113 FADRIQUELAN ET AL VS MONTEREY FOOD CORP., G.R. 178409, JUNE 08, 2011...............................115 MIGUEL DELA BARAIRO, PENA PETITIONER, VS. OFFICE OF THE PRESIDENT AND MST MARINE SERVICES (PHILS,), INC. RESPONDENT, G.R. NO. 189314, JUNE 15, 2011.........................................................115 MAGDALA MULTIPURPOSE & LIVELIHOOD COOPERATIVE VS. KILUSANG MANGGAGAWA NG LGS ET AL, G.R. NOS. 191138-39, OCTOBER 19, 2011.............................................................................................. 116 ABERIA ET AL., VS. NATIONAL LABOR RELATION COMMISSION ET., [G.R. NOS. 154113, 187778, 187861, AND 196156 DECEMBER 7, 2011].................................................................................................... 119 PICOP RESOURCES, INCORPORATED VS DEQUILLA, ET. AL. ; GR NO 172666; DECEMBER 7, 2011....120 C. ALCANTARA & SONS, INC., PETITIONER, VS. COURT OF APPEALS, G.R. NO. 155109, SEPTEMBER 29, 2010, NAGKAHIUSANG MAMUMUO SA ALSONS-SPFL (NAMAAL-SPFL), AND ITS MEMBERS WHOSE NAMES ARE LISTED BELOW, PETITIONERS, VS. C. ALCANTARA & SONS, INC., RESPONDENT. G.R. NO. 179220122 DIGITAL TELECOMMUNICATIONS PHILS INC. vs. DIGITEL EMPLOYEES UNION ET AL..........................124 AUTOMOTIVE ENGINE REBUILDERS vs. PROGRESIBONG UNYON NG MGA MANGGAGAWA SA AER. . .125 HOLY CHILD CATHOLIC SCHOOL vs. HON. STO TOMAS ET AL...........................................................128 VISAYAS COMMUNITY MEDICAL CENTER vs. YBALLE, ET AL..............................................................130 PHILTRANCO SERVICE ENTERPRISES INC. vs. PHILTRANCO WORKERS UNION-ASSOCIATION OF GENUINE LABOR ORGANIZATIONS.................................................................................................................. 131 WESLEYAN UNIVERSITY-PHILS. vs. WESLEYAN UNIVERSITY-PHILS., FACULTY & STAFF ASSOCIATION. 132 TABANGAO SHELL REFINERY EMPLOYEES ASSOCIATION vs. PILIPINAS SHELL PETROLEUM CORP......134 TOPIC 14: UNFAIR LABOR PRACTICE..........................................................................136 STANDARD CHARTERED BANK EMPLOYEES UNION vs. CONFESOR...................................................138 PHILIPPINE CARPET EMPLOYEES ASSOCIATION vs. HON. STO. TOMAS..............................................142 ST. JOHN COLLEGES, INC. vs. ST. JOHN ACADEMY FACULTY EMPLOYEES UNION................................145 SAN MIGUEL FOODS, INC. vs. SAN MIGUEL CORPORATION EMPLOYEES UNION-PTGWO...................148 PUREFOODS CORP. vs. NAGKAKAISANG SAMAHANG MANGGAGAWA NG PUREFOODS RANK AND FILE150 GENERAL SANTOS COCA-COLA PLANT FREE WORKERS UNION-TUPAS vs. CCBPI(GEN. SANTOS CITY) ET ........................................................................................................................................................ 152 DE LA SALLE UNIVERSITY ET AL. vs. DE LA SALLE UNIVERSITY EMPLOYEES ASSOCIATION...............154 TUNAY NA PAGKAKAISA NG MANGGAGAWA SA ASIA BREWERY vs. ASIA BREWERY, INC...................156 MANILA MINING CORP. EMPLOYEES ASSOCIATION-FFW vs. MANILA MINING CORP............................158 PRINCE TRANSPORT ET AL. vs. GARCIA ET AL.................................................................................. 160 PARK HOTEL ET AL. vs. SORIANO ET AL........................................................................................... 162 GOYA, INC. vs. GOYA, INC. EMPLOYEES UNION-FFW.........................................................................162 BAPTISTA ET AL. vs. VILLANUEVA ET AL........................................................................................... 164 TH SHOPFITTERS CORP. ET AL. vs. T&H SHOPFITTERS CORP. UNION................................................166 TOPIC 15: REVISED GUIDELINES OF THE NCMB FOR THE CONDUCT OF VOLUNTARY ARBITRATION PROCEEDINGS.........................................................................................................168 SANYO PHILIPPINE WORKERS UNION-PPSLU vs. CANIZARES............................................................168 NAVARRO III vs. DAMASCO............................................................................................................... 170 SAN MIGUEL CORPORATION vs. NLRC.............................................................................................. 171

4

PANTRANCO NORTH EXPRESS INC. VS. NLRC................................................................................... 175 SILVA ET AL vs. NLRC....................................................................................................................... 177 UNION OF NESTLE WORKERS CAGAYAN DE ORO FACTORY vs. NESTLE PHILIPPINES INC..................182 TABIGUE ET AL. vs. INTERNATIONAL COPRA EXPORT CORPORATION................................................183 SAINT LOUIS UNIVERSITY vs. COBARRUBIAS.................................................................................... 185 TENG vs. PAHAGAC ET AL................................................................................................................ 187 CAONG, JR. vs. BEGUALOS............................................................................................................... 188 ESTATE OF DULAY vs. ABOITIZ JEBSEN MARITIME INC. ET AL...........................................................189 LEPANTO CONSOLIDATED MINING COMPANY vs. THE LEPANTO CAPATAZ UNION..............................192 7K CORP. vs. ALBARICO................................................................................................................... 195

5

TOPIC 12: RIGHT TO SELF-ORGANIZATION

SAN MIGUEL CORP., VS. MANDAUE PACKING PRODUCTS, G.R. NO. 152356, AUGUST 16, 2005 Facts: On 15 June 1998, Respondent union, identifying itself as an affiliate of Federation of Free Workers (FFW), filed a petition for certification election with the DOLE Regional Office. In the petition, respondent stated that it sought to be certified and to represent the permanent rank-and-file monthly paid employees of the petitioner. The following documents were attached to the petition:

(1) a Charter Certificate issued by FFW on 5 June 1998 certifying that respondent as of that date was duly certified as a local or chapter of FFW; (2) a copy of the constitution of respondent prepared by its Secretary, Noel T. Bathan and attested by its President, Wilfred V. Sagun; (3) a list of respondent’s officers and their respective addresses, again prepared by Bathan and attested by Sagun; (4) a certification signifying that respondent had just been organized and no amount had yet been collected from its members, signed by respondent’s treasurer Chita D. Rodriguez and attested by Sagun; and (5) a list of all the rank-and-file monthly paid employees of the Mandaue Packaging Products Plants and Mandaue Glass Plant prepared by Bathan and attested by Sagun.

Petitioner company filed a motion to dismiss the petition for certification election on the sole ground that the respondent union is not listed or included in the roster of legitimate labor organizations.

Respondent then submitted to the BLR the same documents earlier attached to its petition for certification. The accompanying letter stated that such documents were submitted in compliance with the requirements for the creation of a local/chapter pursuant to the Labor Code and its Implementing Rules; and it was hoped that the submissions would facilitate the listing of respondent under the roster of legitimate labor organizations. After which, the Chief of LRD-DOLE Regional Office issued a Certificate of Creation of Local/Chapter certifying that from 30 July 1998, respondent has acquired legal personality as a labor organization or worker’s association, it having submitted all the required documents. Opting not to file a comment on the Motion to Dismiss, respondent instead filed a Position Paper wherein it asserted that it had complied with all the necessary requirements for the conduct of a certification election, and that the ground relied upon in the Motion to Dismiss was a mere technicality.

In turn, petitioner filed a comment, wherein it reiterated that respondent was not a legitimate labor organization at the time of the filing of the petition. Petitioner also propounded that contrary to respondent’s objectives of establishing an organization representing rank-and-file employees, two of respondent’s officers, namely Vice-President Emannuel L. Rosell and Secretary Bathan, were actually supervisory employees. In support of this allegation, petitioner attached various documents evidencing the designation of these two officers in supervisory roles, as well as their exercise of various supervisory functions.[9] Petitioner cited Article 245 of the Labor Code, which provides that supervisory employees shall not be eligible for membership in a labor organization of the rank-and-file employees.

Agreeing with the petitioner company, the Med-Arbiter issued an Order dismissing respondent’s petition for certification election. The sole ground relied upon for the dismissal was the Med-Arbiter’s Opinion that as of the date of filing of the petition on 15 June 1998, respondent did not have the legal personality to file the said petition for certification election.

Respondent union appealed the Med-Arbiter’s order to the DOLE which reversed the same. The DOLE concluded that respondent acquired legal personality as early as 15 June 1998, the date it submitted the required documents, citing Section 3, Rule VI of the New Rules Implementing the Labor Code which deems

6

that a local/chapter acquires legal personality from the date of filing of the complete documentary requirements as mandated in the Implementing Rules. DOLE’s ruling was also affirmed by the CA.

Issues: 1. Whether or not the union has already acquired legal personality at the time of its filing for certification election. 2. Whether or not the union’s president and secretary are supervisory employees and thus barred from membership in that union.

Ruling 1. The SC held that the union has already acquired legal personality at the time of its filing for certification election. Section 3, Rule VI of Department Order No. 9 provides when the local/chapter acquires legal personality.

Section 3.Acquisition of legal personality by local chapter. – A local/chapter constituted in accordance with Section 1 of this Rule shall acquire legal personality from the date of filing of the complete documents enumerated therein. Upon compliance with all the documentary requirements, the Regional Office or Bureau shall issue in favor of the local/chapter a certificate indicating that it is included in the roster of legitimate labor organizations.

It is evident based on this rule that the local/chapter acquires legal personality from the date of the filing of the complete documentary requirements, and not from the issuance of a certification to such effect by the Regional Office or Bureau. On the other hand, a labor organization is deemed to have acquired legal personality only on the date of issuance of its certificate of registration, which takes place only after the Bureau of Labor Relations or its Regional Offices has undertaken an evaluation process lasting up until thirty (30) days, within which period it approves or denies the application. In contrast, no such period of evaluation is provided in Department Order No. 9 for the application of a local/chapter, and more importantly, under it such local/chapter is deemed to acquire legal personality “from the date of filing” of the documents enumerated under Section 1, Rule VI, Book V.

Apart from promoting a policy of affiliation of local unions with national unions, there is a practical reason for sanctioning a less onerous procedure for the registration of a local/chapter, as compared to the national union. The local/chapter relies in part on the legal personality of the federation or national union, which in turn, had already undergone evaluation and approval from the Bureau of Legal Relations or Regional Office. In fact, a federation or national union is required, upon registration, to establish proof of affiliation of at least ten (10) locals or chapters which are duly recognized as the collective bargaining agent in the establishment or industry in which they operate; and the names and addresses of the companies where the locals or chapters operate and the list of all the members in each of the companies. Once the national union or federation acquires legal personality upon the issuance of its certificate or registration, its legal personality cannot be subject to collateral attack.

The fact that the local/chapter acquires legal personality from the moment the complete documentary requirements are submitted seems to imply that the duty of the Bureau or Regional Office to register the local/chapter is merely ministerial.

It is thus very clear that the issuance of the certificate of registration by the Bureau or Regional Office is not the operative act that vests legal personality upon a local/chapter under Department Order No. 9. Such legal personality is acquired from the filing of the complete documentary requirements enumerated in Section 1, Rule VI. Admittedly, the manner by which respondent was deemed to have acquired legal personality by the DOLE and the Court of Appeals was not in strict conformity with the provisions of Department Order No. 9. In the ordinary course, it should have been FFW, and not respondent, which should have submitted the subject documents to the Regional Office. Nonetheless, there is no good reason to deny legal personality or defer its conferral to the local/chapter if it is evident at the onset that the federation or national union itself has already through its own means established the local/chapter. In this case, such is evidenced by the

7

Charter Certificate dated 9 June 1998, issued by FFW, and attached to the petition for certification election. The Charter Certificate expressly states that respondent has been issued the said certificate “to operate as a local or chapter of the [FFW]”. The Charter Certificate expressly acknowledges FFW’s intent to establish respondent as of 9 June 1998. This being the case, the SC consider it permissible for respondent to have submitted the required documents itself to the Regional Office, and proper that respondent’s legal personality be deemed existent as of 15 June 1998, the date the complete documents were submitted. 2. The SC held that they are not supervisory employees and are thus qualified to join the rank-and-file union. Under the law, a managerial employee is “one who is vested with powers or prerogatives to lay down and execute management policies and/or to hire, transfer, suspend, layoff, recall, discharge, assign or discipline employees.” A supervisory employee is “one who, in the interest of the employer, effectively recommends managerial actions if the exercise of such recommendatory authority is not merely routinary or clerical in nature but requires the use of independent judgment.’” Finally, “all employees not falling within the definition of managerial or supervisory employee are considered rank-and-file employees”. It is also wellsettled that the actual functions of an employee, not merely his job title, are determinative in classifying such employee as managerial, supervisory or rank and file. In the case of Emmanuel Rossell, appellant’s evidence shows that he undertakes the filling out of evaluation reports on the performance of mechanics, which in turn are used as basis for reclassification. Given a ready and standard form to accomplish, coupled with the nature of the evaluation, it would appear that his functions are more routinary than recommendatory and hardly leave room for independent judgment. In the case of Noel Bathan, appellant’s evidence does not show his job title although it shows that his recommendations on disciplinary actions appear to have carried some weight on higher management. On this limited point, he may qualify as a supervisory employee within the meaning of the law. This may, however, be outweighed by his other functions which are not specified in the evidence. Assuming that Bathan is a supervisory employee, this does not prove the existence of fraud, false statement or misrepresentation. Because good faith is presumed in all representations, an essential element of fraud, false statement and misrepresentation in order for these to be actionable is intent to mislead by the party making the representation. In this case, there is no proof to show that Bathan, or appellee union for that matter, intended to mislead anyone. If this was appellee union’s intention, it would have refrained from using a more precise description of the organization instead of declaring that the organization is composed of ‘rank and file monthlies’. Hence, the charge of fraud, false statement or misrepresentation cannot be sustained. Petition is denied.

GSIS VS. KAPISANAN NG MGA MANGGAGAWA SA GSIS, G.R. NO. 170132, DECEMBER 6, 2006 Facts: This case has its genesis when the manager of GSIS issued a memorandum directing a number of its employees who are union members to show cause why they should not be charged administratively for their participation in the October 4 to October 7, 2004 mass action. The union’s counsel sought reconsideration of said directive on the ground, among others, that the subject employees resumed work in obedience to the return-to-work order thus issued. The plea for reconsideration was, however, effectively denied by the filing, of administrative charges against some 110 union members for grave misconduct and conduct prejudicial to the best interest of the service. The union then filed with the CA a petition for prohibition against the GSIS on the ground that its members should not be made to explain why they supported their union’s cause since the Civil Service Resolution No. 021316, otherwise known as the Guidelines for Prohibited Mass Action, Section 10 of which exhorts government agencies to “harness all means within their capacity to accord due regard and attention to employees’ grievances and facilitate their speedy and amicable disposition through the use of grievance machinery or any other modes of settlement sanctioned by law and existing civil service rules.” It argued that the organized demonstrating employees did nothing more than air their grievances in the exercise of their “broader rights of free expression” and are, therefore, not amenable to administrative sanctions. On the other hand, petitioners assert that the filing of the formal charges are but a natural consequence of the service-disrupting rallies and demonstrations staged during office hours by the absenting GSIS employees, there being appropriate issuances outlawing such kinds of mass action.

8

The CA ruled in favor of the union and held that the filing of administrative charges against the union members is tantamount to grave abuse of discretion which may be the proper subject of the writ of prohibition.

Issue: Whether or not the mass action staged by or participated in by said GSIS employees partook of a strike or prohibited concerted mass action.

Ruling: The SC held that the mass action staged by or participated in by said GSIS employees partook of a strike or prohibited concerted mass action. It may be that the freedom of expression and assembly and the right to petition the government for a redress of grievances stand on a level higher than economic and other liberties. Any suggestion, however, about these rights as including the right on the part of government personnel to strike ought to be, as it has been, trashed.

The Constitution itself qualifies its exercise with the provision “in accordance with law.” This is a clear manifestation that the state may, by law, regulate the use of this right, or even deny certain sectors such right. Executive Order 180 which provides guidelines for the exercise of the right of government workers to organize, for instance, implicitly endorsed an earlier CSC circular which “enjoins under pain of administrative sanctions, all government officers and employees from staging strikes, demonstrations, mass leaves, walkouts and other forms of mass action which will result in temporary stoppage or disruption of public service” by stating that the Civil Service law and rules governing concerted activities and strikes in government service shall be observed.

The settled rule in this jurisdiction is that employees in the public service may not engage in strikes, mass leaves, walkouts, and other forms of mass action that will lead in the temporary stoppage or disruption of public service. The right of government employees to organize is limited to the formation of unions or associations only, without including the right to strike, adding that public employees going on disruptive unauthorized absences to join concerted mass actions may be held liable for conduct prejudicial to the best interest of the service.

With the view we take of the events that transpired on October 4-7, 2004, what respondent’s members launched or participated in during that time partook of a strike or, what contextually amounts to the same thing, a prohibited concerted activity. The phrase “prohibited concerted activity” refers to any collective activity undertaken by government employees, by themselves or through their employees’ organization, with the intent of effecting work stoppage or service disruption in order to realize their demands or force concessions, economic or otherwise; it includes mass leaves, walkouts, pickets and acts of similar nature. Indeed, for four straight days, participating KMG members and other GSIS employees staged a walk out and waged or participated in a mass protest or demonstration right at the very doorstep of the GSIS main office building. The record of attendance for the period material shows that, on the first day of the protest, 851 employees, or forty eight per cent (48%) of the total number of employees in the main office (1,756) took to the streets during office hours, from 6 a.m. to 2 p.m., leaving the other employees to fend for themselves in an office where a host of transactions take place every business day. On the second day, 707 employees left their respective work stations, while 538 participated in the mass action on the third day. A smaller number, i.e., 306 employees, but by no means an insignificant few, joined the fourth day activity. To say that there was no work disruption or that the delivery of services remained at the usual level of efficiency at the GSIS main office during those four (4) days of massive walkouts and wholesale absences would be to understate things. And to place the erring employees beyond the reach of administrative accountability would be to trivialize the civil service rules, not to mention the compelling spirit of professionalism exacted of civil servants by the Code of Conduct and Ethical Standards for Public Officials and Employees. Ruling of the CA is reversed.

9

DIOKNO VS. CACDAC, G.R. NO. 168475, JULY 4, 2007 Facts: The First Line Association of Meralco Supervisory Employees (FLAMES) is a legitimate labor organization which is the supervisory union of Meralco.Petitioners and private respondents are members of FLAMES. On 1 April 2003, the FLAMES Executive Board created the Committee on Election (COMELEC) for the conduct of its union elections scheduled on 7 May 2003.The COMELEC was composed of petitioner Dante M. Tong as its chairman, and petitioners Jaime C. Mendoza and Romeo M. Macapulay as members. Subsequently, private respondents Jimmy S. Ong, Nardito C. Alvarez, Alfredo J. Escall, and Jaime T. Valeriano filed their respective certificates of candidacy.On 12 April 2003, the COMELEC rejected Jimmy S. Ongs candidacy on the ground that he was not a member of FLAMES.Meanwhile, the certificates of candidacy of Nardito C. Alvarez, Alfredo J. Escall, and Jaime T. Valeriano were similarly rejected on the basis of the exclusion of their department from the scope of the existing collective bargaining agreement (CBA).The employees assigned to the aforesaid department are allegedly deemed disqualified from membership in the union for being confidential employees.

On 24 April 2003, private respondents Jimmy S. Ong, Nardito C. Alvarez, Alfredo J. Escall, Jaime T. Valeriano (Ong, et al.), and a certain Leandro M. Tabilog filed a Petition before the Med-Arbitration Unit of the Department of Labor and Employment (DOLE).

On 30 April 2003, DOLE-NCR Regional Director Alex E. Maraan issued an Order directing DOLE personnel to observe the conduct of the FLAMES election on 7 May 2003. On 2 May 2003, petitioners filed a Petition with the COMELEC seeking the disqualification of private respondents Edgardo Daya, Pablo Lucas, Leandro Tabilog, Reynaldo Espiritu, Jose Vito, Antonio de Luna, Armando Yalung, Edwin Layug, Nards Pabilona, Reynaldo Reyes, Evangeline Escall, Alberto Alcantara, Rogelio Cervitillo, Marcelino Morelos, and Faustino Ermino (Daya, et al.).

On 6 May 2003, the COMELEC issued a Decision, declaring Daya, et al., officially disqualified to run and/or to participate in the 7 May 2003 FLAMES elections.

According to the COMELEC, Daya, et al., violated Article IV, Section 4(a)(6) of the FLAMES Constitution and By-Laws (CBL) by allowing non-members to aid them in their campaign.Their acts of solicitation for support from non-union members were deemed inimical to the interest of FLAMES.

On 7 May 2003, the COMELEC proclaimed the following candidates, including some of herein petitioners as winners of the elections.

On 8 May 2003, private respondents Daya, et al., along with Ong, et al., filed with the Med-Arbitration Unit of the DOLE-NCR, a Petition to: a) Nullify Order of Disqualification; b) Nullify Election Proceedings and Counting of Votes; c) Declare Failure of Election; and d) Declare Holding of New Election to be Controlled and Supervised by the DOLE.

On 14 May 2003, another group led by private respondent Gaudencio Jimenez, Jr., along with private respondents Johnson S. Reyes, Gavino R. Vidanes, Arnaldo G. Tayao, Bonifacio F. Cirujano, Edgardo G. Cadavona, Maximo A. Caoc, Jose O. Maclit, Jr., Luzmindo D. Acorda, Jr., Lemuel R. Ragasa and Gil G. de Vera (Jimenez, et al.) filed a Petition with the Med-Arbitration Unit of the DOLE-NCR against petitioners to nullify the 7 May 2003 election on the ground that the same was not free, orderly, and peaceful.It was docketed as Case No. NCR-OD-0305-004-LRD, which was subsequently consolidated with the Petition of Daya, et al. and the earlier Petition of Ong, et al.

Meanwhile, the records show that a subsequent election was held on 30 June 2004, which was participated in and won by herein private respondents Daya, et al.

10

The Court of Appeals, in the aforesaid case, rendered a Decision dated 12 January 2007, upholding the validity of the 30 June 2004 elections, and the declaration of herein private respondents Daya, et al., as the duly elected winners therein. On 7 July 2003, Med-Arbiter Tranquilino B. Reyes, Jr. issued a Decision in favor of private respondents, Daya, et al.However, the petition of Jimenez, et al.,was dismissed because it was premature, it appearing that the COMELEC had not yet resolved their protest prior to their resort to the Med-Arbiter.Finally, the Petition of Ong, et al., seeking to declare themselves as bona fide members of FLAMES was ordered dismissed.

Aggrieved, petitioners filed an appeal before the Director of the BLR.On 3 December 2003, the Director of the BLR issued a Resolution, affirming in toto the assailed Decision of the Med-Arbiter.Petitioners elevated the case to the Court of Appeals via a Petition for Certiorari.The Court of Appeals found petitioners appeal to be bereft of merit. The appellate court held that the provision relied upon by the COMELEC concerns the dismissal and/or expulsion of union members, which power is vested in the FLAMES Executive Board, and not the COMELEC.It affirmed the finding of the BLR Director that the COMELEC, in disqualifying private respondents Daya, et al., committed a procedural shortcut.

Petitioners sought a reconsideration of the 17 June 2004 Decision of the Court of Appeals, but the same was denied Hence, the instant Petition.

Issue: Whether the Court of Appeals committed grave abuse of discretion when it affirmed the jurisdiction of the BLR to take cognizance of the case and then upheld the ruling of the BLR Director and Med-Arbiter, nullifying the COMELECs order of disqualification of private respondents Daya et al., and annulling the 7 May 2003 FLAMES elections.

Ruling: The Petition is devoid of merit.We affirm the finding of the Court of Appeals upholding the jurisdiction of the BLR.Article 226 of the Labor Code is hereunder reproduced, to wit:

ART. 226. BUREAU OF LABOR RELATIONS. The Bureau of Labor Relations and the Labor Relations Divisions in the regional offices of the Department of Labor shall have original and exclusive authority to act, at their own initiative or upon request of either or both parties, on all inter-union and intra-union conflicts, and all disputes, grievances or problems arising from or affecting labor-management relations in all workplaces whether agricultural or nonagricultural, except those arising from the implementation or interpretation of collective bargaining agreements which shall be the subject of grievance procedure and/or voluntary arbitration.

This Court in Bautista v. Court of Appeals, interpreting Article 226 of the Labor Code, was explicit in declaring that the BLR has the original and exclusive jurisdiction on all inter-union and intra-union conflicts.We said that since Article 226 of the Labor Code has declared that the BLR shall have original and exclusive authority to act on all inter-union and intra-union conflicts, there should be no more doubt as to its jurisdiction.As defined, an intra-union conflict would refer to a conflict within or inside a labor union, while an inter-union controversy or dispute is one occurring or carried on between or among unions. More specifically, an intra-union dispute is defined under Section (z), Rule I of the Rules Implementing Book V of the Labor Code, viz: (z) Intra-Union Dispute refers to any conflict between and among union members, and includes all disputes or grievances arising from any violation of or disagreement over any provision of the constitution and bylaws of a union, including cases arising from chartering or affiliation of labor organizations or from any violation of the rights and conditions of union membership provided for in the Code.

The controversy in the case at bar is an intra-union dispute.There is no question that this is one which involves a dispute within or inside FLAMES, a labor union.At issue is the propriety of the disqualification of private respondents Daya, et al., by the FLAMES COMELEC in the 7 May 2003 elections.It must also be

11

stressed that even as the dispute involves allegations that private respondents Daya, et al., sought the help of non-members of the union in their election campaign to the detriment of FLAMES, the same does not detract from the real character of the controversy.It remains as one which involves the grievance over the constitution and bylaws of a union, and it is a controversy involving members of the union.Moreover, the non-members of the union who were alleged to have aided private respondents Daya, et al., are not parties in the case.We are, therefore, unable to understand petitioners persistence in placing the controversy outside of the jurisdiction of the BLR.The law is very clear. It requires no further interpretation.The Petition which was initiated by private respondents Daya, et al., before the BLR was properly within its cognizance, it being an intra-union dispute. Indubitably, when private respondents Daya, et al.,brought the case to the BLR, it was an invocation of the power and authority of the BLR to act on an intra-union conflict.

The Petition is DENIED. The Decision of the Court of Appeals dated 17 June 2004, and its Resolution dated 10 June 2005 in CA are AFFIRMED. SAN MIGUEL CORP. EMPLOYEES UNION-PTGWO VS. SAN MIGUEL PACKAGING PRODUCTS EMPLOYEES UNION, G.R. NO. 171153, SEPT. 12, 2007 Facts: Petitioner is the incumbent bargaining agent for the bargaining unit comprised of the regular monthly-paid rank and file employees of the three divisions of San Miguel Corporation (SMC), namely, the San Miguel Corporate Staff Unit (SMCSU), San Miguel Brewing Philippines (SMBP), and the San Miguel Packaging Products (SMPP), in all offices and plants of SMC, including the Metal Closure and Lithography Plant in Laguna. It had been the certified bargaining agent for 20 years – from 1987 to 1997.

Respondent is registered as a chapter of Pambansang Diwa ng Manggagawang Pilipino (PDMP). PDMP issued Charter Certificate No. 112 to respondent on 15 June 1999.5 In compliance with registration requirements, respondent submitted the requisite documents to the BLR for the purpose of acquiring legal personality. Upon submission of its charter certificate and other documents, respondent was issued Certificate of Creation of Local or Chapter PDMP-01 by the BLR on 6 July 1999. Thereafter, respondent filed with the Med-Arbiter of the DOLE Regional Officer in the National Capital Region (DOLE-NCR), three separate petitions for certification election to represent SMPP, SMCSU, and SMBP.8 All three petitions were dismissed, on the ground that the separate petitions fragmented a single bargaining unit.9

On 17 August 1999, petitioner filed with the DOLE-NCR a petition seeking the cancellation of respondent's registration and its dropping from the rolls of legitimate labor organizations. In its petition, petitioner accused respondent of committing fraud and falsification, and non-compliance with registration requirements in obtaining its certificate of registration. It raised allegations that respondent violated Articles 239(a), (b) and (c) and 234(c) of the Labor Code. Moreover, petitioner claimed that PDMP is not a legitimate labor organization, but a trade union center, hence, it cannot directly create a local or chapter. On 14 July 2000, DOLE-NCR Regional Director Maximo B. Lim issued an Order dismissing the allegations of fraud and misrepresentation, and irregularity in the submission of documents by respondent. Regional Director Lim further ruled that respondent is allowed to directly create a local or chapter. However, he found that respondent did not comply with the 20% membership requirement and, thus, ordered the cancellation of its certificate of registration and removal from the rolls of legitimate labor organizations. Respondent appealed to the BLR.

While the BLR agreed with the findings of the DOLE Regional Director dismissing the allegations of fraud and misrepresentation, and in upholding that PDMP can directly create a local or a chapter, it reversed the Regional Director's ruling that the 20% membership is a requirement for respondent to attain legal personality as a labor organization. Invoking the power of the appellate court to review decisions of quasi-judicial agencies, petitioner filed with the Court of Appeals a Petition for Certiorari under Rule 65 of the 1997 Rules of Civil Procedure. The Court of Appeals, in a Decision dated 9 March 2005, dismissed the petition and affirmed the Decision of the BLR Hence, this Petition for Certiorari under Rule 45 of the Revised Rules of Court.

12

Issues: 1. Whether or not the private respondent is required to submit the number of employees and names of all its members comprising at least 20% of the employees in the bargaining unit where it seeks to operate

2. Whether or not PDMP as a trade union center is a legitimate labor organization and has the power to create a local or chapter

Ruling:

There is merit in petitioner's contentions.

A legitimate labor organization is defined as "any labor organization duly registered with the Department of Labor and Employment, and includes any branch or local thereof."The mandate of the Labor Code is to ensure strict compliance with the requirements on registration because a legitimate labor organization is entitled to specific rights under the Labor Code, and are involved in activities directly affecting matters of public interest. Registration requirements are intended to afford a measure of protection to unsuspecting employees who may be lured into joining unscrupulous or fly-by-night unions whose sole purpose is to control union funds or use the labor organization for illegitimate ends.Legitimate labor organizations have exclusive rights under the law which cannot be exercised by non-legitimate unions, one of which is the right to be certified as the exclusive representative of all the employees in an appropriate collective bargaining unit for purposes of collective bargaining. The acquisition of rights by any union or labor organization, particularly the right to file a petition for certification election, first and foremost, depends on whether or not the labor organization has attained the status of a legitimate labor organization.

The procedure for registration of a local or chapter of a labor organization is provided in Book V of the Implementing Rules of the Labor Code, as amended by Department Order No. 9 which took effect on 21 June 1997, and again by Department Order No. 40 dated 17 February 2003. The Implementing Rules as amended by D.O. No. 9 should govern the resolution of the petition at bar since respondent's petition for certification election was filed with the BLR in 1999; and that of petitioner on 17 August 1999. The applicable Implementing Rules enunciates a two-fold procedure for the creation of a chapter or a local. The first involves the affiliation of an independent union with a federation or national union or industry union. The second, finding application in the instant petition, involves the direct creation of a local or a chapter through the process of chartering.

A duly registered federation or national union may directly create a local or chapter by submitting to the DOLE Regional Office or to the BLR two copies of the following: (a) A charter certificate issued by the federation or national union indicating the creation or establishment of the local/chapter; (b) The names of the local/chapter's officers, their addresses, and the principal office of the local/chapter; and (c) The local/chapter's constitution and by-laws; Provided, That where the local/chapter's constitution and by-laws is the same as that of the federation or national union, this fact shall be indicated accordingly. All the foregoing supporting requirements shall be certified under oath by the Secretary or the Treasurer of the local/chapter and attested to by its President.

The Implementing Rules stipulate that a local or chapter may be directly created by a federation or national union. A duly constituted local or chapter created in accordance with the foregoing shall acquire legal personality from the date of filing of the complete documents with the BLR. The issuance of the certificate of registration by the BLR or the DOLE Regional Office is not the operative act that vests legal personality

13

upon a local or a chapter under Department Order No. 9. Such legal personality is acquired from the filing of the complete documentary requirements enumerated in Section 1, Rule VI.

Petitioner insists that Section 3 of the Implementing Rules, as amended by Department Order No. 9, violated Article 234 of the Labor Code when it provided for less stringent requirements for the creation of a chapter or local. This Court disagrees.

Article 234 of the Labor Code provides that an independent labor organization acquires legitimacy only upon its registration with the BLR:

Any applicant labor organization, association or group of unions or workers shall acquire legal personality and shall be entitled to the rights and privileges granted by law to legitimate labor organizations upon issuance of the certificate of registration based on the following requirements:

(a) Fifty pesos (P50.00) registration fee; (b) The names of its officers, their addresses, the principal address of the labor organization, the minutes of the organizational meetings and the list of the workers who participated in such meetings; (c) The names of all its members comprising at least twenty percent (20%) of all the employees in the bargaining unit where it seeks to operate; (d) If the applicant union has been in existence for one or more years, copies of its annual financial reports; and (e) Four (4) copies of the constitution and by-laws of the applicant union, minutes of its adoption or ratification, and the list of the members who participated in it. (Italics supplied.)

It is emphasized that the foregoing pertains to the registration of an independent labor organization, association or group of unions or workers. However, the creation of a branch, local or chapter is treated differently. This Court, in the landmark case of Progressive Development Corporation v. Secretary, Department of Labor and Employment, declared that when an unregistered union becomes a branch, local or chapter, some of the aforementioned requirements for registration are no longer necessary or compulsory. Whereas an applicant for registration of an independent union is mandated to submit, among other things, the number of employees and names of all its members comprising at least 20% of the employees in the bargaining unit where it seeks to operate, as provided under Article 234 of the Labor Code and Section 2 of Rule III, Book V of the Implementing Rules, the same is no longer required of a branch, local or chapter. The intent of the law in imposing less requirements in the case of a branch or local of a registered federation or national union is to encourage the affiliation of a local union with a federation or national union in order to increase the local union's bargaining powers respecting terms and conditions of labor.

Subsequently, in Pagpalain Haulers, Inc. v. Trajano where the validity of Department Order No. 9 was directly put in issue, this Court was unequivocal in finding that there is no inconsistency between the Labor Code and Department Order No. 9.As to petitioner's claims that respondent obtained its Certificate of Registration through fraud and misrepresentation, this Court finds that the imputations are not impressed with merit. In the instant case, proof to declare that respondent committed fraud and misrepresentation remains wanting. This Court had, indeed, on several occasions, pronounced that registration based on false and fraudulent statements and documents confer no legitimacy upon a labor organization irregularly recognized, which, at best, holds on to a mere scrap of paper. Under such circumstances, the labor organization, not being a legitimate labor organization, acquires no rights.

This Court emphasizes, however, that a direct challenge to the legitimacy of a labor organization based on fraud and misrepresentation in securing its certificate of registration is a serious allegation which deserves careful scrutiny. Allegations thereof should be compounded with supporting circumstances and evidence. The records of the case are devoid of such evidence. Furthermore, this Court is not a trier of facts, and this doctrine applies with greater force in labor cases. Findings of fact of administrative agencies and quasi-

14

judicial bodies, such as the BLR, which have acquired expertise because their jurisdiction is confined to specific matters, are generally accorded not only great respect but even finality. Still, petitioner postulates that respondent was not validly and legitimately created, for PDMP cannot create a local or chapter as it is not a legitimate labor organization, it being a trade union center.

Petitioner's argument creates a predicament as it hinges on the legitimacy of PDMP as a labor organization. Firstly, this line of reasoning attempts to predicate that a trade union center is not a legitimate labor organization. In the process, the legitimacy of PDMP is being impugned, albeit indirectly. Secondly, the same contention premises that a trade union center cannot directly create a local or chapter through the process of chartering.

Anent the foregoing, as has been held in a long line of cases, the legal personality of a legitimate labor organization, such as PDMP, cannot be subject to a collateral attack. The law is very clear on this matter. Article 212 (h) of the Labor Code, as amended, defines a legitimate labor organization as "any labor organization duly registered with the DOLE, and includes any branch or local thereof." On the other hand, a trade union center is any group of registered national unions or federations organized for the mutual aid and protection of its members; for assisting such members in collective bargaining; or for participating in the formulation of social and employment policies, standards, and programs, and is duly registered with the DOLE in accordance with Rule III, Section 2 of the Implementing Rules.

The Implementing Rules stipulate that a labor organization shall be deemed registered and vested with legal personality on the date of issuance of its certificate of registration. Once a certificate of registration is issued to a union, its legal personality cannot be subject to collateral attack. It may be questioned only in an independent petition for cancellation in accordance with Section 5 of Rule V, Book V of the Implementing Rules. The aforementioned provision is enunciated in the following: Sec. 5.Effect of registration. The labor organization or workers' association shall be deemed registered and vested with legal personality on the date of issuance of its certificate of registration. Such legal personality cannot thereafter be subject to collateral attack, but may be questioned only in an independent petition for cancellation in accordance with these Rules.

PDMP was registered as a trade union center and issued Registration Certificate No. FED-11558-LC by the BLR on 14 February 1991. Until the certificate of registration of PDMP is cancelled, its legal personality as a legitimate labor organization subsists. Once a union acquires legitimate status as a labor organization, it continues to be recognized as such until its certificate of registration is cancelled or revoked in an independent action for cancellation. It bears to emphasize that what is being directly challenged is the personality of respondent as a legitimate labor organization and not that of PDMP. This being a collateral attack, this Court is without jurisdiction to entertain questions indirectly impugning the legitimacy of PDMP.

Corollarily, PDMP is granted all the rights and privileges appurtenant to a legitimate labor organizationand continues to be recognized as such until its certificate of registration is successfully impugned and thereafter cancelled or revoked in an independent action for cancellation.We now proceed to the contention that PDMP cannot directly create a local or a chapter, it being a trade union center.This Court reverses the finding of the appellate court and BLR on this ground, and rules that PDMP cannot directly create a local or chapter.

After an exhaustive study of the governing labor law provisions, both statutory and regulatory, we find no legal justification to support the conclusion that a trade union center is allowed to directly create a local or chapter through chartering. Apropos, we take this occasion to reiterate the first and fundamental duty of this Court, which is to apply the law. The solemn power and duty of the Court to interpret and apply the law does not include the power to correct by reading into the law what is not written therein.

Presidential Decree No. 442, better known as the Labor Code, was enacted in 1972. Being a legislation on social justice, the provisions of the Labor Code and the Implementing Rules have been subject to several amendments, and they continue to evolve, considering that labor plays a major role as a socio-economic force. The Labor Code was first amended by Republic Act No. 6715, and recently, by Republic Act No. 9481.

15

Incidentally, the term trade union center was never mentioned under Presidential Decree No. 442, even as it was amended by Republic Act No. 6715. The term trade union center was first adopted in the

Implementing Rules, under Department Order No. 9.

Culling from its definition as provided by Department Order No. 9, a trade union center is any group of registered national unions or federations organized for the mutual aid and protection of its members; for assisting such members in collective bargaining; or for participating in the formulation of social and employment policies, standards, and programs, and is duly registered with the DOLE in accordance with Rule III, Section 2 of the Implementing Rules. The same rule provides that the application for registration of an industry or trade union center shall be supported by the following: (a) The list of its member organizations and their respective presidents and, in the case of an industry union, the industry where the union seeks to operate; (b) The resolution of membership of each member organization, approved by the Board of Directors of such union; (c) The name and principal address of the applicant, the names of its officers and their addresses, the minutes of its organizational meeting/s, and the list of member organizations and their representatives who attended such meeting/s; and (d) A copy of its constitution and by-laws and minutes of its ratification by a majority of the presidents of the member organizations, provided that where the ratification was done simultaneously with the organizational meeting, it shall be sufficient that the fact of ratification be included in the minutes of the organizational meeting.

Evidently, while a "national union" or "federation" is a labor organization with at least ten locals or chapters or affiliates, each of which must be a duly certified or recognized collective bargaining agent; a trade union center, on the other hand, is composed of a group of registered national unions or federations. The Implementing Rules, as amended by Department Order No. 9, provide that "a duly registered federation or national union" may directly create a local or chapter. The provision reads: Section 1.Chartering and creation of a local/chapter. – A duly registered federation or national union may directly create a local/chapter by submitting to the Regional Office or to the Bureau two (2) copies of the following:

(a) A charter certificate issued by the federation or national union indicating the creation or establishment of the local/chapter; (b) The names of the local/chapter's officers, their addresses, and the principal office of the local/chapter; and (c) The local/chapter's constitution and by-laws; provided that where the local/chapter's constitution and by-laws is the same as that of the federation or national union, this fact shall be indicated accordingly.

All the foregoing supporting requirements shall be certified under oath by the Secretary or the Treasurer of the local/chapter and attested to by its President. Department Order No. 9 mentions two labor organizations either of which is allowed to directly create a local or chapter through chartering – a duly registered federation or a national union. Department Order No. 9 defines a "chartered local" as a labor organization in the private sector operating at the enterprise level that acquired legal personality through a charter certificate, issued by a duly registered federation or national union and reported to the Regional Office in accordance with Rule III, Section 2-E of these Rules. Republic Act No. 9481 or "An Act Strengthening the Workers' Constitutional Right to Self-Organization, Amending for the Purpose Presidential Decree No. 442, As Amended, Otherwise Known as the Labor Code of the Philippines" lapsed into law on 25 May 2007 and became effective on 14 June 2007. This law further amends the Labor Code provisions on Labor Relations. Article 234 now includes the term trade union center, but interestingly, the provision indicating the procedure for chartering or creating a local or chapter, namely Article 234-A, still makes no mention of a "trade union center."

16

Also worth emphasizing is that even in the most recent amendment of the implementing rules, there was no mention of a trade union center as being among the labor organizations allowed to charter. This Court deems it proper to apply the Latin maxim expressio unius est exclusio alterius. Under this maxim of statutory interpretation, the expression of one thing is the exclusion of another. When certain persons or things are specified in a law, contract, or will, an intention to exclude all others from its operation may be inferred. If a statute specifies one exception to a general rule or assumes to specify the effects of a certain provision, other exceptions or effects are excluded. Where the terms are expressly limited to certain matters, it may not, by interpretation or construction, be extended to other matters. Such is the case here. If its intent were otherwise, the law could have so easily and conveniently included "trade union centers" in identifying the labor organizations allowed to charter a chapter or local. Anything that is not included in the enumeration is excluded therefrom, and a meaning that does not appear nor is intended or reflected in the very language of the statute cannot be placed therein. The rule is restrictive in the sense that it proceeds from the premise that the legislating body would not have made specific enumerations in a statute if it had the intention not to restrict its meaning and confine its terms to those expressly mentioned. Expressium facit cessare tacitum. What is expressed puts an end to what is implied. Casus omissus pro omisso habendus est. A person, object or thing omitted must have been omitted intentionally.

Therefore, since under the pertinent status and applicable implementing rules, the power granted to labor organizations to directly create a chapter or local through chartering is given to a federation or national union, then a trade union center is without authority to charter directly.

The ruling of this Court in the instant case is not a departure from the policy of the law to foster the free and voluntary organization of a strong and united labor movement, and thus assure the rights of workers to self-organization. The mandate of the Labor Code in ensuring strict compliance with the procedural requirements for registration is not without reason. It has been observed that the formation of a local or chapter becomes a handy tool for the circumvention of union registration requirements. Absent the institution of safeguards, it becomes a convenient device for a small group of employees to foist a not-sodesirable federation or union on unsuspecting co-workers and pare the need for wholehearted voluntariness, which is basic to free unionism. As a legitimate labor organization is entitled to specific rights under the Labor Code and involved in activities directly affecting public interest, it is necessary that the law afford utmost protection to the parties affected. However, as this Court has enunciated in Progressive Development Corporation v. Secretary of Department of Labor and Employment, it is not this Court's function to augment the requirements prescribed by law. Our only recourse, as previously discussed, is to exact strict compliance with what the law provides as requisites for local or chapter formation.

In sum, although PDMP as a trade union center is a legitimate labor organization, it has no power to directly create a local or chapter. Thus, SMPPEU-PDMP cannot be created under the more lenient requirements for chartering, but must have complied with the more stringent rules for creation and registration of an independent union, including the 20% membership requirement.

Petition is GRANTED. The Decision of the Court of Appeals in is REVERSED and SET ASIDE. The Certificate of Registration of San Miguel Packaging Products Employees Union–Pambansang Diwa ng Manggagawang Pilipino is ORDERED CANCELLED, and SMPPEU-PDMP DROPPED from the rolls of legitimate labor organizations. Costs against petitioner.

DONG SEUNG INC., VS. BUREAU OF LABOR RELATIONS, G.R. NO. 162356, APRIL 14, 2008 Facts:

Petitioner filed with the Department of Labor and Employment (DOLE), Region IV a Petition for cancellation of the union registration of respondent union on the grounds that the List of Officers and Constitution and By-laws which the respondent union attached to its application for union registration contain the union secretary's certification but the same is not under oath, contrary to Section 1, Rule VI of the Implementing Rules of Book V of the Labor Code, as amended by Department Order No. 9, series of 1997; and that, as shown in a Sinumpaang Petisyon, 148 out of approximately 200 employees-members have since

17

denounced respondent union for employing deceit in obtaining signatures to support its registration application.

DOLE (Region IV) Regional Director Ricardo Martinez, Sr. delisted from the roster of legitimate labor organization the Charter Certificate [of] NAMAWU-Local 188. Respondent union appealed to the Bureau of Labor Relations BLR gave due course to the appeal and granted the same, it ordered that NAMAWU-Local 188, shall remain in the roster of legitimate labor organizations. After its motion for reconsideration was denied by the BLR, petitioner filed with the CA a Petition for Certiorari, insisting that the BLR acted with grave abuse of discretion in giving due course to respondent union’s appeal despite its having been filed out of time. The CA dismissed the petition and the motion for reconsideration which was subsequently filed by the petitioner. Issue: Whether or not the CA erred in sustaining the BLR when it declared respondent’s union registration valid.

Ruling:

The BLR found respondent union’s appeal tardy yet gave due course to it on account of its inherent merit. The CA found respondent union’s appeal to have “substantially complied with the requirements provided by law.

The requirement that the union secretary certify under oath all documents and papers filed in support of an application for union registration is imposed by Article 235 of the Labor Code, to wit: Art. 235. Action on application. The Bureau shall act on all applications for registration within thirty (30) days from filing. All requisite documents and papers shall be certified under oath by the secretary or the treasurer of the organization, as the case may be, and attested to by its president. In reversing DOLE Region IV, the BLR cited its Advisory, dated October 14, 1998, which interprets the requirement under Article 235, to wit: Pursuant to Rule XVII, Section 1 of Department Order No. 09, Series of 1997 x x x. [T]he Bureau of Labor Relations is empowered, consistent with the State policy to promote unionism, to “devise or prescribe such forms as are necessary to facilitate the process of registration of labor organizations x x x,” including the chartering of locals or chapters. Accordingly, the Bureau has devised and transmitted to the Regional Offices the appropriate official registration forms, particularly the following: xxxx 5. BLR Reg. Form No. 5-LOC-LO. S. 1998 For Chartering Locals/ Chapters xxxx Part I of each of the first seven forms is a space provided for the notarization of the application x x x. However, considering that applicants are not yet fully familiar with the forms in spite of orientation and seminar conducted, some applications have been submitted without using the forms prescribed by the Bureau. In lieu of submitting a notarized application using the official forms, some applicants comply with the requirements by having their supporting documents separately notarized. To prevent inconvenience to the public, particularly to the applicants, the Regional Offices are hereby advised that applications submitted with supporting documents which are separately notarized need not comply with the notarization requirement under Part I or Part II, as the case may be, of the prescribed forms. x x x Accordingly, the absence of notarization under Part I or Part II of the appropriate forms shall not be a basis for denying applications where it appears that all the required supporting documents have already been notarized or attested. (Emphasis supplied)

The BLR explained that under the foregoing Advisory, the certification issued by respondent union’s secretary may be notarized either separately or along with the main application. The BLR noted that respondent union correctly availed of the second option:

18

A perusal of the registration records of the [respondent] revealed that respondent’s registration application was sufficient in form and substance, having been notarized as provided in the BLR official forms. (Atty. Manuel E. Robles notarized such application on 8 February 1999 at Cavite City.) All the other supporting documents to the charter certificate issued by the National Mines and Allied Workers Union were certified true and correct by the secretary and attested to by the president. Thus, from the standpoint of compliance, [respondent] x x x submitted all the documentary requirements for the creation of a local/chapter in accordance with Section 1, Rule VI, D.O. 9 series of 1997. [28] (Emphasis supplied) Indeed, all that Article 235 requires is that the secretary’s certification be under oath. It does not prescribe a specific manner of its notarization. Based on its interpretation of Article 235, the BLR, in its October 14, 1998 Advisory, allows for the wholesale notarization of a union’s application for registration and recognizes the effects thereof even on the attachments, including the secretary’s certification. This is a reasonable interpretation considering that the form of notarization contemplated in said Advisory adequately serves the purpose of Article 235, which is to forestall fraud and misrepresentation. More importantly, such interpretation of the BLR is accorded great weight by the Court for it is said agency which is vested with authority and endowed with expertise to implement the law in question. The other ground cited by DOLE Region IV in canceling the registration of respondent union is that the latter allegedly committed misrepresentation in securing the signatures of its members. The CA and BLR, on the other hand, assign no credence to the Sinumpaang Petisyon for it is a mere photocopy, the genuineness and due execution of which cannot be reasonably ascertained. Another factor which militates against the veracity of the allegations in the Sinumpaang Petisyon is the lack of particularities on how, when and where respondent union perpetrated the alleged fraud on each member. Such details are crucial for in the proceedings for cancellation of union registration on the ground of fraud or misrepresentation, what needs to be established is that the specific act or omission of the union deprived the complaining employeesmembers of their right to choose.

WHEREFORE, the petition is DENIED.

DEL PILAR ACADEMY ET AL., VS. DEL PILAR ACADEMY EMPLOYEES UNION, G.R. NO. 170112, APRIL 30, 2008 Facts:

Respondent Del Pilar Academy Employees Union (the UNION) is the certified collective bargaining representative of teaching and non-teaching personnel of petitioner Del Pilar Academy (DEL PILAR), an educational institution operating in Imus, Cavite.

On September 15, 1994, the UNION and DEL PILAR entered into a Collective Bargaining Agreement (CBA) granting salary increase and other benefits to the teaching and non-teaching staff.

The UNION then assessed agency fees from non-union employees, and requested DEL PILAR to deduct said assessment from the employees’ salaries and wages. DEL PILAR, however, refused to effect deductions claiming that the non-union employees were not amenable to it.

Traversing the complaint, DEL PILAR denied committing unfair labor practices against the UNION. It justified the non-deduction of the agency fees by the absence of individual check off authorization from the non-union employees. As regards the proposal to amend the provision on summer vacation leave with pay, DEL PILAR alleged that the proposal cannot be considered unfair for it was done to make the provision of the CBA conformable to the DECS’ Manual of Regulations for Private Schools.

The Labor Arbiter ruled in favor of the union that DEL PILAR should have deducted the union fees from the non-union employees citing article 248 of the Labor Code. On appeal, the National Labor Relations Commission (NLRC) affirmed the Arbiter’s ruling. In gist, it upheld the UNION’s right to agency fee, but did

19

not consider DEL PILAR’s failure to deduct the same an unfair labor practice. The UNION’s motion for reconsideration having been denied, it then went to the CA via certiorari. On July 19, 2005, the CA rendered the assailed decision, affirming with modification the resolutions of the NLRC.

Like the Arbiter and the NLRC, the CA upheld the UNION’s right to collect agency fees from non-union employees, but did not adjudge DEL PILAR liable for unfair labor practice. However, it ordered DEL PILAR to deduct agency fees from the salaries of non-union employees.

Issue:

Whether or not the UNION is entitled to collect agency fees from non-union members, and if so, whether an individual written authorization is necessary for a valid check off.

Ruling:

The collection of agency fees in an amount equivalent to union dues and fees, from employees who are not union members, is recognized by Article 248(e) of the Labor Code. When so stipulated in a collective bargaining agreement or authorized in writing by the employees concerned, the Labor Code and its Implementing Rules recognize it to be the duty of the employer to deduct the sum equivalent to the amount of union dues, as agency fees, from the employees' wages for direct remittance to the union. The system is referred to as check off. No requirement of written authorization from the non-union employees is necessary if the non-union employees accept the benefits resulting from the CBA.

DEL PILAR urges this Court to reverse the CA ruling insofar as it ordered the deduction of agency fees from the salaries of non-union employees, arguing that such conclusion proceeds from a misplaced premise that the salary increase arose from the CBA.

The argument cannot be sustained.

Contrary to what DEL PILAR wants to portray, the grant of annual salary increase is not the only provision in the CBA that benefited the non-union employees. The UNION negotiated for other benefits, namely, limitations on teaching assignments to 23 hours per week, additional compensation for overload units or teaching assignments in excess of the 23 hour per week limit, and payment of longevity pay. It also negotiated for entitlement to summer vacation leave with pay for two (2) months for teaching staff who have rendered six (6) consecutive semesters of service. For the non-teaching personnel, the UNION worked for their entitlement to fifteen (15) days leave with pay. These provisions in the CBA surely benefited the non-union employees, justifying the collection of, and the UNION’s entitlement to, agency fees.

Accordingly, no requirement of written authorization from the non-union employees is needed to effect a valid check off. Article 248(e) makes it explicit that Article 241, paragraph (o), [requiring written authorization is inapplicable to non-union members, especially in this case where the non-union employees receive several benefits under the CBA.

As explained by this Court in Holy Cross of Davao College, Inc. v. Hon. Joaquin viz.:

The employee's acceptance of benefits resulting from a collective bargaining agreement justifies the deduction of agency fees from his pay and the union's entitlement thereto. In this aspect, the legal basis of the union's right to agency fees is neither contractual nor statutory, but quasi-contractual, deriving from the established principle that non-union employees may not unjustly enrich themselves by benefiting from employment conditions negotiated by the bargaining union.

20

WHEREFORE, the petition is DENIED. The Decision and Resolution of the Court of Appeals in CA-G.R. SP No. 86868, are AFFIRMED. SO ORDERED.

S.S. VENTURES INTERNATIONAL INC., VS. SS VENTURES LABOR UNION, G.R. NO. 161690, JULY 23, 2008

Facts: Petitioner S.S. Ventures International, Inc. (Ventures), a PEZA-registered export firm with principal place of business at Phase I-PEZA-Bataan Export Zone, Mariveles, Bataan, is in the business of manufacturing sports shoes. Respondent S.S. Ventures Labor Union (Union), on the other hand, is a labor organization registered with the Department of Labor and Employment (DOLE).

On March 21, 2000, the Union filed with DOLE-Region III a petition for certification election in behalf of the rank-and-file employees of Ventures. Five hundred forty two (542) signatures, 82 of which belong to terminated Ventures employees, appeared on the basic documents supporting the petition.

On August 21, 2000, Ventures filed a Petition to cancel the Unions certificate of registration invoking the grounds set forth in Article 239(a) of the Labor Code, the petition alleged the following:

(1) The Union deliberately and maliciously included the names of more or less 82 former employees no longer connected with Ventures in its list of members who attended the organizational meeting and in the adoption/ratification of its constitution and by-laws held on January 9, 2000 in Mariveles, Bataan; and the Union forged the signatures of these 82 former employees to make it appear they took part in the organizational meeting and adoption and ratification of the constitution; (2) The Union maliciously twice entered the signatures of three persons namely: Mara Santos, Raymond Balangbang, and Karen Agunos; (3) No organizational meeting and ratification actually took place; and (4) The Unions application for registration was not supported by at least 20% of the rank-and-file employees of Ventures, or 418 of the total 2,197-employee complement. Since more or less 82 of the 500[3] signatures were forged or invalid, then the remaining valid signatures would only be 418, which is very much short of the 439 minimum (2197 total employees x 20% = 439.4) required by the Labor Code.

In a Decision dated April 6, 2001, Regional Director Ana C. Dione of DOLE-Region III found for Ventures. Aggrieved, the Union interposed a motion for reconsideration, a recourse which appeared to have been forwarded to the Bureau of Labor Relations (BLR). Although it would later find this motion to have been belatedly filed, the BLR, over the objection of Ventures which filed a Motion to Expunge, gave it due course and treated it as an appeal.Despite Venturesmotion to expunge the appeal, the BLR Director rendered on October 11, 2002 a decisionin BLR-A-C-60-6-11-01, granting the Unions appeal and reversing the decision of Dione.

Ventures sought reconsideration of the above decision but was denied by the BLR.Ventures then went to the Court of Appeals (CA) on a petition for certiorari under Rule 65. On October 20, 2003, the CA rendered a Decision, dismissing Ventures petition. Ventures motion for reconsideration met a similar fate.

Hence, this petition for review under Rule 45.

Issue:

21

Whether or not there was fraud or misrepresentation on the part of the Union sufficient to justify cancellation of its registration.

Ruling: The petition lacks merit. The right to form, join, or assist a union is specifically protected by Art. XIII, Section 3 of the Constitution and such right, according to Art. III, Sec. 8 of the Constitution and Art. 246 of the Labor Code, shall not be abridged. Once registered with the DOLE, a union is considered a legitimate labor organization endowed with the right and privileges granted by law to such organization. While a certificate of registration confers a union with legitimacy with the concomitant right to participate in or ask for certification election in a bargaining unit, the registration may be canceled or the union may be decertified as the bargaining unit, in which case the union is divested of the status of a legitimate labor organization. Among the grounds for cancellation is the commission of any of the acts enumerated in Art. 239(a) of the Labor Code, such as fraud and misrepresentation in connection with the adoption or ratification of the unions constitution and like documents. The Court, has in previous cases, said that to decertify a union, it is not enough to show that the union includes ineligible employees in its membership. It must also be shown that there was misrepresentation, false statement, or fraud in connection with the application for registration and the supporting documents, such as the adoption or ratification of the constitution and bylaws or amendments thereto and the minutes of ratification of the constitution or by-laws, among other documents.

Essentially, Ventures faults both the BLR and the CA in finding that there was no fraud or misrepresentation on the part of the Union sufficient to justify cancellation of its registration. In this regard, Ventures makes much of, first, the separate hand-written statements of 82 employees who, in gist, alleged that they were unwilling or harassed signatories to the attendance sheet of the organizational meeting.

We are not persuaded. As aptly noted by both the BLR and CA, these mostly undated written statements submitted by Ventures on March 20, 2001, or seven months after it filed its petition for cancellation of registration, partake of the nature of withdrawal of union membership executed after the Unions filing of a petition for certification election on March 21, 2000. We have in precedent cases said that the employees withdrawal from a labor union made before the filing of the petition for certification election is presumed voluntary, while withdrawal after the filing of such petition is considered to be involuntary and does not affect the same. Now then, if a withdrawal from union membership done after a petition for certification election has been filed does not vitiate such petition, is it not but logical to assume that such withdrawal cannot work to nullify the registration of the union? Upon this light, the Court is inclined to agree with the CA that the BLR did not abuse its discretion nor gravely err when it concluded that the affidavits of retraction of the 82 members had no evidentiary weight.

It cannot be over-emphasized that the registration or the recognition of a labor union after it has submitted the corresponding papers is not ministerial on the part of the BLR. Far from it. After a labor organization has filed the necessary registration documents, it becomes mandatory for the BLR to check if the requirements under Art. 234 of the Labor Code have been sedulously complied with. If the unions application is infected by falsification and like serious irregularities, especially those appearing on the face of the application and its attachments, a union should be denied recognition as a legitimate labor organization. Prescinding from these considerations, the issuance to the Union of Certificate of Registration No. RO300-00-02-UR-0003 necessarily implies that its application for registration and the supporting documents thereof are prima facie free from any vitiating irregularities.

Second, Ventures draws attention to the inclusion of 82 individuals to the list of participants in the January 9, 2000 organizational meeting. Ventures submits that the 82, being no longer connected with the company, should not have been counted as attendees in the meeting and the ratification proceedings immediately afterwards.

The assailed inclusion of the said 82 individuals to the meeting and proceedings adverted to is not really fatal to the Unions cause for, as determined by the BLR, the allegations of falsification of signatures or misrepresentation with respect to these individuals are without basis. The Court need not delve into the question of whether these 82 dismissed individuals were still Union members qualified to vote and affix their signature on its application for registration and supporting documents. Suffice it to say that, as aptly

22

observed by the CA, the procedure for acquiring or losing union membership and the determination of who are qualified or disqualified to be members are matters internal to the union and flow from its right to selforganization.

To our mind, the relevancy of the 82 individuals active participation in the Unions organizational meeting and the signing ceremonies thereafter comes in only for purposes of determining whether or not the Union, even without the 82, would still meet what Art. 234(c) of the Labor Code requires to be submitted, to wit: Art. 234. Requirements of Registration.Any applicant labor organization x x x shall acquire legal personality and shall be entitled to the rights and privileges granted by law to legitimate labor organizations upon issuance of the certificate of registration based on the following requirements:

xxxx

(c) The names of all its members comprising at least twenty percent (20%) of all the employees in the bargaining unit where it seeks to operate.

The bare fact that three signatures twice appeared on the list of those who participated in the organizational meeting would not, to our mind, provide a valid reason to cancel Certificate of Registration No. RO300-00-02-UR-0003. As the Union tenably explained without rebuttal from Ventures, the double entries are no more than normal human error, effected without malice. Even the labor arbiter who found for Ventures sided with the Union in its explanation on the absence of malice.

The cancellation of a unions registration doubtless has an impairing dimension on the right of labor to selforganization. Accordingly, we can accord concurrence to the following apt observation of the BLR: [F]or fraud and misrepresentation [to be grounds for] cancellation of union registration under Article 239 [of the Labor Code], the nature of the fraud and misrepresentation must be grave and compelling enough to vitiate the consent of a majority of union members.

In its Comment, the Union points out that for almost seven (7) years following the filing of its petition, no certification election has yet been conducted among the rank-and-file employees. If this be the case,the delay has gone far enough and can no longer be allowed to continue. The CA is right when it said that Ventures should not interfere in the certification election by actively and persistently opposing the certification election of the Union. A certification election is exclusively the concern of employees and the employer lacks the legal personality to challenge it. In fact, jurisprudence frowns on the employers interference in a certification election for such interference unduly creates the impression that it intends to establish a company union.

WHEREFORE, the petition is DENIED. The Decision and Resolution dated respectively of the CA are AFFIRMED. S.S. Ventures Labor Union shall remain in the roster of legitimate labor organizations, unless it has in the meantime lost its legitimacy for causes set forth in the Labor Code. Costs against petitioner.

INGUILLO ET AL, VS. FIRST PHIL SCALES INC., ET AL., GR NO. 165407, JUNE 5, 2009

Facts: First Philippine Scales, Inc. (FPSI), a domestic corporation engaged in the manufacturing of weighing scales, employed Bergante and Inguillo as assemblers. In 1991, FPSI and First Philippine Scales Industries Labor Union (FPSILU) entered into a Collective Bargaining Agreement, the duration of which was for a period of 5 years starting September 12, 1991 until September 12, 1996. On September 19, 1991, the members of FPSILU ratified the CBA in a document entitled RATIPIKASYON NG KASUNDUAN. Bergante and Inguillo, who were members of FPSILU, signed the said document.

23

During the lifetime of the CBA, Bergante, Inguillo and several FPSI employees joined another union, the Nagkakaisang Lakas ng Manggagawa (NLM), which was affiliated with a federation called KATIPUNAN (NLMKATIPUNAN, for brevity). Subsequently, NLM-KATIPUNAN filed with the Department of Labor and Employment (DOLE) an intra-union dispute against FPSILU and FPSI. In said case, the Med-Arbiter decided in favor of FPSILU. It also ordered the officers and members of NLM-KATIPUNAN to return to FPSILU the amount of P90,000.00 pertaining to the union dues erroneously collected from the employees. Upon finality of the Med-Arbiter's Decision, a Writ of Execution was issued to collect the adjudged amount from NLMKATIPUNAN. However, as no amount was recovered, notices of garnishment were issued to United Coconut Planters Bank and to FPSI for the latter to hold for FPSILU the earnings of Domingo Grutas, Jr. (Grutas) and Inguillo, formerly FPSILU's President and Secretary for Finance, respectively.

The executive board and members of the FPSILU addressed a document denominated as "Petisyon"to FPSI's general manager, Amparo Policarpio (Policarpio), seeking the termination of the services of the following employees, namely: Grutas, Yolanda Tapang, Shirley Tapang, Gerry Trinidad, Gilbert Lucero, Inguillo, Bergante, and Vicente Go, on the following grounds: (1) disloyalty to the Union by separating from it and affiliating with a rival Union, the NLM-KATIPUNAN; (2) dereliction of duty by failing to call periodic membership meetings and to give financial reports; (3) depositing Union funds in the names of Grutas and former Vice-President Yolanda Tapang, instead of in the name of FPSILU, care of the President; (4) causing damage to FPSI by deliberately slowing down production, preventing the Union to even attempt to ask for an increase in benefits from the former; and (5) poisoning the minds of the rest of the members of the Union so that they would be enticed to join the rival union. Inguillo filed with the NLRC a complaint against FPSI and/or Policarpio (respondents) for illegal withholding of salary and damages. On May 16, 1996, respondents terminated the services of the employees mentioned in the "Petisyon."

The following day, two (2) separate complaints for illegal dismissal, reinstatement and damages were filed against respondents by: (1) NLM-KATIPUNAN, Grutas, Trinidad, Bergante, Yolanda Tapang, Go, Shirley Tapang and Lucero (Grutas complaint, for brevity); and (2) Inguillo (Inguillo complaint). Both complaints were consolidated with Inguillo's prior complaint for illegal withholding of salary, which was pending before Labor Arbiter Manuel Manansala. Some of the complainants agreed to amicably settle their cases. Bergante and Inguillo, the remaining complainants, were directed to submit their respective position papers, after which their complaints were submitted for resolution.

The Labor Arbiter dismissed the remaining complaints of Bergante and Inguillo and held that they were not illegally dismissed. He explained that the two clearly violated the Union Security Clause of the CBA when they joined NLM-KATIPUNAN and committed acts detrimental to the interests of FPSILU and respondents. Affirmed by NLRC and when elevated the case to CA, the latter also affirmed the decision of NLRC. CA’s decision: The employer FPSI did nothing but to put in force their agreement when it separated the disaffiliating union members, herein complainants, upon the recommendation of the union. Such a stipulation is not only necessary to maintain loyalty and preserve the integrity of the union, but is allowed by the Magna Carta of Labor when it provided that while it is recognized that an employee shall have the right of self-organization, it is at the same time postulated that such rights shall not injure the right of the labor organization to prescribe its own rules with respect to the acquisition or retention of membership therein. Having ratified their CBA and being then members of FPSILU, the complainants owe fealty and are required under the Union Security clause to maintain their membership in good standing with it during the term thereof, a requirement which ceases to be binding only during the 60-day freedom period immediately preceding the expiration of the CBA, which was not present in this case.

x x x the dismissal of the complainants pursuant to the demand of the majority union in accordance with their union security [clause] agreement following the loss of seniority rights is valid and privileged and does not constitute unfair labor practice or illegal dismissal. Indeed, the Supreme Court has for so long a time already recognized a union security clause in the CBA, like the one at bar, as a specie of closed-shop arrangement and trenchantly upheld the validity of the action of the employer in enforcing its terms as a lawful exercise of its rights and obligations under the contract. The collective bargaining agreement in this case contains a union security clause-a closed-shop agreement.

24

A closed-shop agreement is an agreement whereby an employer binds himself to hire only members of the contracting union who must continue to remain members in good standing to keep their jobs. It is "the most prized achievement of unionism." It adds membership and compulsory dues. By holding out to loyal members a promise of employment in the closed-shop, it welds group solidarity. (National Labor Union v. Aguinaldo's Echague Inc., 97 Phil. 184). It is a very effective form of union security agreement. This Court has held that a closed-shop is a valid form of union security, and such a provision in a collective bargaining agreement is not a restriction of the right of freedom of association guaranteed by the Constitution.Hence, this petition.

Issue: Whether or not the enforcement of the aforesaid Union Security Clause justified herein petitioners' dismissal from the service.

Ruling: Yes, the enforcement justified petitioner’s dismissal. Essentially, the Labor Code of the Philippines has several provisions under which an employee may be validly terminated, namely: (1) just causes under Article 282; (2) authorized causes under Article 283; (3) termination due to disease under Article 284; and (4) termination by the employee or resignation under Article 285. While the said provisions did not mention as ground the enforcement of the Union Security Clause in the CBA, the dismissal from employment based on the same is recognized and accepted in our jurisdiction.

"Union security" is a generic term, which is applied to and comprehends "closed shop," "union shop," "maintenance of membership" or any other form of agreement, which imposes upon employees the obligation to acquire or retain union membership as a condition affecting employment. There is union shop when all new regular employees are required to join the union within a certain period as a condition for their continued employment. There is maintenance of membership shop when employees, who are union members as of the effective date of the agreement, or who thereafter become members, must maintain union membership as a condition for continued employment until they are promoted or transferred out of the bargaining unit or the agreement is terminated. A closed-shop, on the other hand, may be defined as an enterprise in which, by agreement between the employer and his employees or their representatives, no person may be employed in any or certain agreed departments of the enterprise unless he or she is, becomes, and, for the duration of the agreement, remains a member in good standing of a union entirely comprised of or of which the employees in interest are a part.

In their Petition, Bergante and Inguillo assail the legality of their termination based on the Union Security Clause in the CBA between FPSI and FPSILU. Article II of the CBA pertains to Union Security and Representatives, which provides: The Company hereby agrees to a UNION SECURITY [CLAUSE] with the following terms: XXXXXX 5. Any employee/union member who fails to retain union membership in good standing may be recommended for suspension or dismissal by the Union Directorate and/or FPSILU Executive Council for any of the following causes: a) Acts of Disloyalty; b) Voluntary Resignation or Abandonment from the UNION; c) Organization of or joining another labor union or any labor group that would work against the UNION; XXX

XXX

The aforesaid provision requires all members to maintain their membership with FPSILU during the lifetime of the CBA. Records show that Bergante and Inguillo were former members of FPSILU based on their signatures in the document which ratified the CBA. It can also be inferred that they disaffiliated from FPSILU when the CBA was still in force and subsisting, as can be gleaned from the documents relative to the intraunion dispute between FPSILU and NLM-KATIPUNAN. In view of their disaffiliation, as well as other acts

25

allegedly detrimental to the interest of both FPSILU and FPSI, a "Petisyon" was submitted to Policarpio, asking for the termination of the services of employees who failed to maintain their Union membership. The Court is now tasked to determine whether the enforcement of the aforesaid Union Security Clause justified herein petitioners' dismissal from the service.

In the case at bar, in terminating the employment of an employee by enforcing the Union Security Clause, the employer needs only to determine and prove that: (1) the union security clause is applicable; (2) the union is requesting for the enforcement of the union security provision in the CBA; and (3) there is sufficient evidence to support the union's decision to expel the employee from the union or company.

We hold that all the requisites have been sufficiently met and FPSI was justified in enforcing the Union Security Clause, for the following reasons: First. FPSI was justified in applying the Union Security Clause, as it was a valid provision in the CBA, the existence and validity of which was not questioned by either party. Moreover, petitioners were among the 93 employees who affixed their signatures to the document that ratified the CBA. They cannot now turn their back and deny knowledge of such provision.

Second. FPSILU acted on its prerogative to recommend to FPSI the dismissal of the members who failed to maintain their membership with the Union. Aside from joining another rival union, FPSILU cited other grounds committed by petitioners and the other employees which tend to prejudice FPSI’s interests, i.e., dereliction of duty - by failing to call periodic membership meetings and to give financial reports; depositing union funds in the names of Grutas and former Vice-President Yolanda Tapang, instead of in the name of FPSILU care of the President; causing damage to FPSI by deliberately slowing down production, preventing the Union from even attempting to ask for an increase in benefits from the former; and poisoning the minds of the rest of the members of the Union so that they would be enticed to join the rival union.

Third. FPSILU's decision to ask for the termination of the employees in the "Petisyon" was justified and supported by the evidence on record. Bergante and Inguillo were undisputably former members of FPSILU. In fact, Inguillo was the Secretary of Finance, the underlying reason why his salary was garnished to satisfy the judgment of the Med-Arbiter who ordered NLM-KATIPUNAN to return the Union dues it erroneously collected from the employees. Their then affiliation with FPSILU was also clearly shown by their signatures in the document which ratified the CBA. Without a doubt, they committed acts of disloyalty to the Union when they failed not only to maintain their membership but also disaffiliated from it. They abandoned FPSILU and even joined another union which works against the former's interests. This is evident from the intra-union dispute filed by NLM-KATIPUNAN against FPSILU. Once affiliated with NLM-KATIPUNAN, Bergante and Inguillo proceeded to recruit other employees to disaffiliate from FPSILU and even collected Union dues from them.

In Del Monte Philippines, the stipulations in the CBA authorizing the dismissal of employees are of equal import as the statutory provisions on dismissal under the Labor Code, since a CBA is the law between the company and the Union, and compliance therewith is mandated by the express policy to give protection to labor. In Caltex Refinery Employees Association (CREA) v. Brillantes, the Court expounded on the effectiveness of union security clause when it held that it is one intended to strengthen the contracting union and to protect it from the fickleness or perfidy of its own members. For without such safeguards, group solidarity becomes uncertain; the union becomes gradually weakened and increasingly vulnerable to company machinations. In this security clause lies the strength of the union during the enforcement of the collective bargaining agreement. It is this clause that provides labor with substantial power in collective bargaining.

To safeguard the rights of the employees, We have said time and again that dismissals pursuant to union security clauses are valid and legal, subject only to the requirement of due process, that is, notice and hearing prior to dismissal. In like manner, We emphasized that the enforcement of union security clauses is authorized by law, provided such enforcement is not characterized by arbitrariness, and always with due process. There are two (2) aspects which characterize the concept of due process under the Labor Code: one is substantive––whether the termination of employment was based on the provisions of the Labor Code or in

26

accordance with the prevailing jurisprudence; the other is procedural - the manner in which the dismissal was effected. Corollarily, procedural due process in the dismissal of employees requires notice and hearing. The employer must furnish the employee two written notices before termination may be effected. The first notice apprises the employee of the particular acts or omissions for which his dismissal is sought, while the second notice informs the employee of the employer’s decision to dismiss him. The requirement of a hearing, on the other hand, is complied with as long as there was an opportunity to be heard, and not necessarily that an actual hearing was conducted. In the present case, the required two notices that must be given to herein petitioners Bergante and Inguillo were lacking. The records are bereft of any notice that would have given a semblance of substantial compliance on the part of herein respondents. The Court has always underscored the significance of the two-notice rule in dismissing an employee and has ruled in a number of cases that non-compliance therewith is tantamount to deprivation of the employee’s right to due process.

In fine, We hold that while Bergante and Inguillo's dismissals were valid pursuant to the enforcement of Union Security Clause, respondents however did not comply with the requisite procedural due process. As in the case of Agabon v. National Labor Relations Commission, where the dismissal is for a cause recognized by the prevailing jurisprudence, the absence of the statutory due process should not nullify the dismissal or render it illegal, or ineffectual. Accordingly, for violating Bergante and Inguillo's statutory rights, respondents should indemnify them the amount of P30,000.00 each as nominal damages.

STA LUCIA EAST COMMERCIAL CORP., VS. SOLE ET AL., GR NO. 162355, AUGUST 14, 2009 Facts: On 27 February 2001, Confederated Labor Union of the Philippines (CLUP), in behalf of its chartered local, instituted a petition for certification election among the regular rank-and-file employees of Sta. Lucia East Commercial Corporation and its Affiliates. The affiliate companies included in the petition were SLE Commercial, SLE Department Store, SLE Cinema, Robsan East Trading, Bowling Center, Planet Toys, Home Gallery and Essentials. On 21 August 2001, Med-Arbiter Bactin ordered the dismissal of the petition due to inappropriateness of the bargaining unit. CLUP-Sta. Lucia East Commercial Corporation and its Affiliates Workers Union appealed the order of dismissal to this Office on 14 September 2001.On 20 November 2001, CLUP-Sta. Lucia East Commercial Corporation and its Affiliates Workers Union [CLUP-SLECC and its Affiliates Workers Union]moved for the withdrawal of the appeal. On 31 January 2002, this Office granted the motion and affirmed the dismissal of the petition.

In the meantime, on 10 October 2001, [CLUP-SLECC and its Affiliates Workers Union] reorganized itself and re-registered as CLUP-Sta. Lucia East Commercial Corporation Workers Association (herein appellant CLUPSLECCWA), limiting its membership to the rank-and-file employees of Sta. Lucia East Commercial Corporation. It was issued Certificate of Creation of a Local Chapter.

On the same date, [CLUP-SLECCWA] filed the instant petition. It alleged that [SLECC] employs about 115 employees and that more than 20% of employees belonging to the rank-and-file category are its members. [CLUP-SLECCWA] claimed that no certification election has been held among them within the last 12 months prior to the filing of the petition, and while there is another union registered with DOLE-Regional Office No. IV on 22 June 2001 covering the same employees, namely [SMSLEC], it has not been recognized as the exclusive bargaining agent of [SLECCs] employees.

On 22 November 2001, SLECC filed a motion to dismiss the petition.It averred that it has voluntarily recognized [SMSLEC] on 20 July 2001 as the exclusive bargaining agent of its regular rank-and-file employees, and that collective bargaining negotiations already commenced between them.

27

On 29 November 2001, a CBA between [SMSLEC] and [SLECC] was ratified by its rank-and-file employees and registered with DOLE-Regional Office No. IV on 9 January 2002.

Issue:

Whether or not the appellate court committed a reversible error when it affirmed the Secretary’s finding that SLECCs voluntary recognition of SMSLEC was done while a legitimate labor organization was in existence in the bargaining unit.

Ruling:

The petition has no merit. We see no reason to overturn the rulings of the Secretary and of the appellate court.

Legitimate Labor Organization

Article 212(g) of the Labor Code defines a labor organization as any union or association of employees which exists in whole or in part for the purpose of collective bargaining or of dealing with employers concerning terms and conditions of employment. Upon compliance with all the documentary requirements, the Regional Office or Bureau shall issue in favor of the applicant labor organization a certificate indicating that it is included in the roster of legitimate labor organizations. Any applicant labor organization shall acquire legal personality and shall be entitled to the rights and privileges granted by law to legitimate labor organizations upon issuance of the certificate of registration.

Bargaining Unit

The concepts of a union and of a legitimate labor organization are different from, but related to, the concept of a bargaining unit. We explained the concept of a bargaining unit in San Miguel Corporation v. Laguesma, where we stated that: A bargaining unit is a group of employees of a given employer, comprised of all or less than all of the entire body of employees, consistent with equity to the employer, indicated to be the best suited to serve the reciprocal rights and duties of the parties under the collective bargaining provisions of the law.

The fundamental factors in determining the appropriate collective bargaining unit are: (1) the will of the employees (Globe Doctrine);(2) affinity and unity of the employees interest, such as substantial similarity of work and duties, or similarity of compensation and working conditions (Substantial Mutual Interests Rule); (3) prior collective bargaining history; and (4) similarity of employment status. Contrary to petitioners assertion, this Court has categorically ruled that the existence of a prior collective bargaining history is neither decisive nor conclusive in the determination of what constitutes an appropriate bargaining unit.

However, employees in two corporations cannot be treated as a single bargaining unit even if the businesses of the two corporations are related.

A Legitimate Labor Organization Representing an Inappropriate Bargaining Unit

CLUP-SLECC and its Affiliates Workers Unions initial problem was that they constituted a legitimate labor organization representing a non-appropriate bargaining unit. However, CLUP-SLECC and its Affiliates Workers Union subsequently re-registered as CLUP-SLECCWA, limiting its members to the rank-and-file of SLECC.SLECC cannot ignore that CLUP-SLECC and its Affiliates Workers Union was a legitimate labor

28

organization at the time of SLECCs voluntary recognition of SMSLEC. SLECC and SMSLEC cannot, by themselves, decide whether CLUP-SLECC and its Affiliates Workers Union represented an appropriate bargaining unit.

The inclusion in the union of disqualified employees is not among the grounds for cancellation of registration, unless such inclusion is due to misrepresentation, false statement or fraud under the circumstances enumerated in Sections (a) to (c) of Article 239 of the Labor Code. THUS, CLUP-SLECC AND ITS AFFILIATES WORKERS UNION, HAVING BEEN VALIDLY ISSUED A CERTIFICATE OF REGISTRATION, SHOULD BE CONSIDERED AS HAVING ACQUIRED JURIDICAL PERSONALITY WHICH MAY NOT BE ATTACKED COLLATERALLY. THE PROPER PROCEDURE FOR SLECC IS TO FILE A PETITION FOR CANCELLATION OF CERTIFICATE OF REGISTRATIONOF CLUP-SLECC AND ITS AFFILIATES WORKERS UNION AND NOT TO IMMEDIATELY COMMENCE VOLUNTARY RECOGNITION PROCEEDINGS WITH SMSLEC.

SLECCs Voluntary Recognition of SMSLEC

The employer may voluntarily recognize the representation status of a union in unorganized establishments. SLECC WAS NOT AN UNORGANIZED ESTABLISHMENT WHEN IT VOLUNTARILY RECOGNIZED SMSLEC AS ITS EXCLUSIVE BARGAINING REPRESENTATIVE ON 20 JULY 2001.CLUP-SLECC AND ITS AFFILIATES WORKERS UNION FILED A PETITION FOR CERTIFICATION ELECTION ON 27 FEBRUARY 2001 AND THIS PETITION REMAINED PENDING AS OF 20 JULY 2001.THUS, SLECCS VOLUNTARY RECOGNITION OF SMSLEC ON 20 JULY 2001, THE SUBSEQUENT NEGOTIATIONS AND RESULTING REGISTRATION OF A CBA EXECUTED BY SLECC AND SMSLEC ARE VOID AND CANNOT BAR CLUP-SLECCWAS PRESENT PETITION FOR CERTIFICATION ELECTION.

EMPLOYERS PARTICIPATION IN A PETITION FOR CERTIFICATION ELECTION

We find it strange that the employer itself, SLECC, filed a motion to oppose CLUP-SLECCWAs petition for certification election. In petitions for certification election, the employer is a mere bystander and cannot oppose the petition or appeal the Med-Arbiters decision. The exception to this rule, which happens when the employer is requested to bargain collectively, is not present in the case before us.

The petition is DENIED and AFFIRM the Decision promulgated of the Court of Appeal

MARIWASA SIAM CERAMICS INC. VS. SECRETARY OF DOLE, ET AL., G.R. NO. 183317, DECEMBER 21, 2009 Facts:

On May 4, 2005, respondent Samahan Ng Mga Manggagawa Sa Mariwasa Siam Ceramics, Inc. (SMMSCIndependent) was issued a Certificate of Registration as a legitimate labor organization by the Department of Labor and Employment (DOLE), Region IV-A.

On June 14, 2005, petitioner Mariwasa Siam Ceramics, Inc. filed a Petition for Cancellation of Union Registration against respondent, claiming that the latter violated Article 234 of the Labor Code for not complying with the 20% requirement, and that it committed massive fraud and misrepresentation in violation of Article 239 of the same code. On August 26, 2005, the Regional Director of DOLE IV-A issued an Order granting the petition, revoking the registration of respondent, and delisting it from the roster of active labor unions.

29

Aggrieved, respondent appealed to the Bureau of Labor Relations (BLR).

In a Decision dated June 14, 2006, the BLR granted respondent’s appeal. Petitioner filed a Motion for Reconsideration but the BLR denied it in a Resolution dated February 2, 2007. Petitioner sought recourse with the Court of Appeals (CA) through a Petition for Certiorari; but the CA denied the petition for lack of merit. Petitioner’s motion for reconsideration of the CA Decision was likewise denied, hence, this petition based on the following grounds—

Issues: 1. Whether or not private respondent union complied with the 20% membership requirement 2. Whether or not the Court of Appeals seriously erred when it ruled that private respondent union did not commit misrepresentation, fraud or false statement.

Ruling: The petition should be denied.

The petitioner insists that respondent failed to comply with the 20% union membership requirement for its registration as a legitimate labor organization because of the disaffiliation from the total number of union members of 102 employees who executed affidavits recanting their union membership.

It is, thus, imperative that we peruse the affidavits appearing to have been executed by thes affiants.

Evidently, the affidavits were written and prepared in advance, and the pro forma affidavits were ready to be filled out with the employees’ names and signatures. It is worthy to note, however, that the affidavit does not mention the identity of the people who allegedly forced and deceived the affiant into joining the union, much less the circumstances that constituted such force and deceit. Indeed, not only was this allegation couched in very general terms and sweeping in nature, but more importantly, it was not supported by any evidence whatsoever.

In appreciating affidavits of recantation such as these, our ruling in La Suerte Cigar and Cigarette Factory v. Director of the Bureau of Labor Relations11 is enlightening, viz.— On the second issue—whether or not the withdrawal of 31 union members from NATU affected the petition for certification election insofar as the 30% requirement is concerned, We reserve the Order of the respondent Director of the Bureau of Labor Relations, it appearing undisputably that the 31 union members had withdrawn their support to the petition before the filing of said petition. It would be otherwise if the withdrawal was made after the filing of the petition for it would then be presumed that the withdrawal was not free and voluntary. The presumption would arise that the withdrawal was procured through duress, coercion or for valuable consideration. In other words, the distinction must be that withdrawals made before the filing of the petition are presumed voluntary unless there is convincing proof to the contrary, whereas withdrawals made after the filing of the petition are deemed involuntary.

The reason for such distinction is that if the withdrawal or retraction is made before the filing of the petition, the names of employees supporting the petition are supposed to be held secret to the opposite party. Logically, any such withdrawal or retraction shows voluntariness in the absence of proof to the contrary. Moreover, it becomes apparent that such employees had not given consent to the filing of the petition, hence the subscription requirement has not been met.

30

When the withdrawal or retraction is made after the petition is filed, the employees who are supporting the petition become known to the opposite party since their names are attached to the petition at the time of filing. Therefore, it would not be unexpected that the opposite party would use foul means for the subject employees to withdraw their support. In the instant case, the affidavits of recantation were executed after the identities of the union members became public, i.e., after the union filed a petition for certification election on May 23, 2005, since the names of the members were attached to the petition. The purported withdrawal of support for the registration of the union was made after the documents were submitted to the DOLE, Region IV-A. The logical conclusion, therefore, following jurisprudence, is that the employees were not totally free from the employer’s pressure, and so the voluntariness of the employees’ execution of the affidavits becomes suspect.

It is likewise notable that the first batch of 25 pro forma affidavits shows that the affidavits were executed by the individual affiants on different dates from May 26, 2005 until June 3, 2005, but they were all sworn before a notary public on June 8, 2005. Accordingly, we cannot give full credence to these affidavits, which were executed under suspicious circumstances, and which contain allegations unsupported by evidence. At best, these affidavits are selfserving. They possess no probative value.

Nevertheless, even assuming the veracity of the affidavits of recantation, the legitimacy of respondent as a labor organization must be affirmed. While it is true that the withdrawal of support may be considered as a resignation from the union, the fact remains that at the time of the union’s application for registration, the affiants were members of respondent and they comprised more than the required 20% membership for purposes of registration as a labor union. Article 234 of the Labor Code merely requires a 20% minimum membership during the application for union registration. It does not mandate that a union must maintain the 20% minimum membership requirement all throughout its existence.

Respondent asserts that it had a total of 173 union members at the time it applied for registration. Two names were repeated in respondent’s list and had to be deducted, but the total would still be 171 union members. Further, out of the four names alleged to be no longer connected with petitioner, only two names should be deleted from the list since Diana Motilla and T.W. Amutan resigned from petitioner only on May 10, 2005 and May 17, 2005, respectively, or after respondent’s registration had already been granted. Thus, the total union membership at the time of registration was 169. Since the total number of rank-andfile employees at that time was 528, 169 employees would be equivalent to 32% of the total rank-and-file workers complement, still very much above the minimum required by law.

For the purpose of de-certifying a union such as respondent, it must be shown that there was misrepresentation, false statement or fraud in connection with the adoption or ratification of the constitution and by-laws or amendments thereto; the minutes of ratification; or, in connection with the election of officers, the minutes of the election of officers, the list of voters, or failure to submit these documents together with the list of the newly elected-appointed officers and their postal addresses to the BLR. The bare fact that two signatures appeared twice on the list of those who participated in the organizational meeting would not, to our mind, provide a valid reason to cancel respondent’s certificate of registration. The cancellation of a union’s registration doubtless has an impairing dimension on the right of labor to selforganization. For fraud and misrepresentation to be grounds for cancellation of union registration under the Labor Code, the nature of the fraud and misrepresentation must be grave and compelling enough to vitiate the consent of a majority of union members. In this case, we agree with the BLR and the CA that respondent could not have possibly committed misrepresentation, fraud, or false statements. The alleged failure of respondent to indicate with mathematical precision the total number of employees in the bargaining unit is of no moment, especially as it was able to comply with the 20% minimum membership requirement. Even if the total number of rankand-file employees of petitioner is 528, while respondent declared that it should only be 455, it still cannot be denied that the latter would have more than complied with the registration requirement.

31

The petition is DENIED. The assailed Decision and Resolution of the Court of Appeals are AFFIRMED. Costs against petitioner.

GENERAL MILLING CORP VS. CASIO ET AL., GR NO. 149552, MARCH 10, 2010 Facts: The labor union Ilaw at Buklod ng Mangagawa (IBM)-Local 31 Chapter (Local 31) was the sole and exclusive bargaining agent of the rank and file employees of GMC in Lapu-Lapu City. On November 30, 1991, IBMLocal 31, through its officers and board members, namely, respondents Virgilio Pino, Paulino Cabreros, Ma. Luna P. Jumaoas, Dominador Booc, Bartolome Auman, Remegio Cabantan, Fidel Valle, Loreto Gonzaga, Edilberto Mendoza and Antonio Panilag (Pino, et al.), entered into a Collective Bargaining Agreement (CBA) with GMC. The effectivity of the said CBA was retroactive to August 1, 1991. The CBA contained the following union security provisions: Section 3. MAINTENANCE OF MEMBERSHIP – All employees/workers employed by the Company with the exception of those who are specifically excluded by law and by the terms of this Agreement must be members in good standing of the Union within thirty (30) days upon the signing of this agreement and shall maintain such membership in good standing thereof as a condition of their employment or continued employment. Section 6. The Company, upon written request of the Union, shall terminate the services of any employee/worker who fails to fulfill the conditions set forth in Sections 3 and 4 thereof, subject however, to the provisions of the Labor Laws of the Philippines and their Implementing Rules and Regulations. The Union shall absolve the Company from any and all liabilities, pecuniary or otherwise, and responsibilities to any employee or worker who is dismissed or terminated in pursuant thereof. Casio, et al. were regular employees of GMC with daily earnings ranging from P173.75 to P201.50, and length of service varying from eight to 25 years.Casio was elected IBM-Local 31 President for a three-year term in June 1991, while his co-respondents were union shop stewards. In a letter dated February 24, 1992, Rodolfo Gabiana (Gabiana), the IBM Regional Director for Visayas and Mindanao, furnished Casio, et al. with copies of the Affidavits of GMC employees Basilio Inoc and Juan Potot, charging Casio, et al. with “acts inimical to the interest of the union.” Through the same letter, Gabiana gave Casio, et al. three days from receipt thereof within which to file their answers or counteraffidavits. However, Casio, et al. refused to acknowledge receipt of Gabiana’s letter. Subsequently, on February 29, 1992, Pino, et al., as officers and members of the IBM-Local 31, issued a Resolution expelling Casio, et al. from the union. Gabiana then wrote a letter dated March 10, 1992, addressed to Eduardo Cabahug (Cabahug), GMC Vice-President for Engineering and Plant Administration, informing the company of the expulsion of Casio, et al. from the union pursuant to the Resolution dated February 29, 1992 of IBM-Local 31 officers and board members. Gabiana likewise requested that Casio, et al. “be immediately dismissed from their work for the interest of industrial peace in the plant.” Pressured by the threatened filing of a suit for unfair labor practice, GMC acceded to Gabiana’s request to terminate the employment of Casio, et al. GMC issued a Memorandum dated March 24, 1992 terminating the employment of Casio, et al. effective April 24, 1992 and placing the latter under preventive suspension for the meantime. Casio, et al. next sought recourse from the National Labor Relations Commission (NLRC) Regional Arbitration Branch VII by filing on August 3, 1992 a Complaint against GMC and Pino, et al. for unfair labor practice, particularly, the termination of legitimate union officers, illegal suspension, illegal dismissal, and moral and exemplary damages.

Issue: Whether Casio, et al. were illegally dismissed without any valid?

Ruling: In this case, the Voluntary Arbitrator was convinced that Casio, et al. were legally dismissed; while the Court of Appeals believed the opposite, because even though the dismissal of Casio, et al. was made by

32

GMC pursuant to a valid closed shop provision in the CBA, the company still failed to observe the elementary rules of due process. The Court is therefore constrained to take a second look at the evidence on record considering that the factual findings of the Voluntary Arbitrator and the Court of Appeals are contradictory. There are two aspects which characterize the concept of due process under the Labor Code: one is substantive – whether the termination of employment was based on the provision of the Labor Code or in accordance with the prevailing jurisprudence; the other is procedural – the manner in which the dismissal was effected. After a thorough review of the records, the Court agrees with the Court of Appeals. The dismissal of Casio, et al. was indeed illegal, having been done without just cause and the observance of procedural due process. In Alabang Country Club, Inc. v. National Labor Relations Commission, the Court laid down the grounds for which an employee may be validly terminated, thus: Under the Labor Code, an employee may be validly terminated on the following grounds: (1) just causes under Art. 282; (2) authorized causes under Art.283; (3) termination due to disease under Art.284, and (4) termination by the employee or resignation under Art. 285.

Another cause for termination is dismissal from employment due to the enforcement of the union security clause in the CBA. x x x. “Union security” is a generic term, which is applied to and comprehends “closed shop,” “union shop,” “maintenance of membership,” or any other form of agreement which imposes upon employees the obligation to acquire or retain union membership as a condition affecting employment. There is union shop when all new regular employees are required to join the union within a certain period as a condition for their continued employment. There is maintenance of membership shop when employees, who are union members as of the effective date of the agreement, or who thereafter become members, must maintain union membership as a condition for continued employment until they are promoted or transferred out of the bargaining unit or the agreement is terminated. A closed shop, on the other hand, may be defined as an enterprise in which, by agreement between the employer and his employees or their representatives, no person may be employed in any or certain agreed departments of the enterprise unless he or she is, becomes, and, for the duration of the agreement, remains a member in good standing of a union entirely comprised of or of which the employees in interest are a part.

Union security clauses are recognized and explicitly allowed under Article 248(e) of the Labor Code, which provides that: Art. 248. Unfair Labor Practices of Employers. x x x xxxx (e) To discriminate in regard to wages, hours of work, and other terms and conditions of employment in order to encourage or discourage membership in any labor organization. Nothing in this Code or in any other law shall stop the parties from requiring membership in a recognized collective bargaining agent as a condition for employment, except those employees who are already members of another union at the time of the signing of the collective bargaining agreement.

It is State policy to promote unionism to enable workers to negotiate with management on an even playing field and with more persuasiveness than if they were to individually and separately bargain with the employer. For this reason, the law has allowed stipulations for “union shop” and “closed shop” as means of encouraging workers to join and support the union of their choice in the protection of their rights and interest vis-à-vis the employer. Moreover, a stipulation in the CBA authorizing the dismissal of employees are of equal import as the statutory provisions on dismissal under the Labor Code, since “a CBA is the law between the company and the union and compliance therewith is mandated by the express policy to give protection to labor.” In terminating the employment of an employee by enforcing the union security clause, the employer needs only to determine and prove that: (1) the union security clause is applicable; (2) the union is requesting for the enforcement of the union security provision in the CBA; and (3) there is sufficient evidence to support the decision of the union to expel the employee from the union. These requisites constitute just cause for terminating an employee based on the union security provision of the CBA.

33

There is no question that in the present case, the CBA between GMC and IBM-Local 31 included a maintenance of membership and closed shop clause as can be gleaned from Sections 3 and 6 of Article II. IBM-Local 31, by written request, can ask GMC to terminate the employment of the employee/worker who failed to maintain its good standing as a union member.

It is similarly undisputed that IBM-Local 31, through Gabiana, the IBM Regional Director for Visayas and Mindanao, twice requested GMC, in the letters dated March 10 and 19, 1992, to terminate the employment of Casio, et al. as a necessary consequence of their expulsion from the union. It is the third requisite – that there is sufficient evidence to support the decision of IBM-Local 31 to expel Casio, et al. – which appears to be lacking in this case. The failure of GMC to make a determination of the sufficiency of evidence supporting the decision of IBMLocal 31 to expel Casio, et al. is a direct consequence of the non-observance by GMC of procedural due process in the dismissal of employees.

As a defense, GMC contends that as an employer, its only duty was to ascertain that IBM-Local 31 accorded Casio, et al. due process; and, it is the finding of the company that IBM-Local 31 did give Casio, et al. the opportunity to answer the charges against them, but they refused to avail themselves of such opportunity. This argument is without basis. The Court has stressed time and again that allegations must be proven by sufficient evidence because mere allegation is definitely not evidence. The records of this case are absolutely bereft of any supporting evidence to substantiate the bare allegation of GMC that Casio, et al. were accorded due process by IBM-Local 31. There is nothing on record that would indicate that IBM-Local 31 actually notified Casio, et al. of the charges against them or that they were given the chance to explain their side. All that was stated in the IBM-Local 31 Resolution dated February 29, 1992, expelling Casio, et al. from the union, was that “a copy of the said letter complaint [dated February 24, 1992] was dropped or left in front of E. Casio.” It was not established that said letter-complaint charging Casio, et al. with acts inimical to the interest of the union was properly served upon Casio, that Casio willfully refused to accept the said letter-notice, or that Casio had the authority to receive the same letter-notice on behalf of the other employees similarly accused. It’s worthy to note that Casio, et al. were expelled only five days after the issuance of the letter-complaint against them. The Court cannot find proof on record when the three-day period, within which Casio, et al. was supposed to file their answer or counter-affidavits, started to run and had expired. The Court is likewise unconvinced that the said three-day period was sufficient for Casio, et al. to prepare their defenses and evidence to refute the serious charges against them. Contrary to the position of GMC, the acts of Pino, et al. as officers and board members of IBM-Local 31, in expelling Casio, et al. from the union, do not enjoy the presumption of regularity in the performance of official duties, because the presumption applies only to public officers from the highest to the lowest in the service of the Government, departments, bureaus, offices, and/or its political subdivisions.

The twin requirements of notice and hearing constitute the essential elements of procedural due process. The law requires the employer to furnish the employee sought to be dismissed with two written notices before termination of employment can be legally effected: (1) a written notice apprising the employee of the particular acts or omissions for which his dismissal is sought in order to afford him an opportunity to be heard and to defend himself with the assistance of counsel, if he desires, and (2) a subsequent notice informing the employee of the employer’s decision to dismiss him. This procedure is mandatory and its absence taints the dismissal with illegality. Irrefragably, GMC cannot dispense with the requirements of notice and hearing before dismissing Casio, et al. even when said dismissal is pursuant to the closed shop provision in the CBA. The rights of an employee to be informed of the charges against him and to reasonable opportunity to present his side in a controversy with either the company or his own union are not wiped away by a union security clause or a union shop clause in a collective bargaining agreement. An employee is entitled to be protected not only from a company which disregards his rights but also from his own union the leadership of which could yield to the temptation of swift and arbitrary expulsion from membership and hence dismissal from his job.

In the case at bar, Casio, et al. did not receive any other communication from GMC, except the written notice of termination dated March 24, 1992. GMC, by its own admission, did not conduct a separate and independent investigation to determine the sufficiency of the evidence supporting the expulsion of Casio, et al. by IBP-Local 31. It straight away acceded to the demand of IBP-Local 31 to dismiss Casio, et al.

34

In sum, the Court finds that GMC illegally dismissed Casio, et al. because not only did GMC fail to make a determination of the sufficiency of evidence to support the decision of IBM-Local 31 to expel Casio, et al., but also to accord the expelled union members procedural due process, i.e., notice and hearing, prior to the termination of their employment

An employee who is illegally dismissed is entitled to the twin reliefs of full backwages and reinstatement. If reinstatement is not viable, separation pay is awarded to the employee. In awarding separation pay to an illegally dismissed employee, in lieu of reinstatement, the amount to be awarded shall be equivalent to one month salary for every year of service. Under Republic Act No. 6715, employees who are illegally dismissed are entitled to full backwages, inclusive of allowances and other benefits or their monetary equivalent, computed from the time their actual compensation was withheld from them up to the time of their actual reinstatement but if reinstatement is no longer possible, the backwages shall be computed from the time of their illegal termination up to the finality of the decision. Thus, Casio, et al. are entitled to backwages and separation pay considering that reinstatement is no longer possible because the positions they previously occupied are no longer existing, as declared by GMC. Casio, et al., having been compelled to litigate in order to seek redress for their illegal dismissal, are entitled to the award of attorney’s fees equivalent to 10% of the total monetary award. WHEREFORE, the instant petition is hereby DENIED.

THE HERITAGE HOTEL MANILA VS. NATL UNION OF WORKERS IN HOTEL ETC., GR NO. 178296, JANUARY 12, 2011 Facts: On October 11, 1995, respondent filed with the Department of Labor and Employment-National Capital Region (DOLE-NCR) a petition for certification election. The Med-Arbiter granted the petition on February 14, 1996 and ordered the holding of a certification election. On appeal, the DOLE Secretary, in a Resolution dated August 15, 1996, affirmed the Med-Arbiter's order and remanded the case to the Med-Arbiter for the holding of a pre-election conference on February 26, 1997. Petitioner filed a motion for reconsideration, but it was denied on September 23, 1996. The pre-election conference was not held as initially scheduled; it was held a year later, or on February 20, 1998. Petitioner moved to archive or to dismiss the petition due to alleged repeated non-appearance of respondent. The latter agreed to suspend proceedings until further notice. The pre-election conference resumed on January 29, 2000. Subsequently, petitioner discovered that respondent had failed to submit to the Bureau of Labor Relations (BLR) its annual financial report for several years and the list of its members since it filed its registration papers in 1995. Consequently, on May 19, 2000, petitioner filed a Petition for Cancellation of Registration of respondent, on the ground of the non-submission of the said documents. Petitioner prayed that respondent's Certificate of Creation of Local/Chapter be cancelled and its name be deleted from the list of legitimate labor organizations. It further requested the suspension of the certification election proceedings.

Issue: Whether or not the non-submission of financial reports warrant the cancellation of the respondent’s registration

Ruling: No. The respondent's registration as a legitimate labor union should not be cancelled. The Regional Director has ample discretion in dealing with a petition for cancellation of a union's registration, particularly, determining whether the union still meets the requirements prescribed by law. It is sufficient to give the Regional Director license to treat the late filing of required documents as sufficient compliance with the requirements of the law. After all, the law requires the labor organization to submit the annual financial report and list of members in order to verify if it is still viable and financially sustainable as an organization so as to protect the employer and employees from fraudulent or fly-by-night unions. With the submission of the required documents by respondent, the purpose of the law has been achieved, though belatedly.

35

We cannot ascribe abuse of discretion to the Regional Director and the DOLE Secretary in denying the petition for cancellation of respondent's registration. The union members and, in fact, all the employees belonging to the appropriate bargaining unit should not be deprived of a bargaining agent, merely because of the negligence of the union officers who were responsible for the submission of the documents to the BLR.

Labor authorities should, indeed, act with circumspection in treating petitions for cancellation of union registration, lest they be accused of interfering with union activities. In resolving the petition, consideration must be taken of the fundamental rights guaranteed by Article XIII, Section 3 of the Constitution, i.e., the rights of all workers to self-organization, collective bargaining and negotiations, and peaceful concerted activities. Labor authorities should bear in mind that registration confers upon a union the status of legitimacy and the concomitant right and privileges granted by law to a legitimate labor organization, particularly the right to participate in or ask for certification election in a bargaining unit. Thus, the cancellation of a certificate of registration is the equivalent of snuffing out the life of a labor organization. For without such registration, it loses - as a rule - its rights under the Labor Code.

It is worth mentioning that the Labor Code's provisions on cancellation of union registration and on reportorial requirements have been recently amended by Republic Act (R.A.) No. 9481. The amendment sought to strengthen the workers' right to self-organization and enhance the Philippines' compliance with its international obligations as embodied in the International Labour Organization (ILO) Convention No. 87, pertaining to the non-dissolution of workers' organizations by administrative authority. Thus, R.A. No. 9481 inserted in the Labor Code Article 242-A, which provides: ART. 242-A. Reportorial Requirements.-The following are documents required to be submitted to the Bureau by the legitimate labor organization concerned: xxx

xxx

(c) Its annual financial report within thirty (30) days after the close of every fiscal year; and xxx

xxx

Failure to comply with the above requirements shall not be a ground for cancellation of union registration but shall subject the erring officers or members to suspension, expulsion from membership, or any appropriate penalty.

ILO Convention No. 87, which we have ratified in 1953, provides that "workers' and employers' organizations shall not be liable to be dissolved or suspended by administrative authority." The ILO has expressed the opinion that the cancellation of union registration by the registrar of labor unions, which in our case is the BLR, is tantamount to dissolution of the organization by administrative authority when such measure would give rise to the loss of legal personality of the union or loss of advantages necessary for it to carry out its activities, which is true in our jurisdiction. Although the ILO has allowed such measure to be taken, provided that judicial safeguards are in place, i.e., the right to appeal to a judicial body, it has nonetheless reminded its members that dissolution of a union, and cancellation of registration for that matter, involve serious consequences for occupational representation. It has, therefore, deemed it preferable if such actions were to be taken only as a last resort and after exhausting other possibilities with less serious effects on the organization. It is undisputed that appellee failed to submit its annual financial reports and list of individual members in accordance with Article 239 of the Labor Code. However, the existence of this ground should not necessarily lead to the cancellation of union registration. Article 239 recognizes the regulatory authority of the State to exact compliance with reporting requirements. Yet there is more at stake in this case than merely monitoring union activities and requiring periodic documentation thereof. The more substantive considerations involve the constitutionally guaranteed freedom of association and right of workers to self-organization. Also involved is the public policy to promote free trade unionism and collective bargaining as instruments of industrial peace and democracy. An overly stringent interpretation of the statute governing cancellation of union registration without regard to surrounding circumstances cannot be allowed. Otherwise, it would lead to an unconstitutional application of the statute and emasculation of public policy objectives. Worse, it can render nugatory the protection to labor and social justice clauses that pervades the Constitution and the Labor Code.

36

Moreover, submission of the required documents is the duty of the officers of the union. It would be unreasonable for this Office to order the cancellation of the union and penalize the entire union membership on the basis of the negligence of its officers. In National Union of Bank Employees vs. Minister of Labor, the Supreme Court ruled: As aptly ruled by respondent Bureau of Labor Relations Director Noriel: "The rights of workers to selforganization finds general and specific constitutional guarantees. x x x Such constitutional guarantees should not be lightly taken much less nullified. A healthy respect for the freedom of association demands that acts imputable to officers or members be not easily visited with capital punishments against the association itself." At any rate, we note that on 19 May 2000, appellee had submitted its financial statement for the years 1996-1999. With this submission, appellee has substantially complied with its duty to submit its financial report for the said period. To rule differently would be to preclude the union, after having failed to meet its periodic obligations promptly, from taking appropriate measures to correct its omissions. For the record, we do not view with favor appellee's late submission. Punctuality on the part of the union and its officers could have prevented this petition.

LEGEND INTERNATIONAL RESORTS LTD., VS. KILUSANG MANGGAGAWA NG LEGENDA, G.R. NO. 169754, FEB. 23, 2011 Facts: June 6, 2001, KML filed with the Med-Arbitration Unit of the DOLE, San Fernando, Pampanga, a petition for certification election. KML alleged that it is a legitimate labor organization of the rank and file employees of Legend. It was issued its Certification of Registration by DOLE on May 18, 2001. Legend moved to dismiss the petition on the grounds that it is not a legitimate labor organization because its membership is a mixture of rank and file employees and supervisory employees. KML also committed acts of fraud and misrepresentation when it made it appear that certain employees attended its general membership meeting on April 5, 2001 when in reality some of them were either at work, have already resigned, or were abroad.

KML argued that even if the supervisory employees were excluded from membership, the certification election could still proceed because the required number of total rank and file employees necessary is still sustained. It also claimed that its legitimacy as a labor union cannot be attacked collaterally. Med Arbiter judgment September 20, 2001: dismissed KML’s petition for certification election. Since its membership included supervisory employees, it was not a legitimate labor organization. KML was also guilty of fraud and misrepresentation; 70 employees who were claimed to be among those who attended its organizational meeting were either at work or elsewhere.

Office of the Secretary of DOLE May 22, 2002 decision: reversed Med- Arbiter’s decision. KML’s legitimacy as a union cannot be attacked collaterally. The presence of supervisory employees does not ipso facto render the existence of a labor organization illegal. Mixed membership is not one of the grounds for dismissal of a petition for certification election. Ordered the immediate conduct of the certification election. Legend filed a Motion for Reconsideration. It also alleged that it filed a petition for cancellation of union registration of KML which was granted by the DOLE Regional Office, November 7, 2001. MFR was denied in a resolution dated August 20, 2002: a final order of cancellation is required before a petition for certification of election may be dismissed on the ground of lack of legal personality, and that the November 7, 2001 decision was reversed by the BLR March 26, 2002.

CA: held that the issue on the legitimacy of KML as a labor organization has already been settled with finality. The March 26, 2002 decision upholding the legitimacy had long become final and executor for failure of Legend to appeal.KML being a legitimate labor org, it could properly file a petition for certification election. Legend filed MFR stating that it has appealed to the CA the March 26, 2002 decision and is still pending. CA denied MFR. Issues: Whether Legend has timely appealed the March 26, 2002 decision (re: cancellation of union registration)

37

Whether the cancellation of KML’s certificate of registration should retroact to the time of its issuance (it was cancelled in the November 7, 2001 decision) Whether the legitimacy of the legal personality of KML can be collaterally attacked in a petition for certification election

Held: 1.Yes. The March 26, 2002 decision has not yet attained finality considering that it has timely appealed to the CA and which at that time is still pending resolution. Legend timely filed on Sept 6, 2002 a petition for certiorari before the CA assailing the March 26, 2002 decision. On June 30, 2005, CA reversed the March 26, 2002 decision of the BLR and reinstated the November 7, 2001 decision cancelling the certificate of registration of KML. KML’s MRF was denied. KML filed a petition for certiorari before the SC which was denied. KML moved for reconsideration but it was denied with finality. The decision to cancel KML’s certificate of registration became final and executory and entry of judgment was made on July 18, 2006. 2.No. According to ACA vs Calleja, a certification proceeding is not a litigation in the sense that the term is ordinarily understood, but an investigation of a non-adversarial and fact-finding character. An order to hold a certificationelection is proper despite the pendency of the petition for cancellation of the registration certificate of the respondent union. The rationale is that at the time the union filed the petition, it still had the legal personality to perform such act absent an order directing the cancellation. There is no basis for Legend’s assertion that the cancellation of KML’s certificate of registration should retroact to the time of its issuance or that it effectively nullified all of KML’s activities, including its filing of the petition for certification election and its demand to collectively bargain. 3.NO. The legitimacy of the legal personality of KML cannot be collaterally attacked in a petition for certification election proceeding. Such legal personality may not be subject to a collateral attack but only through a separate action instituted particularly for the purpose of assailing it. SC affirmed the May 22, 2002 decision (KML’s legitimacy cannot be attacked collaterally, presence of supervisory employees does not render it illegal) and the August 20, 2002 resolution (a final order of cancellation is required before a petition for certification election may be dismissed on the ground of lack of personality, reversed March 26, 2002 decision)

SAMAHANG MANGGAGAWA SA CHARTER CHEMICAL SOLIDARITY OF UNIONS IN THE PHILS FOR EMPOWERMENT AND REFORMS (SMCC-SUPER) ET AL., VS. CHARTER CHEMICAL AND COATING CORP., G.R. NO. 169717, MARCH 16, 2011 Facts: Petitioner union filed a petition for certification election among the regular rank-and-file employees of the respondent company witht eh Mediation Arbitration Union of the DOLE-NCR. Respondent filed a Motion to Dismiss on the ground that petitioner union is not a legitimate labor organization because of failure to comply with the documentation requirements set by law, and the inclusions of supervisory employees within petitioner union. Med-Arbiter: Dismissed the petition for certification election because petitioner union is not a legitimate labor organization. The charter certificate and "Listahan ng mga Dumalo sa Pangkalahatang Pulong at mga Sumang-ayon at Nagratipika sa Saligang Batas" were not executed under oath and certified by the union secretary and attested to by the union president as required by Section 235 of the Labor Code. It also held that the list of membership of petitioner union consisted of members who performed supervisory functions. Under Article 245 of the Labor Code, said supervisory employees are prohibited from joining petitioner union which seeks to represent the rank-and-file employees of respondent company. DOLE: Granted petitioner union’s petition for certification election. Court of Appeals: Nullified the decision of the DOLE. The appellate court gave credence to the findings of the Med-Arbiter that petitioner union failed to comply with the documentation requirements under the

38

Labor Code. It, likewise, upheld the Med-Arbiter's finding that petitioner union consisted of both rank-andfile and supervisory employees. Moreover, the CA held that the issues as to the legitimacy of petitioner union may be attacked collaterally in a petition for certification election and the infirmity in the membership of petitioner union cannot be remedied through the exclusion-inclusion proceedings in a preelection conference pursuant to the ruling inToyota Motor Philippines v. Toyota Motor Philippines Corporation Labor Union. Thus, considering that petitioner union is not a legitimate labor organization, it has no legal right to file a petition for certification election.

Issues: 1. 2.

Whether the alleged mixture of rank-and-file and supervisory employee[s] of petitioner [union's] membership is [a] ground for the cancellation of petitioner [union's] legal personality and dismissal of [the] petition for certification election. Whether the alleged failure to certify under oath the local charter certificate issued by its mother federation and list of the union membership attending the organizational meeting [is a ground] for the cancellation of petitioner [union's] legal personality as a labor organization and for the dismissal of the petition for certification election.

Ruling: The petition is meritorious. The charter certificate need not be certified under oath by the local union's secretary or treasurer and attested to by its president.

The then prevailing Section 1, Rule VI of the Implementing Rules of Book V, as amended by D.O. No. 9, series of 1997, provides: Section 1. Chartering and creation of a local chapter -- A duly registered federation or national union may directly create a local/chapter by submitting to the Regional Office or to the Bureau two (2) copies of the following: (a) A charter certificate issued by the federation or national union indicating the creation or establishment of the local/chapter; (b) The names of the local/chapter's officers, their addresses, and the principal office of the local/chapter; and (c) The local/chapter's constitution and by-laws provided that where the local/chapter's constitution and by-laws [are] the same as [those] of the federation or national union, this fact shall be indicated accordingly. All the foregoing supporting requirements shall be certified under oath by the Secretary or the Treasurer of the local/chapter and attested to by its President.

As readily seen, the Sama-samang Pahayag ng Pagsapi at Authorization and Listahan ng mga Dumalo sa Pangkalahatang Pulong at mga Sumang-ayon at Nagratipika sa Saligang Batas are not among the documents that need to be submitted to the Regional Office or Bureau of Labor Relations in order to register a labor organization. As to the charter certificate, the above-quoted rule indicates that it should be executed under oath. Petitioner union concedes and the records confirm that its charter certificate was not executed under oath. However, in San Miguel Corporation (Mandaue Packaging Products Plants) v. Mandaue Packing Products Plants-San Miguel Corporation Monthlies Rank-and-File Union-FFW (MPPP-SMPPSMAMRFU-FFW),[22]which was decided under the auspices of D.O. No. 9, Series of 1997, we ruled In San Miguel Foods-Cebu B-Meg Feed Plant v. Hon. Laguesma, 331 Phil. 356 (1996), the Court ruled that it was not necessary for the charter certificate to be certified and attested by the local/chapter officers. Id.While this ruling was based on the interpretation of the previous Implementing Rules provisions which were supplanted by the 1997 amendments, we believe that the same doctrine

39

obtains in this case. Considering that the charter certificate is prepared and issued by the national union and not the local/chapter, it does not make sense to have the local/chapter's officers x x xcertify or attest to a document which they had no hand in the preparation of.

In accordance with this ruling, petitioner union's charter certificate need not be executed under oath. Consequently, it validly acquired the status of a legitimate labor organization upon submission of (1) its charter certificate, (2) the names of its officers, their addresses, and its principal office, and (3) its constitution and by-laws-- the last two requirements having been executed under oath by the proper union officials as borne out by the records. The mixture of rank-and-file and supervisory employees in petitioner union does not nullify its legal personality as a legitimate labor organization. The CA found that petitioner union has for its membership both rank-and-file and supervisory employees. However, petitioner union sought to represent the bargaining unit consisting of rank-and-file employees. Under Article 245 of the Labor Code, supervisory employees are not eligible for membership in a labor organization of rank-and-file employees. It was the Rules and Regulations Implementing R.A. No. 6715 (1989 Amended Omnibus Rules) which supplied the deficiency by introducing the following amendment to Rule II (Registration of Unions): Sec. 2. Who may file. - Any legitimate labor organization or the employer, when requested to bargain collectively, may file the petition. The petition, when filed by a legitimate labor organization, shall contain, among others: xxxx (c) description of the bargaining unit which shall be the employer unit unless circumstances otherwise require; and provided further, that the appropriate bargaining unit of the rank-and-file employees shall not include supervisory employees and/or security guards. (Emphasis supplied)

By that provision, any questioned mingling will prevent an otherwise legitimate and duly registered labor organization from exercising its right to file a petition for certification election.

But the 1989 Amended Omnibus Rules was further amended by Department Order No. 9, series of 1997 (1997 Amended Omnibus Rules). Specifically, the requirement under Sec. 2(c) of the 1989 Amended Omnibus Rules - that the petition for certification election indicate that the bargaining unit of rank-and-file employees has not been mingled with supervisory employees - was removed. Instead, what the 1997 Amended Omnibus Rules requires is a plain description of the bargaining unit. there is a prohibition against the mingling of supervisory and rank-and-file employees in one labor organization, the Labor Code does not provide for the effects thereof. Thus, the Court held that after a labor organization has been registered, it may exercise all the rights and privileges of a legitimate labor organization. Any mingling between supervisory and rank-and-file employees in its membership cannot affect its legitimacy for that is not among the grounds for cancellation of its registration, unless such mingling was brought about by misrepresentation, false statement or fraud under Article 239 of the Labor Code. In San Miguel Corp. (Mandaue Packaging Products Plants) v. Mandaue Packing Products Plants-San Miguel Packaging Products-San Miguel Corp. Monthlies Rank-and-File Union-FFW, the Court explained that since the 1997 Amended Omnibus Rules does not require a local or chapter to provide a list of its members, it would be improper for the DOLE to deny recognition to said local or chapter on account of any question pertaining to its individual members. More to the point is Air Philippines Corporation v. Bureau of Labor Relations, which involved a petition for

40

cancellation of union registration filed by the employer in 1999 against a rank-and-file labor organization on the ground of mixed membership: the Court therein reiterated its ruling in Tagaytay Highlands that the inclusion in a union of disqualified employees is not among the grounds for cancellation, unless such inclusion is due to misrepresentation, false statement or fraud under the circumstances enumerated in Sections (a) and (c) of Article 239 of the Labor Code. As a result, petitioner union was not divested of its status as a legitimate labor organization even if some of its members were supervisory employees; it had the right to file the subject petition for certification election. The legal personality of petitioner union cannot be collaterally attacked by respondent company in the certification election proceedings. Petitioner union correctly argues that its legal personality cannot be collaterally attacked in the certification election proceedings. As we explained in Kawashima: Except when it is requested to bargain collectively, an employer is a mere bystander to any petition for certification election; such proceeding is non-adversarial and merely investigative, for the purpose thereof is to determine which organization will represent the employees in their collective bargaining with the employer. The choice of their representative is the exclusive concern of the employees; the employer cannot have any partisan interest therein; it cannot interfere with, much less oppose, the process by filing a motion to dismiss or an appeal from it; not even a mere allegation that some employees participating in a petition for certification election are actually managerial employees will lend an employer legal personality to block the certification election. The employer's only right in the proceeding is to be notified or informed thereof. SAN MIGUEL FOODS VS. SAN MIGUEL CORP SUPERVISORS AND EXEMPT UNION G.R. NO. 146206 AUGUST 1, 2011 FACTS: A certification election was conducted and on the date of the election, petitioner employer filed the Omnibus Objections and Challenge to Voters, questioning the eligibility to vote by some of its employees on the grounds that some employees do not belong to the bargaining unit which respondent seeks to represent or that there is no existence of employer-employee relationship with petitioner. ISSUE: The ff issues were raised when the case reached the SC: the inclusion of employees in supervisor levels 3 and 4 and the exempt employees in the proposed bargaining unit, thereby allowing their participation in the certification election; the application of the “community or mutuality of interests” test; and the determination of the employees who belong to the category of confidential employees Ruling: 1ST issue (Certain factors, such as specific line of work, working conditions, location of work, mode of compensation, and other relevant conditions do not affect or impede their commonality of interest.)

Petitioner’s contentions are erroneous. In G.R. No. 110399, the Court explained that the employees of San Miguel Corporation Magnolia Poultry Products Plants of Cabuyao, San Fernando, and Otis constitute a single bargaining unit, which is not contrary to the one-company, one-union policy. An appropriate bargaining unit is defined as a group of employees of a given employer, comprised of all or less than all of the entire body of employees, which the collective interest of all the employees, consistent with equity to the employer, indicate to be best suited to serve the reciprocal rights and duties of the parties under the collective bargaining provisions of the law.[21]

In National Association of Free Trade Unions v. Mainit Lumber Development Company Workers Union – United Lumber and General Workers of the Phils,[22] the Court, taking into account the “community or

41

mutuality of interests” test, ordered the formation of a single bargaining unit consisting of the Sawmill Division in Butuan City and the Logging Division in Zapanta Valley, Kitcharao, Agusan [Del] Norte of the Mainit Lumber Development Company. It held that while the existence of a bargaining history is a factor that may be reckoned with in determining the appropriate bargaining unit, the same is not decisive or conclusive. Other factors must be considered. The test of grouping is community or mutuality of interest. This is so because the basic test of an asserted bargaining unit’s acceptability is whether or not it is fundamentally the combination which will best assure to all employees the exercise of their collective bargaining rights.[23] Certainly, there is a mutuality of interest among the employees of the Sawmill Division and the Logging Division. Their functions mesh with one another. One group needs the other in the same way that the company needs them both. There may be differences as to the nature of their individual assignments, but the distinctions are not enough to warrant the formation of a separate bargaining unit.[24]

Thus, applying the ruling to the present case, the Court affirms the finding of the CA that there should be only one bargaining unit for the employees in Cabuyao, San Fernando, and Otis[25] of Magnolia Poultry Products Plant involved in “dressed” chicken processing and Magnolia Poultry Farms engaged in “live” chicken operations. Certain factors, such as specific line of work, working conditions, location of work, mode of compensation, and other relevant conditions do not affect or impede their commonality of interest. Although they seem separate and distinct from each other, the specific tasks of each division are actually interrelated and there exists mutuality of interests which warrants the formation of a single bargaining unit.

Confidential employees are defined as those who (1) assist or act in a confidential capacity, in regard (2) to persons who formulate, determine, and effectuate management policies in the field of labor relations. [26] The two criteria are cumulative, and both must be met if an employee is to be considered a confidential employee - that is, the confidential relationship must exist between the employee and his supervisor, and the supervisor must handle the prescribed responsibilities relating to labor relations. The exclusion from bargaining units of employees who, in the normal course of their duties, become aware of management policies relating to labor relations is a principal objective sought to be accomplished by the “confidential employee rule.”[27]

2nd Issue (PAYROLL MASTER NOT CONFIDENTIAL, PERSONNEL ASSISTANT CONFIDENTIAL!!)

HUMAN

RESOURCE

ASSISTANT

and

A confidential employee is one entrusted with confidence on delicate, or with the custody, handling or care and protection of the employer’s property.[28] Confidential employees, such as accounting personnel, should be excluded from the bargaining unit, as their access to confidential information may become the source of undue advantage.[29] However, such fact does not apply to the position of Payroll Master and the whole gamut of employees who, as perceived by petitioner, has access to salary and compensation data. The CA correctly held that the position of Payroll Master does not involve dealing with confidential labor relations information in the course of the performance of his functions. Since the nature of his work does not pertain to company rules and regulations and confidential labor relations, it follows that he cannot be excluded from the subject bargaining unit.

Corollarily, although Article 245[30] of the Labor Code limits the ineligibility to join, form and assist any labor organization to managerial employees, jurisprudence has extended this prohibition to confidential employees or those who by reason of their positions or nature of work are required to assist or act in a fiduciary manner to managerial employees and, hence, are likewise privy to sensitive and highly confidential records.[31] Confidential employees are thus excluded from the rank-and-file bargaining unit. The rationale for their separate category and disqualification to join any labor organization is similar to the inhibition for managerial employees, because if allowed to be affiliated with a union, the latter might not be assured of their loyalty in view of evident conflict of interests and the union can also become company-denominated with the presence of managerial employees in the union membership.[32] Having access to confidential information, confidential employees may also become the source of undue advantage. Said employees may act as a spy or spies of either party to a collective bargaining agreement. [33]

In this regard, the CA correctly ruled that the positions of Human Resource Assistant and Personnel Assistant belong to the category of confidential employees and, hence, are excluded from the bargaining unit, considering their respective positions and job descriptions. As Human Resource Assistant,

42

[34] the scope of one’s work necessarily involves labor relations, recruitment and selection of employees, access to employees' personal files and compensation package, and human resource management. As regards a Personnel Assistant,[35] one's work includes the recording of minutes for management during collective bargaining negotiations, assistance to management during grievance meetings and administrative investigations, and securing legal advice for labor issues from the petitioner’s team of lawyers, and implementation of company programs. Therefore, in the discharge of their functions, both gain access to vital labor relations information which outrightly disqualifies them from union membership.

The proceedings for certification election are quasi-judicial in nature and, therefore, decisions rendered in such proceedings can attain finality.

It bears stressing that a certification election is the sole concern of the workers; hence, an employer lacks the personality to dispute the same. The general rule is that an employer has no standing to question the process of certification election, since this is the sole concern of the workers. [37] Law and policy demand that employers take a strict, hands-off stance in certification elections. The bargaining representative of employees should be chosen free from any extraneous influence of management. A labor bargaining representative, to be effective, must owe its loyalty to the employees alone and to no other. [38] The only exception is where the employer itself has to file the petition pursuant to Article 258 [39] of the Labor Code because of a request to bargain collectively.[40]

BPI VS. BPI EMPLOYEES UNION-DAVAO CHAPTER, GR NO. 164301, OCTOBER 19, 2011 RESOLUTION ON THE MAIN DECISION OF AUG. 18, 2010 Facts: On March 23, 2000, the Bangko Sentral ng Pilipinas approved the Articles of Merger executed on January 20, 2000 by and between BPI, herein petitioner, and FEBTC. This Article and Plan of Merger was approved by the Securities and Exchange Commission on April 7, 2000. Pursuant to the Article and Plan of Merger, all the assets and liabilities of FEBTC were transferred to and absorbed by BPI as the surviving corporation. FEBTC employees, including those in its different branches across the country, were hired by petitioner as its own employees, with their status and tenure recognized and salaries and benefits maintained.

Respondent BPI Employees Union-Davao Chapter - Federation of Unions in BPI Unibank (hereinafter the "Union," for brevity) is the exclusive bargaining agent of BPI’s rank and file employees in Davao City. The former FEBTC rank-and-file employees in Davao City did not belong to any labor union at the time of the merger. Prior to the effectivity of the merger, or on March 31, 2000, respondent Union invited said FEBTC employees to a meeting regarding the Union Shop Clause (Article II, Section 2) of the existing CBA between petitioner BPI and respondent Union.

The parties both advert to certain provisions of the existing CBA, which includes: Section 2. Union Shop - New employees falling within the bargaining unit as defined in Article I of this Agreement, who may hereafter be regularly employed by the Bank shall, within thirty (30) days after they become regular employees, join the Union as a condition of their continued employment. It is understood that membership in good standing in the Union is a condition of their continued employment with the Bank. After the meeting called by the Union, some of the former FEBTC employees joined the Union, while others refused. Later, however, some of those who initially joined retracted their membership. Respondent Union then sent notices to the former FEBTC employees who refused to join, as well as those who retracted their membership, and called them to a hearing regarding the matter. When these former FEBTC employees refused to attend the hearing, the president of the Union requested BPI to implement the Union Shop Clause of the CBA and to terminate their employment pursuant thereto.

Issue:

43

Whether or not the former FEBTC employees that were absorbed by petitioner upon the merger between FEBTC and BPI should be covered by the Union Shop Clause found in the existing CBA between petitioner and respondent Union.

Ruling: Yes. They are covered by the Union Shop Clause. "Union security" is a generic term which is applied to and comprehends "closed shop," "union shop," "maintenance of membership" or any other form of agreement which imposes upon employees the obligation to acquire or retain union membership as a condition affecting employment. There is union shop when all new regular employees are required to join the union within a certain period for their continued employment. There is maintenance of membership shop when employees, who are union members as of the effective date of the agreement, or who thereafter become members, must maintain union membership as a condition for continued employment until they are promoted or transferred out of the bargaining unit or the agreement is terminated. A closed-shop, on the other hand, may be defined as an enterprise in which, by agreement between the employer and his employees or their representatives, no person may be employed in any or certain agreed departments of the enterprise unless he or she is, becomes, and, for the duration of the agreement, remains a member in good standing of a union entirely comprised of or of which the employees in interest are a part. In the case of Liberty Flour Mills Employees v. Liberty Flour Mills, Inc., we ruled that: It is the policy of the State to promote unionism to enable the workers to negotiate with management on the same level and with more persuasiveness than if they were to individually and independently bargain for the improvement of their respective conditions. To this end, the Constitution guarantees to them the rights "to selforganization, collective bargaining and negotiations and peaceful concerted actions including the right to strike in accordance with law." There is no question that these purposes could be thwarted if every worker were to choose to go his own separate way instead of joining his co-employees in planning collective action and presenting a united front when they sit down to bargain with their employers. It is for this reason that the law has sanctioned stipulations for the union shop and the closed shop as a means of encouraging the workers to join and support the labor union of their own choice as their representative in the negotiation of their demands and the protection of their interest vis-à-vis the employer.

In other words, the purpose of a union shop or other union security arrangement is to guarantee the continued existence of the union through enforced membership for the benefit of the workers. All employees in the bargaining unit covered by a Union Shop Clause in their CBA with management are subject to its terms. However, under law and jurisprudence, the following kinds of employees are exempted from its coverage, namely, employees who at the time the union shop agreement takes effect are bona fide members of a religious organization which prohibits its members from joining labor unions on religious grounds; employees already in the service and already members of a union other than the majority at the time the union shop agreement took effect; confidential employees who are excluded from the rank and file bargaining unit; and employees excluded from the union shop by express terms of the agreement.

When certain employees are obliged to join a particular union as a requisite for continued employment, as in the case of Union Security Clauses, this condition is a valid restriction of the freedom or right not to join any labor organization because it is in favor of unionism. This Court, on occasion, has even held that a union security clause in a CBA is not a restriction of the right of freedom of association guaranteed by the Constitution. Moreover, a closed shop agreement is an agreement whereby an employer binds himself to hire only members of the contracting union who must continue to remain members in good standing to keep their jobs. It is "the most prized achievement of unionism." It adds membership and compulsory dues. By holding out to loyal members a promise of employment in the closed shop, it wields group solidarity.

Indeed, the situation of the former FEBTC employees in this case clearly does not fall within the first three exceptions to the application of the Union Shop Clause discussed earlier. No allegation or evidence of religious exemption or prior membership in another union or engagement as a confidential employee was presented by both parties. The sole category therefore in which petitioner may prove its claim is the fourth recognized exception or whether the former FEBTC employees are excluded by the express terms of the existing CBA between petitioner and respondent.

44

As the Union likewise pointed out in its pleadings, there were benefits under the CBA that the former FEBTC employees did not enjoy with their previous employer. As BPI employees, they will enjoy all these CBA benefits upon their "absorption." Thus, although in a sense BPI is continuing FEBTC’s employment of these absorbed employees, BPI’s employment of these absorbed employees was not under exactly the same terms and conditions as stated in the latter’s employment contracts with FEBTC. This further strengthens the view that BPI and the former FEBTC employees voluntarily contracted with each other for their employment in the surviving corporation. The rationale for upholding the validity of union shop clauses in a CBA, even if they impinge upon the individual employee’s right or freedom of association, is not to protect the union for the union’s sake. Laws and jurisprudence promote unionism and afford certain protections to the certified bargaining agent in a unionized company because a strong and effective union presumably benefits all employees in the bargaining unit since such a union would be in a better position to demand improved benefits and conditions of work from the employer. In the case at bar, since the former FEBTC employees are deemed covered by the Union Shop Clause, they are required to join the certified bargaining agent, which supposedly has gathered the support of the majority of workers within the bargaining unit in the appropriate certification proceeding. Their joining the certified union would, in fact, be in the best interests of the former FEBTC employees for it unites their interests with the majority of employees in the bargaining unit. It encourages employee solidarity and affords sufficient protection to the majority status of the union during the life of the CBA which are the precisely the objectives of union security clauses, such as the Union Shop Clause involved herein. We are indeed not being called to balance the interests of individual employees as against the State policy of promoting unionism, since the employees, who were parties in the court below, no longer contested the adverse Court of Appeals’ decision. Nonetheless, settled jurisprudence has already swung the balance in favor of unionism, in recognition that ultimately the individual employee will be benefited by that policy. In the hierarchy of constitutional values, this Court has repeatedly held that the right to abstain from joining a labor organization is subordinate to the policy of encouraging unionism as an instrument of social justice.

In sum, this Court finds it reasonable and just to conclude that the Union Shop Clause of the CBA covers the former FEBTC employees who were hired/employed by BPI during the effectivity of the CBA in a manner which petitioner describes as "absorption." A contrary appreciation of the facts of this case would, undoubtedly, lead to an inequitable and very volatile labor situation which this Court has consistently ruled against. In the case of former FEBTC employees who initially joined the union but later withdrew their membership, there is even greater reason for the union to request their dismissal from the employer since the CBA also contained a Maintenance of Membership Clause.

A final point in relation to procedural due process, the Court is not unmindful that the former FEBTC employees’ refusal to join the union and BPI’s refusal to enforce the Union Shop Clause in this instance may have been based on the honest belief that the former FEBTC employees were not covered by said clause. In the interest of fairness, we believe the former FEBTC employees should be given a fresh thirty (30) days from notice of finality of this decision to join the union before the union demands BPI to terminate their employment under the Union Shop Clause, assuming said clause has been carried over in the present CBA and there has been no material change in the situation of the parties.

OCTAVIO vs. PHILIPPINE LONG DISTANCE TELEPHONE COMPANY [G.R. No. 175492, February 27, 2013]

Facts: On October 1, 2000, PLDT hired Octavio as Sales System Analyst I on a probationary status. He became a member of GUTS. When Octavio was regularized on January 1, 2001, he was receiving a monthly basic salary of P10,000.00. On February 1, 2002, he was promoted to the position of Sales System Analyst 2 and his salary was increased to P13,730.00. Octavio claimed entitlement to salary increases per the CBAs of 1999-2001 and 2002-2004. He insisted that when he was regularized as a supervisory employee on January 1, 2001, he became entitled to receive the

45

across-the-board increase of P2,500.00 as provided for under the CBA of 1999-2001 which took effect on January 1, 1999. Then pursuant to the CBA of 2002-2004, he should have received an additional increase of P2,000.00 apart from the merit increase of P3,730.00 which was given him due to his promotion on February 1, 2002. However, PLDT unilaterally decided to deem as included in the said P3,730.00 the P2,000.00 across-the-board increase for 2002 as stipulated in the CBA of 2002-2004. This, according to Octavio, amounts to diminution of benefits. Moreover, Octavio averred that the CBA cannot be the subject of further negotiation as it has the force of law between the parties. Finally, Octavio claimed that PLDT committed an act of unfair labor practice because, while it granted the claim for salary increase of 18 supervisory employees who were regularized on January 1, 2002 and onwards, it discriminated against him by refusing to grant him the same salary increase. He thus prayed for an additional award of damages and attorney's fees. PLDT countered that the issues advanced by Octavio had already been resolved by the Union-Management Grievance Committee when it denied his claims through the Committee Resolution. Moreover, the grant of across-the-board salary increase for those who were regularized starting January 1, 2002 and the exclusion thereto of those who were regularized on January 1, 2001, do not constitute an act of unfair labor practice as would result in any discrimination or encourage or discourage membership in a labor organization. In fact, when the Union-Management Grievance Committee came up with the Committee Resolution, they considered the same as the most practicable and reasonable solution for both management and union. At any rate, the said Committee Resolution had already become final and conclusive between the parties for failure of Octavio to elevate the same to the proper forum. In addition, PLDT claimed that the NLRC has no jurisdiction to hear and decide Octavio's claims. The committee failed to reach an agreement. Hence, Management position deemed adopted which is read as follows: xxx B) Mr. Octavio's salary at the time of his promotion and before the conclusion of the GUTS CBA was P10,000.00. C) Upon the effectivity of his promotion on February 1, 2002, his basic monthly salary was adjusted to P13,730.00, the minimum salary of the new position. D) In June 2002, the GUTS-CBA was concluded and Mr. Octavio's basic salary was recomputed to include the P2,000.00 1st year increase retroactive January 2002. The resulting basic salary was P12,000.00. E) Applying the above-mentioned policy, Mr. Octavio's basic salary was adjusted to the minimum salary of the new position, which is P13,730.00. xxx

Aggrieved, Octavio filed before the Arbitration Branch of the NLRC a Complaint for payment of said salary increases. The Labor Arbiter dismissed the Complaint of Octavio and upheld the Committee Resolution. The CA declared the Committee Resolution to be binding on Octavio, he being a member of GUTS, and because he failed to question its validity and enforceability. Issue: a. b. c.

Whether the employer and bargaining representative may amend the provisions of the collective bargaining agreement without the consent and approval of the employees; If so, whether the said agreement is binding [on] the employees; Whether merit increases may be awarded simultaneously with increases given in the Collective

46

Bargaining Agreement

Ruling: The claim of an individual employee for salary increase under the terms of the existing collective bargaining agreement (CBA) is a matter subject of the grievance machinery, it involving the interpretation and implementation of the pertinent provisions of the CBA. From the denial of his claim by the UnionManagement Grievance Committee, his recourse pursuant to the CBA was to elevate his grievance to the Board of Arbitrators for final decision. Here, he instead filed a complaint with the arbitration branch of the NLRC nine months after the resolution of the Union- Management Grievance Committee. His complaint must therefore fail. By failing to abide with the procedure prescribed by the CBA, he is deemed to have waived his right to question the resolution of the Committee. Clearly, he departed from the grievance procedure mandated in the CBA and denied the Board of Arbitrators the opportunity to pass upon a matter over which it has jurisdiction. That has the effect of making the Committee’s resolution binding upon him. Moreover, the Committee’s resolution is not a modification of the CBA. Said resolution is a product of the grievance procedure outlined in the CBA itself. Finally, the denial of his claim for salary increase did not violate Article 100 of the Labor Code against diminution of benefits. Even assuming there is diminution of benefits, Article 100 does not prohibit a union from offering and agreeing to reduce wages and benefits of the employees as the right to free collective bargaining includes the right to suspend it. Bargaining covers a process of finding a reasonable and acceptable solution to stabilize labor-management relations to promote stable industrial peace.

NATIONAL UNION OF BANK EMPLOYEES vs. PHILNABANK EMPLOYEES ASSOCIATION [G.R. No. 174287, August 12, 2013] Facts: Philippine National Bank (PNB) used to be a government-owned and controlled banking institution. Its rankand-file employees, being government personnel, were represented for collective negotiation by the Philnabank Employees Association (PEMA), a public sector union. In 1996, the Securities and Exchange Commission approved its changed status as a private corporation. PEMA affiliated with petitioner National Union of Bank Employees (NUBE), which is a labor federation composed of unions in the banking industry, adopting the name NUBE-PNB Employees Chapter (NUBE-PEC). Later, NUBE-PEC was certified as the sole and exclusive bargaining agent of the PNB rank-and-file employees. A collective bargaining agreement (CBA) was subsequently signed between NUBE-PEC and PNB covering the period of January 1, 1997 to December 31, 2001. Pursuant to Article V on Check-off and Agency Fees of the CBA, PNB shall deduct the monthly membership fee and other assessments imposed by the union from the salary of each union member, and agency fee from the salary of the rank- and-file employees within the bargaining unit who are not union members. Moreover, during the effectivity of the CBA, NUBE, being the Federation union, agreed that PNB shall remit P15.00 of the P65.00 union dues per month collected by PNB from every employee, and that PNB shall directly credit the amount to NUBE’s current account with PNB. Following the expiration of the CBA, the Philnabank Employees Association-FFW (PEMA-FFW) filed on January 2, 2002 a petition for certification election among the rank-and-file employees of PNB. While the petition for certification election was still pending, two significant events transpired – the independent union registration of NUBE- PEC and its disaffiliation with NUBE.

Thereafter, the Board of Directors of NUBE-PEC adopted a Resolution disaffiliating itself from NUBE.

47

On June 25, 2003, NUBE-PEC filed a Manifestation and Motion before the Med-Arbitration Unit of DOLE, praying that, in view of its independent registration as a labor union and disaffiliation from NUBE, its name as appearing in the official ballots of the certification election be changed to "Philnabank Employees Association (PEMA)" or, in the alternative, both parties be allowed to use the name "PEMA" but with PEMAFFW and NUBE-PEC be denominated as "PEMA-Bustria Group" and "PEMA-Serrana Group," respectively. On the same date, PEMA sent a letter to the PNB management informing its disaffiliation from NUBE and requesting to stop, effective immediately, the check-off of the P15.00 due for NUBE. Acting thereon, on July 4, 2003, PNB informed NUBE of PEMA’s letter and its decision to continue the deduction of the P15.00 fees, but stop its remittance to NUBE effective July 2003. PNB also notified NUBE that the amounts collected would be held in a trust account pending the resolution of the issue on PEMA’s disaffiliation.

Issue: Whether PEMA validly disaffiliated itself from NUBE, the resolution of which, in turn, inevitably affects the latter’s right to collect the union dues held in trust by PNB.

Ruling: We deny the petition. Whether there was a valid disaffiliation is a factual issue. It is elementary that a question of fact is not appropriate for a petition for review on certiorari under Rule 45 of the Rules of Court. When supported by substantial evidence, the findings of fact of the CA are conclusive and binding on the parties and are not reviewable by this Court subject to exceptions. However, the Court finds no cogent reason to apply these recognized exceptions. The right of the local union to exercise the right to disaffiliate from its mother union is well settled in this jurisdiction. A local union has the right to disaffiliate from its mother union or declare its autonomy. A local union, being a separate and voluntary association, is free to serve the interests of all its members including the freedom to disaffiliate or declare its autonomy from the federation which it belongs when circumstances warrant, in accordance with the constitutional guarantee of freedom of association. Local unions do not owe their creation and existence to the national federation to which they are affiliated but, instead, to the will of their members. The sole essence of affiliation is to increase, by collective action, the common bargaining power of local unions for the effective enhancement and protection of their interests. Admittedly, there are times when without succor and support local unions may find it hard, unaided by other support groups, to secure justice for themselves. It only gives rise to a contract of agency, where the former acts in representation of the latter. Hence, local unions are considered principals while the federation is deemed to be merely their agent. As such principals, the unions are entitled to exercise the rights and privileges of a legitimate labor organization, including the right to seek certification as the sole and exclusive bargaining agent in the appropriate employer unit.

x x x [A] local union may disaffiliate at any time from its mother federation, absent any showing that the same is prohibited under its constitution or rule. Such, however, does not result in it losing its legal personality altogether.

48

In the case at bar, there is nothing shown in the records nor is it claimed by NUBE that PEMA was expressly forbidden to disaffiliate from the federation nor were there any conditions imposed for a valid breakaway. This being so, PEMA is not precluded to disaffiliate from NUBE after acquiring the status of an independent labor organization duly registered before the DOLE. Also, there is no merit on NUBE’s contention that PEMA’s disaffiliation is invalid for non-observance of the procedure that union members should make such determination through secret ballot and after due deliberation, conformably with Article 241 (d) of the Labor Code, as amended. Conspicuously, other than citing the opinion of a "recognized labor law authority," NUBE failed to quote a specific provision of the law or rule mandating that a local union’s disaffiliation from a federation must comply with Article 241 (d) in order to be valid and effective. Granting, for argument’s sake, that Article 241 (d) is applicable, still, We uphold PEMA’s disaffiliation from NUBE. First, non-compliance with the procedure on disaffiliation, being premised on purely technical grounds cannot rise above the employees’ fundamental right to self-organization and to form and join labor organizations of their own choosing for the purpose of collective bargaining. Second, the Article nonetheless provides that when the nature of the organization renders such secret ballot impractical, the union officers may make the decision in behalf of the general membership. In this case, NUBE did not even dare to contest PEMA’s representation that "PNB employees, from where [PEMA] [derives] its membership, are scattered from Aparri to Jolo, manning more than 300 branches in various towns and cities of the country," hence, "[to] gather the general membership of the union in a general membership to vote through secret balloting is virtually impossible." It is understandable, therefore, why PEMA’s board of directors merely opted to submit for ratification of the majority their resolution to disaffiliate from NUBE. Third, and most importantly, NUBE did not dispute the existence of the persons or their due execution of the document showing their unequivocal support for the disaffiliation of PEMA from NUBE. Consequently, by PEMA's valid disaffiliation from NUBE, the vinculum that previously bound the two entities was completely severed. As NUBE was divested of any and all power to act in representation of PEMA, any act performed by the former that affects the interests and affairs of the latter, including the supposed expulsion of Serrana et al., is rendered without force and effect. Also, in effect, NUBE loses it right to collect all union dues held in its trust by PNB. The moment that PEMA separated from and left NUBE and exists as an independent labor organization with a certificate of registration, the former is no longer obliged to pay dues and assessments to the latter; naturally, there would be no longer any reason or occasion for PNB to continue making deductions. Without said affiliation, the employer has no link to the mother union. The obligation of an employee to pay union dues is coterminous with his affiliation or membership. "The employees' check-off authorization, even if declared irrevocable, is good only as long as they remain members of the union concerned." A contract between an employer and the parent organization as bargaining agent for the employees is terminated bv the disaffiliation ofthe local of which the employees are members. On the other hand, it was entirely reasonable for PNB to enter into a CBA with PEMA as represented by Serrana et al. Since PEMA had validly separated itself from NUBE, there would be no restrictions which could validly hinder it from collectively bargaining with PNB.

TOPIC 13: RIGHTS OF LEGITIMATE LABOR ORGANIZATION

49

RIVERA VS. ESPIRITU, G.R. NO. 135547, JANUARY 23, 2002 Facts:

On June 5, 1998, PAL pilots affiliated with the Airline Pilots Association of the Philippines (ALPAP) went on a three-week strike, causing serious losses to the financially beleaguered flag carrier. As a result, PAL’s financial situation went from bad to worse. Faced with bankruptcy, PAL adopted a rehabilitation plan and downsized its labor force by more than one-third.

On July 22, 1998, PALEA went on strike to protest the retrenchment measures adopted by the airline, which affected 1,899 union members. The strike ended four days later, when PAL and PALEA agreed to a more systematic reduction in PAL’s work force and the payment of separation benefits to all retrenched employees.

On August 28, 1998, then President Estrada issued Administrative Order No. 16 creating an Inter-Agency Task Force (Task Force) to address the problems of the ailing flag carrier. The Task Force was composed of the Departments of Finance, Labor and Employment, Foreign Affairs, Transportation and Communication, and Tourism, together with the Securities and Exchange Commission (SEC). It was “empowered to summon all parties concerned for conciliation, mediation (for) the purpose of arriving at a total and complete solution of the problem.” Conciliation meetings were then held between PAL management and the three unions representing the airline’s employees, with the Task Force as mediator.

PAL management submitted to the Task Force an offer by private respondent Lucio Tan, Chairman and Chief Executive Officer of PAL, of a plan to transfer shares of stock to its employees. The pertinent portion of said plan, apart from transferring shares of stocks of the company to employees reads that:

xxx 3. In order for PAL to attain (a) degree of normalcy while we are tackling its problems, we would request for a suspension of the Collective Bargaining Agreements (CBAs) for 10 years.

This offer was rejected by the members of PALEA, a duly organized registered union of PAL. After a few more negotiations, and mounting pressure on the worker’s due to PAL’s imminent closure, the following was agreed upon and further ratified by PALEA and PAL:

“x x x 4. To assure investors and creditors of industrial peace, PALEA agrees, subject to the ratification by the general membership, (to) the suspension of the PAL-PALEA CBA for a period of ten (10) years, provided the following safeguards are in place: a. PAL shall continue recognizing PALEA as the duly certified bargaining agent of the regular rank-and-file ground employees of the Company; b. The ‘union shop/maintenance of membership’ provision under the PAL-PALEA CBA shall be respected. x x x.”

Seven officers and members of PALEA filed a petition to annul the agreement entered into between PAL and PALEA on the ground that it violates the constitutional rights of employees to self-organization and collective bargaining, being founded on public policy, may not be waived, nor waiver, ratified.

Issue:

50

Whether or not the PAL-PALEA agreement, stipulating the suspension of the PAL-PALEA CBA unconstitutional and contrary to public policy.

Ruling:

The SC held that the agreement is both constitutional and in consonance with public policy.

Petitioners claim the agreement’s unconstitutionality saying that violates the worker’s right to selforganization and collective bargaining hence violating the “protection to labor” policy laid down by the Constitution:

ART. 253-A. Terms of a Collective Bargaining Agreement. – Any Collective Bargaining Agreement that the parties may enter into shall, insofar as the representation aspect is concerned, be for a term of five (5) years.x x x All other provisions of the Collective Bargaining Agreement shall be renegotiated not later than three (3) years after its execution.

Under this provision, insofar as representation is concerned, a CBA has a term of five years, while the other provisions, except for representation, may be negotiated not later than three years after the execution. Petitioners submit by agreeing to a 10-year suspension, PALEA, in effect, abdicated the workers’ constitutional right to bargain for another CBA at the mandated time. This argument is devoid of merit.

A CBA is “a contract executed upon request of either the employer or the exclusive bargaining representative incorporating the agreement reached after negotiations with respect to wages, hours of work and all other terms and conditions of employment, including proposals for adjusting any grievances or questions arising under such agreement.”

The assailed PAL-PALEA agreement was the result of voluntary collective bargaining negotiations undertaken in the light of the severe financial situation faced by the employer, with the peculiar and unique intention of not merely promoting industrial peace at PAL, but preventing the latter’s closure.

The primary purpose of a CBA is the stabilization of labor-management relations in order to create a climate of a sound and stable industrial peace.

The SC found no conflict between said agreement and Article 253-A of the Labor Code. Article 253-A has a two-fold purpose. One is to promote industrial stability and predictability. Inasmuch as the agreement sought to promote industrial peace at PAL during its rehabilitation, said agreement satisfies the first purpose of Article 253-A. The other is to assign specific timetables wherein negotiations become a matter of right and requirement. Nothing in Article 253-A, prohibits the parties from waiving or suspending the mandatory timetables and agreeing on the remedies to enforce the same.

In the instant case, it was PALEA, as the exclusive bargaining agent of PAL’s ground employees, that voluntarily entered into the CBA with PAL and opted for the 10-year suspension of the CBA. Either case it was the union’s exercise of its right to collective bargaining. The right to free collective bargaining, after all, includes the right to suspend it.

Petitioners further allege that the 10-year suspension of the CBA under the PAL-PALEA agreement virtually installed PALEA as a company union for said period, amounting to unfair labor practice.

The questioned proviso of the agreement reads: a. PAL shall continue recognizing PALEA as the duly certified-bargaining agent of the regular rank-and-file ground employees of the Company;

51

The aforesaid provision clearly only shows the intent of the parties to maintain “union security” during the period of the suspension of the CBA. Its objective is to assure the continued existence of PALEA during the said period. We are unable to declare the objective of union security an unfair labor practice. It is State policy to promote unionism to enable workers to negotiate with management on an even playing field and with more persuasiveness than if they were to individually and separately bargain with the employer. For this reason, the law has allowed stipulations for “union shop” and “closed shop” as means of encouraging workers to join and support the union of their choice in the protection of their rights and interests vis-à-vis the employer.

Petitioners’ contention that the agreement installs PALEA as a virtual company union is also untenable. Under Article 248 (d) of the Labor Code, a company union exists when the employer acts “[t]o initiate, dominate, assist or otherwise interfere with the formation or administration of any labor organization, including the giving of financial or other support to it or its organizers or supporters.” The case records are bare of any showing of such acts by PAL.

We also do not agree that the agreement violates the five-year representation limit mandated by Article 253-A. Under said article, the representation limit for the exclusive bargaining agent applies only when there is an extant CBA in full force and effect. In the instant case, the parties agreed to suspend the CBA and put in abeyance the limit on the representation period.

In sum, we are of the view that the PAL-PALEA agreement dated September 27, 1998, is a valid exercise of the freedom to contract. Under the principle of inviolability of contracts guaranteed by the Constitution, the contract must be upheld.

Petition is dismissed.

SAN MIGUEL CORP., VS. NLRC, G.R. NO. 119293, JUNE 10, 2003 Facts:

Petitioner San Miguel Corporation (SMC) and respondent Ilaw at Buklod ng Manggagawa (IBM), exclusive bargaining agent of petitioner’s daily-paid rank and file employees, executed a Collective Bargaining Agreement (CBA) under which they agreed to submit all disputes to grievance and arbitration proceedings. The CBA also included a mutually enforceable no-strike no-lockout agreement.

On April 11, 1994, IBM, through its vice president Colomeda, filed with the NCMB a notice of strike against petitioner for allegedly committing: (1) illegal dismissal of union members, (2) illegal transfer, (3) violation of CBA, (4) contracting out of jobs being performed by union members, (5) labor-only contracting, (6) harassment of union officers and members, (7) non-recognition of duly-elected union officers, and (8) other acts of unfair labor practice. The next day, IBM filed another notice of strike, this time through its president Galvez, raising similar grounds: (1) illegal transfer, (2) labor-only contracting, (3) violation of CBA, (4) dismissal of union officers and members, and (5) other acts of unfair labor practice. The Galvez group subsequently requested the NCMB to consolidate its notice of strike with that of the Colomeda group, to which the latter opposed, alleging Galvez’s lack of authority in filing the same.

Petitioner company thereafter filed a Motion for Severance of Notices of Strike with Motion to Dismiss, on the grounds that the notices raised non-strikeable issues and that they affected four corporations which are separate and distinct from each other. The NCMB thereafter found that the real issues involved are nonstrikeable. Hence it issued separate letter-orders to both union groups, converting their notices of strike into preventive mediation.

52

On May 16, 1994, while separate preventive mediation conferences were ongoing, the Colomeda group filed with the NCMB a notice of holding a strike vote. Petitioner opposed by filing a Manifestation and Motion to Declare Notice of Strike Vote Illegal. NCMB Director Ubaldo in response issued another letter to the Colomeda Group reiterating the conversion of the notice of strike into a case of preventive mediation and emphasizing the findings that the grounds raised center only on an intra-union conflict, which is not strikeable.

Meanwhile, on May 23, 1994, the Galvez group filed its second notice of strike against petitioner, where additional grounds were set forth therein, including discrimination, coercion of employees, illegal lockout and illegal closure. The NCMB however found these grounds to be mere amplifications of those alleged in the first notice that the group filed. It therefore ordered the consolidation of the second notice with the preceding one that was earlier reduced to preventive mediation. On the same date, the group likewise notified the NCMB of its intention to hold a strike vote on May 27, 1994.

On May 27, 1994, the Colomeda group notified the NCMB of the results of their strike vote, which favored the holding of a strike. In reply, NCMB issued a letter again advising them that by virtue of the PAL v. Drilon ruling, their notice of strike is deemed not to have been filed, consequently invalidating any subsequent strike for lack of compliance with the notice requirement. Despite this and the pendency of the preventive mediation proceedings, on June 4, 1994, IBM went on strike. The strike paralyzed the operations of petitioner, causing it losses allegedly worth P29.98 million in daily lost production.

Two days after the declaration of strike, petitioner company filed with NLRC an amended Petition for Injunction with Prayer for the Issuance of Temporary Restraining Order, Free Ingress and Egress Order and Deputization Order. After due hearing and ocular inspection, the NLRC on June 13, 1994 resolved to issue a temporary restraining order (TRO) directing free ingress to and egress from petitioner’s plants, without prejudice to the union’s right to peaceful picketing and continuous hearings on the injunction case.

To minimize further damage to itself, petitioner on June 16, 1994, entered into a Memorandum of Agreement (MOA) with the respondent-union, calling for a lifting of the picket lines and resumption of work in exchange of “good faith talks” between the management and the labor management committees. The MOA, signed in the presence of Department of Labor and Employment (DOLE) officials, expressly stated that cases filed in relation to their dispute will continue and will not be affected in any manner whatsoever by the agreement. The picket lines ended and work was then resumed.

Respondent thereafter moved to reconsider the issuance of the TRO, and sought to dismiss the injunction case in view of the cessation of its picketing activities as a result of the signed MOA. It argued that the case had become moot and academic there being no more prohibited activities to restrain, be they actual or threatened.[21] Petitioner, however, opposed and submitted copies of flyers being circulated by IBM, as proof of the union’s alleged threat to revive the strike.The NLRC did not rule on the opposition to the TRO and allowed it to lapse.

On November 29, 1994, the NLRC issued the challenged decision, denying the petition for injunction for lack of factual basis. It found that the circumstances at the time did not constitute or no longer constituted an actual or threatened commission of unlawful acts. It likewise denied petitioner’s motion for reconsideration in its resolution dated February 1, 1995. Hence, this petition.

Issue:

Whether or not it was appropriate for the NLRC to deny the petitioner company’s petition for injunction.

Ruling:

53

The SC held that it was incorrect for the NLRC to deny the petitioner company’s petition for injunction.

Article 254 of the Labor Code provides that no temporary or permanent injunction or restraining order in any case involving or growing out of labor disputes shall be issued by any court or other entity except as otherwise provided in Articles 218 and 264 of the Labor Code. Under the first exception, Article 218 (e) of the Labor Code expressly confers upon the NLRC the power to “enjoin or restrain actual and threatened commission of any or all prohibited or unlawful acts, or to require the performance of a particular act in any labor dispute which, if not restrained or performed forthwith, may cause grave or irreparable damage to any party or render ineffectual any decision in favor of such party x x x.” The second exception, on the other hand, is when the labor organization or the employer engages in any of the “prohibited activities” enumerated in Article 264.

Pursuant to Article 218 (e), the coercive measure of injunction may also be used to restrain an actual or threatened unlawful strike. In the case of San Miguel Corporation v. NLRC, where the same issue of NLRC’s duty to enjoin an unlawful strike was raised, we ruled that the NLRC committed grave abuse of discretion when it denied the petition for injunction to restrain the union from declaring a strike based on nonstrikeable grounds. Further, in IBM v. NLRC, we held that it is the “legal duty and obligation” of the NLRC to enjoin a partial strike staged in violation of the law. Failure promptly to issue an injunction by the public respondent was likewise held therein to be an abuse of discretion.

In the case at bar, petitioner sought a permanent injunction to enjoin the respondent’s strike. A strike is considered as the most effective weapon in protecting the rights of the employees to improve the terms and conditions of their employment. However, to be valid, a strike must be pursued within legal bounds. One of the procedural requisites that Article 263 of the Labor Code and its Implementing Rules prescribe is the filing of a valid notice of strike with the NCMB. Imposed for the purpose of encouraging the voluntary settlement of disputes, this requirement has been held to be mandatory, the lack of which shall render a strike illegal.

In the present case, NCMB converted IBM’s notices into preventive mediation as it found that the real issues raised are non-strikeable. Such order is in pursuance of the NCMB’s duty to exert “all efforts at mediation and conciliation to enable the parties to settle the dispute amicably,” and in line with the state policy of favoring voluntary modes of settling labor disputes. In accordance with the Implementing Rules of the Labor Code, the said conversion has the effect of dismissing the notices of strike filed by respondent. When the NCMB ordered the preventive mediation on May 2, 1994, respondent had thereupon lost the notices of strike it had filed. Subsequently, however, it still defiantly proceeded with the strike while mediation was ongoing, and notwithstanding the letter-advisories of NCMB warning it of its lack of notice of strike.

The NCMB having no coercive powers of injunction, petitioner sought recourse from the public respondent. The NLRC issued a TRO only for free ingress to and egress from petitioner’s plants, but did not enjoin the unlawful strike itself. It ignored the fatal lack of notice of strike, and five months after came out with a decision summarily rejecting petitioner’s cited jurisprudence.

Public respondent, in its decision, ruled that there was a lack of factual basis in issuing the injunction. Contrary to the NLRC’s finding, we find that at the time the injunction was being sought, there existed a threat to revive the unlawful strike as evidenced by the flyers then being circulated by the IBM-NCR Council which led the union. These flyers were not denied by respondent, and were dated June 19, 1994, just a day after the union’s manifestation with the NLRC that there existed no threat of commission of prohibited activities.

Moreover, it bears stressing that Article 264(a) of the Labor Code explicitly states that a declaration of strike without first having filed the required notice is a prohibited activity, which may be prevented through an injunction in accordance with Article 254. Clearly, public respondent should have granted the injunctive relief to prevent the grave damage brought about by the unlawful strike.

54

Also noteworthy is public respondent’s disregard of petitioner’s argument pointing out the union’s failure to observe the CBA provisions on grievance and arbitration.

The Sc added that it cannot sanction the respondent-union’s brazen disregard of legal requirements imposed purposely to carry out the state policy of promoting voluntary modes of settling disputes. The state’s commitment to enforce mutual compliance therewith to foster industrial peace is affirmed by no less than our Constitution. Trade unionism and strikes are legitimate weapons of labor granted by our statutes. But misuse of these instruments can be the subject of judicial intervention to forestall grave injury to a business enterprise.

WHEREFORE, the instant petition is hereby GRANTED.

SAMAHANG MANGGAGAWA SA SULPICIO LINES VS. SULPICIO LINES, G.R. NO. 140992, MARCH 25, 2004 Facts:

Due to a CBA deadlock, petitioner union filed with the NCMB a Notice of Strike. Respondent company, on its part filed with the DOLE a petition praying that the SOLE assume jurisdiction over the controversy. The SOLE then issued an order assuming jurisdiction over the labor dispute, enjoined any strike or lockout and directed the parties to cease and desist from committing any and all acts that might exacerbate the situation.

Meanwhile, on May 20, 1994, petitioner filed with the NCMB a second notice of strike alleging that respondent company committed acts constituting unfair labor practice amounting to union busting. Provoked by respondent’s alleged unfair labor practice/s, petitioner union immediately conducted a strike vote. Thus, on May 20, 1994, about 9:30 o’clock in the morning, 167 rank-and-file employees, officers and members of petitioner, did not report for work and instead gathered in front of Pier 12, North Harbor at Manila.

The SOLE again issued an order directing the striking employees to return to work; and certifying the labor dispute to the NLRC for compulsory arbitration. Meanwhile, respondent company filed with the NLRC a complaint for “illegal strike/clearance for termination.”

On September 29, 1995, the NLRC issued a Resolution declaring the strike of petitioner’s officers and members illegal, with notice to respondent of the option to terminate their (petitioner’s officers) employment. In the same Resolution, the NLRC dismissed petitioner’s complaint against respondent.

Issue:

Whether or not the one-day work stoppage or petitioner’s officers and members was an illegal strike.

Ruling:

The SC held that the one-day work stoppage or petitioner’s officers and members was an illegal strike.

Articles 263 and 264 of the Labor Code, as amended, provide:

55

“ART. 263. STRIKES, PICKETING AND LOCKOUTS. xxx (c) In cases of bargaining deadlocks, the duly certified or recognized bargaining agent may file a notice of strike x x x with the Ministry (now Department) at least 30 days before the intended date thereof. In cases of unfair labor practice, the period of notice shall be 15 days and in the absence of a duly certified or recognized bargaining agent, the notice of strike may be filed by any legitimate labor organization in behalf of its members. However, in case of dismissal from employment of union officers duly elected in accordance with the union constitution and by-laws, which may constitute union busting where the existence of the union is threatened, the 15-day cooling-off period shall not apply and the union may take action immediately. xxx (f) A decision to declare a strike must be approved by a majority of the total union membership in the bargaining unit concerned, obtained by secret ballot in meetings or referenda called for that purpose. x x x. The decision shall be valid for the duration of the dispute based on substantially the same grounds considered when the strike or lockout vote was taken. The Ministry (now Department) may at its own initiative or upon the request of any affected party, supervise the conduct of the secret balloting. In every case, the union x x x shall furnish the Ministry (now Department) the results of the voting at least seven days before the intended strike or lockout, subject to the cooling-off period herein provided. x x x. ART. 264. PROHIBITED ACTIVITIES. (a) No labor organization or employer shall declare a strike or lockout without first having bargained collectively in accordance with Title VII of this Book or without first having filed the notice required in the preceding article or without the necessary strike or lockout vote first having been obtained and reported to the Ministry (now Department). x x x.”

Following are the Implementing Guidelines of the above provisions issued by the Department of Labor and Employment: 1. A strike shall be filed with the Department of Labor and Employment at least 15 days if the issues raised are unfair labor practice or at least 30 days if the issue involved bargaining deadlock. However, in case of dismissal from employment of union officers duly elected in accordance with the union constitution and bylaws, which may constitute union busting where the existence of the union is threatened, the 15-day cooling-off period shall not apply and the union may take action immediately; 2. The strike shall be supported by a majority vote of the members of the union obtained by secret ballot in a meeting called for the purpose; and 3. A strike vote shall be reported to the Department of Labor and Employment at least seven (7) days before the intended strike.

There is no showing that the petitioner union observed the 7-day strike ban; and that the results of the strike vote were submitted by petitioners to the Department of Labor and Employment at least seven (7) days before the strike. The SC thus held that for failing to comply with the mandatory requirements of Article 263 (c) and (f) of the Labor Code, the strike mounted by petitioner union on May 20, 1994 is illegal.

The rule now is that such requirements as the filing of a notice of strike, strike vote, and notice given to the Department of Labor are mandatory in nature. Thus, even if the union acted in good faith in the belief that the company was committing an unfair labor practice, if no notice of strike and a strike vote were conducted, the said strike is illegal.” In a desperate attempt to justify its position, petitioner insists that what transpired on May 20, 1994 was not a strike but merely a “one-day work absence” or a “simple act of absenteeism”. The SC was not convinced. A strike, as defined in Article 212 (o) of the Labor Code, as amended, means “any temporary stoppage of work by the concerted action of employees as a result of an industrial or labor dispute.” The term “strike” shall comprise not only concerted work stoppages, but also slowdowns, mass

56

leaves, sitdowns, attempts to damage, destroy or sabotage plant equipment and facilities, and similar activities.

The basic elements of a strike are present in the case at bar. First, petitioner’s officers and members numbering 167, in a concerted manner, did not report for work on May 20, 1994; second, they gathered in front of respondent’s office at Pier 12, North Harbor at Manila to participate in a strike voting conducted by petitioner; and third, such union activity was an aftermath of petitioner’s second notice of strike by reason of respondent’s unfair labor practice/s. Clearly, what transpired then was a strike because the cessation of work by petitioner’s concerted action resulted from a labor dispute.

Invoking compassion, petitioner pleads that its officers who participated in the one-day strike should not be dismissed from the service, considering that respondent’s business activities were not interrupted, much less paralyzed. While we sympathize with their plight, however, we must take care that in the contest between labor and capital, the results achieved are fair and in conformity with the law.

Petition is denied.

NOTRE DAME OF GREATER MANILA VS. LAGUESMA, G.R.NO. 149833, JUNE 29, 2004 Facts:

Respondent union filed with the Med-Arbitration Branch, National Capital Region, (DOLE) a petition for direct certification as the sole and exclusive bargaining agent or certification election among the rank and file employees of petitioner NDGM. The Med-Arbiter issued an order granting the petition for certification election and to undertake a pre-election conference.

During one of the pre-election conferences, petitioner NDGM filed a motion to include probationary and substitute employees in the list of qualified voters. This motion was denied by the Med-Arbiter by handwritten notation on the motion itself. NDGM filed an appeal from the said handwritten ‘order’ of the Med-Arbiter. Despite the appeal of petitioner, the Med-Arbiter ordered that a certification election be conducted, wherein the “Yes” vote won.

Petitioner NDGM filed a written notice of protest against the conduct and results of the certification of election, which was opposed by private respondent NDGMTEU. The said union, a few days after, filed a motion to certify the said union as the exclusive bargaining agent of petitioner was filed. Thereafter, the Med-Arbiter issued an order which certified private respondent NDGMTEU as the sole and exclusive bargaining agent of all the rank-and-file employees of petitioner and accordingly dismissed petitioner’s protest.

The petitioner then filed an appeal of the said order but the DOLE dismissed the appeal for lack of merit. Its motion for reconsideration was likewise denied.

Upon appeal with the CA, it held that the Med-Arbiter’s notation on petitioner’s motion was not an order that could be the subject of an appeal to the SOLE. Thus, the CA held that staying the holding of the certification election was unnecessary. The CA added that that petitioner had no standing to question the qualification of the workers who should be included in the list of voters because, in the process of choosing their collective bargaining representative, the employer was definitely an intruder.

Issue:

57

Whether or not the holding of the certification election was stayed by petitioner’s appeal of the medarbiter’s notation on the Motion to Include the Probationary and Substitute Employees in the List of Qualified Voters.

Ruling: The SC held that the holding of the certification election was not stayed by petitioner’s appeal of the medarbiter’s notation on the Motion. Under Article 259 of the Labor Code, “any party to an election may appeal the order or results of the election as determined by the Med-Arbiter directly to the Secretary of Labor and Employment on the grounds that the rules and regulations or parts thereof established by the Secretary of Labor and Employment for the conduct of the election have been violated. Such appeal shall be decided within fifteen (15) calendar days.” This provision is supplemented by Section 10 of Rule V of Book Five of the 1992 Omnibus Rules Implementing the Labor Code. Stating that such appeal stays the holding of a certification election, the later provision reads: “Sec. 10. Decision of the Secretary final and inappealable.– The Secretary shall have fifteen (15) calendar days within which to decide the appeal from receipt of the records of the case. The filing of the appeal from the decision of the Med-Arbiter stays the holding of any certification election. The decision of the Secretary shall be final and inappealable.” Petitioner argues that the med-arbiter’s handwritten notation denying its Motion was the order referred to by Article 259. Hence, petitioner insists that its appeal of the denial should have stayed the holding of the certification election. Petitioner is mistaken. Article 259 clearly speaks of the “order x x x of the election.” Hence, the Article pertains, not just to any of the med-arbiter’s orders like the subject notation, but to the order granting the petition for certification election -- in the present case, that which was issued on November 18, 1991. This is an unmistakable inference from a reading of Sections 6 and 7 of the implementing rules: “SEC. 6. Procedure.– Upon receipt of a petition, the Regional Director shall assign the case to a Med-Arbiter for appropriate action. The Med-Arbiter, upon receipt of the assigned petition, shall have twenty (20) working days from submission of the case for resolution within which to dismiss or grant the petition. In a petition filed by a legitimate organization involving an unorganized establishment, the Med-Arbiter shall immediately order the conduct of a certification election.

“In a petition involving an organized establishment or enterprise where the majority status of the incumbent collective bargaining union is questioned through a verified petition by a legitimate labor organization, the Med-Arbiter shall immediately order the certification election by secret ballot if the petition is filed x x x. “SEC. 7 . Appeal.– Any aggrieved party may appeal the order of the Med-Arbiter to the Secretary on the ground that the rules and regulations or parts thereof established by the Secretary for the conduct of election have been violated.

Not all the orders issued by a med-arbiter are appealable. In fact, interlocutory orders issued by the medarbiter prior to the grant or denial of the petition, including orders granting motions for intervention issued after an order calling for a certification election, shall not be appealable. However, any issue arising therefrom may be raised in the appeal on the decision granting or denying the petition. The intention of the law is to limit the grounds for appeal that may stay the holding of a certification election. This intent is manifested by the issuance of Department Order No. 40. Under the new rules, an appeal of a med-arbiter’s order to hold a certification election will not stay the holding thereof where the employer company is an unorganized establishment, and where no union has yet been duly recognized or certified as a bargaining representative. This new rule, therefore, decreases or limits the appeals that may impede the selection by employees of their bargaining representative. Expediting such selection process advances the primacy of free collective bargaining, in accordance with the State’s policy to “promote and emphasize the primacy of free collective bargaining”; and “to ensure the participation of workers in decision and policy-making processes affecting their rights, duties and welfare.”

58

Consequently, the appeal of the med-arbiter’s January 13, 1992 handwritten notation -- pertaining to the incidental matter of the list of voters -- should not stay the holding of the certification election. More important, unless it filed a petition for a certification election pursuant to Article 258 of the Labor Code, the employer has no standing to question the election, which is the sole concern of the workers. The Labor Code states that any party to an election may appeal the decision of the med-arbiter. Petitioner was not such a party to the proceedings, but a stranger which had no right to interfere therein.

Petition is denied.

MANILA DIAMOND HOTEL EMPLOYEES UNION VS. COURT OF APPEALS, G.R. NO. 140518, DECEMBER 16, 2004 Facts:

On November 11, 1996, Petitioner union filed a petition for a certification election so that it may be declared the exclusive bargaining representative of the Hotel’s employees for the purpose of collective bargaining. The petition was dismissed by the DOLE on January 15, 1997. After a few months, however, on August 25, 1997, petitioner union sent a letter to the Hotel informing it of its desire to negotiate for a collective bargaining agreement. In reply, respondent hotel write to the union stating that the Hotel cannot recognize it as the employees’ bargaining agent since its petition for certification election had been earlier dismissed by the DOLE. On that same day, the Hotel received a letter from the Union stating that they were not giving the Hotel a notice to bargain, but that they were merely asking for the Hotel to engage in collective bargaining negotiations with the Union for its members only and not for all the rank and file employees of the Hotel.

On September 18, 1997, the Union announced that it was taking a strike vote. A Notice of Strike was thereafter filed on September 29, 1997, with the NCMB for the Hotel’s alleged “refusal to bargain” and for alleged acts of unfair labor practice. The NCMB summoned both parties and held a series of dialogues, the first of which was on October 6, 1997. On November 29, 1997, however, the Union staged a strike against the Hotel. Numerous confrontations between the two parties followed, creating an obvious strain between them. The Hotel claims that the strike was illegal and it had to dismiss some employees for their participation in the allegedly illegal concerted activity. The Union, on the other hand, accused the Hotel of illegally dismissing the workers.

The SOLE then assumed jurisdiction over the labor dispute, and issued an order certifying the same to the NLRC for compulsory arbitration. The striking officers and members of the union were also directed to return to work and the Hotel to accept them back under the same terms and conditions prevailing prior to the strike. The parties are enjoined from committing any act that may exacerbate the situation. The union members reported for work the day after in received the order but the Hotel refused to accept the returning workers and instead filed a Motion for Reconsideration of the SOLE’s Order.

Thereafter, the SOLE issued another order modifying the order earlier issued. Instead of an actual return to work, the SOLE directed that the strikers be reinstated only in the payroll. The Union moved for the reconsideration of this Order, but its motion was denied. On appeal, the CA affirmed the order of the SOLE for payroll reinstatement. The CA held that the challenged order is merely an error of judgment and not a grave abuse of discretion and that payroll reinstatement is not prohibited by law, but may be “called for” under certain circumstances. Issue: Whether or not the order of the SOLE for payroll reinstatement is “called for the circumstance” of this case. Ruling: The SC held that there is no showing that the facts called for payroll reinstatement as an alternative remedy. A strained relationship between the striking employees and management is no reason for payroll reinstatement in lieu of actual reinstatement. Petitioner correctly points out that labor disputes naturally

59

involve strained relations between labor and management, and that in most strikes, the relations between the strikers and the non-strikers will similarly be tense. Bitter labor disputes always leave an aftermath of strong emotions and unpleasant situations. Nevertheless, the government must still perform its function and apply the law, especially if, as in this case, national interest is involved.

As a general rule, the State encourages an environment wherein employers and employees themselves must deal with their problems in a manner that mutually suits them best. This is the basic policy embodied in Article XIII, Section 3 of the Constitution, which was further echoed in Article 211 of the Labor Code. Hence, a voluntary, instead of compulsory, mode of dispute settlement is the general rule.

However, Article 263, paragraph (g) of the Labor Code, which allows the Secretary of Labor to assume jurisdiction over a labor dispute involving an industry indispensable to the national interest, provides an exception:

(g) When, in his opinion, there exists a labor dispute causing or likely to cause a strike or lockout in an industry indispensable to the national interest, the Secretary of Labor and Employment may assume jurisdiction over the dispute and decide it or certify the same to the Commission for compulsory arbitration. Such assumption or certification shall have the effect of automatically enjoining the intended or impending strike or lockout as specified in the assumption or certification order. If one has already taken place at the time of assumption or certification, all striking or locked out employees shall immediately return to work and the employer shall immediately resume operations and readmit all workers under the same terms and conditions prevailing before the strike or lockout. x xx x This provision is viewed as an exercise of the police power of the State. A prolonged strike or lockout can be inimical to the national economy and, therefore, the situation is imbued with public necessity and involves the right of the State and the public to self-protection. Under Article 263(g), all workers must immediately return to work and all employers must readmit all of them under the same terms and conditions prevailing before the strike or lockout. This Court must point out that the law uses the precise phrase of “under the same terms and conditions,” revealing that it contemplates only actual reinstatement. This is in keeping with the rationale that any work stoppage or slowdown in that particular industry can be inimical to the national economy. It is clear that Article 263(g) was not written to protect labor from the excesses of management, nor was it written to ease management from expenses, which it normally incurs during a work stoppage or slowdown. It was an error on the part of the Court of Appeals to view the assumption order of the Secretary as a measure to protect the striking workers from any retaliatory action from the Hotel. This Court reiterates that this law was written as a means to be used by the State to protect itself from an emergency or crisis. It is not for labor, nor is it for management. It is, therefore, evident from the foregoing that the Secretary’s subsequent order for mere payroll reinstatement constitutes grave abuse of discretion amounting to lack or excess of jurisdiction. Indeed, this Court has always recognized the “great breadth of discretion” by the Secretary once he assumes jurisdiction over a labor dispute. However, payroll reinstatement in lieu of actual reinstatement is a departure from the rule in these cases and there must be showing of special circumstances rendering actual reinstatement impracticable, as in the UST case aforementioned, or otherwise not conducive to attaining the purpose of the law in providing for assumption of jurisdiction by the Secretary of Labor and Employment in a labor dispute that affects the national interest. None appears to have been established in this case. Even in the exercise of his discretion under Article 236(g), the Secretary must always keep in mind the purpose of the law. Time and again, this Court has held that when an official by-passes the law on the asserted ground of attaining a laudable objective, the same will not be maintained if the intendment or purpose of the law would be defeated.

Petition is granted.

UNIVERSITY OF IMMACULATE CONCEPCION VS. SEC. OF LABOR, G.R. NO. 151379, JANUARY 14, 2005 Facts:

60

During the negotiations for the CBA between petitioner school and its union, one item was left unresolved and this was the inclusion or exclusion of the following positions in the scope of the bargaining unit:

a.

Secretaries

b.

Registrars

c.

Accounting Personnel

d.

Guidance Counselors

This matter was submitted for voluntary arbitration. The panel of voluntary arbitrators rendered a decision excluding the above-mentioned secretaries, registrars, chief of the accounting department, cashiers and guidance counselors from the coverage of the bargaining unit, while included the accounting clerks and the accounting staff member in the bargaining unit. The union moved for the reconsideration of the above decision. Pending, however, the resolution of its motion, it filed a notice of strike with the NCMB on the grounds of bargaining deadlock and unfair labor practice. During the thirty (30) day cooling-off period, two union members were dismissed by petitioner. Consequently, the union went on strike. Thereafter, the SOLE issued an order assuming jurisdiction over the labor dispute, and all workers were directed to return to work within twenty-four (24) hours upon receipt of this Order and for Management to accept them back under the same terms and conditions prevailing prior to the strike. Parties were further directed to cease and desist from committing any or all acts that might exacerbate the situation. Parties were also directed to submit their respective position papers. Anent the motion for reconsideration filed by the union, the panel of voluntary arbitrators denied the same. Petitioner school then furnished copies of the panel’s denial of the motion for reconsideration and the Decision to the individual respondents. Thereafter, the petitioner school gave the abovementioned individual respondents two choices: to resign from the union and remain employed as confidential employees or resign from their confidential positions and remain members of the union. The petitioner school relayed to these employees that they could not remain as confidential employees and at the same time as members or officers of the Union. However, the individual respondents remained steadfast in their claim that they could still retain their confidential positions while being members or officers of the Union. Hence, the petitioner school sent notices of termination to the individual respondents. The union then filed another notice of strike, this time citing as a reason the petitioner school’s termination of the individual respondents. The union alleged that the school’s act of terminating the individual respondents is in violation of the Order of the SOLE. The SOLE issued another Order reiterating the directives contained in the earlier Order. The SOLE also stated therein that the effects of the termination from employment of these individual respondents be suspended pending the determination of the legality thereof. Hence, the school was directed to reinstate the individual respondents under the same terms and conditions prevailing prior to the labor dispute. The school, thereafter, moved to reconsider the aforesaid 2 nd order of the SOLE. It argued that the SOLE’s Order directing the reinstatement of the individual respondents would render nugatory the decision of the panel of voluntary arbitrators to exclude them from the collective bargaining unit. However, the school’s motion was denied by the SOLE, wherein the latter declared that the decision of the panel of voluntary arbitrators to exclude the individual respondents from the collective bargaining unit did not authorize the school to terminate their employment. With its 2 nd MR denied, the school filed another which made the SOLE modify the two previous orders. The 3 rd order of the SOLE directed the school to place the 12 terminated employees under payroll reinstatement until the validity of their termination is finally resolved. On appeal, the CA dismissed the petition of the school for lack of merit and likewise denied its MR. Hence, this petition. The school contends that the Secretary cannot take cognizance of an issue involving employees who are not part of the bargaining unit. It insists that since the individual respondents had already been excluded from the bargaining unit by a final and executory order by the panel of voluntary arbitrators, then they cannot be covered by the Secretary’s assumption order. Issues: 1. Whether or not the SOLE can take cognizance of an issue involving employees who are not part of the bargaining unit. 2. Whether or not the order of the SOLE for payroll reinstatement is “called for the circumstance” of this case.

61

Ruling:

1. The SC held that the contention of the school is incorrect. Although the SC it recognizes the exercise of management prerogatives and it often declines to interfere with the legitimate business decisions of the employer, this privilege is not absolute, but subject to exceptions. One of these exceptions is when the Secretary of Labor assumes jurisdiction over labor disputes involving industries indispensable to the national interest under Article 263(g) of the Labor Code, which states:

(g) When, in his opinion, there exists a labor dispute causing or likely to cause a strike or lockout in an industry indispensable to the national interest, the Secretary of Labor and Employment may assume jurisdiction over the dispute and decide it or certify the same to the Commission for compulsory arbitration. Such assumption or certification shall have the effect of automatically enjoining the intended or impending strike or lockout as specified in the assumption or certification order. If one has already taken place at the time of assumption or certification, all striking or locked out employees shall immediately return to work and the employer shall immediately resume operations and readmit all workers under the same terms and conditions prevailing before the strike or lockout. x x x When the Secretary of Labor ordered the school to suspend the effect of the termination of the individual respondents, the Secretary did not exceed her jurisdiction, nor did the Secretary gravely abuse the same. It must be pointed out that one of the substantive evils which Article 263(g) of the Labor Code seeks to curb is the exacerbation of a labor dispute to the further detriment of the national interest. The University’s act of suspending and terminating union members and the Union’s act of filing another Notice of Strike after this Office has assumed jurisdiction are certainly in conflict with the status quo ante. By any standards, these acts will not in any way help in the early resolution of the labor dispute. It is clear that the actions of both parties merely served to complicate and aggravate the aready strained labormanagement relations.

Indeed, it is clear that the act of the school of dismissing the individual respondents from their employment became the impetus for the union to declare a second notice of strike. It is not a question anymore of whether or not the terminated employees, the individual respondents herein, are part of the bargaining unit. Any act committed during the pendency of the dispute that tends to give rise to further contentious issues or increase the tensions between the parties should be considered an act of exacerbation and should not be allowed. 2. The SC held that the order of the SOLE for payroll reinstatement is “called for the circumstance” of this case. Article 263(g) of the Labor Code states that all workers must immediately return to work and all employers must readmit all of them under the same terms and conditions prevailing before the strike or lockout. The phrase “under the same terms and conditions” makes it clear that the norm is actual reinstatement. This is consistent with the idea that any work stoppage or slowdown in that particular industry can be detrimental to the national interest. As an exception to the rule, payroll reinstatement must rest on special circumstances that render actual reinstatement impracticable or otherwise not conducive to attaining the purposes of the law. The “superseding circumstances” mentioned by the SOLE no doubt refer to the final decision of the panel of arbitrators as to the confidential nature of the positions of the twelve private respondents, thereby rendering their actual and physical reinstatement impracticable and more likely to exacerbate the situation. The payroll reinstatement in lieu of actual reinstatement ordered in these cases, therefore, appears justified as an exception to the rule until the validity of their termination is finally resolved. This Court sees no grave abuse of discretion on the part of the SOLE in ordering the same. The decision of the CA is affirmed.

CAPITOL MEDICAL CENTER VS. NLRC, G.R. NO. 147080, APRIL 26, 2005 Facts: After the respondent union was able to get a Supreme Court decision dismissing its rival union’s petition for certification election was dismissed and ordering petitioner company to negotiate a CBA with respondent union as the certified bargaining agent of the rank-and-file employees, said union requested for a meeting

62

with the petitioner company to discuss matters pertaining to a negotiation for a CBA, conformably with the decision of the Court. However, petitioner company rejected the proposed meeting, on the claim that it was a violation of Republic Act No. 6713 and that the Union was not a legitimate one. Petitioner company also filed a Petition for the Cancellation of the Union’s Certificate of Registration with the DOLE on the ground that the union failed for several years to submit financial statements as required by law, and the union was engaged in a strike which has been declared illegal by the NLRC. Apparently unaware of the petition, the Union reiterated its proposal for CBA negotiations but to no avail. Instead of filing a motion with the SOLE for the enforcement of the SC decision, the union filed a Notice of Strike with the NCMB on the grounds of refusal to bargain, coercion on employees and interference/restraint to self-organization. After a series of fruitless conferences before the NCMB, petitioner company filed a letter with the NCMB requesting that the Notice of Strike be dismissed because the union failed to furnish the agency with a copy of a notice of meeting where the strike vote was conducted. Thereafter, the Union submitted to the NCMB the minutes of the alleged strike vote purportedly held on November 10, 1997 at the parking lot in front of the petitioner’s premises, at the corner of Scout Magbanua Street and Panay Avenue, Quezon City. It appears that 178 out of the 300 union members participated therein, and the results were as follows: 156 members voted to strike; 14 members cast negative votes; and eight votes were spoiled. On November 28, 1997, the officers and members of the Union staged a strike. Subsequently, on December 1, 1997, the Union filed an ex parte motion with the DOLE, praying for its assumption of jurisdiction over the dispute. The Union likewise prayed for the imposition of appropriate legal sanctions, not limited to contempt and other penalties, against the hospital director/president and other responsible corporate officers for their continuous refusal, in bad faith, to bargain collectively with the Union, to adjudge the same hospital director/president and other corporate officers guilty of unfair labor practices, and for other just, equitable and expeditious reliefs in the premises. The SOLE then assumed jurisdiction over the labor dispute and directed all striking workers to return to work within 24 hours and the management to resume normal operations and accept back all striking workers under the same terms and conditions prevailing before the strike. Further, parties are directed to cease and desist from committing any act that may exacerbate the situation; and for both parties to submit their respective proposals for the CBA. In obedience to the order of the SOLE, the officers and members of the Union stopped their strike and returned to work. For its part, the petitioner filed a petition to declare the strike illegal with the NLRC where it appended affidavits of a number of persons who where in at the corner of Scout Magbanua Street and Panay Avenue, Quezon City on November 10, 1997, and attested that no secret balloting took place in the said premises. Another also attested that his signature was forged while others declared that they were not union members yet their signatures were included in the strike vote attendance. In their position paper, the respondents appended the joint affidavit of the Union president and those members who alleged that they had cast their votes during the strike vote held on November 10, 1997. In the meantime, the Regional Director of the DOLE denied the petition of petitioner for the cancellation of the respondent Union’s certificate of registration. As to the petition of petitioner to declare the union’s striker illegal, the Labor Arbiter ruled in the affirmative and held that the strike staged by respondent union was illegal and subsequently, the union officers lost their employment status with petitioner. The Labor Arbiter ruled that no voting had taken place on November 10, 1997; moreover, no notice of such voting was furnished to the NCMB at least twenty-four (24) hours prior to the intended holding of the strike vote. On appeal, the NLRC reversed the ruling of the LA and petitioner’s MR was likewise denied. Petitioner elevated its cause to the CA which in turn dismissed the petition and affirmed the ruling of the NLRC. Hence, this petition. The petitioner asserts that the NLRC and the CA erred in holding that the submission of a notice of a strike vote to the Regional Branch of the NCMB as required by Section 7, Rule XXII of the Omnibus Rules Implementing the Labor Code, is merely directory and not mandatory. The use of the word “shall” in the rules, the petitioner avers, indubitably indicates the mandatory nature of the respondent Union’s duty to submit the said notice of strike vote. Issue: Whether or not the strike staged by the union was legal. Ruling: The SC held that the strike staged by the union was illegal. The respondent Union failed to comply with the second paragraph of Section 10, Rule XXII of the Omnibus Rules of the NLRC which reads:

63

Section 10.Strike or lockout vote. – A decision to declare a strike must be approved by a majority of the total union membership in the bargaining unit concerned obtained by secret ballot in meetings or referenda called for the purpose. A decision to declare a lockout must be approved by a majority of the Board of Directors of the employer, corporation or association or the partners obtained by a secret ballot in a meeting called for the purpose. The regional branch of the Board may, at its own initiative or upon the request of any affected party, supervise the conduct of the secret balloting. In every case, the union or the employer shall furnish the regional branch of the Board and notice of meetings referred to in the preceding paragraph at least twentyfour (24) hours before such meetings as well as the results of the voting at least seven (7) days before the intended strike or lockout, subject to the cooling-off period provided in this Rule. Although the second paragraph of Section 10 of the said Rule is not provided in the Labor Code of the Philippines, nevertheless, the same was incorporated in the Omnibus Rules Implementing the Labor Code and has the force and effect of law. Aside from the mandatory notices embedded in Article 263, paragraphs (c) and (f) of the Labor Code, a union intending to stage a strike is mandated to notify the NCMB of the meeting for the conduct of strike vote, at least twenty-four (24) hours prior to such meeting. Unless the NCMB is notified of the date, place and time of the meeting of the union members for the conduct of a strike vote, the NCMB would be unable to supervise the holding of the same, if and when it decides to exercise its power of supervision. In National Federation of Labor v. NLRC, the Court enumerated the notices required by Article 263 of the Labor Code and the Implementing Rules, which include the 24-hour prior notice to the NCMB: 1) A notice of strike, with the required contents, should be filed with the DOLE, specifically the Regional Branch of the NCMB, copy furnished the employer of the union; 2) A cooling-off period must be observed between the filing of notice and the actual execution of the strike thirty (30) days in case of bargaining deadlock and fifteen (15) days in case of unfair labor practice. However, in the case of union busting where the union’s existence is threatened, the cooling-off period need not be observed. … 4) Before a strike is actually commenced, a strike vote should be taken by secret balloting, with a 24hour prior notice to NCMB. The decision to declare a strike requires the secret-ballot approval of majority of the total union membership in the bargaining unit concerned. 5) The result of the strike vote should be reported to the NCMB at least seven (7) days before the intended strike or lockout, subject to the cooling-off period. A union is mandated to notify the NCMB of an impending dispute in a particular bargaining unit via a notice of strike. Thereafter, the NCMB, through its conciliator-mediators, shall call the parties to a conference at the soonest possible time in order to actively assist them in exploring all possibilities for amicable settlement. In the event of the failure in the conciliation/mediation proceedings, the parties shall be encouraged to submit their dispute for voluntary arbitration. However, if the parties refuse, the union may hold a strike vote, and if the requisite number of votes is obtained, a strike may ensue. The purpose of the strike vote is to ensure that the decision to strike broadly rests with the majority of the union members in general and not with a mere minority, and at the same time, discourage wildcat strikes, union bossism and even corruption. A strike vote report submitted to the NCMB at least seven days prior to the intended date of strike ensures that a strike vote was, indeed, taken. In the event that the report is false, the seven-day period affords the members an opportunity to take the appropriate remedy before it is too late. The 15 to 30 day cooling-off period is designed to afford the parties the opportunity to amicably resolve the dispute with the assistance of the NCMB conciliator/mediator, while the seven-day strike ban is intended to give the DOLE an opportunity to verify whether the projected strike really carries the imprimatur of the majority of the union members.

The requirement of giving notice of the conduct of a strike vote to the NCMB at least 24 hours before the meeting for the said purpose is designed to (a) inform the NCMB of the intent of the union to conduct a strike vote; (b) give the NCMB ample time to decide on whether or not there is a need to supervise the conduct of the strike vote to prevent any acts of violence and/or irregularities attendant thereto; and (c) should the NCMB decide on its own initiative or upon the request of an interested party including the employer, to supervise the strike vote, to give it ample time to prepare for the deployment of the requisite personnel, including peace officers if need be. Unless and until the NCMB is notified at least 24 hours of the union’s decision to conduct a strike vote, and the date, place, and time thereof, the NCMB cannot determine for itself whether to supervise a strike vote meeting or not and insure its peaceful and regular conduct. The failure of a union to comply with the requirement of the giving of notice to the NCMB at least

64

24 hours prior to the holding of a strike vote meeting will render the subsequent strike staged by the union illegal.

Petition is granted.

PLDT VS. MANGGAGAW NG KOMUNIKASYON SA PILIPINAS, G.R. NO. 162783, JULY 14, 2005 Facts:

Petitioner PLDT is a domestic corporation engaged in the telecommunications business. Private respondent Manggagawa ng Komunikasyon sa Pilipinas (MKP) is a labor union of rank and file employees in PLDT. The members of respondent union learned that a redundancy program would be implemented by the petitioner. Thereupon it filed a Notice of Strike with the NCM.

A number of conciliation meetings, conducted by the NCMB, were held between the parties. However, these efforts proved futile. On 23 December 2002, the private respondent staged a strike. On 31 December 2002, three hundred eighty three (383) union members were terminated from service pursuant to PLDT’s redundancy program. On 02 January 2003, the SOLE issued an Order, a portion of the which reads: That PLDT’s operations is impressed with public and national interest as communication plays a vital role in furtherance of trade, commerce, and industry specially at this time of globalized economy where information is vital to economic survival. Work stoppage at PLDT will also adversely effect the ordinary dayto-day life of the public in areas of its franchise. Communication is also a component of state security.

Thus, the SOLE certified the labor dispute at the PLDT to the NLRC for compulsory arbitration pursuant to Art. 263(g) of the Labor Code and ordered all striking workers to return to work within twenty four (24) hours from receipt of the order, except those who were terminated due to redundancy. The employer is hereby enjoined to accept the striking workers under the same terms and conditions prevailing prior to the strike and the parties are likewise directed to cease and desist from committing any act that might worsen the situation.

As the private respondent had no other plain, speedy and adequate remedy in the ordinary course of law, it filed a petition for certiorari and mandamus under Rule 65 of the 1997 Rules on Civil Procedure before the Court of Appeals. The Court of Appeals granted the petition and the order of the Secretary was set aside. Hence this petition.

Issue:

Whether the order of the Secretary of DOLE excluding from the return-to-work order the workers dismissed due to the redundancy program of petitioner, are valid or not?

Ruling:

The SC held that the order of the SOLE is invalid.

When the Secretary exercises the powers granted by Article 263(g) of the Labor Code, he is, indeed, granted great breadth of discretion. However, the application of this power is not without limitation, lest the Secretary would be above the law. Discretion is defined as the act or the liberty to decide, according to the principles of justice and one’s ideas of what is right and proper under the circumstances, without wilfullness or favor. Where anything is left to any person to be done according to his discretion, the law intends it must be done with a sound discretion, and according to law. From the foregoing, it is quite apparent that no

65

matter how broad the exercise of discretion is, the same must be within the confines of law. Thus, the wide latitude of discretion given the Secretary under Art. 263(g) shall and must be within the sphere of law.

As Article 263(g) is clear and unequivocal in stating that ALL striking or locked out employees shall immediately return to work and the employer shall immediately resume operations and readmit ALL workers under the same terms and conditions prevailing before the strike or lockout, then the unmistakable mandate must be followed by the Secretary.

Assumption of jurisdiction over a labor dispute, or as in this case the certification of the same to the NLRC for compulsory arbitration, always co-exists with an order for workers to return to work immediately and for employers to readmit all workers under the same terms and conditions prevailing before the strike or lockout. Time and again, this Court has held that when an official bypasses the law on the asserted ground of attaining a laudable objective, the same will not be maintained if the intendment or purpose of the law would be defeated.

The Records would show that the strike occurred on 23 December 2002. Article 263(g) directs that the employer must readmit all workers under the same terms and conditions prevailing before the strike. Since the strike was held on the aforementioned date, then the condition prevailing before it, which was the condition present on 22 December 2002, must be maintained. Undoubtedly, on 22 December 2002, the members of the private respondent who were dismissed due to alleged redundancy were still employed by the petitioner and holding their respective positions. This is the status quo that must be maintained.

The decision of the CA is affirmed.

LIGHT RAILWAY TRANSIT VS. VENUS, JR., G.R. NO. 163782, MARCH 24, 2006 Facts:

Petitioner LTRA, a government corporation, constructed a light rail transit system from Monumento in Kalookan City to Baclaran in Parañaque, Metro Manila. LTRA then, contracted petitioner METRO to operate and manage the Metro manila Light Rail Transit System. METRO then hired its own employees including herein private respondents. Petitioner METRO thereafter entered into a 10-year CBA with Pinag-isang Lakas ng Manggagawa sa METRO, Inc. – National Federation of Labor, otherwise known as PIGLAS-METRO, INC. – NFL – KMU (Union), the certified exclusive collective bargaining representative of the rank-and-file employees of petitioner METRO. However, in 1989, LTRA purchased the shares of stocks in petitioner METRO. LTRA and METRO continued with their distinct and separate juridical personalities. Hence, when the CBA expired, they renewed the same, initially on a yearly basis, and subsequently on a monthly basis. In July 2000, the union filed a Notice of Strike with the NCMB against METRO on account of a deadlock in the CBA. On the same day, the Union struck. The power supply switches in the different light rail transit substations were turned off. The members of the Union picketed the various substations. They completely paralyzed the operations of the entire light rail transit system. As the strike adversely affected the mobility of the commuting public, the SOLE issued on that same day an assumption of jurisdiction order directing all the striking employees “to return to work immediately upon receipt of this Order and for the Company to accept them back under the same terms and conditions of employment prevailing prior to the strike.” The SOLE’s order of assumption of jurisdiction was personally served on the Union through its members and officers, but the latter refused to receive the same. The sheriffs thus posted the Order in the different stations/terminals of the light rail transit system. Further, the Order of assumption of jurisdiction was published in newspapers. Despite the issuance, posting, and publication of the assumption of jurisdiction and return to work order, the Union officers and members, including herein private respondent workers, failed to return to work. Thus, private respondents were considered dismissed from employment. In the meantime, the Agreement for the Management and Operation of the Metro Manila Light Rail Transit System between petitioners LRTA and METRO expired. The Board of Directors of petitioner LRTA decided not to renew the contract with petitioner METRO and directed the LRTA management instead to

66

immediately take over the management and operation of the light rail transit system to avert the mass transportation crisis. Private respondents then filed a complaint for illegal dismissal before NLRC and impleaded both petitioners LRTA and METRO. The LA ruled in favor of the private respondents and held that they were illegally dismissed. On appeal, the NLRC found that the striking workers failed to heed the return to work order and reversed and set aside the decision of the labor arbiter. The suit against LRTA was dismissed since “LRTA is a government-owned and controlled corporation created by virtue of Executive Order No. 603 with an original charter”10 and “it had no participation whatsoever with the termination of complainants’ employment. The private respondents’ MR was also denied by the NLRC. The CA, however, reversed the ruling of the NLRC and declared the workers’ dismissal as illegal, pierced the veil of separate corporate personality and held the LRTA and METRO as jointly liable for back wages. Hence, this petition. Petitioner METRO maintains that private respondent workers were not illegally dismissed but should be deemed to have abandoned their jobs after defying the assumption of jurisdiction and return-to-work order issued by the Labor Secretary. Private respondent workers, on the other hand, submit that they could not immediately return to work as the light rail transit system had ceased its operations. Issue: Whether or not private respondents were illegally dismissed. Ruling: The SC held that private respondents were not illegally dismissed. In Batangas Laguna Tayabas Bus Co. v. NLRC, The SC said that the five-day period for the strikers to obey the Order of the Secretary of Justice and return to work was not sufficient as “some of them may have left Metro Manila and did not have enough time to return during the period given by petitioner, which was only five days.” In Batangas Laguna Tayabas Bus Co., it was further held – The contention of the petitioner that the private respondents abandoned their position is also not acceptable. An employee who forthwith takes steps to protest his lay-off cannot by any logic be said to have abandoned his work. For abandonment to constitute a valid cause for termination of employment, there must be a deliberate, unjustified refusal of the employee to resume his employment. This refusal must be clearly established. As we stressed in a recent case, mere absence is not sufficient; it must be accompanied by overt acts unerringly pointing to the fact that the employee simply does not want to work anymore. In the instant case, private respondent workers could not have defied the return-to-work order of the SOLE simply because they were dismissed immediately, even before they could obey the said order. The records show that the assumption of jurisdiction and return-to-work order was issued by the SOLE on July 25, 2000. The said order was served and posted by the sheriffs of the Department of Labor and Employment the following day, on July 26, 2000. Further, the said order of assumption of jurisdiction was duly published on July 27, 2000, in the Philippine Daily Inquirer and the Philippine Star. On the same day also, on July 27, 2000, private respondent workers were dismissed. Neither could they be considered as having abandoned their work. If petitioner METRO did not dismiss the strikers right away, and instead accepted them back to work, the management agreement between petitioners LRTA and METRO could still have been extended and the workers would still have had work to return to. Thus petitioner METRO is guilty of illegal dismissal. Decision of the CA is affirmed.

UNIVERSITY OF SAN AGUSTIN EMPLOYEES UNION-FFW VS. COURT OF APPEALS, G.R. NO. 169632, MARCH 28, 2006 Facts:

On July 27, 2000, the parties entered into a 5-year CBA, which, among other things, provided that the economic provisions thereof shall have a period of three (3) years or up to 2003. Complementary to said provisions is Section 3 of Article VIII of the CBA providing for salary increases for School Years (SY) 20002003, such increase to take the form of either a lump sum or a percentage of the tuition incremental proceeds (TIP). The CBA contained a “no strike, no lockout” clause and a grievance machinery procedure to resolve management-labor disputes, including a voluntary arbitration mechanism should the grievance

67

committee fail to satisfactorily settle such disputes. Pursuant to the CBA, the parties commenced negotiations for the economic provisions for the remaining two years, i.e., SY2003-2004 and SY2004-2005. During the negotiations, the parties could not agree on the manner of computing the TIP, thus the need to undergo preventive mediation proceedings before the NCMB. Despite preventive mediation proceedings, the issue was not resolved, which prompted the union to declare a bargaining deadlock grounded on the parties’ failure to arrive at a mutually acceptable position on the manner of computing the 70% of the net TIP to be allotted for salary and other benefits for SY2003-2004 and SY2004-2005. Thereafter, the Union filed a Notice of Strike before the NCMB which was expectedly opposed by the University in a Motion to Strike Out Notice of Strike and to Refer the Dispute to Voluntary Arbitration, invoking the “No strike, no lockout” clause of the parties’ CBA. The NCMB, however, failed to resolve the University’s motion. The parties then made a joint request for the SOLE to assume jurisdiction over the dispute. The SOLE then assumed jurisdiction over the dispute and strictly enjoined and the parties are directed to cease and desist from committing any act that might exacerbate the situation. The parties were also directed to submit their respective position papers. On September 19, 2003, the Union staged a strike. At 6:45 a.m. of the same day, Sheriffs Francisco L. Reyes and Rocky M. Francisco had arrived at San Agustin University to serve the SOLE’s order on the Union. When the sheriffs tried to serve the order to the union’s vice president, the same refused, on the ground that only the union president us authorized to receive such order. The sheriffs explained to the VP that even if she refused to acknowledge receipt of the order, the same would be considered served. Sheriff Reyes further informed the Union that once the sheriffs post the order, it would be considered received by the Union. Thereafter, the sheriffs posted copies of the order at the main gate of the university, main entrance, and at the union’s office. They also served the order to the university. Notwithstanding the sheriffs’ advice as to the legal implication of the Union’s refusal to be served with the order, the Union went ahead with the strike. At around 5:25 p.m., the Union president arrived at the respondent University’s premises and received the order from the sheriffs. On September 24, 2003, the University filed a Petition to Declare Illegal Strike and Loss of Employment Status at the NLRC. The university requested that this case be consolidated with the earlier case pending before the SOLE, which the LA granted. The LA, thereafter, rendered a decision directed to conclude a memorandum of agreement embodying the foregoing dispositions to be appended to the current CBA, and dismissing the university’s petition to declare the strike illegal. The University moved for a reconsideration of the said decision but its motion was denied by the SOLE. The university then elevated the issue to the CA, which granted its appeal. While the CA affirmed the rest of the SOLE’s decision on the economic issues, particularly the formula to be used in computing the share of the employees in the tuition fee increase for Academic Year 2003-2004, it, however, reversed the SOLE’s ruling as to the legality of the September 19, 2003 strike and consequently, the union officers are deemed to have lost their employment status. While both parties sought reconsideration of the CA’s decision, the university served notices of termination to the union officers who were declared by the CA as deemed to have lost their employment status. On the same day – April 7, 2005 – in response to the University’s action, the Union filed with the NCMB a second notice of strike, this time on ground of alleged union busting. On April 22, 2005, the parties again took initial steps to negotiate the new CBA but said attempts proved futile. Hence, on April 25, 2005, the Union went on strike. In reaction, the University notified the Union that it was pulling out of the negotiations because of the strike. In the meantime, acting on the parties’ MRs, the CA found merit on the university’s MR and thereby resolved that the economic issues on the ground that said issues were proper subject of the grievance machinery as embodied in the parties’ CBA. Consequently, the CA directed the parties to refer the economic issues of the CBA to voluntary arbitration. The CA, however, stood firm in its finding that the strike conducted by the petitioner Union was illegal and its officers were deemed to have lost their employment status. Hence, this petition. Issue: Whether or not the strike staged by the union was illegal. Ruling: The SC held that the strike was illegal.

The SC found the CA’s conclusions to be well supported by evidence, particularly the Sheriff’s Report. As we see it, the SOLE was remiss in disregarding the sheriff’s report. It bears stressing that said report is an official statement by the sheriff of his acts under the writs and processes issued by the court, in this case, the SOLE, in obedience to its directive and in conformity with law. In the absence of contrary evidence, a presumption exists that a sheriff has regularly performed his official duty. To controvert the presumption arising therefrom, there must be clear and convincing evidence.

68

To be sure, the Union was not able to sufficiently dispute the truth of the narration of facts contained in the sheriff’s report. Hence, it was not unreasonable for the CA to conclude that there was a deliberate intent by the Union and its officers to disregard the AJO and proceed with their strike, which, by their act of disregarding said AJO made said strike illegal. The AJO was issued by the SOLE pursuant to Article 263(g) of the Labor Code, which reads: Art. 263. Strikes, picketing, and lockouts. - … (g) When, in his opinion, there exists a labor dispute causing or likely to cause a strike or lockout in an industry indispensable to the national interest, the Secretary of Labor and Employment may assume jurisdiction over the dispute and decide it or certify the same to the Commission for compulsory arbitration. Such assumption or certification shall have the effect of automatically enjoining the intended or impending strike or lockout as specified in the assumption or certification order. If one has already taken place at the time of assumption or certification, all striking or locked out employees shall immediately return to work and the employer shall immediately resume operations and readmit all workers under the same terms and conditions prevailing before the strike or lockout. The Secretary of Labor and Employment or the Commission may seek the assistance of law enforcement agencies to ensure compliance with this provision as well as with such orders as he may issue to enforce the same. Conclusively, when the SOLE assumes jurisdiction over a labor dispute in an industry indispensable to national interest or certifies the same to the NLRC for compulsory arbitration, such assumption or certification shall have the effect of automatically enjoining the intended or impending strike or lockout. Moreover, if one had already taken place, all striking workers shall immediately return to work and the employer shall immediately resume operations and readmit all workers under the same terms and conditions prevailing before the strike or lockout. When the Secretary exercises these powers, he is granted great breadth of discretion in order to find a solution to a labor dispute. The most obvious of these powers is the automatic enjoining of an impending strike or lockout or the lifting thereof if one has already taken place. Assumption of jurisdiction over a labor dispute, or the certification of the same to the NLRC for compulsory arbitration, always co-exists with an order for workers to return to work immediately and for employers to readmit all workers under the same terms and conditions prevailing before the strike or lockout. Thus, the SC saw no reversible error in the CA’s finding that the strike of September 19, 2003 was illegal. Consequently, the Union officers were deemed to have lost their employment status for having knowingly participated in said illegal act.

Petition is denied.

SUKHOTAI CUISINE & RESTAURANT VS. COURT OF APPEALS, G.R. NO. 150437, JULY 17, 2006 Facts:

Sometime in March 1998, the majority of the employees of the petitioner company organized themselves into a union which affiliated with the Philippine Labor Alliance Council (PLAC), and was designated as PLAC Local 460 Sukhothai Restaurant Chapter (Union).

On December 3, 1998, private respondent union filed a Notice of Strike with the NCMB on the ground of unfair labor practice, and particularly, acts of harassment, fault-finding, and union busting through coercion and interference with union affairs. On December 10, 1998, in a conciliation conference, the representatives of the petitioner agreed and guaranteed that there will be no termination of the services of private respondents during the pendency of the case, with the reservation of the management prerogative to issue memos to erring employees for the infraction, or violation of company policies. On the following day, or on December 11, 1998, a Strike Vote was conducted and supervised by NCMB personnel, and the results of the vote were submitted to the NCMB on December 21, 1998. On January 21, 1999, the petitioner and the Union entered into a Submission Agreement, thereby agreeing to submit the issue of unfair labor practice – the subject matter of the foregoing Notice of Strike and the Strike Vote – for voluntary arbitration with a view to prevent the strike. On March 24, 1999, during the pendency of the voluntary arbitration proceedings, the petitioner, through its president, Ernesto Garcia, dismissed Eugene Lucente, a union member, due to an alleged petty quarrel with a co-employee in February 1999. In view of this termination, private respondent Union filed with the NLRC a complaint for illegal dismissal. In the morning of June 24, 1999, private respondent Jose Lanorias, a union member, was

69

relieved from his post, and his employment as cook, terminated. Subsequently, respondent Billy Bacus, the union vice-president, conferred with Ernesto Garcia and protested Lanorias’s dismissal. Shortly thereafter, respondents staged a “wildcat strike.” On June 25, 1999, a Notice of Strike was re-filed by the private respondents and the protest, according to the respondents, was converted into a “sit-down strike.” On the next day, or on June 26, 1999, the same was transformed into an “actual strike.” On June 29, 1999, the petitioner filed a complaint for illegal strike with the NLRC against private respondents, seeking to declare the strike illegal, and to declare respondents, who participated in the commission of illegal acts, to have lost their employment status. Having arrived at no amicable settlement, the parties submitted their position papers, together with supporting documents, affidavits of witnesses, and photographs, in compliance with the orders of the Labor Arbiter. On October 12, 1999, the Labor Arbiter rendered a Decision declaring the strike illegal, and the employment of union officers and all individual respondents are deemed validly terminated in accordance with law. The LA held that the Union failed to comply with the mandatory requisites for a lawful strike; that the issuance of memos by the petitioner to instill discipline on erring employees is a lawful exercise of management prerogative and do not amount to acts of unfair labor practice; that, instead of resorting to a strike, private respondents should have availed of the proper legal remedies such as the filing of complaints for illegal suspension or illegal dismissal with the NLRC; that, the root causes of the controversy are the petition for certification election and petition for cancellation of union registration which were then pending before the Department of Labor as well as the issue on unfair labor practice then pending before the voluntary arbitrator, and, hence, the parties should have awaited the resolution of the cases in the proper fora; and that even if private respondents complied with all the requisites of a valid strike, the strike is still illegal due to the commission of prohibited acts, including the obstruction of free ingress and egress of the premises, intimidation, and threat inflicted upon non-striking employees. On appeal by the union, the NLRC reversed the decision of the LA and dismissed the complaint of illegal strike. In overruling the Labor Arbiter, the NLRC held that the petitioner is guilty of union busting; that the petitioner violated the Submission Agreement dated December 10, 1998 in that no termination shall be effected during the voluntary arbitration proceedings and, hence, the strike was justified; that the Notice of Strike and Strike Vote dated December 3, 1998 and December 11, 1998, respectively, are applicable to the strike of June 24, 25, and 26, 1999 since the same issues of unfair labor practice were involved and that unfair labor practices are continuing offenses; that even if the foregoing Notice of Strike and Strike Vote were not applicable, the Union may take action immediately since the petitioner is guilty of union busting; and that the re-filing of a Notice of Strike on June 25, 1999 cured the defect of non-compliance with the mandatory requirements. After its MR was denied by the NLRC, petitioner company appealed to the CA, which in turn, was denied. Hence, this petition. Respondents insist that the filing of the Notice of Strike on December 3, 1998, the Strike Vote of December 11, 1998, the submission of the results of the vote to the NCMB on December 21, 1998, and their observation of the 15-day cooling-off period in case of unfair labor practice as well as the seven-day reporting period of the results of the strike vote, all satisfy the mandatory requirements under Article 263[9] of the Labor Code and are applicable to the June 1999 strike. In support of this theory, respondents invoke Article 263(f) in that the decision to strike is valid for the duration of the dispute based on substantially the same grounds considered when the strike vote was taken, thus, there is no need to repeat the process. Furthermore, according to the respondents, even assuming for the sake of argument that the Notice of Strike and Strike Vote in December 1998 cannot be made to apply to the concerted actions in June 1999, these requirements may nonetheless be dispensed with since the petitioner is guilty of union busting and, hence, the Union can take action immediately.

Issues: Whether or not the strike staged by the private respondents is illegal. Whether or not private respondents were deemed to have lost their employment status by participating in the commission of illegal acts during the strike. Ruling 1. The SC held that the strike staged by the private respondents is illegal. The undisputed fact is that at the time the strike was staged in June 1999, voluntary arbitration between the parties was ongoing by virtue of the January 21, 1999 Submission Agreement. The issue to be resolved under those proceedings pertained to the very same issues stated in the Notice of Strike of December 3, 1998: the commission of unfair labor practices, such as acts of harassment, fault-finding, and union busting through coercion and interference with union affairs.

70

Article 264 of the Labor Code provides that “no strike or lockout shall be declared after assumption of jurisdiction by the President or the Secretary or after certification or submission of the dispute to compulsory or voluntary arbitration or during the pendency of cases involving the same grounds for the strike or lockout.”

This Court has held that strikes staged in violation of agreements providing for arbitration are illegal, since these agreements must be strictly adhered to and respected if their ends are to be achieved. The rationale of the prohibition under Article 264 is that once jurisdiction over the labor dispute has been properly acquired by competent authority, that jurisdiction should not be interfered with by the application of the coercive processes of a strike. Indeed it is among the chief policies of the State to promote and emphasize the primacy of free collective bargaining and negotiations, including voluntary arbitration, mediation, and conciliation, as modes of settling labor, or industrial disputes. The principle behind labor unionism in private industry is that industrial peace cannot be secured through compulsion by law. Relations between private employers and their employees rest on an essentially voluntary basis, subject to the minimum requirements of wage laws and other labor and welfare legislation.

The alleged dismissals of Lucente and respondent Lanorias, both union members, which allegedly triggered the wildcat strike, are not sufficient grounds to justify the radical recourse on the part of the private respondents. The questions that surround their dismissal, as private respondents so affirm, are connected to the alleged breach of the “guarantee” by the petitioner not to dismiss its employees during the pendency of the arbitration case, the very questions which they also link to the other incidents of unfair labor practices allegedly committed by the petitioner—these matters should have been raised and resolved in the voluntary arbitration proceedings that were commenced precisely to address them. On the other hand, if private respondents believed that the disciplinary measures had nothing to do with the issues under arbitration, then they should have availed of the appropriate remedies under the Labor Code, such as the institution of cases of illegal dismissal or, by agreement of the parties, the submission of the cases to the grievance machinery of the CBA, if one is available, so that they may be subjected to separate voluntary arbitration proceedings, or simply seek to terminate the pending voluntary arbitration case and complete the mandatory procedure for a lawful strike. Private respondents should have availed themselves of any of these alternative remedies instead of resorting to a drastic and unlawful measure, specifically, the holding a wildcat strike. And because of the fact that the Union was fully aware that the arbitration proceedings were pending, good faith cannot be invoked as a defense.

For failing to exhaust all steps in the arbitration proceedings by virtue of the Submission Agreement, in view of the proscription under Article 264 of the Labor Code, and the prevailing state policy as well as its underlying rationale, this Court declares that the strike staged by the private respondents is illegal.

2. The SC held that private respondents were validly terminated. In the determination of the liabilities of the individual respondents, the applicable provision is Article 264(a) of the Labor Code:

Art. 264. Prohibited Activities – (a) x x x xxxx x x x x Any union officer who knowingly participates in an illegal strike and any worker or union officer who knowingly participates in the commission of illegal acts during a strike may be declared to have lost his employment status: Provided, That mere participation of a worker in a lawful strike shall not constitute sufficient ground for termination of his employment, even if a replacement had been hired by the employer during such lawful strike.

The effects of such illegal strikes, outlined in Article 264, make a distinction between workers and union officers who participate therein: an ordinary striking worker cannot be terminated for mere participation in an illegal strike. There must be proof that he or she committed illegal acts during a strike. A union officer, on the other hand, may be terminated from work when he knowingly participates in an illegal strike, and like other workers, when he commits an illegal act during a strike. In all cases, the striker must be identified. But proof beyond reasonable doubt is not required. Substantial evidence available under the attendant circumstances, which may justify the imposition of the penalty of dismissal, may suffice. Liability

71

for prohibited acts is to be determined on an individual basis as was clearly provided by the evidence submitted by the company. Thus, the Labor Arbiter is correct in ruling that the employment of all individual private respondents are deemed validly terminated.

PHILCOM EMPLOYEES UNION VS. PHIL. GLOBAL COMMUNICATION, G.R. NO. 144315, JULY 17, 2006 Facts:

During the course of the negotiations for the renewal of its CBA, the union filed with the NCMB a Notice of Strike due to perceived unfair labor practice committed by the company. In view of the filing of the Notice of Strike, the company suspended negotiations on the CBA which moved the union to file another Notice of Strike, on the ground of bargaining deadlock. At a conciliation conference held at the NCMB, the parties agreed to consolidate the two (2) Notices of Strike filed by the union and to maintain the status quo during the pendency of the proceedings.

However, while the union and the company officers and representatives were meeting, the remaining union officers and members staged a strike at the company premises, barricading the entrances and egresses thereof and setting up a stationary picket at the main entrance of the building. The following day, the company immediately filed a petition for the Secretary of Labor and Employment to assume jurisdiction over the labor dispute in accordance with Article 263(g) of the Labor Code.

Thereafter, the SOLE issued an order assuming jurisdiction over the dispute, enjoining any strike or lockout, whether threatened or actual, directing the parties to cease and desist from committing any act that may exacerbate the situation, directing the striking workers to return to work within twenty-four (24) hours from receipt of the Secretary’s Order and for management to resume normal operations, as well as accept the workers back under the same terms and conditions prior to the strike. The parties were likewise required to submit their respective position papers and evidence within ten (10) days from receipt of said order. A second order was issued reiterating the previous directive to all striking employees to return to work immediately. The union filed a Motion for Reconsideration assailing, among others, the authority of the SOLE to assume jurisdiction over the labor dispute. Said motion was denied. As directed, the parties submitted their respective position papers. In its position paper, the union raised the issue of the alleged unfair labor practice of the company, such as contractualization, disallowance of union leave, non-implementation of employees’ benefits, etc. The company, on the other hand, raised in its position paper the sole issue of the illegality of the strike staged by the union. Based on the position papers, the SOLE held that the union’s allegation of unfair labor practices were not within the legal connotation of Article 248 of the Labor Code, but were actually mere grievances which should have been processed through the grievance machinery or voluntary arbitration outlined under the CBA. The SOLE further ruled that the strike conducted by the union was illegal. Upon appeal to the CA, the appellate court denied the union’s petition and affirmed the ruling of the SOLE. Issue:

Whether or not the SOLE can take cognizance of the issue of the legality of a strike notwithstanding the absence of any proper petition to declare the strike illegal. Ruling: The SC held that the SOLE properly took cognizance of the issue on the legality of the strike. Since the very reason of the Secretary’s assumption of jurisdiction was PEU’s declaration of the strike, any issue regarding the strike is not merely incidental to, but is essentially involved in, the labor dispute itself.

Article 263(g) of the Labor Code provides:

72

When, in his opinion, there exists a labor dispute causing or likely to cause a strike or lockout in an industry indispensable to the national interest, the Secretary of Labor and Employment may assume jurisdiction over the dispute and decide it or certify the same to the Commission for compulsory arbitration. Such assumption or certification shall have the effect of automatically enjoining the intended or impending strike or lockout as specified in the assumption or certification order. If one has already taken place at the time of assumption or certification, all striking or locked out employees shall immediately return to work and the employer shall immediately resume operations and readmit all workers under the same terms and conditions prevailing before the strike or lockout. The Secretary of Labor and Employment or the Commission may seek the assistance of law enforcement agencies to ensure the compliance with this provision as well as with such orders as he may issue to enforce the same. x x x x. The powers granted to the Secretary under Article 263(g) of the Labor Code have been characterized as an exercise of the police power of the State, with the aim of promoting public good.[16] When the Secretary exercises these powers, he is granted “great breadth of discretion” in order to find a solution to a labor dispute. The most obvious of these powers is the automatic enjoining of an impending strike or lockout or its lifting if one has already taken place.

It is of no moment that PEU never acquiesced to the submission for resolution of the issue on the legality of the strike. PEU cannot prevent resolution of the legality of the strike by merely refusing to submit the issue for resolution. It is also immaterial that this issue, as PEU asserts, was not properly submitted for resolution of the Secretary. The authority of the Secretary to assume jurisdiction over a labor dispute causing or likely to cause a strike or lockout in an industry indispensable to national interest includes and extends to all questions and controversies arising from such labor dispute. The power is plenary and discretionary in nature to enable him to effectively and efficiently dispose of the dispute.

Petition is dismissed.

FAR EASTERN UNIVERSITY-DR. NICANOR REYES MEDICAL FOUNDATION VS. FEU-NRMF EMPLOYEES ASSO., G.R. NO. 168362, OCTOBER 12, 2006 Facts:

In view of the forthcoming expiry of its CBA with the petitioner company, respondent union, on 21 March 1996, sent a letter-proposal to petitioner company stating therein their economic and non-economic proposals for the negotiation of the new CBA. On 8 May 1996, petitioner FEU-NRMF sent a letter-reply flatly rejecting respondent union’s demands and proposed to maintain the same provisions of the old CBA. Petitioner FEU-NRMF reasoned that due to financial constraints, it cannot afford to accede to a number of their demands for educational and death benefits, uniforms, longetivity pay, meal allowance and special pay, but nevertheless gave an assurance that it will seriously consider their proposal on salary increase. In an effort to arrive at a compromise, subsequent conciliation proceedings were conducted before the National Conciliation and Mediation Board - National Capital Region (NCMB-NCR) but because of the unyielding stance of both parties, the negotiation failed.

Consequently, respondent union filed a Notice of Strike before the NCMB on the ground of bargaining deadlock. A strike vote was conducted and after the 30-day cooling off period and the 7-day strike ban, the union, on 6 September 1996, staged a strike. Before the strike was conducted, respondent union offered a skeletal force of nursing and health personnel who will man the hospital’s operation for the duration of the strike. For reasons unknown to respondent union, however, petitioner FEU-NRMF failed or refused to accept the offer. For its part, petitioner FEU-NRMF, on 29 August 1996, filed a Petition for the Assumption of Jurisdiction or for Certification of Labor Dispute with the NLRC, underscoring the fact that it is a medical institution engaged in the business of providing health care for its patients.

The SOLE then granted the petition and thus issued an Order assuming jurisdiction over the labor dispute, thereby prohibiting any strike or lockout whether actual or impending, and enjoining the parties from committing any acts which may exacerbate the situation. On 6 September 1996, the NLRC process server,

73

certified that, on 5 September 1996 at around 4:00 P.M., he attempted to serve a copy of the Assumption of Jurisdiction Order to the union officers but since no one was around at the strike area, he just posted copies of the said Order at several conspicuous places within the premises of the hospital. Claiming that they had no knowledge that the Secretary of Labor already assumed jurisdiction over the pending labor dispute as they were not able to receive a copy of the Assumption of Jurisdiction Order, striking employees continued holding a strike until 12 September 1996.

On 12 September 1996, the Secretary of Labor issued another Order directing all the striking employees to return to work and the petitioner FEU-NRMF to accept them under the same terms and conditions prevailing before the strike. Accordingly, on 13 September 1996, a Return to Work Agreement was executed by the disputing parties, whereby striking employees agreed to return to their work and the petitioner FEU-NRMF undertook to accept them under status pro ante. On the same day, the striking employees returned to their respective stations.

Subsequently, petitioner FEU-NRMF filed a case before the NLRC, contending that respondent union staged the strike in defiance of the Assumption of Jurisdiction Order; hence, it was illegal. Further, the said strike was conducted in a deleterious and prejudicial manner, endangering the lives of the patients confined at the hospital. In order to redress the wrongful and illegal acts of the respondent union, petitioner FEU-NRMF prayed for the declaration that the strike is illegal and, resultantly, for the dismissal of the striking employees and decertification of the respondent union, plus damages.

In contrast, respondent union avers that petitioner FEU-NRMF refused to bargain collectively despite hefty financial gains and, thus, guilty of surface bargaining. Before staging a strike, respondent union complied with the procedural requirements by filing a notice of strike and strike vote with the NCMB-NCR. The thirtyday cooling off period and the seven-day strike ban was also fully observed. Respondent union also offered a skeletal work force but it was refused by petitioner FEU-NRMF. The strike was conducted in a peaceful and orderly manner where striking employees merely sat down outside the hospital’s premises with their placards airing their grievances. Petitioner FEU-NRMF’s allegation of sabotage, therefore, was merely concocted. Finally, respondent maintained that they did not defy any order of the Secretary of Labor because neither its officers nor its members were able to receive a copy of the same.

The Labor Arbiter rendered a decision declaring the strike illegal and dismissing the union officers for conducting the strike in defiance of the Assumption of Jurisdiction Order.

On appeal, the NLRC affirmed in toto the decision of the LA and, thus, upheld the illegality of the strike and loss of employment status of the union officers. The NLRC found that during the conciliation proceedings before the NCMB-NCR, the union officers admitted that they were aware that the Secretary of Labor issued an Assumption of Jurisdiction Order which enjoined the strike they were conducting. There was, therefore, an utter defiance of the said Order, making the strike illegal. The union officers’ dismissal is thus warranted. The union’s MR was likewise denied.

The union brought its petition to the CA, which reversed the ruling of the NLRC. The appellate court found that no personal service was validly effected by the process server that could bind the striking employees. The petitioner’s MR was also denied by the CA. Hence, this petition.

Issue:

Whether the service of the Assumption of Jurisdiction Order was validly effected by the process server so as to bind the respondent union and hold them liable for the acts committed subsequent to the issuance of the said Order.

Ruling:

74

The SC held that the service of the Assumption of Jurisdiction Order was not validly effected by the process server.

It can be inferred from the certification/proof of service that the process server resorted to posting the Order when personal service was rendered impossible since the striking employees were not present at the strike area. This mode of service, however, is not sanctioned by either the NLRC Revised Rules of Procedure or the Revised Rules of Court. The pertinent provisions of the NLRC Revised Rules of Procedure read:

Section 6.Service of Notices and Resolutions. (a) Notices or summons and copies of orders, shall be served on the parties to the case personally by the Bailiff or duly authorized public officer within three (3) days from receipt thereof or by registered mail; Provided that in special circumstances, service of summons may be effected in accordance with the pertinent provisions of the Rules of Court; Provided further, that in cases of decisions and final awards, copies thereof shall be served on both parties and their counsel or representative by registered mail; Provided further, that in cases where a party to a case or his counsel on record personally seeks service of the decision upon inquiry thereon, service to said party shall be deemed effected upon actual receipt thereof; Provided finally, that where parties are so numerous, service shall be made on counsel and upon such number of complainants, as may be practicable, which shall be considered substantial compliance with Article 224(a) of the Labor Code, as amended.

An Order issued by the SOLE assuming jurisdiction over the labor dispute is not a final judgment for it does not dispose of the labor dispute with finality. Consequently, the rule on service of summons and orders, and not the proviso on service of decisions and final awards, governs the service of the Assumption of Jurisdiction Order.

Under the NLRC Revised Rules of Procedure, service of copies of orders should be made by the process server either personally or through registered mail. However, due to the urgent nature of the Assumption of Jurisdiction Order and the public policy underlying the injunction carried by the issuance of the said Order, service of copies of the same should be made in the most expeditious and effective manner, without any delay, ensuring its immediate receipt by the intended parties as may be warranted under the circumstances. Accordingly, in this case, personal service is the proper mode of serving the Assumption of Jurisdiction Order.

It is also provided under the same rules that in special circumstances, service of summons may be effected in accordance with the pertinent provisions of the Rules of Court. Parenthetically, the manner upon which personal service may be made is prescribed by the following provisions of the Revised Rules of Court:

Rule 13. Filing and Service of Pleadings, Judgments And Other Papers.

Section 6.Personal service. – Service of the papers may be made by delivering personally a copy to the party or his counsel, or by leaving it in his office with his clerk or with a person having charge thereof. if no person is found in his office, or his office is not known, or he has no office, then by leaving a copy, between the hours of eight in the morning and six in the evening, at the party’s or counsel’s residence, if known, with a person of sufficient age and discretion then residing therein.

Let it be recalled that the process server merely posted copies of the Assumption of Jurisdiction Order in conspicuous places in the hospital. Such posting is not prescribed by the rules, nor is it even referred to when the said rules enumerated the different modes of effecting substituted service, in case personal service is impossible by the absence of the party concerned.

Clearly, personal service effectively ensures that the notice desired under the constitutional requirement of due process is accomplished. If, however, efforts to find the party concerned personally would make prompt service impossible, service may be completed by substituted service, that is, by leaving a copy,

75

between the hours of eight in the morning and six in the evening, at the party’s or counsel’s residence, if known, with a person of sufficient age and discretion then residing therein.

Substituted service derogates the regular method of personal service. It is therefore required that statutory restrictions for effecting substituted service must be strictly, faithfully and fully observed. Failure to comply with this rule renders absolutely void the substituted service along with the proceedings taken thereafter. The underlying principle of this rigid requirement is that the person, to whom the orders, notices or summons are addressed, is made to answer for the consequences of the suit even though notice of such action is made, not upon the party concerned, but upon another whom the law could only presume would notify such party of the pending proceedings.

Applying this principle in the case at bar, presumption of receipt of the copies of the Assumption of Jurisdiction Order could not be lightly inferred from the circumstances considering the adverse effect in case the parties failed to heed to the injunction directed by such Order. Worthy to note that in a number of cases, we have ruled that defiance of the assumption and return-to-work orders of the Secretary of Labor after he has assumed jurisdiction is a valid ground for the loss of employment status of any striking union officer or member.[26] Employment is a property right of which one cannot be deprived of without due process. Due process here would demand that the respondent union be properly notified of the Assumption of Jurisdiction Order of the Secretary of Labor enjoining the strike and requiring its members to return to work. Thus, there must be a clear and unmistakable proof that the requirements prescribed by the Rules in the manner of effecting personal or substituted service had been faithfully complied with. Merely posting copies of the Assumption of Jurisdiction Order does not satisfy the rigid requirement for proper service outlined by the above stated rules. Needless to say, the manner of service made by the process server was invalid and irregular. Respondent union could not therefore be adjudged to have defied the said Order since it was not properly apprised thereof. Accordingly, the strike conducted by the respondent union was valid under the circumstances.

Petition is denied.

GSIS VS. KAPISANAN NG MGA MANGGAGAWA SA GSIS, G.R. NO. 170132, DECEMBER 6, 2006 Facts:

This case has its genesis when the manager of GSIS issued a memorandum directing a number of its employees who are union embers to show cause why they should not be charged administratively for their participation in the October 4 to October 7, 2004 mass action. The union’s counsel sought reconsideration of said directive on the ground, among others, that the subject employees resumed work in obedience to the return-to-work order thus issued. The plea for reconsideration was, however, effectively denied by the filing, of administrative charges against some 110 union members for grave misconduct and conduct prejudicial to the best interest of the service. The union then filed with the CA a petition for prohibition against the GSIS on the ground that its members should not be made to explain why they supported their union’s cause since the Civil Service Resolution No. 021316, otherwise known as the Guidelines for Prohibited Mass Action, Section 10 of which exhorts government agencies to “harness all means within their capacity to accord due regard and attention to employees’ grievances and facilitate their speedy and amicable disposition through the use of grievance machinery or any other modes of settlement sanctioned by law and existing civil service rules.” It argued that the organized demonstrating employees did nothing more than air their grievances in the exercise of their “broader rights of free expression” and are, therefore, not amenable to administrative sanctions. On the other hand, petitioners assert that the filing of the formal charges are but a natural consequence of the service-disrupting rallies and demonstrations staged during office hours by the absenting GSIS employees, there being appropriate issuances outlawing such kinds of mass action.

The CA ruled in favor of the union and held that the filing of administrative charges against the union members is tantamount to grave abuse of discretion which may be the proper subject of the writ of prohibition.

76

Issue: Whether or not the mass action staged by or participated in by said GSIS employees partook of a strike or prohibited concerted mass action. Ruling: The SC held that the mass action staged by or participated in by said GSIS employees partook of a strike or prohibited concerted mass action. It may be that the freedom of expression and assembly and the right to petition the government for a redress of grievances stand on a level higher than economic and other liberties. Any suggestion, however, about these rights as including the right on the part of government personnel to strike ought to be, as it has been, trashed.

The Constitution itself qualifies its exercise with the provision “in accordance with law.” This is a clear manifestation that the state may, by law, regulate the use of this right, or even deny certain sectors such right. Executive Order 180 which provides guidelines for the exercise of the right of government workers to organize, for instance, implicitly endorsed an earlier CSC circular which “enjoins under pain of administrative sanctions, all government officers and employees from staging strikes, demonstrations, mass leaves, walkouts and other forms of mass action which will result in temporary stoppage or disruption of public service” by stating that the Civil Service law and rules governing concerted activities and strikes in government service shall be observed.

The settled rule in this jurisdiction is that employees in the public service may not engage in strikes, mass leaves, walkouts, and other forms of mass action that will lead in the temporary stoppage or disruption of public service. The right of government employees to organize is limited to the formation of unions or associations only, without including the right to strike, adding that public employees going on disruptive unauthorized absences to join concerted mass actions may be held liable for conduct prejudicial to the best interest of the service.

With the view we take of the events that transpired on October 4-7, 2004, what respondent’s members launched or participated in during that time partook of a strike or, what contextually amounts to the same thing, a prohibited concerted activity. The phrase “prohibited concerted activity” refers to any collective activity undertaken by government employees, by themselves or through their employees’ organization, with the intent of effecting work stoppage or service disruption in order to realize their demands or force concessions, economic or otherwise; it includes mass leaves, walkouts, pickets and acts of similar nature. Indeed, for four straight days, participating KMG members and other GSIS employees staged a walk out and waged or participated in a mass protest or demonstration right at the very doorstep of the GSIS main office building. The record of attendance for the period material shows that, on the first day of the protest, 851 employees, or forty eight per cent (48%) of the total number of employees in the main office (1,756) took to the streets during office hours, from 6 a.m. to 2 p.m., leaving the other employees to fend for themselves in an office where a host of transactions take place every business day. On the second day, 707 employees left their respective work stations, while 538 participated in the mass action on the third day. A smaller number, i.e., 306 employees, but by no means an insignificant few, joined the fourth day activity.

To say that there was no work disruption or that the delivery of services remained at the usual level of efficiency at the GSIS main office during those four (4) days of massive walkouts and wholesale absences would be to understate things. And to place the erring employees beyond the reach of administrative accountability would be to trivialize the civil service rules, not to mention the compelling spirit of professionalism exacted of civil servants by the Code of Conduct and Ethical Standards for Public Officials and Employees.

Ruling of the CA is reversed.

77

BIFLEX PHILS., INC., LABOR UNION VS. FILFLEX INDUSTRIAL & MFG., CORP., G.R. NO. 155679, DECEMBER 19, 2006 Facts:

Petitioners Patricia Villanueva, Emilia Bandola, Raquel Cruz, Delia Relato, Regina Castillo, Lolita delos Angeles, Marissa Villoria, Marita Antonio, Lolita Lindio, Eliza Caraulia, and Liza Sua were officers of Biflex (Phils.) Inc. Labor Union. On the other hand, petitioners Myrna dela Torre, Avelina Añonuevo, Bernice Borcelo, Narlie Yagin, Evelyn Santillan, Leony Serdoncilo, Trinidad Cuya, Andrea Lumibao, Gynie Arneo, Elizabeth Capellan, Josephine Detosil, Zenaida Francisco, and Florencia Anago were officers of Filflex Industrial and Manufacturing Labor Union. The two petitioner-unions, which are affiliated with National Federation of Labor Unions (NAFLU), are the respective collective bargaining agents of the employees of corporations.

Respondents Biflex (Phils.) Inc. and Filflex Industrial and Manufacturing Corporation (respondents) are sister companies engaged in the garment business. Situated in one big compound along with another sister company, General Garments Corporation (GGC), they have a common entrance.

On October 24, 1990, the labor sector staged a welga ng bayan to protest the accelerating prices of oil. On even date, petitioner-unions, led by their officers, herein petitioners, staged a work stoppage which lasted for several days, prompting respondents to file on October 31, 1990 a petition to declare the work stoppage illegal for failure to comply with procedural requirements. On November 13, 1990, respondents resumed their operations. Petitioners, claiming that they were illegally locked out by respondents, assert that aside from the fact that the welga ng bayan rendered it difficult to get a ride and the apprehension that violence would erupt between those participating in the welga and the authorities, respondents’ workers were prevented from reporting for work. Petitioners further assert that respondents were “slighted” by the workers’ no-show, and as a punishment, the workers as well as petitioners were barred from entering the company premises. On their putting up of tents, tables and chairs in front of the main gate of respondents’ premises, petitioners, who claim that they filed a notice of strike on October 31, 1990, explain that those were for the convenience of union members who reported every morning to check if the management would allow them to report for work. Respondents, on the other hand, maintain that the work stoppage was illegal since the following requirements for the staging of a valid strike were not complied with: (1) filing of notice of strike; (2) securing a strike vote, and (3) submission of a report of the strike vote to the Department of Labor and Employment. The Labor Arbiter decided in favor of the respondents and held that the strike was illegal. Consequently, their following officers are declared to have lost their employment status. Respondents thereupon terminated the employment of petitioners.

On appeal, the NLRC reversed the ruling of the Labor Arbiter, it holding that there was no strike to speak of as no labor or industrial dispute existed between the parties. It accordingly ordered respondents to reinstate petitioners to their former positions, without loss of seniority rights, and with full backwages from the date of their termination. On respondents’ petition for certiorari, the Court of Appeals reversed that of the NLRC and reinstated that of the Labor Arbiter.

Issue: Whether or not the strike staged by the union was illegal. Ruling:

The SC held that the strike was illegal.

78

That petitioners staged a work stoppage on October 24, 1990 in conjunction with the welga ng bayan organized by the labor sector to protest the accelerating prices of oil, it is not disputed. Stoppage of work due to welga ng bayan is in the nature of a general strike, an extended sympathy strike. It affects numerous employers including those who do not have a dispute with their employees regarding their terms and conditions of employment.

Employees who have no labor dispute with their employer but who, on a day they are scheduled to work, refuse to work and instead join a welga ng bayan commit an illegal work stoppage.

Even if petitioners’ joining the welga ng bayan were considered merely as an exercise of their freedom of expression, freedom of assembly or freedom to petition the government for redress of grievances, the exercise of such rights is not absolute. For the protection of other significant state interests such as the “right of enterprises to reasonable returns on investments, and to expansion and growth” enshrined in the 1987 Constitution must also be considered, otherwise, oppression or self-destruction of capital in order to promote the interests of labor would be sanctioned. And it would give imprimatur to workers’ joining demonstrations/rallies even before affording the employer an opportunity to make the necessary arrangements to counteract the implications of the work stoppage on the business, and ignore the novel “principle of shared responsibility between workers and employers” aimed at fostering industrial peace.

There being no showing that petitioners notified respondents of their intention, or that they were allowed by respondents, to join the welga ng bayan on October 24, 1990, their work stoppage is beyond legal protection.

Even assuming arguendo that in staging the strike, petitioners had complied with legal formalities, the strike would just the same be illegal, for by blocking the free ingress to and egress from the company premises, they violated Article 264(e) of the Labor Code which provides that “no person engaged in picketing shall … obstruct the free ingress to or egress from the employer’s premises for lawful purposes, or obstruct public thoroughfares.” In fine, the legality of a strike is determined not only by compliance with its legal formalities but also by the means by which it is carried out.

Petitioners, being union officers, should thus bear the consequences of their acts of knowingly participating in an illegal strike, conformably with the third paragraph of Article 264 (a) of the Labor Code which provides:

. . . Any union officer who knowingly participates in an illegal strike and any worker or union officer who knowingly participates in the commission of illegal acts during a strike may be declared to have lost his employment status: Provided, That mere participation of a worker in a lawful strike shall not constitute sufficient ground for termination of his employment, even if a replacement had been hired by the employer during such lawful strike.

The SC, passing on the use of the word “may” in the immediately quoted provision, held that “the law . . . grants the employer the option of declaring a union officer who participated in an illegal strike as having lost his employment.” Reinstatement of a striker or retention of his employment, despite his participation in an illegal strike, is a management prerogative which this Court may not supplant.

Petition is denied.

MANILA HOTEL EMPLOYEES ASSOCIATION VS. MANILA HOTEL CORP., G.R. NO. 154591, MARCH 5, 2007, CITING GRAND BOULEVARD HOTEL VS. DACANAY, G.R. NO. 153665, JULY 18, 2003 Facts:

79

On 11 November 1999, the MHEA filed a Notice of Strike with the NCMB Manila Hotel on the grounds of unfair labor practices. Upon the petition of Manila Hotel, the SOLE certified the labor dispute to the NLRC for compulsory arbitration pursuant to Article 263(g) of the Labor Code. Specifically, the Order enjoined any strike or lockout and the parties were ordered to cease and desist from committing any acts that may exacerbate the situation. MHEA filed a Motion for Reconsideration dated 29 November 1999 assailing the validity of said Order. During the conferences at the NLRC, the parties were advised of the certification order, which prohibited them from taking any action that would exacerbate the situation.

On 10 February 2000, the MHEA conducted a strike despite the clear terms of the Order issued by the SOLE, and despite the repeated reminders thereof. Manila Hotel then filed a complaint with prayer for injunction and or TRO, alleging that MHEA conducted an illegal strike, blocked all ingress and egress of the hotel premises, harassed and intimidated company officers, non-striking employees, customers and suppliers. In addition, it sought a declaration that the strike was illegal and that, consequently, the striking employees lost their employment.

The NLRC issued an Order directing the striking workers to return to work immediately and the hotel to accept them back under the same terms and conditions of employment. The NLRC further instructed the parties to submit proof of compliance with the instant order immediately after the lapse of twenty-four hours. The NLRC received a copy of the Compliance filed by Manila Hotel on 14 February 2000, manifesting that only six striking employees complied with the return-to-work Order and were reinstated. The other striking employees had openly defied the said Order.

In response to the NLRC’s return-to-work order, the union filed a motion alleging that the NLRC had not acquired jurisdiction over the labor dispute pending the resolution of the MR filed questioning the order of the SOLE. The union claimed that said motion had prevented the said Order of the SOLE from becoming final and executory.

The NLRC ruled that the strike held by the union was illegal for its defiance of the return-to-work order. . However, it determined that only the union officers were deemed to have lost their employment. It ruled that there was no evidence showing who among the striking employees were actually notified of the returnto-work order, and therefore, such employees have not forfeited their employment. But in view of the antagonism on both sides, the NLRC awarded a severance pay equivalent to one-month salary to the returning union members for every year of service, instead of ordering Manila Hotel to reinstate them.

Issue:

Whether or not the strike held by petitioner was illegal.

Ruling:

The SC held that it was illegal.

MHEA members seek their reinstatement after participating in an illegal strike, that is, a strike that was conducted after receiving an Order of assumptionby the SOLE certifying the dispute to the NLRC for compulsory arbitration. Worse still, the strikers failed to comply with the 11 February 2000 return-to-work Order, issued by the NLRC, despite receipt thereof. The law explicitly prohibits such acts.

ART. 263. STRIKES, PICKETING, AND LOCKOUTS xxxx (g) When, in his opinion there exists a labor dispute causing or likely to cause a strike or lockout in an industry indispensable to the national interest, the Secretary of Labor and Employment may assume jurisdiction over the dispute and decide it or certify the same to the Commission for compulsory arbitration.

80

Such assumption or certification shall have the effect of automatically enjoining the intended or impending strike or lockout as specified in the assumption or certification order. If one has already taken place at the time of the assumption or certification, all striking or locked out employees shall immediately return to work and the employer shall immediately resume operations and readmit all workers under the same terms and conditions prevailing before the strike or lockout. The Secretary of Labor and Employment or the Commission may seek the assistance of law enforcement agencies to ensure compliance with this provision as well as with such orders as he may issue to enforce the same.

ART. 264. PROHIBITED ACTIVITIES (a)

xxxx

No strike or lockout shall be declared after assumption of jurisdiction by the President or the Minister or after certification or submission of the dispute to compulsory or voluntary arbitration or during the pendency of cases involving the same grounds for the strike or lockout.

More to the point, the Court has consistently ruled in a long line of cases spanning several decades that once the SOLE assumes jurisdiction over a labor dispute, such jurisdiction should not be interfered with by the application of the coercive processes of a strike or lockout. Defiance of the assumption order or a return-to work order by a striking employee, whether a union officer or a member, is an illegal act and, therefore, a valid ground for loss of employment status.

The assumption of jurisdiction by the SOLE over labor disputes causing or likely to cause a strike or lockout in an industry indispensable to the national interest is in the nature of a police power measure. The allegation[ that the strikers relied on their honest belief that the filing of a Motion for Reconsideration of the Order, issued by the SOLE on 24 November 1999, entitled them to participate in a strike, cannot be sustained. A return-to-work order is immediately executory notwithstanding the filing of a motion for reconsideration. It must be strictly complied with even during the pendency of any petition questioning its validity. The very nature of a return-to-work order issued in a certified case lends itself to no other construction. The certification attests to the urgency of the matter, affecting as it does an industry indispensable to the national interest. The order is issued in the exercise of the court’s compulsory power of arbitration, and therefore must be obeyed until set aside. To say that its [return-to-work order] effectivity must await affirmance on a motion for reconsideration is not only to emasculate it but indeed to defeat its import, for by then the deadline fixed for the return to work would, in the ordinary course, have already passed and hence can no longer be affirmed insofar as the time element it concerned. Returning to work in this situation is not a matter of option or voluntariness but of obligation. The worker must return to his job together with his co-workers so the operations of the company can be resumed and it can continue serving the public and promoting its interest. This extraordinary authority given to the Secretary of Labor is aimed at arriving at a peaceful and speedy solution to labor disputes, without jeopardizing national interests. Regardless therefore of their motives, or the validity of their claims, the striking workers must cease and/or desist from any and all acts that tend to, or undermine this authority of the Secretary of Labor, once an assumption and/or certification order is issued. They cannot, for instance, ignore return-to-work orders, citing unfair labor practices on the part of the company, to justify their action.

Petition is denied.

FACULTY ASSOCIATION OF MAPUA INSTITUTE OF TECHNOLOGY VS. COURT OF APPEALS, G.R. NO. 164060, JUNE 15, 2007 Facts: Mapua Institute Technology (MIT) presented a new faculty ranking and compensation system with the Faculty Association of Mapua Institute of Technology (FAMIT), to which the latter agreed for adoption and implementation in the CBA on the condition that there should be no diminution in rank and pay of the faculty member. The parties then entered a new CBA which incorporated the new ranking for the college faculty.

81

When the CBA took effect, the VP for Academic Affairs issued a memo to all deans and subject chairs to evaluate the faculty using the new ranking system to evaluate and re-rank the faculty. The following factors were considered and given points; (1) educational attainment, (2) professional honors received, (3) relevant training, (4) relevant professional experience, (5) scholarly work & creative efforts, (6) award winning works, (7) officership in relevant technical & professional organizations, (8) administrative positions held. One month after, MIT called FAMIT’s attention to certain omissions in the CBA and requested for an amendment but the latter rejected the proposal. According to FAMIT, changes proposed would constitute a violation of the ratified CBA and would result in diminution of rank and benefits of the FAMIT college faculty. It further argued that the proposed amendment revised the point ranges earlier agreed upon by the parties and expands the 19 faculty ranks to 23.

Meanwhile, MIT adopted a new formula for determining the pay rates of the high school faculty: Rate/Load x Total Teaching Load = Salary where total teaching load equals number of classes multiplied by hours of service per week divided by 3 hours (as practiced, one unit subject is equal to 3 hours service). FAMIT opposed the formula and averred that MIT did not implement Section 2 of Article VI, which states:

ARTICLE VI General Wage Clause xxxx Section 2. The INSTITUTE shall pay the following rate per load for high school faculty according to corresponding faculty rank, to wit: · 25% increase in per rate/load for all high school faculty members effective November 2000; · 10% increase in per rate/load for all permanent high school faculty members effective June 2001.8 (Emphasis supplied.) MIT maintained that it was within its right to change the pay formula used. Hence, together with the issue pertaining to the ranking of the college faculty, FAMIT brought the matter to the National Conciliation and Mediation Board for mediation. Proceedings culminated in the submission of the case to the Panel of Voluntary Arbitrators for resolution. The Panel of Voluntary Arbitrators ruled in favor of the petitioner.

Issues: (1) Is MIT’s new proposal, regarding faculty ranking and evaluation, lawful and consistent with the ratified CBA? (2) Is MIT’s development of a new pay formula for the high school department, without the knowledge of FAMIT, lawful and consistent with the ratified CBA?

Ruling: NO. MIT’s new proposal, regarding faculty ranking and evaluation, is not lawful and not consistent with the CBA. Considering the submissions of the parties, in the light of the existing CBA, we find that the new point range system proposed by MIT is an unauthorized modification of Annex "C" of the 2001 CBA. It is made up of a faculty classification that is substantially different from the one originally incorporated in the current CBA between the parties. Thus, the proposed system contravenes the existing provisions of the CBA, hence, violative of the law between the parties. xxx…xxx…xxx… xxx…xxx…xxx… Until a new CBA is executed by and between the parties, they are duty-bound to keep the status quo and to continue in full force and effect the terms and conditions of the existing agreement. The law does not provide for any exception nor qualification on which economic provisions of the existing agreement are to retain its force and effect. Therefore, it must be understood as encompassing all the terms and conditions in the said agreement.13

82

The CBA during its lifetime binds all the parties. The provisions of the CBA must be respected since its terms and conditions "constitute the law between the parties." Those who are entitled to its benefits can invoke its provisions. In the event that an obligation therein imposed is not fulfilled, the aggrieved party has the right to go to court and ask redress.14 The CBA is the norm of conduct between petitioner and private respondent and compliance therewith is mandated by the express policy of the law.15

NO. MIT cannot unilaterally adopt a new formula. In our view, there is no room for unilateral change of the formula by MIT. Needless to stress, the Labor Code is specific in enunciating that in case of doubt in the interpretation of any law or provision affecting labor, such should be interpreted in favor of labor. Xxx…xxx…xxx…

PILIPINO TELEPHONE CORP., VS. PILIPINO TELEPHONE EMPLOYEES ASSO. G.R. NO.160058, JUNE 22, 2007 Facts: As the CCB between the Union and Pilipino Telephone Corp., the Union submitted to the company its proposals for renegotiation of the non-representation aspectsof their CBA. As there was a standstill on several issues, the parties submitted their dispute to the National Conciliation and Mediation Board (NCMB) for preventive mediation but it failed. Thereafter, the Union filed a notice of strike with NCMB for unfair labor practice due to the alleged acts of "restraint and coercion of union members and interference with their right to self-organization". The Sec. of Labor assumed jurisdiction over the labor dispute. Then, the Union filed a second notice of strike on the ground of (a) union busting for alleged refusal of the company to turn over union funds and (b) the mass promotion of union members during the CBA negotiation, allegedly aimed at excluding them from the bargaining unit during the CBA negotiation. On the same day, the Union went on strike. Sec. of Labor directed the union to return to work. Later on, the Comapny filed with the NLRC a petition to declare the said union strike illegal where decision was rendered declaring said strike illegal and some members to have lost their employment status and others suspended. On appeal to NLRC, the decision was affirmed. Thus, Union officers filed a petition for certiorari . CA modified the ruling of NLRC by suspending UNION OFFICERS instead of dismissing them. Both parties appealed

Issues: a) the legality of the Union's strike and b) the penalty to be imposed on the Union officers, if any.

Ruling: On Legality of Strike: SC SAID IT WAS ILLEGAL FOR FAILURE TO COMPLY WITH PROCEDURAL REQUIREMENTS Article 263 of the Labor Code, as amended by Republic Act (R.A.) No. 6715,22 and Rule XXII, Book V of the Omnibus Rules Implementing the Labor Code outline the following procedural requirements for a valid strike: 1) A notice of strike, with the required contents, should be filed with the DOLE, specifically the Regional Branch of the NCMB, copy furnished the employer of the union; 2) A cooling-off period must be observed between the filing of notice and the actual execution of the strike thirty (30) days in case of bargaining deadlock and fifteen (15) days in case of unfair labor practice. However, in the case of union busting where the union's existence is threatened, the cooling-off period need not be observed. xxx xxx xxx

83

4) Before a strike is actually commenced, a strike vote should be taken by secret balloting, with a 24-hour prior notice to NCMB. The decision to declare a strike requires the secret-ballot approval of majority of the total union membership in the bargaining unit concerned. 5) The result of the strike vote should be reported to the NCMB at least seven (7) days before the intended strike or lockout, subject to the cooling-off period.23 It is settled that these requirements are mandatory in nature and failure to comply therewith renders the strike illegal.

In the case at bar, the Union staged the strike on the same day that it filed its second notice of strike. The Union violated the seven-day strike ban. This requirement should be observed to give the Department of Labor and Employment (DOLE) an opportunity to verify whether the projected strike really carries the approval of the majority of the union members.

Moreover, we agree with the CA that there was no union busting which would warrant the non-observance of the cooling-off period. To constitute union busting under Article 263 of the Labor Code, there must be: 1) a dismissal from employment of union officers duly elected in accordance with the union constitution and by-laws; and 2) the existence of the union must be threatened by such dismissal. In the case at bar, the second notice of strike filed by the Union merely assailed the "mass promotion" of its officers and members during the CBA negotiations. Surely, promotion is different from dismissal The Union's reliance on Bacus v. Ople,28 Panay Electric Company v. NLRC29 and PNOC Dockyard and Engineering Corporation v. NLRC30 is likewise unavailing.

Nowhere in Panay Electric Company and PNOC Dockyard and Engineering Corporation did the Court rule that the procedural requirements for a valid strike may be dispensed with if the striking workers believed in good faith that the company was committing acts of unfair labor practice. In both cases, the striking union members complied with the procedural requirements for a valid strike. It is correct that this Court, in Bacus, held that "a strike staged by the workers inspired by good faith does not automatically make the same illegal," but said case was decided before the effectivity of R.A. No. 6715 on March 21, 1989. We have ruled that with the enactment of R.A. No. 6715, the requirements as to the filing of a notice of strike, strike vote, and notice given to the DOLE are mandatory in nature.

ON PENALTY OF DISMISSAL: SC SAID CA ERRED IN LOWERING THE PENALTY Article 264 of the Labor Code further provides: Art. 264. Prohibited activities.— x x x Any workers whose employment has been terminated as a consequence of an unlawful lockout shall be entitled to reinstatement with full back wages. Any union officer who knowingly participates in illegal strike and any worker or union officer who knowingly participates in the commission of illegal acts during a strike may be declared to have lost his employment status: Provided, that mere participation of a worker in a lawful strike shall not constitute sufficient ground for termination of his employment, even if a replacement had been hired by the employer during such lawful strike. x x x

We have explained the meaning of this provision as follows: The effects of illegal strikes, as outlined in Article 264 of the Labor Code, make a distinction between ordinary workers and union officers who participate therein. Under established jurisprudence, a union officer may be terminated from employment for knowingly participating in an illegal strike. The fate of union members is different. Mere participation in an illegal strike is not a sufficient ground for termination of the services of the union members. The Labor Code protects ordinary, rank-and-file union members who participated in such a strike from losing their jobs provided that they did not commit illegal acts during the strike. It cannot be overemphasized that strike, as the most preeminent economic weapon of the workers to force management to agree to an equitable sharing of the joint product of labor and capital, exert some disquieting effects not only on the relationship between labor and management, but also on the general

84

peace and progress of society and economic well-being of the State. This weapon is so critical that the law imposes the supreme penalty of dismissal on union officers who irresponsibly participate in an illegal strike and union members who commit unlawful acts during a strike. The responsibility of the union officers, as main players in an illegal strike, is greater than that of the members as the union officers have the duty to guide their members to respect the law. The policy of the state is not to tolerate actions directed at the destabilization of the social order, where the relationship between labor and management has been endangered by abuse of one party's bargaining prerogative, to the extent of disregarding not only the direct order of the government to maintain the status quo, but the welfare of the entire workforce though they may not be involved in the dispute. The grave penalty of dismissal imposed on the guilty parties is a natural consequence, considering the interest of public welfare.

LANDTEX INDUSTRIES VS. CA, G.R. NO. 150278, AUGUST 9, 2007 Facts: Landtex, a sole proprietorship owned by Alex and managed by William Go, is a business enterprise engaged in the manufacture of garments. Salvador Ayson worked in Landtex as knitting operator and was an officer of Landtex Industries Workers Union Federation of Free Workers which had an existing collective bargaining agreement (CBA) with Landtex. Ayson received from Landtex a letter which stated that he committed acts contrary to company policies. The letter required him to explain why no disciplinary actions should be taken against him for spreading damaging rumors about the personal life of an unspecified person (William Go), and for having an altercation with one of the company’s owners when he was asked to submit an ID picture. Ayson denied the allegations. Landtex decided to conduct an investigation in view of Ayson’s denials. Ayson had a meeting with the counsel of Landtex and the second meeting was rescheduled for Ayson’s failure to attend. Ayson received a notice stating his termination due to his lack of cooperation during the investigations. Despite such notice, he still reported to work. The union president requested Landtex for a formal dialogue regarding Ayson’s case. Landtex reaffirmed its decision terminating Ayson in the meetings with the union. Landetx and the union agreed to refer the matter to a third party in accordance with the provisions of law and CBA. Landtex expected that the issue be referred to the National Conciliation and Mediation Board (NCMB) for the selection of voluntary arbitrator. Ayson and the Union, however, filed a complaint before the Labor Arbiter. The labor Arbiter ruled in favor of Ayson. The labor arbiter declared that despite union manifestation of its desire to refer Ayson’s case to a third party in accordance with law and CBA, this manifestation did not affect Landtex’s termination of Ayson’s employment. His termination thus properly falls under the jurisdiction of the Labor Arbiter. The NLRC agreed with Landtex and Wiliam Go’s argument that Ayson’s case falls within the original and exclusive jurisdiction of the voluntary arbitrators, as provided in Article 261 of the Labor Code. The appellate court sustained the jurisdiction of the labor arbiter. It further stated that the records are bereft of any showing that grievance mediation had been undertaken so as to thresh out any disciplinary measure against Ayson. The appellate court found that Ayson was illegally dismissed because his termination was characterized by bad faith, and wanton and reckless exercise of management prerogative. Thus, Landtex and William Go filed a petition to the Supreme Court.

Issues: (1) Whether or not the Labor arbiter has jurisdiction over the case. (2) Whether or not the meeting between Landtex and the Union constitute grievance machinery as mandated by the CBA. (3) Whether or not Ayson was validly dismissed.

Ruling: (1) Article 261 of the Labor Code provides that voluntary arbitrators shall have original and exclusive jurisdiction to hear and decide all unresolved grievances arising from the interpretation or implementation

85

of the Collective Bargaining Agreement and those arising from the interpretation or enforcement of company personnel policies. On the other hand, a reading of Article 217 in conjunction with Article 262 shows that termination disputes fall under the jurisdiction of the labor arbiter unless the union and the company agree that termination disputes should be submitted to voluntary arbitration. Such agreement should be clear and unequivocal. Existing law is an intrinsic part of a valid contract without need for the parties to expressly refer to it. Thus, the original and exclusive jurisdiction of the labor arbiter over unfair labor practices, termination disputes, and claims for damages cannot be arrogated into the powers of voluntary arbitrators in the absence of an express agreement between the union and the company. In the present case, the CBA between Landtex and the union does not clearly state that termination disputes, as opposed to mere disciplinary actions, are covered by the CBA. The CBA defined a grievance as “one that arises from the interpretation or implementation of this Agreement, including disciplinary action imposed on any covered employee.” The CBA did not explicitly state that termination disputes should be submitted to the grievance machinery. (2) We find nothing in the records which shows that the meetings between the union and Landtex already constitute the grievance machinery as mandated by the CBA. The meetings happened only after the effectivity of Ayson’s termination. The meetings did not comply with the requisite number of participants. The CBA mandated that there should be three representatives each from the union and Landtex but there were seven union members and two Landtex representatives who attended the meetings. More importantly, there was nothing in the minutes that shows that the attendees constituted a Management-Employee Committee. Finally, the appellate court is correct in stating that if Landtex really believed that the labor arbiter did not have jurisdiction over the present case, then Landtex should have filed a motion to dismiss in accordance with Section 15, Rule V of The New Rules of Procedure of the NLRC.[26] Instead of filing a motion to dismiss,Landtex participated in the proceedings before the labor arbiter. Had Landtex immediately filed a motion to dismiss, the labor arbiter would have determined the issue outright before proceeding with hearing the case. In the present case, Landtex raised the issue of jurisdiction only after the labor arbiter required the parties to submit their position papers. (3) The requisites for a valid dismissal are (1) the dismissal must be for any of the causes expressed in Article 282 of the Labor Code, and (2) the opportunity to be heard and to defend oneself. Landtex and William Go assert that Ayson’s termination was for a just cause as defined in Article 282 of the Labor Code; hence, the two-notice rule should be followed. Procedural due process in the dismissal of employees requires notice and hearing. The employer must furnish the employee two written notices before termination may be effected. The first notice apprises the employee of the particular acts or omissions for which his dismissal is sought, while the second notice informs the employee of the employer’s decision to dismiss him.[37] In the present case, Landtex more than complied with the two-notice rule. The requirement of a hearing, on the other hand, is complied with as long as there was an opportunity to be heard, and not necessarily that an actual hearing was conducted. In the present case, Landtex scheduled three meetings before terminating Ayson. However, Landtex failed to understand the law’s purpose in requiring the opportunity to be heard. Landtex scheduled meetings with Ayson but these meetings were not free from arbitrariness. Ayson could not adequately defend himself from Landtex’s and William Go’s accusations. No witness was ever presented against Ayson, hence Ayson could not test the veracity of their claims. Unsubstantiated suspicions, accusations, and conclusions of the employer are not sufficient to justify an employee’s dismissal. The employer must prove by substantial evidence the facts and incidents upon which the accusations are made. In Philippine Associated Smelting and Refining Corporation (PASAR) v. NLRC,[40] we ruled that the mere conduct of an investigation and the statements of the company’s security guard are not enough to establish the validity of the charge of wrongdoing against the dismissed employees. It is not enough for an employer who wishes to dismiss an employee to charge him with wrongdoing. The validity of the charge must be established in a manner consistent with due process. A suspicion or belief no matter how sincerely felt cannot substitute for factual findings carefully established through an orderly procedure.

SAN MIGUEL FOODS INC., VS. SAN MIGUEL CORP EMPLOYEES UNION-PTGWO , G.R. NO. 168569, OCTOBER 5, 2007 Facts:

86

On November 9, 1992, some employees of San Miguel Foods, Inc’s Finance Department, through the Union represented by Edgar Moraleda, brought a grievance against Finance Manager Gideon Montesa (Montesa), for "discrimination, favoritism, unfair labor practices, promoting divisiveness, etc, before SMFI Plant Operations Manager George Nava in accordance with Step 1 of the grievance machinery. The Union sought the "1. review, evaluat[ion] & upgrad[ing of] all Finance staff and 2. promot[ion of] G.Q. Montesa to other SMC affiliate[s] & subsidiaries." SMFI informed the Union that it planned to address the grievance through a "work management review" to be completed by March 1993. However, it was not completed by March 1993, prompting the Union to, elevate the grievance to Step 2. Almost 9 months after the grievance meeting was held or on October 6, 1993, SMFI rendered a "Decision on Step 1 Grievance" stating that it was still in the process of completing the "work management review," hence, the Union’s requests could not be granted. The Union filed a complaint on October 20, 1993 before the NLRC, Arbitration Branch, against SMFI, its President Amadeo P. Veloso, and its Finance Manager Montesa for "unfair labor practice, [and] unjust discrimination in matters of promotion . . . " It prayed that SMFI et al. be ordered to promote the therein named employees "with the corresponding pay increases, etc.” SMFI et al. filed a motion to dismiss,8contending that the issues raised in the complaint were grievance issues and, therefore, "should be resolved in the grievance machinery. The Labor Arbiter granted SMFI et al.’s motion to dismiss and ordered the remand of the case to the grievance machinery for completion of the proceedings. On appeal, to the NLRC by "Motion for Reconsideration/Appeal", such was granted and accordingly ordered the Labor Arbiter to continue the proceedings on the Union’s complaint. Hence, SMFI filed a petition for certiorari with SC which they referred the case to the CA pursuant to St. Martin Funeral Homes v. NLRC. Court of Appeals denied SMFI et al.’s petition for certiorari, it holding that the Labor Arbiter has jurisdiction over the complaint of the Union, they having violated the seniority rule under the CBA by appointing and promoting certain employees which amounted to a ULP.

Issues: Whether or not the labor arbiter has jurisdiction over the case. Whether or not SMFI is guilty of unfair labor practice.

Ruling: Section 1 of Rule 8 of the Rules of Court should thus not be strictly applied to a case filed before a Labor Arbiter. In determining jurisdiction over a case, allegations made in the complaint, as well as those in the position paper, may thus be considered. SMFI argues that the allegations in the Union’s complaint filed before the Labor Arbiter do not establish a cause of action for ULP, the Union having merely contended that SMFI was guilty thereof without specifying the ultimate facts upon which it was based. It cites Section 1 of Rule 8 of the Rules of Court as applying suppletorily to the proceedings before the Labor Arbiter. Thus, SMFI concludes that the Labor Arbiter has no jurisdiction over its complaint. The jurisdiction of Labor Arbiters, enumerated in Article 217 of the Labor Code, includes complaints for ULP. Indeed, the particular acts of ULP alleged to have been committed by SMFI were not specified; neither were the ultimate facts in support thereof. In its Position Paper, however, the Union detailed the particular acts of ULP attributed to SMFI and the ultimate facts in support thereof. Section 7, Rule V of the New Rules of Procedure of the NLRC provides: The proceedings before the Labor Arbiter shall be non-litigious in nature. The technicalities of law and procedure and the rules obtaining in the courts of law shall not strictly apply thereto. The Labor Arbiter may avail himself of all reasonable means to ascertain the facts of the controversy speedily, including ocular inspection and examination of well-informed persons. Section 1 of Rule 8 of the Rules of Court should thus not be strictly applied to a case filed before a Labor Arbiter. In determining jurisdiction over a case, allegations made in the complaint, as well as those in the position paper, may thus be considered. SMFI guilty of ULP but only on the ground of violation of the CBA Agreement.

87

ULP on the ground of discrimination which must allege that that they were done to encourage or discourage membership in a labor organization. Based on Art. 248. Unfair labor practices of employers. – It shall be unlawful for an employer to commit any of the following unfair labor practices: (e) To discriminate in regard to wages, hours of work, and other terms and conditions of employment in order to encourage or discourage membership in any labor organization. On the questioned promotions, the Union did not allege that they were done to encourage or discourage membership in a labor organization. In fact, those promoted were members of the complaining Union. The promotions do not thus amount to ULP under Article 248(e) of the Labor Code. ULP on the ground of violation of Collective Bargaining Agreement – (1) gross violation of the CBA; AND (2) the violation pertains to the economic provisions of the CBA. As for the alleged ULP committed under Article 248(i), for violation of a CBA, this Article is qualified by Article 261 of the Labor Code, the pertinent portion of which latter Article reads: xxx violations of a Collective Bargaining Agreement, except those which are gross in character, shall no longer be treated as unfair labor practice and shall be resolved as grievances under the Collective Bargaining Agreement. For purposes of this article, gross violations of Collective Bargaining Agreement shall mean flagrant and/or malicious refusal to comply with the economic provisions of such agreement. In Silva v. NLRC, for a ULP case to be cognizable by the Labor Arbiter, and the NLRC to exercise its appellate jurisdiction, the allegations in the complaint should show prima facie the concurrence of two things, namely: (1) gross violation of the CBA; AND (2) the violation pertains to the economic provisions of the CBA. First, Thegrievance machinery provision in the CBA is not an economic provision, however, hence, the second requirement for a Labor Arbiter to exercise jurisdiction of a ULP is not present. Second, the Union alleges that violated the Job Security provision in the CBA, specifically the seniority rule, in that SMFI "appointed less senior employees to positions at its Finance Department, consequently intentionally by-passing more senior employees who are deserving of said appointment." Since the seniority rule in the promotion of employees has a bearing on salary and benefits, it may, following a liberal construction (following the rule on construction in favor of labor) of Article 261 of the Labor Code, be considered an "economic provision" of the CBA. the Union charges SMFI to have promoted less senior employees, thus bypassing others who were more senior and equally or more qualified. It may not be seriously disputed that this charge is a gross or flagrant violation of the seniority rule under the CBA, a ULP over which the Labor Arbiter has jurisdiction. The Court of Appeals having affirmed the NLRC decision finding that the Labor Arbiter has jurisdiction over the Union’s complaint and thus remanding it to the Labor Arbiter for continuation of proceedings thereon, the appellate court’s said finding may be taken to have been made only for the purpose of determining jurisdiction. WHEREFORE, the Petition is DENIED.

TOYOTA MOTOR PHILS WORKERS ASSO.VS. NLRC, G.R. NO. 158786, OCTOBER 19, 2007 Facts: Toyota Motor Phils.Workers Assoc. (Union) is a legitimate labor organization duly registered with the Department of Labor and Employment (DOLE) and is the sole and exclusive bargaining agent of all Toyota rank and file employees. The Union applied and was granted the immediate holding of a certification election ( the most democratic and expeditious method by which the laborers can freely determine the union that shall act as their representative in their dealings with the establishment where they are working. It is the appropriate means whereby controversies and disputes on representation may be laid to rest, by the unequivocal vote of the employees themselves) and was later

88

recognized by the NCMB as the sole and exclusive bargaining agent of all the Toyota rank and file employees. Toyota challenged said Order via an appeal to the DOLE Secretary. Pending Toyota’s appeal, the Union submitted its CBA proposals to Toyota which the latter denied. On February 22 & 23, 2001, Despite denial of the Union’s request, more than 200 employees staged mass actions on in front of the BLR and the DOLE offices, to protest the partisan and anti-union stance of Toyota which resulted to substantial company losses. On March 16, 2001, Toyota terminated the employment of 227 employees. In reaction to the dismissal of its union members and officers, the Union went on strike on March 17, 2001. Subsequently, from March 28, 2001 to April 12, 2001, the Union intensified its strike by barricading the gates of Toyota’s Bicutan and Sta. Rosa plants. The strikers prevented workers who reported for work from entering the plants. On April 10, 2001, the DOLE Secretary assumed jurisdiction over the labor dispute and issued an Order certifying the labor dispute to the NLRC. In said Order, the DOLE Secretary directed all striking workers to return to work at their regular shifts by April 16, 2001. On the other hand, it ordered Toyota to accept the returning employees under the same terms and conditions obtaining prior to the strike or at its option, put them under payroll reinstatement. Meanwhile, on May 23, 2001, at around 12:00 noon, despite the issuance of the DOLE Secretary’s certification Order, several payroll-reinstated members of the Union staged a protest rally in front of Toyota’s Bicutan Plant bearing placards and streamers in defiance of the April 10, 2001 Order. Then, on May 28, 2001, around forty-four (44) Union members staged another protest action in front of the Bicutan Plant. At the same time, some twenty-nine (29) payroll-reinstated employees picketed in front of the Santa Rosa Plant’s main entrance, and were later joined by other Union members. When the parties were asked to submit their position papers, only Toyota heeded the order and the NLRC declared the strikes illegal. Issues: (1) Whether the mass actions committed by the Union on different occasions are illegal strikes; (2) Extent of the liabilities of union members participating in the strike (3) Whether separation pay should be awarded to the Union members who participated in the illegal strikes. Ruling: On the 1st issue: Noted authority on labor law, Ludwig Teller, lists six (6) categories of an illegal strike, viz: (1) [when it] is contrary to a specific prohibition of law, such as strike by employees performing governmental functions; or (2) [when it] violates a specific requirement of law, [such as Article 263 of the Labor Code on the requisites of a valid strike]; or (3) [when it] is declared for an unlawful purpose, such as inducing the employer to commit an unfair labor practice against non-union employees; or (4) [when it] employs unlawful means in the pursuit of its objective, such as a widespread terrorism of non-strikers [for example, prohibited acts under Art. 264(e) of the Labor Code]; or (5) [when it] is declared in violation of an existing injunction, [such as injunction, prohibition, or order issued by the DOLE Secretary and the NLRC under Art. 263 of the Labor Code]; or (6) [when it] is contrary to an existing agreement, such as a no-strike clause or conclusive arbitration clause.

A strike means any temporary stoppage of work by the concerted action of employees as a result of an industrial or labor dispute. A labor dispute, in turn, includes any controversy or matter concerning terms or conditions of employment or the association or representation of persons in negotiating, fixing, maintaining, changing, or arranging the terms and conditions of employment, regardless of whether the disputants stand in the proximate relation of the employer and the employee. The term “strike” has been elucidated to encompass not only concerted work stoppages, but also slowdowns, mass leaves, sit-downs, attempts to damage, destroy, or sabotage plant equipment and facilities, and similar activities.

89

While we recognize the right of every person or a group to peaceably assemble and petition the government for redress of grievances, the exercise of such right is governed by existing laws, rules and regulations. There are prerequisites for a valid strike under Art. 263 of the Labor Code, and the Union failed to comply with the following requirements: (1) a notice of strike filed with the DOLE 30 days before the intended date of strike, or 15 days in case of unfair labor practice; (2) strike vote approved by a majority of the total union membership in the bargaining unit concerned obtained by secret ballot in a meeting called for that purpose; and (3) notice given to the DOLE of the results of the voting at least seven days before the intended strike.

These requirements are mandatory and the failure of a union to comply with them renders the strike illegal. The evident intention of the law in requiring the strike notice and the strike-vote report is to reasonably regulate the right to strike, which is essential to the attainment of legitimate policy objectives embodied in the law With respect to the strikes committed from March 17 to April 12, 2001, those were initially legal as the legal requirements were met. However, on March 28 to April 12, 2001, the Union barricaded the gates of the Bicutan and Sta. Rosa plants and blocked the free ingress to and egress from the company premises. This is a palpable violation of Art. 264(e), which proscribes acts of violence, coercion, or intimidation, or which obstruct the free ingress to and egress from the company premises.

While it may be conceded that there was no work disruption in the two Toyota plants, the fact still remains that the Union and its members picketed and performed concerted actions in front of the Company premises. This is a patent violation of the assumption of jurisdiction and certification Order of the DOLE Secretary, which ordered the parties “to cease and desist from committing any act that might lead to the worsening of an already deteriorated situation.” While there are no work stoppages, the pickets and concerted actions outside the plants have a demoralizing and even chilling effect on the workers inside the plants and can be considered as veiled threats of possible trouble to the workers when they go out of the company premises after work and of impending disruption of operations to company officials and even to customers in the days to come. Union officers are liable for unlawful strikes or illegal acts during a strike

Under Art. 264 (a) of the Labor Code, It is clear that the responsibility of union officials is greater than that of the members. They are tasked with the duty to lead and guide the membership in decision making on union activities in accordance with the law, government rules and regulations, and established labor practices. The leaders are expected to recommend actions that are arrived at with circumspection and contemplation, and always keep paramount the best interests of the members and union within the bounds of law. If the strike is legal at the beginning and the officials commit illegal acts during the duration of the strike, then they cannot evade personal and individual liability for said acts.

Member’s liability depends on participation in illegal acts Art. 264(a) of the Labor Code provides that a member is liable when he knowingly participates in an illegal act “during a strike.” While the provision is silent on whether the strike is legal or illegal, we find that the same is irrelevant. As long as the members commit illegal acts, in a legal or illegal strike, then they can be terminated However, when union members merely participate in an illegal strike without committing any illegal act, are they liable? This was squarely answered in Gold City Integrated Port Service, Inc. v. NLRC, where it was held that an ordinary striking worker cannot be terminated for mere participation in an illegal strike. This was an affirmation of the rulings in Bacus v. Ople and Progressive Workers Union v. Aguas, where it was held that though the strike is illegal, the ordinary member who merely participates in the strike should not be meted loss of employment on the considerations of compassion and good faith and in view of the security of tenure provisions under the Constitution. In Esso Philippines, Inc. v. Malayang

90

Manggagawa sa Esso (MME), it was explained that a member is not responsible for the union’s illegal strike even if he voted for the holding of a strike which became illegal. Noted labor law expert, Professor Cesario A. Azucena, Jr., traced the history relating to the liability of a union member in an illegal strike, starting with the “rule of vicarious liability,” thus: Under [the rule of vicarious liability], mere membership in a labor union serves as basis of liability for acts of individuals, or for a labor activity, done on behalf of the union. The union member is made liable on the theory that all the members are engaged in a general conspiracy, and the unlawful acts of the particular members are viewed as necessary incidents of the conspiracy. Thus, the rule on vicarious liability of a union member was abandoned and it is only when a striking worker “knowingly participates in the commission of illegal acts during a strike” that he will be penalized with dismissal. Now, what are considered “illegal acts” under Art.264(a)? No precise meaning was given to the phrase “illegal acts.” It may encompass a number of acts that violate existing labor or criminal laws, such as the following: (1) Violation of Art. 264(e) of the Labor Code which provides that “[n]o person engaged in picketing shall commit any act of violence, coercion or intimidation or obstruct the free ingress to or egress from the employer’s premises for lawful purposes, or obstruct public thoroughfares”; (2)

Commission of crimes and other unlawful acts in carrying out the strike; and

(3) Violation of any order, prohibition, or injunction issued by the DOLE Secretary or NLRC in connection with the assumption of jurisdiction/certification Order under Art. 263(g) of the Labor Code. As earlier explained, this enumeration is not exclusive and it may cover other breaches of existing laws. In the cases at bench, the individual respondents participated in several mass actions, viz: (1)

The rallies held at the DOLE and BLR offices on February 21, 22, and 23, 2001;

(2)

The strikes held on March 17 to April 12, 2001; and

(3)

The rallies and picketing on May 23 and 28, 2001 in front of the Toyota Bicutan and Sta. Rosa plants.

Association of Independent Unions in the Philippines v. NLRC lays down the rule on the liability of the union members: An ordinary striking employee cannot be terminated for mere participation in an illegal strike. There must be proof that he committed illegal acts during the strike and the striker who participated in the commission of illegal act[s] must be identified. But proof beyond reasonable doubt is not required. Substantial evidence available under the circumstances, which may justify the imposition of the penalty of dismissal, may suffice. In the landmark case of Ang Tibay vs. CIR, the court ruled “Not only must there be some evidence to support a finding or conclusion, but the evidence must be ‘substantial.’ Substantial evidence is more than a mere scintilla. It means such relevant evidence that a reasonable mind might accept as sufficient to support a conclusion.

2nd issue: The general rule is that when just causes for terminating the services of an employee under Art. 282 of the Labor Code exist, the employee is not entitled to separation pay. The apparent reason behind the forfeiture of the right to termination pay is that lawbreakers should not benefit from their illegal acts. The dismissed employee, however, is entitled to “whatever rights, benefits and privileges [s/he] may have under the applicable individual or collective bargaining agreement with the employer or voluntary employer policy or practice” or under the Labor Code and other existing laws. This means that the employee, despite the dismissal for a valid cause, retains the right to receive from the employer benefits provided by law, like accrued service incentive leaves. With respect to benefits granted by the CBA provisions and voluntary management policy or practice, the entitlement of the dismissed employees to the benefits depends on the stipulations of the CBA or the company rules and policies.

As in any rule, there are exceptions. One exception where separation pay is given even though an employee is validly dismissed is when the court finds justification in applying the principle of social justice well entrenched in the 1987 Constitution. The new Constitution contains a separate article devoted to the promotion of social justice and human rights with a separate sub-topic for labor. Article XIII expressly recognizes the vital role of labor, hand in hand with management, in the advancement of the national

91

economy and the welfare of the people in general. The categorical mandates in the Constitution for the improvement of the lot of the workers are more than sufficient basis to justify the award of separation pay in proper cases even if the dismissal be for cause. We hold that henceforth separation pay shall be allowed as a measure of social justice only in those instances where the employee is validly dismissed for causes other than serious misconduct or those reflecting on his moral character. Where the reason for the valid dismissal is, for example, habitual intoxication or an offense involving moral turpitude, like theft or illicit sexual relations with a fellow worker, the employer may not be required to give the dismissed employee separation pay, or financial assistance, or whatever other name it is called, on the ground of social justice. A contrary rule would, as the petitioner correctly argues, have the effect, of rewarding rather than punishing the erring employee for his offense.

There are two exceptions when the NLRC or the courts should not grant separation pay based on social justice: Serious misconduct (which is the first ground for dismissal under Art. 282); or Acts that reflect on the moral character of the employee. What is unclear is whether the ruling likewise precludes the grant of separation pay when the employee is validly terminated from work on grounds laid down in Art. 282 of the Labor Code other than serious misconduct. In all of the foregoing situations, the Court declined to grant termination pay because the causes for dismissal recognized under Art. 282 of the Labor Code were serious or grave in nature and attended by willful or wrongful intent or they reflected adversely on the moral character of the employees. We therefore find that in addition to serious misconduct, in dismissals based on other grounds under Art. 282 like willful disobedience, gross and habitual neglect of duty, fraud or willful breach of trust, and commission of a crime against the employer or his family, separation pay should not be conceded to the dismissed employee. In analogous causes for termination like inefficiency, drug use, and others, the NLRC or the courts may opt to grant separation pay anchored on social justice in consideration of the length of service of the employee, the amount involved, whether the act is the first offense, the performance of the employee and the like, using the guideposts enunciated in PLDT (see full text for case reference) on the propriety of the award of separation pay. The union officers who participated in and the union members who committed illegal acts during the illegal strike have lost their employment status. In this case, the strike was held illegal because it violated agreements providing for arbitration. Again, there was no award of separation pay nor financial assistance. Lastly, Even though strikes and lockouts have been recognized as effective bargaining tools, it is an antiquated notion that they are truly beneficial, as they only provide short-term solutions by forcing concessions from one party; but staging such strikes would damage the working relationship between employers and employees, thus endangering the business that they both want to succeed. PETITION DISMISSED. YOKOHAMA TIRE PHILS VS. YOKOHAMA EMPLOYEES UNION, G.R. NO. 159553, DECEMBER 10, 2007 Facts: On October 7, 1999, respondent Yokohama Employees Union (Union) filed a petition for certification election among the rank-and-file employees of Yokohama. Upon appeal from the Med-Arbiter’s order dismissing the petition, the Secretary of the Department of Labor and Employment (DOLE) ordered an election with (1) “Yokohama Employees’ Union” and (2) “No Union” as choices. The election held on November 23, 2001 yielded the following result: YOKOHAMA EMPLOYEES UNION – 131, NO UNION - 117, SPOILED - 2; Total votes cast = 250. Yokohama challenged 78 votes cast by dismissed employees. On the other hand, the Union challenged 68 votes cast by newly regularized rank-and-file employees and another five (5) votes by alleged supervisor-trainees. Yokohama formalized its protest and raised as an issue the eligibility to vote of the 78 dismissed employees, while the Union submitted only a handwritten manifestation during the election.

Issue:

92

Whether or not it is proper to appreciate the votes of the dismissed employees

Ruling: Yes. The votes of the dismissed employees shall be appreciated. Section 2, Rule XII (of the rules implementing Book V of the Labor Code), the rule in force during the November 23, 2001 certification election clearly, unequivocally and unambiguously allows dismissed employees to vote during the certification election if the case they filed contesting their dismissal is still pending at the time of the election.

Here, the votes of employees with illegal dismissal cases were challenged by petitioner although their cases were still pending at the time of the certification election on November 23, 2001. These cases were filed on June 27, 2001 and the appeal of the Labor Arbiter’s February 28, 2003 Decision was resolved by the NLRC only on August 29, 2003. Even the new rule has explicitly stated that without a final judgment declaring the legality of dismissal, dismissed employees are eligible or qualified voters. Thus,

RULE IX CONDUCT OF CERTIFICATION ELECTION Section 5. Qualification of voters; inclusion-exclusion. – . . . An employee who has been dismissed from work but has contested the legality of the dismissal in a forum of appropriate jurisdiction at the time of the issuance of the order for the conduct of a certification election shall be considered a qualified voter, unless his/her dismissal was declared valid in a final judgment at the time of the conduct of the certification election. xxxx Thus, we find no reversible error on the part of the DOLE Acting Secretary and the Court of Appeals in ordering the appreciation of the votes of the dismissed employees. Finally, we need not resolve the other issues for being moot. The 68 votes of the newly regularized rankand-file employees, even if counted in favor of “No Union,” will not materially alter the result. There would still be 208 votes in favor of respondent and 189 votes in favor of “No Union.” We also note that the certification election is already a fait accompli, and clearly petitioner’s rank-and-file employees had chosen respondent as their bargaining representative.

PHIL. AIRLINES INC. VS. PHIL AIRLINES EMPLOYEES ASSOCIATION, G.R. NO. 142399, MARCH 12, 2008 Facts: On 6 February 1987, petitioner PAL and respondent PALEA entered into a CBA covering the period of 19861989. Part of said agreement required petitioner PAL to pay its rank and file employees the following bonuses: Section 4. 13th Month Pay (Mid-year Bonus) A 13 th month pay, equivalent to one month’s current basic pay, consistent with the existing practice shall be paid in advance in May. Section 5. Christmas Bonus The equivalent of one month’s current basic pay as of November 30, shall be paid in December as a Christmas bonus. Payment may be staggered in two (2) stages. It is distinctly understood that nothing herein contained shall be construed to mean that the Company may not at its sole discretion give an additional amount or increase the Christmas bonus. On 22 April 1988, prior to the payment of the 13th month pay (mid-year bonus), petitioner PAL released a guideline implementing the aforequoted provision, to wit:

1)Eligibility

93

a) Ground employees in the general payroll who are regular as of April 30, 1988; b)Other ground employees in the general payroll, not falling within category a) above shall receive their 13th Month Pay on or before December 24, 1988;

2) Amount a)For category a) above, one month basic salary as of April 30, 1988; b)Employees covered under 1 b) above shall be paid not less than 1/12 of their basic salary for every month of service within the calendar year.

3)Payment Date: May 9, 1988 for category 1 a) above. Respondent PALEA assailed the implementation of the foregoing guideline on the ground that all employees of PAL, regular or non-regular, must be paid their 13th month pay. In response thereto, petitioner PAL informed respondent PALEA that rank and file employees who were regularized after 30 April 1988 were not entitled to the 13th month pay as they were already given their Christmas bonuses on 9 December 1988 per the Implementing Rules of Presidential Decree No. 851.

Disagreeing with petitioner PAL, respondent PALEA filed a labor complaint [16] for unfair labor practice against petitioner PAL before the NLRC on 1 March 1989. The complaint interposed that “the cut-off period for regularization should not be used as the parameter for granting [the] 13 th month pay considering that the law does not distinguish the status of employment but (sic) the law covers all employees. petitioner PAL countered that those rank and file employees who were not regularized by 30 April of a particular year are, in principle, not denied their 13 th month pay considering they receive said mandatory bonus in the form of the Christmas Bonus; that the Christmas Bonus given to all its employees is deemed a compliance with Presidential Decree No. 851 and the latter’s implementing rules. It argues that 1) the CBA does not apply to non-regular employees such that any benefits arising from said agreement cannot be made to apply to them, including the mid- year bonus; and 2) it has always been the company practice not to extend the mid-year bonus to those employees who have not attained regular status prior to the month of May, when payment of the particular bonus accrues.

Issue: Whether or not employees regularized after 30 April 1988 are entitled to 13 th month pay or mid-year bonus

Ruling: Be that as it may, a cursory reading of the 1986-1989 CBA of the parties herein will instantly reveal that Art. I, Sec. 3 of said agreement made its provision applicable to all employees in the bargaining unit. Section 3.Application. All the terms and conditions of employment of employees within the bargaining unit are embodied in this Agreement, xxx without distinguishing between regular and non-regular employees. As succinctly put by respondent PALEA in its Memorandum: All employees in (sic) PAL are entitled to the same benefit as they are within the same collective bargaining unit and the entitlement to such benefit spills over to even non-union members. It is a well-settled doctrine that the benefits of a CBA extend to the laborers and employees in the collective bargaining unit, including those who do not belong to the chosen bargaining labor organization. [32] Otherwise, it would be a clear case of discrimination. Hence, to be entitled to the benefits under the CBA, the employees must be members of the bargaining unit, but not necessarily of the labor organization designated as the bargaining agent. A “bargaining unit” has been defined as a group of employees of a given employer, comprised of all or less than all of the entire body of employees, which the collective interest of all the employees, consistent with equity to the employer, indicates to be the best suited to serve the reciprocal rights and duties of the parties under the collective bargaining provisions of the law. [33] At this point, the allegation of petitioner PAL that the nonregular employees do not belong to the collective bargaining unit and are thus not covered by the CBA is unjustified and unsubstantiated. It is apparent to us that petitioner PAL excludes certain employees from the benefits of the CBA only because they have not yet achieved regular status by the cut-off date, 30 April

94

1988. There is no showing that the non-regular status of the concerned employees by said cut-off date sufficiently distinguishes their interests from those of the regular employees so as to exclude them from the collective bargaining unit and the benefits of the CBA.

Having ruled that the benefits provided by the subject CBA are applicable even to non-regular employees who belong to the bargaining unit concerned, the next and crucial query to be addressed is whether the 13th month pay or mid- year bonus can be equated to the Christmas bonus. Xxx

It must be stressed that in the 1986-1989 CBA, petitioner PAL agreed to pay its employees 1) the 13 th month pay or the mid-year bonus, and 2) the Christmas bonus. The 13 th month pay, guaranteed by Presidential Decree No. 851, is explicitly covered or provided for as the mid-year bonus in the CBA, while the Christmas bonus is evidently and distinctly a separate benefit. Petitioner PAL may not be allowed to brush off said distinction, and unilaterally and arbitrarily declare that for non-regular employees, their Christmas bonus is the same as or equivalent to the 13th month pay. Xxx

The non-regular rank and file employees of petitioner PAL as of 30 April 1988, are not actually seeking more benefits than what the other member-employees of the same bargaining unit are already enjoying. They are only requesting that all members of the bargaining unit be treated equally and afforded the same privileges and benefits as agreed upon between respondent PALEA and petitioner PAL in the CBA. Petitioner PAL is committing a patent act of inequity that is grossly prejudicial to the non-regular rank and file employees there being no rational basis for withholding from the latter the benefit of a Christmas bonus besides the 13th month pay or mid-year bonus, while the same is being granted to the other rank and file employees of petitioner PAL who have been regularized as of 30 April 1988, although both types of employees are members of the same bargaining unit. As it had willfully and intentionally agreed to under the terms of the CBA, petitioner PAL must pay its regular and non-regular employees who are members of the bargaining unit represented by respondent PALEA their 13 th month pay or mid-year bonus separately from and in addition to their Christmas bonus.

A collective bargaining agreement refers to a negotiated contract between a legitimate labor organization and the employer concerning wages, hours of work and all other terms and conditions of employment in a bargaining unit.[40] As in all other contracts, the parties to a CBA may establish such stipulations, clauses, terms and conditions as they may deem convenient, provided these are not contrary to law, morals, good customs, public order or public policy. [41] Thus, where the CBA is clear and unambiguous, it becomes the law between the parties, and compliance therewith is mandated by the express policy of the law.

STEEL CORP. VS. SCP EMPLOYEES UNION-NFL G.R. NO. 169829-30, APRIL 16, 2008 Facts: Petitioner Steel Corporation of the Philippines (SCP) is engaged in manufacturing construction materials, supplying approximately 50% of the domestic needs for roofing materials. SCP-Federated Union of the Energy Leaders – General and Allied Services (FUEL-GAS) filed a petition for Certification Election in its bid to represent the rank-and-file employees of the petitioner. Respondent SCP Employees Union (SCPEU) – National Federation of Labor Unions (NAFLU) intervened, seeking to participate and be voted for in such election but the same was denied for having been filed out of time. Later on, a consent election was conducted, with “FUEL-GAS” and “NO UNION” as choices. Said election was however declared a failure because less than a majority of the rank-and-file employees cast their votes. Later on, NAFLU, the mother federation of respondent SCPEU, filed a petition for Certification Election for and on behalf of its affiliate, seeking to represent the rank-and-file employees of petitioner. The Med-Arbiter granted the petition for certification election filed by NAFLU and further ordered the conduct of the election with “NAFLU” and “NO UNION” as choices. Both petitioner and FUEL-GAS appealed to the Secretary of Labor, which appeals were later consolidated. DOLE Undersecretary rendered a consolidated decision ordering the conduct of a certification election with “SCPEU,” respondent and “NO UNION” as choices. Subsequent motions for reconsideration were denied. Unsatisfied, petitioner and FUEL-GAS appealed to the CA. However, the certification election, as ordered by

95

the Med-Arbiter, proceeded. FUEL-GAS participated without prejudice to the decision of the CA in its pending petition. In said election, respondent emerged as winner; hence, an election protest filed by FUELGAS. Meanwhile, the Undersecretary rendered a Decision certifying respondent as the exclusive bargaining agent of petitioner’s employees. Petitioner and FUEL-GAS timely filed motions for reconsideration of the aforesaid decision.

As a consequence of its certification as the exclusive bargaining agent, respondent sent to petitioner CBA proposals. Petitioner, however, held in abeyance any action on the proposals in view of its pending motion for reconsideration. Finding no justification in petitioner’s refusal to bargain with it, respondent filed a Notice of Strike with the NCMB.

Later, respondent filed another Notice of Strike alleging as grounds, petitioner’s refusal to bargain and union busting. The notice was later dismissed and respondent was enjoined from holding a strike. Later, respondent filed another Notice of Strike on the grounds of refusal to bargain and union busting. Respondent thereafter went on strike. However, the Labor Secretary certified the dispute to the NLRC and directed the employees to return to work.

Issue: Whether or not the strike staged by respondent union is illegal

Ruling: Whether or not respondent is the recognized collective bargaining agent had been finally resolved in the negative. Consequently, as correctly concluded by the CA, it could not compel petitioner to bargain with it. Thus, the only issues left for determination are: the validity of the strike participated in by the officers of the respondent union; and the validity of their termination from employment by reason of such participation. The strike is a legitimate weapon in the human struggle for a decent existence. It is considered as the most effective weapon in protecting the rights of the employees to improve the terms and conditions of their employment. But to be valid, a strike must be pursued within legal bounds. The right to strike as a means for the attainment of social justice is never meant to oppress or destroy the employer. The law provides limits for its exercise. In the instant case, the strike undertaken by the officers of respondent union is patently illegal for the following reasons: (1) it is a union-recognition-strike which is not sanctioned by labor laws; (2) it was undertaken after the dispute had been certified for compulsory arbitration; and (3) it was in violation of the Secretary's return-to-work order.

Respondent's notices of strike were founded on petitioner's continued refusal to bargain with it. It thus staged the strike to compel petitioner to recognize it as the collective bargaining agent, making it a unionrecognition-strike. As its legal designation implies, this kind of strike is calculated to compel the employer to recognize one's union and not other contending groups, as the employees' bargaining representative to work out a collective bargaining agreement despite the striking union's doubtful majority status to merit voluntary recognition and lack of formal certification as the exclusive representative in the bargaining unit.

The certification election that was conducted where respondent emerged as winner, not having been recognized as valid, it has no authority to represent the rank and file employees of petitioner. Thus, it could not ask petitioner to bargain with it. As the issue of its identity had been the subject of a separate case which had been settled by the court with finality,61 petitioner cannot, therefore, be faulted in refusing to bargain. Neither could this Court sustain respondent's imputation of unfair labor practice and union busting against petitioner. With more reason, this Court cannot sustain the validity of the strike staged on such basis. Even if this Court were to uphold the validity of respondent's purpose or objective in staging a strike, still, the strike would be declared illegal for having been conducted in utter defiance of the Secretary's return-towork order and after the dispute had been certified for compulsory arbitration. Although ostensibly there

96

were several notices of strike successively filed by respondent, these notices were founded on substantially the same grounds – petitioner's continued refusal to recognize it as the collective bargaining representative.

Article 263(g) of the Labor Code provides: When, in his opinion, there exists a labor dispute causing or likely to cause a strike or lockout in an industry indispensable to the national interest, the Secretary of Labor and Employment may assume jurisdiction over the dispute and decide it or certify the same to the Commission for compulsory arbitration. Such assumption or certification shall have the effect of automatically enjoining the intended or impending strike or lockout as specified in the assumption or certification order. If one has already taken place at the time of assumption or certification, all striking or locked out employees shall immediately return to work and the employer shall immediately resume operations and readmit all workers under the same terms and conditions prevailing before the strike or lockout. The Secretary of Labor and Employment or the Commission may seek the assistance of law enforcement agencies to ensure the compliance with this provision as well as with such orders as he may issue to enforce the same. x x x.

The powers granted to the Secretary under Article 263(g) of the Labor Code have been characterized as an exercise of the police power of the State, aimed at promoting the public good. When the Secretary exercises these powers, he is granted "great breadth of discretion" to find a solution to a labor dispute. The most obvious of these powers is the automatic enjoining of an impending strike or lockout or its lifting if one has already taken place.

The moment the Secretary of Labor assumes jurisdiction over a labor dispute in an industry indispensable to national interest, such assumption shall have the effect of automatically enjoining the intended or impending strike. It was not even necessary for the Secretary of Labor to issue another order directing a return to work. The mere issuance of an assumption order by the Secretary of Labor automatically carries with it a return-to-work order, even if the directive to return to work is not expressly stated in the assumption order. A return-to-work order imposes a duty that must be discharged more than it confers a right that may be waived. While the workers may choose not to obey, they do so at the risk of severing their relationship with their employer.

Says the Labor Code: Art. 264. Prohibited activities. – xxx No strike or lockout shall be declared after assumption of jurisdiction by the President or the Secretary or after certification or submission of the dispute to compulsory or voluntary arbitration or during the pendency of cases involving the same grounds for the strike or lockout. Returning to work in this situation is not a matter of option or voluntariness but of obligation. The worker must return to his job together with his co-workers so that the operations of the company can be resumed and it can continue serving the public and promoting its interest. This extraordinary authority given to the Secretary of Labor is aimed at arriving at a peaceful and speedy solution to labor disputes, without jeopardizing national interests. Regardless of their motives, or the validity of their claims, the striking workers must cease and/or desist from any and all acts that undermine or tend to undermine this authority of the Secretary of Labor, once an assumption and/or certification order is issued. They cannot, for instance, ignore return-to-work orders, citing unfair labor practices on the part of the company, to justify their action. Respondent, in the instant case, after the assumption of jurisdiction and certification of the dispute to the NLRC for compulsory arbitration, filed notices of strike and staged the strike obviously contrary to the provisions of labor laws. Worse, it filed not one but several notices of strike which resulted in two certified cases which were earlier consolidated. These disputes could have been averted had respondent respected the CA's decision. That way, the collective bargaining agent would have been determined and petitioner could have been compelled to bargain. Respondent, through its officers, instead opted to use the weapon of strike to force petitioner to recognize it as the bargaining agent. The strike, having been staged after the dispute had been certified for arbitration and contrary to the return-to-work order, became a prohibited activity, and was thus illegal.

97

Strikes exert disquieting effects not only on the relationship between labor and management, but also on the general peace and progress of society, not to mention the economic well-being of the State. It is a weapon that can either breathe life to or destroy the union and members in their struggle with management for a more equitable due of their labors. Hence, the decision to wield the weapon of strike must therefore rest on a rational basis, free from emotionalism, unswayed by the tempers and tantrums of a few, and firmly focused on the legitimate interest of the union which should not however be antithetical to the public welfare. In every strike staged by a union, the general peace and progress of society and public welfare are involved. Having settled that the subject strike was illegal, this Court shall now determine the proper penalty to be imposed on the union officers who knowingly participated in the strike.

Article 264 of the Labor Code further provides: Art. 264. Prohibited activities.— x x x Any workers whose employment has been terminated as a consequence of an unlawful lockout shall be entitled to reinstatement with full back wages. Any union officer who knowingly participates in an illegal strike and any worker or union officer who knowingly participates in the commission of illegal acts during a strike may be declared to have lost his employment status: Provided, that mere participation of a worker in a lawful strike shall not constitute sufficient ground for termination of his employment, even if a replacement had been hired by the employer during such lawful strike. x x x.

It bears stressing that the law makes a distinction between union members and union officers. A worker merely participating in an illegal strike may not be terminated from employment. It is only when he commits illegal acts during a strike that he may be declared to have lost employment status. For knowingly participating in an illegal strike or participating in the commission of illegal acts during a strike, the law provides that a union officer may be terminated from employment. The law grants the employer the option of declaring a union officer who participated in an illegal strike as having lost his employment. It possesses the right and prerogative to terminate the union officers from service. Otherwise, the workers will simply refuse to return to their work and cause a standstill in the company operations while retaining the positions they refuse to discharge and preventing management from filling up their positions. WHEREFORE, the petition is partly GRANTED.

STANDARD CHARTERED BANK EMPLOYEES UNION VS. STANDARD CHARTERED BANK ET AL., G.R. NO. 161933, APRIL 22, 2008 Facts: Petitioner and the Standard Chartered Bank (Bank) began negotiating for a new Collective Bargaining Agreement (CBA) in May 2000 as their 1998-2000 CBA already expired. Due to a deadlock in the negotiations, petitioner filed a Notice of Strike prompting the Secretary of Labor and Employment to assume jurisdiction over the labor dispute. On May 31, 2001, Secretary Patricia A. Sto. Tomas of the Department of Labor and Employment (DOLE) issued an Order for the parties to execute their CBA.

Issue: Whether the Bank's Chief Cashiers and Assistant Cashiers, personnel of the Telex Department and HR staff are confidential employees, such that they shall be excluded in the bargaining unit

Ruling: The CBA provisions in dispute are the exclusion of certain employees from the appropriate bargaining unit and the adjustment of remuneration for employees serving in an acting capacity for one month. In their proposal, petitioner sought the exclusion of only the following employees from the appropriate bargaining unit – all managers who are vested with the right to hire and fire employees, confidential

98

employees, those with access to labor relations materials, Chief Cashiers, Assistant Cashiers, personnel of the Telex Department and one Human Resources (HR) staff.

The Secretary's disposition of the issues raised by petitioner were affirmed by the CA. The Court sustains the CA. Whether or not the employees sought to be excluded from the appropriate bargaining unit are confidential employees is a question of fact, which is not a proper issue in a petition for review under Rule 45 of the Rules of Court. This holds more true in the present case in which petitioner failed to controvert with evidence the findings of the Secretary and the CA.

The disqualification of managerial and confidential employees from joining a bargaining unit for rank and file employees is already well-entrenched in jurisprudence. While Article 245 of the Labor Code limits the ineligibility to join, form and assist any labor organization to managerial employees, jurisprudence has extended this prohibition to confidential employees or those who by reason of their positions or nature of work are required to assist or act in a fiduciary manner to managerial employees and hence, are likewise privy to sensitive and highly confidential records.

In this case, the question that needs to be answered is whether the Bank's Chief Cashiers and Assistant Cashiers, personnel of the Telex Department and HR staff are confidential employees, such that they should be excluded. As regards the qualification of bank cashiers as confidential employees, National Association of Trade Unions (NATU) – Republic Planters Bank Supervisors Chapter v. Torres declared that they are confidential employees having control, custody and/or access to confidential matters, e.g., the branch's cash position, statements of financial condition, vault combination, cash codes for telegraphic transfers, demand drafts and other negotiable instruments, pursuant to Sec. 1166.4 of the Central Bank Manual regarding joint custody, and therefore, disqualified from joining or assisting a union; or joining, assisting or forming any other labor organization. Golden Farms, Inc. v. Ferrer-Calleja meanwhile stated that “confidential employees such as accounting personnel, radio and telegraph operators who, having access to confidential information, may become the source of undue advantage. Said employee(s) may act as spy or spies of either party to a collective bargaining agreement.”

Finally, in Philips Industrial Development, Inc. v. National Labor Relations Commission, the Court designated personnel staff, in which human resources staff may be qualified, as confidential employees because by the very nature of their functions, they assist and act in a confidential capacity to, or have access to confidential matters of, persons who exercise managerial functions in the field of labor relations.

Petitioner insists that the foregoing employees are not confidential employees; however, it failed to buttress its claim. Aside from its generalized arguments and despite the Secretary's finding that there was no evidence to support it, petitioner still failed to substantiate its claim. Petitioner did not even bother to state the nature of the duties and functions of these employees, depriving the Court of any basis on which it may be concluded that they are indeed confidential employees. As aptly stated by the CA: While We agree that petitioner's proposed revision is in accordance with the law, this does not necessarily mean that the list of exclusions enumerated in the 1998-2000 CBA is contrary to law. As found by public respondent, petitioner failed to show that the employees sought to be removed from the list of exclusions are actually rank and file employees who are not managerial or confidential in status and should, accordingly, be included in the appropriate bargaining unit.

Absent any proof that Chief Cashiers and Assistant Cashiers, personnel of the Telex department and one (1) HR Staff have mutuality of interest with the other rank and file employees, then they are rightfully excluded from the appropriate bargaining unit. x x x21(Emphasis supplied) Petitioner cannot simply rely on jurisprudence without explaining how and why it should apply to this case. Allegations must be supported by evidence. In this case, there is barely any at all. WHEREFORE, the petition is DENIED.

99

SAMAHAN NG MGA MANGGAGAWA SA SAMMA-LAKAS SA INDUSTRIYA NG KAPATIRANG HALIGI NG ALYANSA (SAMMA-LIKHA) VS. SAMMA CORP., G.R. NO. 167141, MAR. 13, 2009 Facts: Petitioner Samahan ng mga Manggagawa sa Samma– Lakas sa Industriya ng Kapatirang Haligi ng Alyansa (SAMMA-LIKHA) filed a petition for certification election on July 24, 2001 in the Department of Labor and Employment (DOLE), Regional Office IV. It claimed that: (1) it was a local chapter of the LIKHA Federation, a legitimate labor organization registered with the DOLE; (2) it sought to represent all the rankand-file employees of respondent Samma Corporation; (3) there was no other legitimate labor organization representing these rank-and-file employees; (4) respondent was not a party to any collective bargaining agreement and (5) no certification or consent election had been conducted within the employer unit for the last 12 months prior to the filing of the petition.

Respondent moved for the dismissal of the petition arguing that (1) LIKHA Federation failed to establish its legal personality; (2) petitioner failed to prove its existence as a local chapter; (3) it failed to attach the certificate of non-forum shopping and (4) it had a prohibited mixture of supervisory and rankand-file employees. In an order dated November 12, 2002, med-arbiter Arturo V. Cosuco ordered the dismissal of the petition on the following grounds: (1) lack of legal personality for failure to attach the certificate of registration purporting to show its legal personality; (2) prohibited mixture of rank-and-file and supervisory employees and (3) failure to submit a certificate of non-forum shopping.

Issues: 1. Whether a certificate for non-forum shopping is required in a petition for certification election; 2. Whether petitioner had the legal personality to file the petition for certification election.

Ruling: No requirement of certificate of non-forum shopping. The requirement for a certificate of non-forum shopping refers to complaints, counter-claims, cross-claims, petitions or applications where contending parties litigate their respective positions regarding the claim for relief of the complainant, claimant, petitioner or applicant. A certification proceeding, even though initiated by a "petition," is not a litigation but an investigation of a non-adversarial and fact-finding character.

… [The] rationale for the requirement of a certification against forum shopping is to apprise the Court of the pendency of another action or claim involving the same issues in another court, tribunal or quasi-judicial agency, and thereby precisely avoid the forum shopping situation. Filing multiple petitions or complaints constitutes abuse of court processes, which tends to degrade the administration of justice, wreaks havoc upon orderly judicial procedure, and adds to the congestion of the heavily burdened dockets of the courts. Furthermore, the rule proscribing forum shopping seeks to promote candor and transparency among lawyers and their clients in the pursuit of their cases before the courts to promote the orderly administration of justice, prevent undue inconvenience upon the other party, and save the precious time of the courts. It also aims to prevent the embarrassing situation of two or more courts or agencies rendering conflicting resolutions or decisions upon the same issue.

The same situation holds true for a petition for certification election. Under the omnibus rules implementing the Labor Code as amended by D.O. No. 9, it is supposed to be filed in the Regional Office which has jurisdiction over the principal office of the employer or where the bargaining unit is principally situated. The rules further provide that where two or more petitions involving the same bargaining unit are filed in one Regional Office, the same shall be automatically consolidated. Hence, the filing of multiple suits and the possibility of conflicting decisions will rarely happen in this proceeding and, if it does, will be easy to discover.

100

Notably, under the Labor Code and the rules pertaining to the form of the petition for certification election, there is no requirement for a certificate of non-forum shopping either in D.O. No. 9, series of 1997 or in D.O. No. 40-03, series of 2003 which replaced the former.

Considering the nature of a petition for certification election and the rules governing it, we therefore hold that the requirement for a certificate of non-forum shopping is inapplicable to such a petition. Legal Personality of Petitioner

Petitioner argues that the erroneous inclusion of one supervisory employee in the union of rank-and-file employees was not a ground to impugn its legitimacy as a legitimate labor organization which had the right to file a petition for certification election.

We agree.

LIKHA was granted legal personality as a federation under certificate of registration no. 92-1015-03211638-FED-LC. Subsequently, petitioner as its local chapter was issued its charter certificate no. 2-01.With certificates of registration issued in their favor, they are clothed with legal personality as legitimate labor organizations: Section 5.Effect of registration. – The labor organization or workers’ association shall be deemed registered and vested with legal personality on the date of issuance of its certificate of registration. Such legal personality cannot thereafter be subject to collateral attack, but may be questioned only in an independent petition for cancellation in accordance with these Rules.

Section 3.Acquisition of legal personality by local chapter. - A local/chapter constituted in accordance with Section 1 of this Rule shall acquire legal personality from the date of filing of the complete documents enumerated therein. Upon compliance with all the documentary requirements, the Regional Office or Bureau of Labor Relations shall issue in favor of the local/chapter a certificate indicating that it is included in the roster of legitimate labor organizations.

Such legal personality cannot thereafter be subject to collateral attack, but may be questioned only in an independent petition for cancellation of certificate of registration. Unless petitioner’s union registration is cancelled in independent proceedings, it shall continue to have all the rights of a legitimate labor organization, including the right to petition for certification election.

Furthermore, the grounds for dismissal of a petition for certification election based on the lack of legal personality of a labor organization are the following: (a) petitioner is not listed by the Regional Office or the Bureau of Labor Relations in its registry of legitimate labor organizations or (b) its legal personality has been revoked or cancelled with finality in accordance with the rules.

As mentioned, respondent filed a petition for cancellation of the registration of petitioner on December 14, 2002. In a resolution dated April 14, 2003, petitioner’s charter certificate was revoked by the DOLE. But on May 6, 2003, petitioner moved for the reconsideration of this resolution. Neither of the parties alleged that this resolution revoking petitioner’s charter certificate had attained finality. However, in this petition, petitioner prayed that its charter certificate be "reinstated in the roster of active legitimate labor [organizations]." This cannot be granted here. To repeat, the proceedings on a petition for cancellation of registration are independent of those of a petition for certification election. This case originated from the latter. If it is shown that petitioner’s legal personality had already been revoked or cancelled with finality in accordance with the rules, then it is no longer a legitimate labor organization with the right to petition for a certification election.

A Final Note

101

Respondent, as employer, had been the one opposing the holding of a certification election among its rankand-file employees. This should not be the case. We have already declared that, in certification elections, the employer is a bystander; it has no right or material interest to assail the certification election. WHEREFORE, the petition is hereby GRANTED.

HOTEL ENTERPRISES OF THE PHILS., (HYATT REGENCY) VS. SAMAHAN NG MGA MANGGAGAWA SA HYATT-(NUWHRAIN) G.R. NO. 165756, JUNE 5, 2009

Facts: Samahan ng mga Manggagawa sa Hyatt-National Union of Workers in the Hotel Restaurant and Allied Industries (Union) is the certified collective bargaining agent of the rank-and-file employees of Hyatt Regency Manila, a hotel owned by petitioner Hotel Enterprises of the Philippines, Inc. (HEPI). In 2001, HEPI’s hotel business suffered a slump due to the local and international economic slowdown. After implementing several schemes to recover their losses, HEPI decided to implement a downsizing scheme by removing positions identified as redundant. The effect was to be a reduction of the hotel’s rankand file employees from the agreed number of 248, from their CBA, down to just 150. The Union opposed the downsizing plan because they did not believe the financial report stating that the hotel was incurring heavy financial losses, and for being violative of the CBA. Despite its opposition, a list of the positions declared redundant and to be contracted out was given by the management to the Union. Notices of termination were, likewise, sent to 48 employees whose positions were to be retrenched or declared as redundant. The Union filed a notice of strike based on unfair labor practice (ULP) against HEPI. A petition was likewise filed by HEPI to declare the strike illegal. The NLRC decided in favor of HEPI. They gave credence to the financial report that the hotel had incurred huge financial losses necessitating the adoption of a downsizing scheme. Thus, NLRC declared the strike illegal, suspended all Union officers for a period of six (6) months without pay, and dismissed the ULP charge against HEPI.

Issue: Whether or not the Union staged a valid strike.

Ruling: The Constitution affords full protection to labor, but the policy is not to be blindly followed at the expense of capital. Always, the interests of both sides must be balanced in light of the evidence adduced and the peculiar circumstances surrounding each case.

The requisites for a valid strike are: (a) a notice of strike filed with the DOLE 30 days before the intended date thereof or 15 days in case of ULP; (b) a strike vote approved by a majority of the total union membership in the bargaining unit concerned obtained by secret ballot in a meeting called for that purpose; and (c) a notice to the DOLE of the results of the voting at least seven (7) days before the intended strike. The requirements are mandatory and failure of a union to comply therewith renders the strike illegal.

In this case, respondent fully satisfied the procedural requirements prescribed by law: a strike notice filed on April 12, 2002; a strike vote reached on April 25, 2002; notification of the strike vote filed also on April 25, 2002; conciliation proceedings conducted on May 8, 20002; and the actual strike on May 10, 2002.

Substantively, however, there appears to be a problem. A valid and legal strike must be based on "strikeable" grounds, because if it is based on a "non-strikeable" ground, it is generally deemed an illegal strike. Corollarily, a strike grounded on ULP is illegal if no acts constituting ULP actually exist. As an exception, even if no such acts are committed by the employer, if the employees believe in good faith that ULP actually exists, then the strike held pursuant to such belief may be legal. As a general rule, therefore,

102

where a union believes that an employer committed ULP and the surrounding circumstances warranted such belief in good faith, the resulting strike may be considered legal although, subsequently, such allegations of unfair labor practices were found to be groundless.

Here, respondent Union went on strike in the honest belief that petitioner was committing ULP after the latter decided to downsize its workforce contrary to the staffing/manning standards adopted by both parties under a CBA forged only four (4) short months earlier. The belief was bolstered when the management hired 100 contractual workers to replace the 48 terminated regular rank-and-file employees who were all Union members. Indeed, those circumstances showed prima facie that the hotel committed ULP. Thus, even if technically there was no legal ground to stage a strike based on ULP (because the downzising/retrenchment scheme by the employer was declared valid), since the attendant circumstances support the belief in good faith that petitioner’s retrenchment scheme was structured to weaken the bargaining power of the Union, the strike, by exception, may be considered legal.

Because of this, we view the NLRC’s decision to suspend all the Union officers for six (6) months without pay to be too harsh a punishment. A suspension of two (2) months without pay should have been more reasonable and just. Be it noted that the striking workers are not entitled to receive strike-duration pay, the ULP allegation against the employer being unfounded.

TEODORICO S. MIRANDA, JR. VS. ASIAN TERMINALS, INC. (ATI) AND COURT OF APPEALS GR NO. 174316 JUNE 23, 2009

Facts:

Petitioner Teodorico S. Miranda, Jr. was employed by respondent ATI in 1991 as Checker I. He also became a member of the Associated Port Checkers and Workers Union (APCWU or the union). On April 10, 1992, the petitioner, who was then the Vice President of the union, was appointed to the position of Shop Steward which is a union position under the payroll of the company. The Collective Bargaining Agreement (CBA) between the union and ATI provided for the appointment of a Shop Steward from among the union members, upon the recommendation of the union president. The Shop Steward is a field representative of both the company and the union and acts as an independent arbiter of all complaints brought to his attention.[

On December 28, 1993, Roger P. Silva, the President of APCWU, wrote a letter to the petitioner regarding the recall of his designation as the union Shop Steward. The union president explained that the petitioner was recalled as union Shop Steward due to loss of trust and confidence in him, pursuant to the “Agreement Amending the MPSI (Marina Port Services, Inc.) - APCWU CBA.” The letter further stated that the petitioner refused to heed the union president’s reminders concerning his “chronic absenteeism” that “is hurting the interest of the Union members as they are left with no responsible union officer when summoned for investigation concerning alleged infractions of company rules.”

Upon the conclusion of the investigation, the grievance committee issued its report recommending to ATI the recall of the petitioner as Shop Steward and for his reversion to his former position of Checker I, in accordance with the CBA. The petitioner questioned his recall as union Shop Steward, and the union president, Roger P. Silva, issued a letter which reasoned that the petitioner’s recall as Shop Steward was pursuant to Section 13 of the Agreement Amending the MPSI-APCWU CBA, amending Section 2, Article V of the MPSI-APCWU CBA which required that the term of office of the Shop Steward shall be based on trust and confidence and favorable recommendation of the duly elected president of the Union.

The petitioner argues that he is entitled to claim reinstatement as Shop Steward as well as the payment of his backwages pending the respondent’s appeal. He further contends that the Court of Appeals erred in dismissing his consolidated petitions which prayed for the enforcement of his reinstatement as Shop Steward for being moot and academic.

103

The respondent, on the other hand, maintains that both the NLRC and the Court of Appeals relied on substantial evidence in arriving at their decision that the consolidated petitions are already moot and academic in view of the previous reinstatement of the petitioner to Checker I and his retrenchment and separation from ATI since October 31, 2001.

Issues:

(1) Whether or not the petitioner should be reinstated to the position of Shop Steward (2) Whether or not the Labor Arbiter has jurisdiction over the dispute

Ruling:

A cursory look at the responsibilities of a shop steward leads to the conclusion that it is a position within the union, and not within the company. A shop steward is appointed by the union in a shop, department, or plant and serves as representative of the union, charged with negotiating and adjustment of grievances of employees with the supervisor of the employer. He is the representative of the union members in a building or other workplace. Black's Law Dictionary defines a shop steward as a union official elected to represent members in a plant or particular department. His duties include collection of dues, recruitment of new members and initial negotiations for the settlement of grievances. The shop steward is responsible for receiving complaints and grievances of the employees and for bringing these complaints to the immediate supervisor of the employee concerned. If the grievance is not settled through the efforts of the shop steward, it is referred to the grievance committee.

It is quite clear that the jurisdiction of shop stewards and the supervisors includes the determination of the issues arising from the interpretation or even implementation of a provision of the CBA, or from any order or memorandum, circular or assignments issued by the appropriate authority in the establishment. In fine, they are part and parcel of the continuous process of grievance resolution designed to preserve and maintain peace among the employees and their employer. They occupy positions of trust and laden with awesome responsibilities. Since the Shop Steward is a union position, the controversy surrounding his recall from his position as Shop Steward becomes a dispute within the union.

An "Internal Union Dispute" or intra-union conflict refers to a conflict within or inside a labor union. It includes all disputes or grievances arising from any violation of or disagreement over any provision of the constitution and by-laws of a union, including any violation of the rights and conditions of union membership provided for in the Code. [ Article 226 of the Labor Code of the Philippines vests on the Bureau of Labor Relations and the Labor Relations Division jurisdiction to act on all inter-union or intra-union conflicts.

The actions of the petitioner bolster the conclusion that his grievances were directed against the union and not the respondent company, making the dispute an intra-union dispute. The first Complaints filed by the petitioner were against the union and the Union President for illegal recall of his designation as Shop Steward. A Complaint was then filed before the DOLE Med-Arbiter praying for reinstatement to union Shop Steward and for the award of the salary differential while he was allegedly illegally demoted. But the money claims could not be brought before the union since the salaries of the petitioner were paid by the respondent company; thus, a Complaint for illegal demotion amounting to constructive dismissal was filed before the Labor Arbiter, against the union, union president and this time including respondent company and the president of the company.

Notwithstanding the determination of the Med-Arbiter, as affirmed by the Secretary of Labor, that the petitioner should be reinstated to the position of Shop Steward, which is binding on this Court, the petitioner could not be reinstated to the position of Shop Steward because his eventual separation from respondent ATI made reinstatement unfeasible. Employment with respondent ATI and membership in the union are required in order to occupy the position of Shop Steward. But the petitioner is neither a member of the union nor employed with respondent ATI. He was already retrenched from respondent ATI since October 21, 2001, and his retrenchment was finally settled through the execution of a Quit Claim and

104

Release which was executed before the Second Division of the NLRC in NLRC CA No. 032809-02. The Quit Claim and Release provides that in consideration of the receipt ofP367,500.00, the petitioner discharges respondent ATI and its officers from any claims arising from his retrenchment, without prejudice to the present labor case filed by the petitioner.

It may seem that the outcome of this case provides no relief for the petitioner despite his invalid removal from the position of union Shop Steward, but the reinstatement of the petitioner could not be forced into the present circumstances because the petitioner is no longer employed by the respondent company. It is a fact that we cannot avoid and must consider in resolving this case. He was already compensated for his retrenchment from ATI, and he released respondent ATI from any and all claims or liability with respect to his separation from employment due to retrenchment. To order the respondent company to reinstate the petitioner to his employment in ATI would render the Quit Claim and Release nugatory.

The events which have taken place during the pendency of the case have rendered the present petition moot and academic.

NATIONAL UNION OF WORKERS IN HOTELS RESTAURANTS AND ALLIED INDUSTRIES-MANILA PAVILION HOTEL CHAPTER VS. SOLE, ET AL. G.R. NO. 181531, JULY 31, 2009 Facts: A certification election was conducted on June 16, 2006 among the rank-and-file employees of respondent Holiday Inn Manila Pavilion Hotel (the Hotel) with the following results:

In view of the significant number of segregated votes, contending unions, petitioner, NUHWHRAIN-MPHC, and respondent Holiday Inn Manila Pavillion Hotel Labor Union (HIMPHLU), referred the case back to Med-Arbiter Ma. Simonette Calabocal to decide which among those votes would be opened TOTAL VOTES CAST = 346 and tallied. Eleven (11) votes were initially segregated because they were cast by dismissed employees, albeit the legality of their dismissal was still pending before the Court of Appeals. Six other votes were segregated because the employees who cast NUWHRAIN-MPHC = 151 them were already occupying supervisory positions at the time of the election. Still five other votes were segregated on the ground that they were cast by probationary employees and, pursuant to the existing Collective Bargaining Agreement (CBA), such employees cannot vote. It bears noting early on, however, that the vote of one Jose Gatbonton (Gatbonton), a probationary employee, was counted.

EMPLOYEES IN VOTERS’ LIST =

353

Petitioner, which garnered 151 votes, appealed to the Secretary of Labor and Employment (SOLE), arguing that the votes of the probationary employees should have been opened considering that probationary employee Gatbonton’s vote was tallied. And petitioner averred that respondent HIMPHLU, which garnered 169 votes, should not be immediately certified as the bargaining agent, as the opening of the 17 segregated ballots would push the number of valid votes cast to 338 (151 + 169 + 1 + 17), hence, the 169 votes which HIMPHLU garnered would be one vote short of the majority which would then become 169. By the assailed Resolution of January 22, 2007, the Secretary of Labor and Employment (SOLE), through then Acting Secretary Luzviminda Padilla, affirmed the Med-Arbiter’s Order. The Order granting the petition for the conduct of the certification election, the six probationary employees were not yet hired, hence, they could not vote.

In fine, the SOLE concluded that the certification of HIMPHLU as the exclusive bargaining agent was proper.

Issues:

105

The relevant issues for resolution then are first, whether employees on probationary status at the time of the certification elections should be allowed to vote, and second, whether HIMPHLU was able to obtain the required majority for it to be certified as the exclusive bargaining agent.

Ruling: On the first issue, the Court rules in the affirmative. The inclusion of Gatbonton’s vote was proper not because it was not questioned but because probationary employees have the right to vote in a certification election. The votes of the six other probationary employees should thus also have been counted. As Airtime Specialists, Inc. v. Ferrer-Calleja holds: In a certification election, all rank and file employees in the appropriate bargaining unit, whether probationary or permanent are entitled to vote. This principle is clearly stated in Art. 255 of the Labor Code which states that the "labor organization designated or selected by the majority of the employees in an appropriate bargaining unit shall be the exclusive representative of the employees in such unit for purposes of collective bargaining." Collective bargaining covers all aspects of the employment relation and the resultant CBA negotiated by the certified union binds all employees in the bargaining unit. Hence, all rank and file employees, probationary or permanent, have a substantial interest in the selection of the bargaining representative. The Code makes no distinction as to their employment status as basis for eligibility in supporting the petition for certification election. The law refers to "all" the employees in the bargaining unit. All they need to be eligible to support the petition is to belong to the "bargaining unit." (Emphasis supplied)

Rule II, Sec. 2 of Department Order No. 40-03, series of 2003, which amended Rule XI of the Omnibus Rules Implementing the Labor Code, provides: Rule II Section 2. Who may join labor unions and workers' associations. - All persons employed in commercial, industrial and agricultural enterprises, including employees of government owned or controlled corporations without original charters established under the Corporation Code, as well as employees of religious, charitable, medical or educational institutions whether operating for profit or not, shall have the right to self-organization and to form, join or assist labor unions for purposes of collective bargaining: provided, however, that supervisory employees shall not be eligible for membership in a labor union of the rank-and-file employees but may form, join or assist separate labor unions of their own. Managerial employees shall not be eligible to form, join or assist any labor unions for purposes of collective bargaining. Alien employees with valid working permits issued by the Department may exercise the right to selforganization and join or assist labor unions for purposes of collective bargaining if they are nationals of a country which grants the same or similar rights to Filipino workers, as certified by the Department of Foreign Affairs.

For purposes of this section, any employee, whether employed for a definite period or not, shall beginning on the first day of his/her service, be eligible for membership in any labor organization. All other workers, including ambulant, intermittent and other workers, the self-employed, rural workers and those without any definite employers may form labor organizations for their mutual aid and protection and other legitimate purposes except collective bargaining.

The provision in the CBA disqualifying probationary employees from voting cannot override the Constitutionally-protected right of workers to self-organization, as well as the provisions of the Labor Code and its Implementing Rules on certification elections and jurisprudence thereon.

Prescinding from the principle that all employees are, from the first day of their employment, eligible for membership in a labor organization, it is evident that the period of reckoning in determining who shall be included in the list of eligible voters is, in cases where a timely appeal has been filed from the Order of the MedArbiter, the date when the Order of the Secretary of Labor and Employment, whether affirming or denying the appeal, becomes final and executory.

106

The filing of an appeal to the SOLE from the Med-Arbiter’s Order stays its execution, in accordance with Sec. 21, and rationally, the Med-Arbiter cannot direct the employer to furnish him/her with the list of eligible voters pending the resolution of the appeal.

During the pendency of the appeal, the employer may hire additional employees. To exclude the employees hired after the issuance of the Med-Arbiter’s Order but before the appeal has been resolved would violate the guarantee that every employee has the right to be part of a labor organization from the first day of their service. In the present case, records show that the probationary employees, including Gatbonton, were included in the list of employees in the bargaining unit submitted by the Hotel on May 25, 2006 in compliance with the directive of the Med-Arbiter after the appeal and subsequent motion for reconsideration have been denied by the SOLE, rendering the Med-Arbiter’s August 22, 2005 Order final and executory 10 days after the March 22, 2007 Resolution (denying the motion for reconsideration of the January 22 Order denying the appeal), and rightly so. Because, for purposes of self-organization, those employees are, in light of the discussion above, deemed eligible to vote. A certification election is the process of determining the sole and exclusive bargaining agent of the employees in an appropriate bargaining unit for purposes of collective bargaining. Collective bargaining, refers to the negotiated contract between a legitimate labor organization and the employer concerning wages, hours of work and all other terms and conditions of employment in a bargaining unit.

The significance of an employee’s right to vote in a certification election cannot thus be overemphasized for he has considerable interest in the determination of who shall represent him in negotiating the terms and conditions of his employment.

Even if the Implementing Rules gives the SOLE 20 days to decide the appeal from the Order of the MedArbiter, experience shows that it sometimes takes months to be resolved. To rule then that only those employees hired as of the date of the issuance of the Med-Arbiter’s Order are qualified to vote would effectively disenfranchise employees hired during the pendency of the appeal. More importantly, reckoning the date of the issuance of the Med-Arbiter’s Order as the cut-off date would render inutile the remedy of appeal to the SOLE. But while the Court rules that the votes of all the probationary employees should be included, under the particular circumstances of this case and the period of time which it took for the appeal to be decided, the votes of the six supervisory employees must be excluded because at the time the certification elections was conducted, they had ceased to be part of the rank and file, their promotion having taken effect two months before the election.

2nd Issue: As to whether HIMPHLU should be certified as the exclusive bargaining agent, the Court rules in the negative. It is well-settled that under the so-called "double majority rule," for there to be a valid certification election, majority of the bargaining unit must have voted AND the winning union must have garnered majority of the valid votes cast.

Prescinding from the Court’s ruling that all the probationary employees’ votes should be deemed valid votes while that of the supervisory employees should be excluded, it follows that the number of valid votes cast would increase – from 321 to 337. Under Art. 256 of the Labor Code, the union obtaining the majority of the valid votes cast by the eligible voters shall be certified as the sole and exclusive bargaining agent of all the workers in the appropriate bargaining unit. This majority is 50% + 1. Hence, 50% of 337 is 168.5 + 1 or at least 170. HIMPHLU obtained 169 while petitioner received 151 votes. Clearly, HIMPHLU was not able to obtain a majority vote. The position of both the SOLE and the appellate court that the opening of the 17 segregated ballots will not materially affect the outcome of the certification election as for, so they contend, even if such member were all in favor of petitioner, still, HIMPHLU would win, is thus untenable.

107

It bears reiteration that the true importance of ascertaining the number of valid votes cast is for it to serve as basis for computing the required majority, and not just to determine which union won the elections. The opening of the segregated but valid votes has thus become material. To be sure, the conduct of a certification election has a two-fold objective: to determine the appropriate bargaining unit and to ascertain the majority representation of the bargaining representative, if the employees desire to be represented at all by anyone. It is not simply the determination of who between two or more contending unions won, but whether it effectively ascertains the will of the members of the bargaining unit as to whether they want to be represented and which union they want to represent them. Having declared that no choice in the certification election conducted obtained the required majority, it follows that a run-off election must be held to determine which between HIMPHLU and petitioner should represent the rank-and-file employees.

A run-off election refers to an election between the labor unions receiving the two (2) highest number of votes in a certification or consent election with three (3) or more choices, where such a certified or consent election results in none of the three (3) or more choices receiving the majority of the valid votes cast; provided that the total number of votes for all contending unions is at least fifty percent (50%) of the number of votes cast.8 With 346 votes cast, 337 of which are now deemed valid and HIMPHLU having only garnered 169 and petitioner having obtained 151 and the choice "NO UNION" receiving 1 vote, then the holding of a run-off election between HIMPHLU and petitioner is in order.

A. SORIANO AVIATION VS. EMPLOYEES ASSOCIATION OF A. SORIANO AVIATION ET AL. G.R. NO. 166879, AUG. 14, 2009

Facts: A. Soriano Aviation (petitioner or the company) and respondent Employees Association of A. Soriano Aviation (the Union), the duly-certified exclusive bargaining agent of the rank and file employees of petitioner, entered into a Collective Bargaining Agreement (CBA) which included a “No-Strike, No-Lock-out” clause. On May 1 & 12, and June 12, 1997, which were legal holidays and peak season for the company, eight mechanics-members of respondent Union refused to render overtime work. Petitioner treated the refusal to work as a concerted action which is a violation of the “No-Strike, No-Lockout” clause in the CBA. It thus meted the workers a 30-day suspension. It also filed a complaint for illegal strike.

The attempted settlement between the parties having been futile, the Union filed a Notice of Strike with the National Conciliation and Mediation Board on October 3, 1997, attributing to petitioner the following acts: (1) union busting, (2) illegal dismissal of union officer, (3) illegal suspension of eight mechanics, (4) violation of memorandum of agreement, (5) coercion of employees and interrogation of newly-hired mechanics with regard to union affiliation, (6) discrimination against the aircraft mechanics, (7) harassment through systematic fault-finding, (8) contractual labor, and (9) constructive dismissal of the Union President, Julius Vargas. As despite conciliation no amicable settlement of the dispute was arrived at, the Union went on strike on October 22, 1997.

The Labor Arbiter declared that the newly implemented work-shift schedule was a valid exercise of management prerogative and the refusal of herein individual respondents to work on three consecutive holidays was a form of protest by the Union, hence, deemed a concerted action. Noting that the Union failed to comply with the formal requirements prescribed by the Labor Code in the holding of strike, the strike was declared illegal. The Labor Arbiter declared the second strike illegal. The Labor Arbiter went on to hold that the Union deliberately resorted to the use of violent and unlawful acts in the course of the “second strike,” hence, the individual respondents were deemed to have lost their employment.

NLRC affirmed in toto the Labor Arbiter’s decision.

108

The Court of Appeals reversed and set aside the NLRC ruling, holding that the acts of violence committed by the Union members in the course of the strike were not serious or pervasive to call for loss of employment of the striking employees. Specifically, the appellate court noted that at the time petitioner filed its complaint in June 1998, almost eight months had already elapsed from the commencement of the strike and, in the interim, the alleged acts of violence were committed only during nine non-consecutive days.

Issue: In issue then is whether the strike staged by respondents is illegal due to the alleged commission of illegal acts and violation of the “No Strike-No Lockout” clause of the CBA and, if in the affirmative, whether individual respondents are deemed to have lost their employment status on account thereof.

Ruling: The Court rules in the affirmative. The Court notes that, as found by the Labor Arbiter in NLRC Case No. 07-05409-97, the first strike or the mechanics’ refusal to work on 3 consecutive holidays was prompted by their disagreement with the management-imposed new work schedule. Having been grounded on a non-strikeable issue and without complying with the procedural requirements, then the same is a violation of the “No Strike-No Lockout Policy” in the existing CBA. Respecting the second strike, where the Union complied with procedural requirements, the same was not a violation of the “No Strike- No Lockout” provisions, as a “No Strike-No Lockout” provision in the Collective Bargaining Agreement (CBA) is a valid stipulation but may be invoked only by employer when the strike is economic in nature or one which is conducted to force wage or other concessions from the employer that are not mandated to be granted by the law. It would be inapplicable to prevent a strike which is grounded on unfair labor practice. In the present case, the Union believed in good faith that petitioner committed unfair labor practice when it went on strike on account of the 30-day suspension meted to the striking mechanics, dismissal of a union officer and perceived union-busting, among others.

The Court holds that the second strike became invalid due to the commission of illegal action in its course. It is hornbook principle that the exercise of the right of private sector employees to strike is not absolute. Thus Section 3 of Article XIII of the Constitution provides: SECTION 3. x x x It shall guarantee the rights of all workers to self-organization, collective bargaining and negotiations and peaceful concerted activities, including the right to strike in accordance with law. They shall be entitled to security of tenure, humane conditions of work, and a living wage. They shall also participate in policy and decision-making processes affecting their rights and benefits as may be provided by law. (Emphasis and underscoring supplied) Indeed, even if the purpose of a strike is valid, the strike may still be held illegal where the means employed are illegal. Thus, the employment of violence, intimidation, restraint or coercion in carrying out concerted activities which are injurious to the right to property renders a strike illegal. And so is picketing or the obstruction to the free use of property or the comfortable enjoyment of life or property, when accompanied by intimidation, threats, violence, and coercion as to constitute nuisance.

Apropos is the following ruling in Sukhothai Cuisine v. Court of Appeals: Well-settled is the rule that even if the strike were to be declared valid because its objective or purpose is lawful, the strike may still be declared invalid where the means employed are illegal. Among such limits are the prohibited activities under Article 264 of the Labor Code, particularly paragraph (e), which states that no person engaged in picketing shall: a) commit any act of violence, coercion, or intimidation or b) obstruct the free ingress to or egress from the employer's premises for lawful purposes, or c) obstruct public thoroughfares.

109

The following acts have been held to be prohibited activities: where the strikers shouted slanderous and scurrilous words against the owners of the vessels; where the strikers used unnecessary and obscene language or epithets to prevent other laborers to go to work, and circulated libelous statements against the employer which show actual malice; where the protestors used abusive and threatening language towards the patrons of a place of business or against co-employees, going beyond the mere attempt to persuade customers to withdraw their patronage; where the strikers formed a human cordon and blocked all the ways and approaches to the launches and vessels of the vicinity of the workplace and perpetrated acts of violence and coercion to prevent work from being performed; and where the strikers shook their fists and threatened non-striking employees with bodily harm if they persisted to proceed to the workplace. Permissible activities of the picketing workers do not include obstruction of access of customers.

The appellate court found in the present case, as in fact it is not disputed, that the Union committed illegal acts during the strike. The Union members’ repeated name-calling, harassment and threats of bodily harm directed against company officers and non-striking employees and, more significantly, the putting up of placards, banners and streamers with vulgar statements imputing criminal negligence to the company, which put to doubt reliability of its operations, come within the purview of illegal acts under Art. 264 and jurisprudence. That the alleged acts of violence were committed in nine non-consecutive days during the almost eight months that the strike was on-going does not render the violence less pervasive or widespread to be excusable. Nowhere in Art. 264 does it requires that violence must be continuous or that it should be for the entire duration of the strike.

The appellate court took against petitioner its filing of its complaint to have the strike declared illegal almost eight months from the time it commenced. Art. 264 does not, however, state for purposes of having a strike declared as illegal that the employer should immediately report the same. It only lists what acts are prohibited. It is thus absurd to expect an employer to file a complaint at the first instance that an act of violence is alleged to be committed, especially, as in the present case, when an earlier complaint to have the refusal of the individual respondents to work overtime declared as an illegal strike was still pending — an issue resolved in its favor only on September 25, 1998.

The records show that the Union went on strike on October 22, 1997, and the first reported harassment incident occurred on October 29, 1997, while the last occurred in January, 1998. Those instances may have been sporadic, but as found by the Labor Arbiter and the NLRC, the display of placards, streamers and banners even up to the time the appeal was being resolved by the NLRC works against the Union’s favor. The acts complained of including the display of placards and banners imputing criminal negligence on the part of the company and its officers, apparently with the end in view of intimidating the company’s clientele, are, given the nature of its business, that serious as to make the “second strike” illegal. Specifically with respect to the putting up of those banners and placards, coupled with the name-calling and harassment, the same indicates that it was resorted to coerce the resolution of the dispute – the very evil which Art. 264 seek to prevent.

2nd Issue: As to the issue strike, Sukhothai instructs:

of

loss

of

employment

of

those

who

participated

in

the

illegal

In the determination of the liabilities of the individual respondents, the applicable provision is Article 264(a) of the Labor Code: Art. 264. Prohibited Activities – (a) x x x x x x x Any union officer who knowingly participates in an illegal strike and any worker or union officer who knowingly participates in the commission of illegal acts during an illegal strike may be declared to have lost his employment status: Provided, That mere participation of a worker in a lawful strike shall not constitute sufficient ground for termination of his employment, even if a replacement had been hired by the employer during such lawful strike. xxxx

110

In Samahang Manggagawa sa Sulpicio Lines, Inc.-NAFLU v. Sulpicio Lines, Inc., this Court explained that the effects of such illegal strikes, outlined in Article 264, make a distinction between workers and union officers who participate therein: an ordinary striking worker cannot be terminated for mere participation in an illegal. There must be proof that he or she committed illegal acts during a strike. A union officer, on the other hand, may be terminated from work when he knowingly participates in an illegal strike, and like other workers, when he commits an illegal act during an illegal strike. In all cases, the striker must be identified. But proof beyond reasonable doubt is not required. Substantial evidence available under the attendant circumstances, which may justify the imposition of the penalty of dismissal, may suffice. The liability for prohibited acts has thus to be determined on an individual basis. A perusal of the Labor Arbiter’s Decision, which was affirmed in toto by the NLRC, shows that on account of the staging of the illegal strike, individual respondents were all deemed to have lost their employment, without distinction as to their respective participation. Of the participants in the illegal strike, whether they knowingly participated in the illegal strike in the case of union officers or knowingly participated in the commission of violent acts during the illegal strike in the case of union members, the records do not indicate. While respondent Julius Vargas was identified to be a union officer, there is no indication if he knowingly participated in the illegal strike. The Court not being a trier of facts, the remand of the case to the NLRC is in order only for the purpose of determining the status in the Union of individual respondents and their respective liability, if any.

YSS EMPLOYEES UNION-PHIL TRANSPORT AND GENERAL WORKERS ORGANIZATION VS. YSS LABORATORIES INC. G.R. NO. 155125, DECEMBER 4, 2009

Facts: YSS Laboratories is a domestic corporation engaged in the pharmaceutical business. YSSEU is a duly registered labor organization and the sole and exclusive bargaining representative of the rank and file employees of YSS Laboratories. In order to arrest escalating business losses, YSS Laboratories implemented a retrenchment program which affected 11 employees purportedly chosen in accordance with the reasonable standards established by the company. Of the 11 employees sought to be retrenched, nine were officers and members of YSSEU. Initially, these employees were given the option to avail themselves of the early retirement program of the company. When no one opted to retire early, YSS Laboratories exercised its option to terminate the services of its employees as allegedly authorized under Article 283 of the Labor Code. Thus, copies of the Notices of Termination were filed with DOLE on 19 March 2001 and were served to concerned employees on 20 March 2001.

Claiming that YSS Laboratories was guilty of discrimination and union-busting in carrying out the said retrenchment program, YSSEU decided to hold a strike. After the necessary strike vote was taken under the supervision of the National Conciliation Mediation Board – National Capital Region (NCMB-NCR), YSSEU staged a strike on 20 April 2001.

In order to forge a compromise, a number of conciliation proceedings were conducted by the NCMB-NCR, but these efforts proved futile since the parties’ stance was unbending. This prompted the Secretary of Labor to finally intervene in order to put an end to a prolonged labor dispute. Underscoring the government’s policy of preserving economic gains and employment levels, the Secretary of Labor deemed that the continuation of the labor dispute was inimical to national interest. Thus, in an Order dated 11 May 2001, the Secretary of Labor certified the labor dispute to the National Labor Relations Commission (NLRC) for compulsory arbitration. Accordingly, all striking workers were thereby directed to return to work within 24 hours from their receipt of the said Order, and YSS Laboratories to accept them under the terms and conditions prevailing before the strike.

YSS Laboratories, however, refused to fully comply with the directive of the Secretary of Labor. In its Urgent Motion for Reconsideration, YSS Laboratories argued that nine union officers and members who were previously terminated from service pursuant to a valid retrenchment should be excluded from the operation of the return-to-work order. It also asserted that the union officers who participated in the purported illegal

111

strike should likewise not be allowed to be back to their employment for they were deemed to have already lost their employment status.

Issues: I. WHETHER OR NOT THE SECRETARY OF LABOR GRAVELY ABUSED ITS DISCRETION IN CERTIFYING THE LABOR DISPUTE TO THE NLRC FOR COMPULSORY ARBITRATION. II. WHETHER OR NOT THE RETRENCHED EMPLOYEES SHOULD BE EXCLUDED FROM THE OPERATION OF THE RETURN TO WORK ORDER.

Ruling: The petition is impressed with merit.

The Orders dated 11 May 2001 and 9 June 2001 of the Secretary of Labor, certifying the labor dispute involving the herein parties to the NLRC for compulsory arbitration, and enjoining YSSEU to return to work and YSS Laboratories to admit them under the same terms and conditions prevailing before the strike, were issued pursuant to Article 263(g) of the Labor Code. Said provision reads: Art. 263. Strikes, picketing, and lockouts. xxxx (g) When, in his opinion, there exists a labor dispute causing or likely to cause a strike or lockout in an industry indispensable to the national interest, the Secretary of Labor and Employment may assume jurisdiction over the dispute and decide it or certify the same to the Commission for compulsory arbitration. Such assumption or certification shall have the effect of automatically enjoining the intended or impending strike or lockout as specified in the assumption or certification order. If one has already taken place at the time of assumption or certification, all striking or locked out employees shall immediately return to work and the employer shall immediately resume operations and readmit all workers under the same terms and conditions prevailing before the strike or lockout. The Secretary of Labor and Employment or the Commission may seek the assistance of law enforcement agencies to ensure compliance with this provision as well as with such orders as he may issue to enforce the same.

After martial law was lifted and democracy was restored, the assumption of jurisdiction in Art. 263(g) has now been viewed as an exercise of the police power of the State with the aim of promoting the common good. The grant of these plenary powers to the Secretary of Labor makes it incumbent upon him to bring about soonest, a fair and just solution to the differences between the employer and the employees, so that the damage such labor dispute might cause upon the national interest may be minimized as much as possible, if not totally averted, by avoiding stoppage of work or any lag in the activities of the industry or the possibility of those contingencies that might cause detriment to the national interest.

In order to effectively achieve such end, the assumption or certification order shall have the effect of automatically enjoining the intended or impending strike or lockout. Moreover, if one has already taken place, all striking workers shall immediately return to work, and the employer shall immediately resume operations and readmit all workers under the same terms and conditions prevailing before the strike or lockout. YSS Laboratories’ vigorous insistence on the exclusion of the retrenched employees from the coverage of the return-to-work order seriously impairs the authority of the Secretary of Labor to forestall a labor dispute that he deems inimical to the national economy. The Secretary of Labor is afforded plenary and broad powers, and is granted great breadth of discretion to adopt the most reasonable and expeditious way of writing finis to the labor dispute.

Accordingly, when the Secretary of Labor directed YSS Laboratories to accept all the striking workers back to work, the Secretary did not exceed his jurisdiction, or gravely abuse the same. It is significant at this point to point out that grave abuse of discretion implies a capricious and whimsical exercise of judgment. Thus, an act may be considered as committed in grave abuse of discretion when the same is performed in a capricious or whimsical exercise of judgment, which is equivalent to lack of jurisdiction. The

112

abuse of discretion must be so patent and gross as to amount to an evasion of a positive duty or to a virtual refusal to perform a duty enjoined by law, or to act at all in contemplation of law, as where the power is exercised in an arbitrary and despotic manner by reason of passion or personal hostility. In the case at bar, there is no showing that the assailed orders were issued in an arbitrary or despotic manner. The Orders dated 11 May 2001 and 9 June 2001 were issued by the Secretary of Labor, with the end in view of preserving the status quo ante while the main issues of the validity of the retrenchment and legality of the strike were being threshed out in the proper forum. This was done for the promotion of the common good, considering that a lingering strike could be inimical to the interest of both employer and employee. The Secretary of Labor acts to maintain industrial peace. Thus, his certification for compulsory arbitration is not intended to interfere with the management’s rights but to obtain a speedy settlement of the dispute.

By harping on the validity of the retrenchment and on the exclusion of the retrenched employees from the coverage of the return-to-work order, YSS Laboratories undermines the underlying principle embodied in Article 263(g) of the Labor Code on the settlement of labor disputes -- that assumption and certification orders are executory in character and are to be strictly complied with by the parties, even during the pendency of any petition questioning their validity. Regardless therefore of its motives, or of the validity of its claims, YSS Laboratories must readmit all striking employees and give them back their respective jobs. Accepting back the workers in this case is not a matter of option, but of obligation mandated by law for YSS Laboratories to faithfully comply with. Its compulsory character is mandated, not to cater to a narrow segment of society, or to favor labor at the expense of management, but to serve the greater interest of society by maintaining the economic equilibrium.

Instructive is the ruling of this Court in Philippine Airlines Employees Association v. Philippine Airlines, Inc.: The very nature of a return-to-work order issued in a certified case lends itself to no other construction. The certification attests to the urgency of the matter, affecting as it does an industry indispensable to the national interest. The order is issued in the exercise of the court’s compulsory power of arbitration, and therefore must be obeyed until set aside. x x x.

Certainly, the determination of who among the strikers could be admitted back to work cannot be made to depend upon the discretion of employer, lest we strip the certification or assumption-of-jurisdiction orders of the coercive power that is necessary for attaining their laudable objective. The return-to-work order does not interfere with the management’s prerogative, but merely regulates it when, in the exercise of such right, national interests will be affected. The rights granted by the Constitution are not absolute. They are still subject to control and limitation to ensure that they are not exercised arbitrarily. The interests of both the employers and employees are intended to be protected and not one of them is given undue preference. WHEREFORE, premises considered, the instant Petition is GRANTED.

GOVERNMENT SERVICE INSURANCE SYSTEM (GSIS) VS. DINNAH VILLAVIZA, ELIZABETH DUQUE, ADRONICO A. ECHAVEZ, RODEL RUBIO, ROWENA THERESE B. GRACIA, PILAR LAYCO, AND ANTONIO JOSE LEGARDA, G.R. NO. 180291, JULY 27, 2010

Facts:

Petitioner Winston Garcia (PGM Garcia), as President and General Manager of the GSIS, filed separate formal charges against respondents Dinnah Villaviza, Elizabeth Duque, Adronico A. Echavez, Rodel Rubio, Rowena Therese B. Gracia, Pilar Layco, and Antonio Jose Legarda for Grave Misconduct and/or Conduct Prejudicial to the Best Interest of the Service pursuant to the Rules of Procedure in Administrative Investigation (RPAI) of GSIS Employees and Officials, III, D, (1, c, f) in relation to Section 52A (3), (20), Rule IV, of the Uniform Rules on Administrative Cases in the Civil Service (URACCS), in accordance with Book V of the Administrative Code of 1987, committed as follows:

That on 27 May 2005, respondent, wearing red shirt together with some employees, marched to or appeared simultaneously at or just outside the office of the Investigation Unit in a mass demonstration/rally of protest and support for Messrs. Mario Molina and Albert Velasco, the latter having surreptitiously entered the GSIS premises;

113

xxx

xxx

xxx

That some of these employees badmouthed the security guards and the GSIS management and defiantly raised clenched fists led by Atty. Velasco who was barred by Hearing Officer Marvin R. Gatpayat in an Order dated 24 May 2005 from appearing as counsel for Atty. Molina pursuant to Section 7 (b) (2) of R.A. 6713 otherwise known as the Code of Conduct and Ethical Standards for Public Officials and Employees; That respondent, together with other employees in utter contempt of CSC Resolution No. 021316, dated 11 October 2002, otherwise known as Omnibus Rules on Prohibited Concerted Mass Actions in the Public Sector caused alarm and heightened some employees and disrupted the work at the Investigation Unit during office hours. Respondents Duque, Echavez, Rubio, Gracia, Layco, and Legarda, together with two others, submitted a letter-explanation to Atty. Barbo dated June 6, 2005. Denying that there was a planned mass action, the respondents explained that their act of going to the office of the GSIS-IU was a spontaneous reaction after learning that their former union president was there. Aside from some of them wanting to show their support, they were interested in that hearing as it might also affect them. PGM Garcia then filed the above-mentioned formal charges for Grave Misconduct and/or Conduct Prejudicial to the Best Interest of the Service against each of the respondents, all dated June 4, 2005. Respondents were again directed to submit their written answers under oath within three (3) days from receipt thereof.5 None was filed. On June 29, 2005, PGM Garcia issued separate but similarly worded decisions finding all seven (7) respondents guilty of the charges and meting out the penalty of one (1) year suspension plus the accessory penalties appurtenant thereto. On appeal, the Civil Service Commission (CSC) found the respondents guilty of the lesser offense of Violation of Reasonable Office Rules and Regulations and reduced the penalty to reprimand. The CSC ruled that respondents were not denied their right to due process but there was no substantial evidence to hold them guilty of Conduct Prejudicial to the Best Interest of the Service.

Issue: Whether or not respondents' actions on May 27, 2005 amounted to a "prohibited concerted activity or mass action."

Ruling:

As defined in Section 5 of CSC Resolution No. 02-1316 which serves to regulate the political rights of those in the government service, the concerted activity or mass action proscribed must be coupled with the "intent of effecting work stoppage or service disruption in order to realize their demands of force concession." Wearing similarly colored shirts, attending a public hearing at the GSIS-IU office, bringing with them recording gadgets, clenching their fists, some even badmouthing the guards and PGM Garcia, are acts not constitutive of an (i) intent to effect work stoppage or service disruption and (ii) for the purpose of realizing their demands of force concession.

Precisely, the limitations or qualifications found in Section 5 of CSC Resolution No. 02-1316 are there to temper and focus the application of such prohibition. Not all collective activity or mass undertaking of government employees is prohibited. Otherwise, we would be totally depriving our brothers and sisters in the government service of their constitutional right to freedom of expression.

Government workers, whatever their ranks, have as much right as any person in the land to voice out their protests against what they believe to be a violation of their rights and interests. Civil Service does not deprive them of their freedom of expression. It would be unfair to hold that by joining the government service, the members thereof have renounced or waived this basic liberty. This freedom can be reasonably regulated only but can never be taken away.

114

A review of PGM Garcia's formal charges against the respondents reveals that he himself was not even certain whether the respondents and the rest of the twenty or so GSIS employees who were at the GSIS-IU office that fateful day marched there or just simply appeared there simultaneously.14 Thus, the petitioners were not even sure if the spontaneous act of each of the twenty or so GSIS employees on May 27, 2005 was a concerted one. The report of Manager Nagtalon of the GSIS-SD which was the basis for PGM Garcia's formal charges reflected such uncertainty.

In the recent case of GSIS v. Kapisanan ng mga Manggagawa sa GSIS,16 the Court upheld the position of petitioner GSIS because its employees, numbering between 300 and 800 each day, staged a walkout and participated in a mass protest or demonstration outside the GSIS for four straight days. We cannot say the same for the 20 or so employees in this case. To equate their wearing of red shirts and going to the GSIS-IU office for just over an hour with that four-day mass action in Kapisanan ng mga Manggagawa sa GSIS case and to punish them in the same manner would most certainly be unfair and unjust.

Recent analogous decisions in the United States, while recognizing the government's right as an employer to lay down certain standards of conduct, tend to lean towards a broad definition of "public concern speech" which is protected by their First Amendment. One such case is that of Scott v. Meters.17 In said case, the New York Transit Authority (NYTA), responsible for operation of New York City's mass transit service, issued a rule prohibiting employees from wearing badges or buttons on their uniforms. A number of union members wore union buttons promoting their opposition to a collective bargaining agreement. Consequently, the NYTA tried to enforce its rule and threatened to subject these union members to discipline. The court, though recognizing the government's right to impose reasonable restrictions, held that the NYTA's rule was "unconstitutionally overboard."

Thus, respondents' freedom of speech and of expression remains intact, and CSC's Resolution No. 02-1316 defining what a prohibited concerted activity or mass action has only tempered or regulated these rights. Measured against that definition, respondents' actuations did not amount to a prohibited concerted activity or mass action. The CSC and the CA were both correct in arriving at said conclusion.

PICOP RESOURCES INC. VS. TANECA ET. AL., G.R. NO. 160828, 09 AUGUST 2010 Facts: On February 13, 2001, respondents Tañeca and 14 others filed a Complaint for unfair labor practice, illegal dismissal and money claims against petitioner PICOP Resources , Inc. (PRI). Respondents were regular rank-and-file employees of PRI and bona fide members of Nagkahiusang Mamumuo sa PRI Southern Philippines Federation of Labor (NAMAPRI-SPFL), which is the collective bargaining agent for the rank-andfile employees of petitioner PRI. PRI has a collective bargaining agreement (CBA) with NAMAPRI-SPFL for a period of five (5) years from May 22, 1995 until May 22, 2000. On May 16, 2000, Atty. Proculo P. Fuentes, VP of PRI sent a letter to the management of PRI demanding the termination of employees who allegedly campaigned for, supported and signed the Petition for Certification Election of the Federation of Free Workers Union (FFW) during the effectivity of the CBA. NAMAPRI-SPFL considered said act of campaigning for and signing the petition for certification election of FFW as an act of disloyalty and a valid basis for termination for a cause in accordance with its Constitution and By-Laws, and the terms and conditions of the CBA. On October 16, 2000, PRI served notices of termination for causes to the 31 out of the 46 employees whom NAMAPRIL-SPFL sought to be terminated on the ground of "acts of disloyalty" committed against it. A Notice was also served on the DOLE, Caraga Region. Respondents alleged that none of them ever withdrew their membership or submitted to PRI any union dues and check-off disauthorizations against NAMAPRI-SPFL. They claimed that they continue to remain on record as bona fide members. They insisted that mere affixation of signature on such authorization was not per se an act of disloyalty. They claimed that while it may be true that they signed the said authorization before the start of the freedom period, the petition of FFW was only filed with the DOLE on May 18, 2000, or 58 days after the start of the freedom period.

115

Respondents, likewise, argued that at the time NAMAPRI-SPFL demanded their termination, it was no longer the bargaining representative of the rank-and-file workers of PRI, because the CBA had already expired on May 22, 2000. Hence, there could be no justification in PRI's act of dismissing respondents due to acts of disloyalty. Respondents asserted that the act of PRI in giving in to the wishes of the Union in discharging them on the ground of disloyalty to the Union amounted to interference with, restraint or coercion of respondents' exercise of their right to self-organization. The act indirectly required petitioners to support and maintain their membership with NAMAPRI-SPFL as a condition for their continued employment. The Labor Arbiter declared the respondents' dismissal to be illegal and ordered PRI to reinstate respondents to their former or equivalent positions without loss of seniority rights and to jointly and solidarily pay their backwages. PRI and NAMAPRI-SPFL appealed to the NLRC which reversed the decision of the Labor Arbiter; thus, declaring the dismissal of respondents from employment as legal. Respondents filed a motion for reconsideration, but it was denied for lack of merit. Respondents filed a petition for certiorari under Rule 65 before the CA. The CA reversed and set aside the assailed Resolutions of the NLRC and reinstated the Decision of the Labor Arbiter. Issues: Whether or not an error in the interpretation of law fall within the ambit of Extraordinary Remedy of Certiorari under Rule 65. Whether or not a CBA can be given its effectivity in all its Terms and Conditions, even beyond the 5-year period when no CBA has yet been entered into, possessing just cause to terminate on the grounds herein. Ruling:

Issue 1: PRI perceived an error in the mode of appeal by respondents in assailing the decision of the NLRC. It claimed that assuming that the NLRC erred in its judgment on the legal issues, its error, if any, is not tantamount to abuse of discretion falling within the ambit of Rule 65. WRONG.

The power of the CA to review NLRC decisions via Rule 65 or Petition for Certiorari has been settled as early as in our decision in St. Martin Funeral Home v. NLRC.cThis Court held that the proper vehicle for such review was a Special Civil Action for Certiorari under Rule 65 of the Rules of Court, and that this action should be filed in the CA in strict observance of the doctrine of the hierarchy of courts.cra1aw

Issue 2: PRI argued that the dismissal of the respondents was valid and legal. It claimed to have acted in good faith at the instance of the incumbent union pursuant to the Union Security Clause of the CBA. Citing Article 253 of the Labor Code, PRI contends that as parties to the CBA, they are enjoined to keep the status quo and continue in full force and effect the terms and conditions of the existing CBA during the 60-day period until a new agreement is reached by the parties. UNTENABLE. "Union security" is a generic term, which is applied to and comprehends "closed shop," "union shop," "maintenance of membership," or any other form of agreement which imposes upon employees the obligation to acquire or retain union membership as a condition affecting employment. However, in terminating the employment of an employee by enforcing the union security clause, the employer needs to determine and prove that: (1) the union security clause is applicable; (2) the union is requesting for the enforcement of the union security provision in the CBA; and (3) there is sufficient evidence to support the decision of the union to expel the employee from the union. These requisites constitute just cause for terminating an employee based on the union security provision of the CBA. As to the first requisite, there is no question that the CBA between PRI and respondents included a union security clause. Following the same provision, PRI, upon written request from the Union, can indeed terminate the employment of the employee who failed to maintain its good standing as a union member. Secondly, it is likewise undisputed that NAMAPRI-SPFL, in two (2) occasions demanded from PRI, in their letters, to terminate the employment of respondents due to their acts of disloyalty to the Union. However, as to the third requisite, there is no sufficient evidence to support the decision of PRI to terminate the employment of the respondents.

116

PRI alleged that respondents were terminated based on the alleged acts of disloyalty. It contends that their acts are a violation of the Union Security Clause, as provided in their CBA.UNCONVINCED. The mere signing of the authorization in support of the Petition before the "freedom period," is not sufficient ground to terminate the employment. Nothing in the records would show that respondents failed to maintain their membership in good standing in the Union. Respondents did not resign or withdraw their membership from the Union to which they belong. Respondents continued to pay their union dues and never joined the FFW. An "authorization letter to file a petition for certification election" is different from an actual "Petition for Certification Election." Likewise, as per records, it was clear that the actual Petition for Certification Election of FFW was filed within the ambit of the freedom period. Strictly speaking, what is prohibited is the filing of a petition for certification election outside the 60-day freedom period.

It can be said that while it is incumbent for the employer to continue to recognize the majority status of the bargaining agent even after the expiration of the freedom period, they could only do so when no petition for certification election was filed. The reason is, with a pending petition for certification, any such agreement entered into by management with a labor organization is fraught with the risk that such a labor union may not be chosen thereafter as the collective bargaining representative. The provision for statusquo is conditioned on the fact that no certification election was filed during the freedom period.

Time and again, we have ruled that we adhere to the policy of enhancing the welfare of the workers. Their freedom to choose who should be their bargaining representative is of paramount importance.

We will emphasize anew that the power to dismiss is a normal prerogative of the employer, but with limitations. The employer is bound to exercise caution in terminating the services of his employees especially so when it is made upon the request of a labor union pursuant to the CBA. Dismissals must not be arbitrary and capricious. Due process must be observed in dismissing an employee, because it affects not only his position but also his means of livelihood. Employers should, therefore, respect and protect the rights of their employees, which include the right to labor.

WHEREFORE, the petition is DENIED.

INSULAR HOTEL EMPLOYEES UNION-NFL VS. WATERFRONT INSULAR HOTEL-DAVAO, G.R. NO. 174040-41, SEPTEMBER 22, 2010 Facts: On November 6, 2000, respondent sent the DOLE Region XI, a Notice of Suspension of Operations, notifying the same that it will suspend its operations for a period of six months due to severe and serious business losses. In said notice, respondent assured the DOLE that if the company could not resume its operations within the six-month period, the company would pay the affected employees all the benefits legally due to them.

During the period of the suspension, Domy R. Rojas, President of Davao Insular Hotel Free Employees Union (DIHFEU-NFL), the recognized labor organization in Waterfront Davao, sent respondent a number of letters asking management to reconsider its decision. The letters signified, among others, the gesture of support of the union for the economic solutions of the company, which included, suspension of the CBA for ten years (no strike, no lock-out shall be enforced); payment of benefits be on a staggered basis or as available; overtime hours rendered shall be off-set as practiced; reduce leave benefits; emergency leave and birthday leave be waived, and multi-tasking of employees, among others.

After series of negotiations, respondent Hotel and DIHFEU-NFL signed a Memorandum of Agreement wherein Hotel agreed to re-open subject to certain concessions. Accordingly, Hotel downsized its manpower structure to 100 rank-and-file employees and a new pay scale was also prepared. The retained employees

117

individually signed a “Reconfirmation of Employment”, which embodied the new terms and conditions of their continued employment.

On June 15, 2001, respondent resumed its business operations.

On August 22, 2002, certain Darius Joves and Debbie Planas, claiming to be local officers of the National Federation of Labor (NFL), filed a Notice of Mediation before the NCMB-Regional Office stating that the Union involved was “DARIUS JOVES/DEBBIE PLANAS ET. AL, National Federation of Labor.” The issue raised in said Notice was the “Diminution of wages and other benefits through unlawful Memorandum of Agreement.”

The NCMB called Joves and Hotel to a conference to a possibility of settling the conflict. In the said conference, Hotel and petitioner IHEU-NFL, represented by Joves, signed a Submission Agreement wherein they chose Alfredo C. Olvida to act as voluntary arbitrator. Submitted for the resolution of Olvida was the determination of WON there was a diminution of wages and other benefits through an unlawful MOA. In support of his authority to file the complaint, Joves, assisted by Atty. Danilo Cullo (Cullo), presented several Special Powers of Attorney (SPA) which were, however, undated and unnotarized.

Hotel filed with the NCMB a Manifestation with Motion for a Second Preliminary Conference, raising the following grounds: (1) The persons who filed the instant complaint in the name of the Insular Hotel Employees Union-NFL have no authority to represent the Union; (2) The individuals who executed the special powers of attorney in favor of the person who filed the instant complaint have no standing to cause the filing of the instant complaint; and (3) The existence of an intra-union dispute renders the filing of the instant case premature.

On September 16, 2002, a second preliminary conference was conducted in the NCMB, where Cullo denied any existence of an intra-union dispute among the members of the union. Cullo, however, confirmed that the case was filed not by the IHEU-NFL but by the NFL. When asked to present his authority from NFL, Cullo admitted that the case was, in fact, filed by individual employees named in the SPAs. The hearing officer directed both parties to elevate the aforementioned issues to AVA Olvida.

The case was docketed and referred to Olvida. Respondent again raised its objections, arguing that the persons who signed the complaint were not the authorized representatives of the Union nor were they parties to the MOA. AVA Olvida directed respondent to file a formal motion to withdraw its submission to voluntary arbitration. Respondent filed its Motion to Withdraw. Cullo then filed an Opposition. In said Opposition, Cullo reiterated that the complainants were not representing the union but filing the complaint through their appointed attorneys-in-fact to assert their individual rights as workers who are entitled to the benefits granted by law and stipulated in the collective bargaining agreement.

On March 18, 2003, AVA Olvida ruled that respondent was correct when it raised its objection to NFL as proper party-complainant. The proper party-complainant is INSULAR HOTEL EMPLOYEES UNION-NFL, the recognized and incumbent bargaining agent of the rank-and-file employees of the respondent hotel. In the submission agreement of the parties dated August 29, 2002, the party complainant written is INSULAR HOTEL EMPLOYEES UNION-NFL and not the NATIONAL FEDERATION OF LABOR and 79 other members. However, since the NFL is the mother federation of the local union, and signatory to the existing CBA, it can represent the union, the officers, the members or union and officers or members, as the case may be, in all stages of proceedings in courts or administrative bodies provided that the issue of the case will involve labor-management relationship like in the case at bar.

Later, respondent filed a Motion for Inhibition alleging AVA Olvida's bias and prejudice towards the cause of the employees. AVA Olvida voluntarily inhibited himself out of “delicadeza” and ordered the remand of the case to the NCMB. The NCMB then required the parties to appear before the conciliator for the selection of a new voluntary arbitrator. Respondent, however, maintained its stand that the NCMB had no jurisdiction over the case. Consequently, at the instance of Cullo, the NCMB approved ex parte the selection of AVA Montejo as the new voluntary arbitrator.

118

On April 5, 2004, AVA Montejo rendered a Decision ruling in favor of Cullo, declaring the Memorandum of Agreement in question as invalid as it is contrary to law and public policy; declaring that there is a diminution of the wages and other benefits of the Union members and officers under the said invalid MOA, among others. Both parties appealed the Decision of AVA Montejo to the CA. Cullo only assailed the Decision in so far as it did not categorically order respondent to pay the covered workers their differentials in wages reckoned from the effectivity of the MOA up to the actual reinstatement of the reduced wages and benefits. Respondent, for its part, questioned among others the jurisdiction of the NCMB. Respondent maintained that the MOA it had entered into with the officers of the Union was valid. Both cases were consolidated by the CA. CA rendered a ruling in favor of respondent Hotel. which was, however, denied.

Aggrieved, Cullo filed a Motion for Reconsideration,

Issues: WON the Accredited Voluntary Arbitrator has no jurisdiction over the case, simply because the Notice of Mediation does not mention the name of the local union but only the affiliate federation. WON the diminution of the benefits enjoyed by the employees is allowable.

Ruling: Cullo contends that the CA committed error when it ruled that the voluntary arbitrator had no jurisdiction over the case simply because the Notice of Mediation did not state the name of the local union thereby disregarding the Submission Agreement which states the names of local union as IHEU-NFL. While it is undisputed that a submission agreement was signed by respondent and “IHEU-NFL,” represented by Joves and Cullo, there are two circumstances which affect its validity: first, the Notice of Mediation was filed by a party who had no authority to do so; second, that respondent had persistently objected questioning the authority of Joves, Cullo and the individual members of the Union to file the complaint before the NCMB.

Who may file a notice or declare a strike or lockout or request preventive mediation. Any certified or duly recognized bargaining representativemay file a notice or declare a strike or request for preventive mediation in cases of bargaining deadlocks and unfair labor practices. It is clear that only a certified or duly recognized bargaining agent may file a notice or request for preventive mediation. It is curious that even Cullo himself admitted, in a number of pleadings, that the case was filed not by the Union but by individual members thereof. Clearly, therefore, the NCMB had no jurisdiction to entertain the notice filed before it. A local union does not owe its existence to the federation with which it is affiliated. It is a separate and distinct voluntary association owing its creation to the will of its members. Mere affiliation does not divest the local union of its own personality, neither does it give the mother federation the license to act independently of the local union. It only gives rise to a contract of agency, where the former acts in representation of the latter. Hence, local unions are considered principals while the federation is deemed to be merely their agent. We hold that the voluntary arbitrator had no jurisdiction over the case. While we commend NFL's zealousness in protecting the rights of lowly workers, We cannot, however, allow it to go beyond what it is empowered to do. Anent the second issue, the same is again without merit. Clearly, the prohibition against elimination or diminution of benefits set out in Article 100 of the Labor Code is specifically concerned with benefits already enjoyed at the time of the promulgation of the Labor Code. Article 100 does not, in other words, purport to apply to situations arising after the promulgation date of the Labor Code. A CBA is “a contract executed upon request of either the employer or the exclusive bargaining representative incorporating the agreement reached after negotiations with respect to wages, hours of work and all other terms and conditions of employment, including proposals for adjusting any grievances or questions arising under such agreement.” The primary purpose of a CBA is the stabilization of labormanagement relations in order to create a climate of a sound and stable industrial peace. In construing a CBA, the courts must be practical and realistic and give due consideration to the context in which it is

119

negotiated and the purpose which it is intended to serve. The right to free collective bargaining, after all, includes the right to suspend it. Stipulated in each Reconfirmation of Employment were the new salary and benefits scheme. In addition, it bears to stress that specific provisions of the new contract also made reference to the MOA. Thus, the individual members of the union cannot feign knowledge of the execution of the MOA. Each contract was freely entered into and there is no indication that the same was attended by fraud, misrepresentation or duress. To this Court's mind, that signing should, therefore, be deemed an implied ratification by the Union members of the MOA. It is iniquitous to receive benefits from a CBA and later on disclaim its validity.

While the terms of the MOA undoubtedly reduced the salaries and certain benefits previously enjoyed by the members of the Union, it cannot escape this Court's attention that it was the execution of the MOA which paved the way for the re-opening of the hotel, notwithstanding its financial distress. More importantly, the execution of the MOA allowed respondents to keep their jobs.

Withal, while the scales of justice usually tilt in favor of labor, the peculiar circumstances herein prevent this Court from applying the same in the instant petition. Even if our laws endeavor to give life to the constitutional policy on social justice and on the protection of labor, it does not mean that every labor dispute will be decided in favor of the workers. The law also recognizes that management has rights which are also entitled to respect and enforcement in the interest of fair play.

WHEREFORE, premises considered, the petition is DENIED.

CITREK EMPLOYEES LABOY UNION-FFW VS. CITREK ELECTRONICS INC. G.R. NO. 190515, 15 NOVEMBER 2010 Facts:

Cirtek Electronics, Inc. (respondent), an electronics and semi-conductor firm situated inside the Laguna Technopark, had an existing CBA with Cirtek Employees Labor Union-Federation of Free Workers (petitioner) for the period January 1, 2001 up to December 31, 2005. Prior to the 3rd year of the CBA, the parties renegotiated its economic provisions but failed to reach a settlement, particularly on the issue of wage increases. Petitioner thereupon declared a bargaining deadlock and filed a Notice of Strike with the NCMBRegional Office on April 26, 2004. Respondent, upon the other hand, filed a Notice of Lockout on June 16, 2004. While the conciliation proceedings were ongoing, respondent placed seven union officers including the President, a Vice President, the Secretary and the Chairman of the Board of Directors under preventive suspension for allegedly spearheading a boycott of overtime work. The officers were eventually dismissed from employment, prompting petitioner to file another Notice of Strike which was, after conciliation meetings, converted to a voluntary arbitration case. The dismissal of the officers was later found to be legal, hence, petitioner appealed.

In the meantime, as amicable settlement of the CBA was deadlocked, petitioner went on strike on June 20, 2005. By Order dated June 23, 2005, the Secretary of Labor assumed jurisdiction over the controversy and issued a Return to Work Order which was complied with. Before the SOLE could rule on the controversy, respondent created a Labor Management Council (LMC) through which it concluded with the remaining officers of petitioner a Memorandum of Agreement providing for daily wage increases of P6.00 per day effective January 1, 2004 and P9.00 per day effective January 1, 2005. Petitioner submitted the MOA via Motion and Manifestation to the Secretary of Labor, alleging that the remaining officers signed the MOA under respondent’s assurance that should the Secretary order a higher award of wage increase, respondent would comply.

120

By Orderdated March 16, 2006, the Secretary of Labor resolved the CBA deadlock by awarding a wage increase of from P6.00 to P10.00 per day effective January 1, 2004 and from P9.00 to P15.00 per day effective January 1, 2005, and adopting all other benefits as embodied in the MOA.

Respondent moved for a reconsideration of the Decision. Reconsideration of the Decision was denied by Resolution of August 12, 2008, hence, respondent filed a petition for certiorari before the Court of Appeals. The CA favored respondent. Issues: Whether or not the SOLE is authorized to give an award higher than that agreed upon in the MOA; Whether or not the MOA was entered into and ratified by the remaining officers of petitioner under the condition, which was not incorporated in the MOA, that respondent would honor the SOLE’s award in the event that it is higher.

Ruling:

The Court resolves both issues in the affirmative. It is well-settled that the SOLE, in the exercise of his power to assume jurisdiction under Art. 263 (g)of the Labor Code, may resolve all issues involved in the controversy including the award of wage increases and benefits. While an arbitral award cannot per se be categorized as an agreement voluntarily entered into by the parties because it requires the intervention and imposing power of the State thru the Secretary of Labor when he assumes jurisdiction, the arbitral award can be considered an approximation of a collective bargaining agreement which would otherwise have been entered into by the parties, hence, it has the force and effect of a valid contract obligation.

That the arbitral award was higher than that which was purportedly agreed upon in the MOA is of no moment. For the SOLE, in resolving the CBA deadlock, is not limited to considering the MOA as basis in computing the wage increases. He could, as he did, consider the financial documents submitted by respondent as well as the parties’ bargaining history and respondent’s financial outlook and improvements as stated in its website.

While a contract constitutes the law between the parties, this is so in the present case with respect to the CBA, not to the MOA in which even the union’s signatories had expressed reservations thereto. But even assuming arguendo that the MOA is treated as a new CBA, since it is imbued with public interest, it must be construed liberally and yield to the common good.

While the terms and conditions of a CBA constitute the law between the parties, it is not, however, an ordinary contract to which is applied the principles of law governing ordinary contracts. A CBA, as a labor contract within the contemplation of Article 1700 of the Civil Code of the Philippines which governs the relations between labor and capital, is not merely contractual in nature but impressed with public interest, thus, it must yield to the common good. As such, it must be construed liberally rather than narrowly and technically, and the courts must place a practical and realistic construction upon it, giving due consideration to the context in which it is negotiated and purpose which it is intended to serve.

The petition is GRANTED. The Decisions of the Court of Appeals are REVERSED and SET ASIDE and the Order dated March 16, 2006 and Resolution dated August 12, 2008 of the Secretary of Labor are REINSTATED.

NOTE: A motion for reconsideration was filed by respondent. A resolution to that effect was issued on June 06, 2011 (G.R. No. 190515, 06 June 2011). The SC denied said motion. SOLIDBANK CORP. VS. GRAMIER, ET. AL, , G.R. NO. 159460-61, NOVEMBER 15, 2010 FACTS:

121

Sometime in October 1999, petitioner Solidbank and respondent Solidbank Employees’ Union (Union) were set to renegotiate the economic provisions of their 1997-2001 Collective Bargaining Agreement (CBA) to cover the remaining two years thereof. Negotiations commenced on November 17, 1999 but seeing that an agreement was unlikely, the Union declared a deadlock on December 22, 1999 and filed a Notice of Strike on December 29, 1999. Sensing the impending strike, the Secretary of Labor and Employment “assumed jurisdiction” over the labor dispute, pursuant to Article 263 (g) of the Labor Code, as amended. The assumption order dated January 18, 2000 directed the parties "to cease and desist from committing any and all acts that might exacerbate the situation. In his Order4 dated March 24, 2000, Secretary Laguesma resolved all economic and non-economic issues submitted by the parties.

Not satisfied with the Secretary’s ruling, on April 3, 2000, the union officers and members decided to protest the same by holding a rally in front of the SOLE office and the bank’s head offices, simultaneous to their filing of MR of the March 24, 2000 order. An overwhelming majority of the employees, including the respondents, joined the “mass leave” and “protest action”. The bank’s provincial branches in Cebu, Iloilo, Bacolod and Naga followed suit and "boycotted regular work." As a result of the employees’ concerted actions, Solidbank’s business operations were paralyzed. The protest action lasted for three days, from April 3 until April 5, 2000.

The president of Solidbank issued a memorandum addressed to all employees to show cause why they should not be dismissed for participating in an illegal strike. On April 5, 2000, Vistan (president of Solidbank) issued another memorandum, this time declaring that the bank is prepared to take back employees who will report for work starting April 6, 2000 "provided these employees were/are not part of those who led or instigated or coerced their co-employees into participating in this illegal act." Out of the 712 employees who took part in the three-day work boycott, a total of 513 returned to work and were accepted by the bank. The remaining 199 employees insisted on defying Vistan’s directive, which included herein respondents Ernesto U. Gamier, Elena R. Condevillamar, Janice L. Arriola and Ophelia C. De Guzman.

Thus, they were dismissed. Respondents Gamier, Condevillamar, Arriola and De Guzman filed separate complaints for illegal dismissal, moral and exemplary damages and attorney’s fees. Respondent Union joined by the 129 dismissed employees filed a separate suit against petitioners for illegal dismissal, unfair labor practice and damages.

ISSUE/s:

The fundamental issues to be resolved in this controversy are: (1) whether the protest rally and concerted work abandonment/boycott staged by the respondents violated the Order dated January 18, 2000 of the Secretary of Labor; (2) whether the respondents were validly terminated; and (3) whether the respondents are entitled to separation pay or financial assistance.

HELD:

I. Article 212 of the Labor Code, as amended, defines strike as any temporary stoppage of work by the concerted action of employees as a result of an industrial or labor dispute. A labor dispute includes any controversy or matter concerning terms and conditions of employment or the association or representation of persons in negotiating, fixing, maintaining, changing or arranging the terms and conditions of employment, regardless of whether or not the disputants stand in the proximate relation of employers and employees.41 The term "strike" shall comprise not only concerted work stoppages, but also slowdowns, mass leaves, sitdowns, attempts to damage, destroy or sabotage plant equipment and facilities and similar activities.42 Thus, the fact that the conventional term "strike" was not used by the striking employees to describe their common course of action is inconsequential, since the substance of the situation, and not its appearance, will be deemed to be controlling.43 After a thorough review of the records, we hold that the CA patently erred in concluding that the concerted mass actions staged by respondents cannot be considered a strike but a legitimate exercise of the

122

respondents’ right to express their dissatisfaction with the Secretary’s resolution of the economic issues in the deadlocked CBA negotiations with petitioners. It must be stressed that the concerted action of the respondents was not limited to the protest rally infront of the DOLE Office on April 3, 2000. Respondent Union had also picketed the Head Office and Paseo de Roxas Branch. About 712 employees, including those in the provincial branches, boycotted and absented themselves from work in a concerted fashion for three continuous days that virtually paralyzed the employer’s banking operations. Considering that these mass actions stemmed from a bargaining deadlock and an order of assumption of jurisdiction had already been issued by the Secretary of Labor to avert an impending strike, there is no doubt that the concerted work abandonment/boycott was the result of a labor dispute.

The right to strike, while constitutionally recognized, is not without legal constrictions.46 Article 264 (a) of the Labor Code, as amended, provides: Art. 264. Prohibited activities. – (a) x x x No strike or lockout shall be declared after assumption of jurisdiction by the President or the Secretary or after certification or submission of the dispute to compulsory or voluntary arbitration or during the pendency of cases involving the same grounds for the strike or lockout. The Court has consistently ruled that once the Secretary of Labor assumes jurisdiction over a labor dispute, such jurisdiction should not be interfered with by the application of the coercive processes of a strike or lockout.47 A strike that is undertaken despite the issuance by the Secretary of Labor of an assumption order and/or certification is a prohibited activity and thus illegal.48 Article 264 (a) of the Labor Code, as amended, also considers it a prohibited activity to declare a strike "during the pendency of cases involving the same grounds for the same strike."49 There is no dispute that when respondents conducted their mass actions on April 3 to 6, 2000, the proceedings before the Secretary of Labor were still pending as both parties filed motions for reconsideration of the March 24, 2000 Order. Clearly, respondents knowingly violated the aforesaid provision by holding a strike in the guise of mass demonstration simultaneous with concerted work abandonment/boycott.

II. The court also ruled that notwithstanding the illegality of the strike, they cannot sanction petitioner’s act of indiscriminately terminating the services of individual respondents who admitted joining the mass actions and who have refused to comply with the offer of the management to report back to work.

The court reiterates the distinction between the liabilities of a union officer and a union member when they participate in an illegal strike, thus:

Art. 264. Prohibited activities.— x x x xxxx Any worker whose employment has been terminated as a consequence of an unlawful lockout shall be entitled to reinstatement with full back wages. Any union officer who knowingly participates in an illegal strike and any worker or union officer who knowingly participates in the commission of illegal acts during a strike may be declared to have lost his employment status: Provided, That mere participation of a worker in a lawful strike shall not constitute sufficient ground for termination of his employment, even if a replacement had been hired by the employer during such lawful strike. xxxx The foregoing shows that the law makes a distinction between union officers and members. For knowingly participating in an illegal strike or participating in the commission of illegal acts during a strike, the law provides that a union officer may be terminated from employment. The law grants the employer the option of declaring a union officer who participated in an illegal strike as having lost his employment. It possesses the right and prerogative to terminate the union officers from service.50 However, a worker merely participating in an illegal strike may not be terminated from employment. It is only when he commits illegal acts during a strike that he may be declared to have lost employment status.51 We have held that the responsibility of union officers, as main players in an illegal strike, is greater than that of the members and, therefore, limiting the penalty of dismissal only for the former for participation in an illegal strike is in order.52Hence, with respect to respondents who are union officers, the validity of their termination by petitioners cannot be questioned. Being fully aware that the proceedings

123

before the Secretary of Labor were still pending as in fact they filed a motion for reconsideration of the March 24, 2000 Order, they cannot invoke good faith as a defense.

FADRIQUELAN ET AL VS MONTEREY FOOD CORP., G.R. 178409, JUNE 08, 2011 Facts: On April 30, 2002 the three-year collective bargaining agreement or CBA between the union Bukluran ng Manggagawa sa Monterey-Ilaw at Buklod ng Manggagawa (the union) and Monterey Foods Corporation (the company) expired. On March 28, 2003 after the negotiation for a new CBA reached a deadlock, the union filed a notice of strike with the National Conciliation and Mediation Board (NCMB). To head off the strike, on April 30, 2003 the company filed with the DOLE a petition for assumption of jurisdiction over the dispute in view of its dire effects on the meat industry. In an Order dated May 12, 2003, the DOLE Secretary assumed jurisdiction over the dispute and enjoined the union from holding any strike. It also directed the union and the company to desist from taking any action that may aggravate the situation. On May 21, 2003 the union filed a second notice of strike before the NCMB on the alleged ground that the company committed unfair labor practices. On June 10, 2003 the company sent notices to the union officers, charging them with intentional acts of slowdown. Six days later or on June 16 the company sent new notices to the union officers, informing them of their termination from work for defying the DOLE Secretary’s assumption order. Issues: 1. Whether or not the CA erred in holding that slowdowns actually transpired at the company’s farms; and 2. Whether or not the CA erred in holding that union officers committed illegal acts that warranted their dismissal from work. Held: First. The law is explicit: no strike shall be declared after the Secretary of Labor has assumed jurisdiction over a labor dispute. A strike conducted after such assumption is illegal and any union officer who knowingly participates in the same may be declared as having lost his employment. Here, what is involved is a slowdown strike. Unlike other forms of strike, the employees involved in a slowdown do not walk out of their jobs to hurt the company. They need only to stop work or reduce the rate of their work while generally remaining in their assigned post. The union of course argues that it merely held assemblies to inform members of the developments in the CBA negotiation, not protest demonstrations over it. But as the CA correctly observed, if the meetings had really been for the stated reason, why did the union officers and members from separate company farms choose to start and end their meetings at the same time and on the same day? And if they did not intend a slowdown, why did they not hold their meetings after work. Second. A distinction exists, however, between the ordinary workers’ liability for illegal strike and that of the union officers who participated in it. The ordinary worker cannot be terminated for merely participating in the strike. There must be proof that he committed illegal acts during its conduct. On the other hand, a union officer can be terminated upon mere proof that he knowingly participated in the illegal strike. In termination cases, the dismissed employee is not required to prove his innocence of the charges against him. The burden of proof rests upon the employer to show that the employee’s dismissal was for just cause. The employer’s failure to do so means that the dismissal was not justified. Here, the company failed to show that all 17 union officers deserved to be dismissed.

MIGUEL DELA BARAIRO, PENA PETITIONER, VS. OFFICE OF THE PRESIDENT AND MST MARINE SERVICES (PHILS,), INC. RESPONDENT, G.R. NO. 189314, JUNE 15, 2011 Facts: Miguel Barairo (petitioner) was hired by respondent MST Marine Services (Phils.) Inc., (MST) for its principal, TSM International, Ltd., as Chief Mate of the vessel Maritina, for a contract period of six months. He boarded

124

the vessel and discharged his duties on July 23, 2004 but was relieved on August 28, 2004 ostensibly for transfer to another vessel, Solar. Petitioner thus disembarked in Manila on August 29. 2004.

Petitioner was later to claim that he was not paid the promised "stand-by fee" in lieu of salary that he was to receive while awaiting transfer to another vessel as in fact the transfer never materialized. On October 20, 2004, petitioner signed a new Contract of Employment for a six-month deployment as Chief Mate of M/T Haruna vessel. He was paid a 1-month “stand-by fee” in connection with the Maritina contract. Petitioner boarded the M/T Haruna on October 31, 2004 then he disembarked a week later as MST claimed that his boarding of M/T Haruna was a "sea trial" which, MST maintains, was priorly made known to him on a "stand-by fee”. MST soon informed petitioner that he would be redeployed to the M/T Haruna on November 30, 2004, but petitioner refused, prompting MST to file a complaint for breach of contract against him before the Philippine Overseas Employment Administration (POEA). Petitioner claimed, however, that he was placed on "forced vacation” when he was made to disembark from the M/T Haruna, and that not wanting to experience a repetition of the previous "termination" of his employment aboard the Maritina, he refused to be redeployed to the M/T Haruna. However, the petitioner was found guilty of the said subject matter of the complaint thus was penalized by the POEA Administrator with one year suspension from overseas deployment. Petitioner appealed said order of the POEA to the Secretary of Labor, who noting that it was petitioner's first offense, modified the POEA Order by shortening the period of suspension from one year to six months. Petitioner then appealed the order to the Office of the President (OP), who in turn dismissed the case for on the ground of lack of jurisdiction.

Issue: Whether or not the OP’s dismissal of the case was proper. Ruling: It was proper because for this case the appropriate remedy to question the decisions or orders of the Secretary of Labor is via Petition for Certiorari under Rule 65 not via an appeal to the OP. For appeals to the OP in labor cases have indeed been eliminated, except those involving national interest over which the President may assume jurisdiction. The rationale behind this development is grounded on the Doctrine of Qualified Political Agency. Also, the Petitioner's appeal of the Secretary of Labor's Decision to the Office of the President did not toll the running of the period, hence, the assailed Decisions of the Secretary of Labor are deemed to have attained finality. The Court concurred with the POEA Administrator and the Secretary of Labor that petitioner's refusal to board the M/T Haruna constituted unjustified breach of his contract of employment. That petitioner believed that respondent company violated his rights when the period of his earlier Maritina contract was not followed and his "stand-by fees" were not fully paid did not justify his refusal to abide by the valid and existing Haruna contract requiring him to serve aboard M/T Haruna. For, as noted in the assailed DOLE Order, "if petitioner's rights have been violated as he claims, he has various remedies under the contract which he did not avail of. Parenthetically, the Undersecretary of Labor declared that the real reason petitioner refused to rejoin Haruna is that he left the Philippines on to join MT Adriatiki, a vessel of another maiming agency, which declaration petitioner has not refuted.

MAGDALA MULTIPURPOSE & LIVELIHOOD COOPERATIVE VS. KILUSANG MANGGAGAWA NG LGS ET AL, G.R. NOS. 191138-39, OCTOBER 19, 2011 Facts: KMLMS filed a notice of strike on March 5, 2002 and conducted its strike-vote on April 8, 2002. However, KMLMS only acquired legal personality when its registration as an independent labor organization was granted on April 9, 2002 by the Department of Labor and Employment.

125

On April 19, 2002, it became officially affiliated as a local chapter of the Pambansang Kaisahan ng Manggagawang Pilipino when its application was granted by the Bureau of Labor Relations. Thereafter, on May 6, 2002, KMLMS — now a legitimate labor organization (LLO) — staged a strike where several prohibited and illegal acts were committed by its participating members. On the ground of lack of valid notice of strike, ineffective conduct of a strike-vote and commission of prohibited and illegal acts, petitioners filed their Petition to Declare the Strike of May 6, 2002 Illegal before the NLRC Regional Arbitration Board (RAB) No. IV in Quezon City.

Ruling of Labor Arbiter The May 6, 2002 strike is illegal and 41 workers are declared to have lost their employment.

Ruling of the NLRC Affirmed the Labor Arbiter’s decision but declared additional seven workers to have lost their employment

Ruling of the CA Affirmed the decision of the NLRC.

Issues: (1) WON the strike was illegal (2) WON the union members are validly terminated from their employment.

Ruling: (1) There is no question that the May 6, 2002 strike was illegal, first, because when KMLMS filed the notice of strike on March 5 or 14, 2002, it had not yet acquired legal personality and, thus, could not legally represent the eventual union and its members. And second, similarly when KMLMS conducted the strikevote on April 8, 2002, there was still no union to speak of, since KMLMS only acquired legal personality as an independent LLO only on April 9, 2002 or the day after it conducted the strike-vote. These factual findings are undisputed and borne out by the records.

Consequently, the mandatory notice of strike and the conduct of the strike-vote report were ineffective for having been filed and conducted before KMLMS acquired legal personality as an LLO, violating Art. 263 (c), (d) and (f) of the Labor Code and Rule XXII, Book V of the Omnibus Rules Implementing the Labor Code.

Art. 263(c): Controversy

What to File

Who may File

Where File

Bargaining Deadlock

Notice of Strike

Duly Certified or Recognized Bargaining Agent

Ministry

30 days before the intended date of strike

Employer

Ministry

30 days before the intended lockout

Duly Certified or Recognized Bargaining Agent

Ministry

15 days before the intended date of strike

Notice Lockout Unfair Practice

Labor

Notice of Strike

of

In absence of the above: Any Legitimate Labor behalf of its members

to

Period of Notice

in

126

On the other hand, Rule XXII, Book V of the Omnibus Rules Implementing the Labor Code likewise pertinently provides: RULE XXII CONCILIATION, STRIKES AND LOCKOUTS xxx xxx xxx SEC. 6.Who may declare a strike or lockout. — Any certified or duly recognized bargaining representative may declare a strike in cases of bargaining deadlocks and unfair labor practices. The employer may declare a lockout in the same cases. In the absence of a certified or duly recognized bargaining representative, any legitimate labor organization in the establishment may declare a strike but only on grounds of unfair labor practice.

It is, thus, clear that the filing of the notice of strike and the conduct of the strike-vote by KMLMS did not comply with the aforequoted mandatory requirements of law and its implementing rules. Consequently, the May 6, 2002 strike is illegal.

(2) There is likewise no dispute that when the May 6, 2002 illegal strike was conducted, the members of respondent KMLMS committed prohibited and illegal acts which doubly constituted the strike illegal.

Art. 264. Prohibited Acts: (a) Any union officer who knowingly participates in an illegal strike and any worker or union officer who knowingly participates in the commission of illegal acts during a strike may be declared to have lost his employment status: Provided, That mere participation of a worker in a lawful strike shall not constitute sufficient ground for termination of his employment, even if a replacement had been hired by the employer during such lawful strike. (b) No person engaged in picketing shall commit any act of violence, coercion or intimidation or obstruct the free ingress to or egress from the employer's premises for lawful purposes, or obstruct public thoroughfares.

Here, the striking workers committed acts of (1) interference by obstructing the free ingress to or egress from petitioners' compound and (2) coercion and intimidation.

Art. 264 of the Code present a substantial distinction of the consequences of an illegal strike between union officers and mere members of the union: Persons Involved

Ground for Termination

Union Officers

Knowingly participating in an illegal strike

Union Members

Committed prohibited and illegal acts during the strike and there is substantial evidence or proof of their participation (mere participation is not a ground)

A total of 34 union members have been declared to have lost their employment due to their commission of prohibited and illegal acts during the illegal strike of May 6, 2002, 27 of which are those from the judgment of the Labor Arbiter.

72 additional union members are however found by the court to have committed illegal acts to which petitioners have substantially proven their identity. There was patent misappreciation of evidence both by the LA and the NLRC, and was not corrected by the CA, in having a view that there is no substantial proof of the identity of the other 72 striking union members who committed prohibited and illegal activities.

127

ABERIA ET AL., VS. NATIONAL LABOR RELATION COMMISSION ET., [G.R. NOS. 154113, 187778, 187861, AND 196156 DECEMBER 7, 2011] Facts:

The consolidated cases involve the legality of mass termination of hospital employees who participated in strike and picketing activities.

Metro Cebu Community Hospital, Inc. (MCCHI), presently known as the Visayas Community Medical Center (VCMC), is a non-stock, non-profit corporation organized under the laws of the Republic of the Philippines. It operates the Metro Cebu Community Hospital (MCCH), a tertiary medical institution located at Osmeña Boulevard, Cebu City. MCCH is owned by the United Church of Christ in the Philippines (UCCP) and Rev. Gregorio P. Iyoy is the Hospital Administrator.The National Federation of Labor (NFL) is the exclusive bargaining representative of the rank-and-file employees of MCCHI. Under the previous Collective Bargaining Agreements (CBAs), the signatories were Ciriaco B. Pongasi, Sr. for MCCHI, and Atty. Armando M. Alforque (NFL Legal Counsel) and Paterno A. Lumapguid as President of NFL-MCCH Chapter. In the CBA effective from January 1994 until December 31, 1995, the signatories were Sheila E. Buot as Board of Trustees Chairman, Rev. Iyoy as MCCH Administrator and Atty. Fernando Yu as Legal Counsel of NFL, while Perla Nava, President of Nagkahiusang Mamumuo sa MCCH (NAMA-MCCH-NFL) signed the Proof of Posting. Nava wrote Rev. Iyoy expressing the union’s desire to renew the CBA, attaching to her letter a statement of proposals signed/endorsed by 153 union members. Nava subsequently requested that the following employees be allowed to avail of one-day union leave with pay. However, MCCHI returned the CBA proposal for Nava to secure first the endorsement of the legal counsel of NFL as the official bargaining representative of MCCHI employees.

Meanwhile, Atty. Alforque informed MCCHI that the proposed CBA submitted by Nava was never referred to NFL and that NFL has not authorized any other legal counsel or any person for collective bargaining negotiations. The collection of union fees (check-off) was temporarily suspended by MCCHI in view of the existing conflict between the federation and its local affiliate. Thereafter, MCCHI attempted to take over the room being used as union office but was prevented to do so by Nava and her group who protested these actions and insisted that management directly negotiate with them for a new CBA. MCCHI referred the matter to Atty. Alforque, NFL’s Regional Director, and advised Nava that their group is not recognized by NFL. The next day, several union members led by Nava and her group launched a series of mass actions such as wearing black and red armbands/headbands, marching around the hospital premises and putting up placards, posters and streamers. Atty. Alforque immediately disowned the concerted activities being carried out by union members which are not sanctioned by NFL. MCCHI directed the union officers led by Nava to submit within 48 hours a written explanation why they should not be terminated for having engaged in illegal concerted activities amounting to strike, and placed them under immediate preventive suspension. Responding to this directive, Nava and her group denied there was a temporary stoppage of work, explaining that employees wore their armbands only as a sign of protest and reiterating their demand for MCCHI to comply with its duty to bargain collectively. Rev. Iyoy, having been informed that Nava and her group have also been suspended by NFL, directed said officers to appear before his office for investigation in connection with the illegal strike wherein they reportedly uttered slanderous and scurrilous words against the officers of the hospital, threatening other workers and forcing them to join the strike. Said union officers, however, invoked the grievance procedure provided in the CBA to settle the dispute between management and the union.

The Department of Labor and Employment (DOLE) Regional Office No. 7 issued certifications stating that there is nothing in their records which shows that NAMA-MCCH-NFL is a registered labor organization, and that said union submitted only a copy of its Charter Certificate. MCCHI then sent individual notices to all union members asking them to submit within 72 hours a written explanation why they should not be terminated for having supported the illegal concerted activities of NAMA-MCCH-NFL which has no legal personality as per DOLE records. Thereafter, several complaints for illegal dismissal and unfair labor practice were filed by the terminated employees against MCCHI, Rev. Iyoy, UCCP and members of the Board of Trustees of MCCHI.

128

Issues: 1. Whether or not declaring private respondents MCCHI guilty of unfair labor practice and union busting.

2. Whether or not the union officers and members were guilty of illegal strike and picketing activities.

Held: 1. MCCHI not guilty of unfair labor practice Art. 248 (g) of the Labor Code, as amended, makes it an unfair labor practice for an employer "[t]o violate the duty to bargain collectively" as prescribed by the Code. The applicable provision in this case is Art. 253 which provides: ART. 253. Duty to bargain collectively when there exists a collective bargaining agreement. – When there is a collective bargaining agreement, the duty to bargain collectively shall also mean that neither party shall terminate nor modify such agreement during its lifetime. However, either party can serve a written notice to terminate or modify the agreement at least sixty (60) days prior to its expiration date. It shall be the duty of both parties to keep the status quo and to continue in full force and effect the terms and conditions of the existing agreement during the 60-day period and/or until a new agreement is reached by the parties. NAMA-MCCH-NFL charged MCCHI with refusal to bargain collectively when the latter refused to meet and convene for purposes of collective bargaining, or at least give a counter-proposal to the proposed CBA the union had submitted and which was ratified by a majority of the union membership. MCCHI, on its part, deferred any negotiations until the local union’s dispute with the national union federation (NFL) is resolved considering that the latter is the exclusive bargaining agent which represented the rank-and-file hospital employees in CBA negotiations since 1987. Records of the NCMB and DOLE Region 7 confirmed that NAMAMCCH-NFL had not registered as a labor organization, having submitted only its charter certificate as an affiliate or local chapter of NFL. Not being a legitimate labor organization, NAMA-MCCH-NFL is not entitled to those rights granted to a legitimate labor organization under Art. 242, specifically: (a) To act as the representative of its members for the purpose of collective bargaining; (b) To be certified as the exclusive representative of all the employees in an appropriate collective bargaining unit for purposes of collective bargaining; 2. NAMA-MCCH-NFL and its union officers and members are guilty of illegal strike and picketing. Art. 263 (b) of the Labor Code, as amended, provides: ART. 263. Strikes, picketing and lockouts. – x x x (b) Workers shall have the right to engage in concerted activities for purposes of collective bargaining or for their mutual benefit and protection. The right of legitimate labor organizations to strike and picket and of employers to lockout, consistent with the national interest, shall continue to be recognized and respected. However, no labor union may strike and no employer may declare a lockout on grounds involving interunion and intra-union disputes. As borne by the records, NAMA-MCCH-NFL was not a duly registered or an independently registered union at the time it filed the notice of strike and when it conducted the strike. It could not then legally represent the union members. Consequently, the mandatory notice of strike and the conduct of the strike vote report were ineffective for having been filed and conducted by NAMA-MCCH-NFL which has no legal personality as a legitimate labor organization, in violation of Art. 263 (c), (d) and (f) of the Labor Code and Rule XXII, Book V of the Omnibus Rules Implementing the Labor Code. The findings of the Executive Labor Arbiter and NLRC, as sustained by the appellate court, clearly established that the striking union members created so much noise, disturbance and obstruction that the local government authorities eventually ordered their removal for being a public nuisance. This was followed by an injunction from the NCMB enjoining the union leaders from further blocking the free ingress to and egress from the hospital, and from committing threats, coercion and intimidation against non-striking employees and patients/vehicles desiring to enter for the purpose of seeking medical treatment/confinement. By then, the illegal strike had lasted for almost five months. The Supreme Court rule for MCCHI.

PICOP RESOURCES, INCORPORATED VS DEQUILLA, ET. AL. ; GR NO 172666; DECEMBER 7, 2011

129

FACTS

Ricardo Dequilla, Cesar Atienza and Aniceto Orbeta (private respondents) were regular rank-and-file employees of Picop Resources, Inc. (PICOP) and members of the NAMAPRI-SPFL, a duly registered labor organization and existing bargaining agent of the PICOP rank-and-file employees. PICOP and NAMAPRI-SPFL had a collective bargaining agreement (CBA) which would expire on May 22, 2000.

On May 16, 2000, the late Atty. Proculo P. Fuentes, Jr. (Atty. Fuentes), then National President of the Southern Philippines Federation of Labor (SPFL), advised the PICOP management to terminate about 800 employees due to acts of disloyalty, specifically, for allegedly campaigning, supporting and signing a petition for the certification of a rival union, the Federation of Free Workers Union (FFW) before the 60-day "freedom period" and during the effectivity of the CBA. Such acts of disloyalty were construed to be a valid cause for termination under the terms and conditions of the CBA. Based on the CBA, the freedom period would start on March 22, 2000.

The employees were given an opportunity to be heard and subsequently on October 16, 2000, PICOP served a notice of termination due to acts of disloyalty to 31 employees including herein private respondents. Thus, they filed a complaint for Unfair Labor Practice and Illegal Dismissal with money claims, damages and attorney’s fees.

ISSUE

Whether an existing CBA can be given full force and effect in its terms and conditions, including its union security clause, even beyond the five year period when no new CBA has been entered into.

RULING

The petition merits a denial.

There is no question that in the CBA entered into by the parties, there is a union security clause. The clause imposes upon the workers the obligation to join and maintain membership in the company’s recognized union as a condition for employment. There is no dispute that private respondents were members of NAMAPRI-SPFL who were terminated by PICOP due to alleged acts of disloyalty. It is basic in labor jurisprudence that the burden of proof rests upon management to show that the dismissal of its worker was based on a just cause. When an employer exercises its power to terminate an employee by enforcing the union security clause, it needs to determine and prove the following: (1) the union security clause is applicable; (2) the union is requesting for the enforcement of the union security provision in the CBA; and (3) there is sufficient evidence to support the decision of the union to expel the employee from the union. In this case, the resolution thereof hinges on whether PICOP was able to show sufficient evidence to support the decision of the union to expel private respondents from it. PICOP basically contends that private respondents were justly terminated from employment for campaigning, supporting and signing a petition for the certification of FFW, a rival union, before the 60-day "freedom period" and during the effectivity of the CBA. Their acts constitute an act of disloyalty against the union which is valid cause for termination pursuant to the Union Security Clause in the CBA. The Court finds Itself unable to agree. PICOP failed to show in detail how private respondents campaigned and supported FFW. Their mere act of signing an authorization for a petition for certification election before the freedom period does not necessarily demonstrate union disloyalty. It is far from being within the definition of "acts of disloyalty.” The act of "signing an authorization for a petition for certification election" is not disloyalty to the union per se considering that the petition for certification election itself was filed during the freedom period which started on March 22, 2000. Moreover, as correctly ruled by the CA, the records are bereft of proof of any contemporaneous acts of resignation or withdrawal of union membership or non-payment of union dues.

130

Neither is there proof that private respondents joined FFW. The fact is, private respondents remained in good standing with their union, NAMAPRI-SPFL.

We are constrained to believe that an "authorization letter to file a petition for certification election" is different from an actual "Petition for Certification Election." Likewise, as per records, it was clear that the actual Petition for Certification Election of FFW was filed only on May 18, 2000. Thus, it was within the ambit of the freedom period which commenced from March 21, 2000 until May 21, 2000. Strictly speaking, what is prohibited is the filing of a petition for certification election outside the 60-day freedom period. This is not the situation in this case. If at all, the signing of the authorization to file a certification election was merely preparatory to the filing of the petition for certification election, or an exercise of respondents’ right to selforganization.

Moreover, PRI anchored their decision to terminate respondents’ employment on Article 253 of the Labor Code which states that "it shall be the duty of both parties to keep the status quo and to continue in full force and effect the terms and conditions of the existing agreement during the 60-day period and/or until a new agreement is reached by the parties." It claimed that they are still bound by the Union Security Clause of the CBA even after the expiration of the CBA; hence, the need to terminate the employment of respondents. Petitioner's reliance on Article 253 is misplaced. Following Article 256, at the expiration of the freedom period, PRI's obligation to recognize NAMAPRI-SPFL as the incumbent bargaining agent does not hold true when petitions for certification election were filed, as in this case.

Moreover, the last sentence of Article 253 which provides for automatic renewal pertains only to the economic provisions of the CBA, and does not include representational aspect of the CBA. An existing CBA cannot constitute a bar to a filing of a petition for certification election. When there is a representational issue, the status quo provision in so far as the need to await the creation of a new agreement will not apply. Otherwise, it will create an absurd situation where the union members will be forced to maintain membership by virtue of the union security clause existing under the CBA and, thereafter, support another union when filing a petition for certification election. If we apply it, there will always be an issue of disloyalty whenever the employees exercise their right to self-organization. The holding of a certification election is a statutory policy that should not be circumvented, or compromised.

C.ALCANTARA & SONS, INC., PETITIONER, VS. COURT OF APPEALS, G.R. NO. 155109, SEPTEMBER 29, 2010, NAGKAHIUSANG MAMUMUO SA ALSONS-SPFL (NAMAAL-SPFL), AND ITS MEMBERS WHOSE NAMES ARE LISTED BELOW, PETITIONERS, VS. C. ALCANTARA & SONS, INC., RESPONDENT. G.R. NO. 179220 This case is about a) the consequences of an illegally staged strike upon the employment status of the union officers and its ordinary members and b) the right of reinstated union members to go back to work pending the company’s appeal from the order reinstating them.

Facts: C. Alcantara & Sons, Inc., (the Company) is a domestic corporation engaged in the manufacture and processing of plywood. Nagkahiusang Mamumuo sa Alsons-SPFL (the Union) is the exclusive bargaining agent of the Company’s rank and file employees. The other parties to these cases are the Union officers 1 and their striking members.2

The Company and the Union entered into a Collective Bargaining Agreement (CBA) that bound them to hold no strike and no lockout in the course of its life. At some point the parties began negotiating the economic provisions of their CBA but this ended in a deadlock, prompting the Union to file a notice of strike. After efforts at conciliation by the Department of Labor and Employment (DOLE) failed, the Union conducted a strike vote that resulted in an overwhelming majority of its members favoring it. The Union reported the strike vote to the DOLE and, after the observance of the mandatory cooling-off period, went on strike.

131

During the strike, the Company filed a petition for the issuance of a writ of preliminary injunction with prayer for the issuance of a temporary restraining order (TRO) Ex Parte3 with the National Labor Relations Commission (NLRC) to enjoin the strikers from intimidating, threatening, molesting, and impeding by barricade the entry of non-striking employees at the Company’s premises. The NLRC first issued a 20-day TRO and, after hearing, a writ of preliminary injunction, enjoining the Union and its officers and members from performing the acts complained of. But several attempts to implement the writ failed. Only the intervention of law enforcement units made such implementation possible. Meantime, the Union filed a petition4 with the Court of Appeals (CA), questioning the preliminary injunction order. On February 8, 1999 the latter court dismissed the petition. The Union did not appeal from such dismissal.

On June 29, 1999 the Labor Arbiter rendered a decision, 6 declaring the Union’s strike illegal for violating the CBA’s no strike, no lockout, provision. As a consequence, the Labor Arbiter held that the Union officers should be deemed to have forfeited their employment with the Company and that they should pay actual damages of P3,825,000.00 plus 10% interest and attorney’s fees. With respect to the striking Union members, finding no proof that they actually committed illegal acts during the strike, the Labor Arbiter ordered their reinstatement without backwages. The Labor Arbiter denied the Union’s counterclaim for lack of merit.

On November 8, 1999 the NLRC rendered a decision, 8 affirming that of the Labor Arbiter insofar as the latter declared the strike illegal, ordered the Union officers terminated, and directed them to pay damages to the Company. The NLRC ruled, however, that the Union members involved, who were identified in the proceedings held in the case, should also be terminated for having committed prohibited and illegal acts.

The Union filed a petition for certiorari 9 with the CA, questioning the NLRC decision. Finding merit in the petition, the CA rendered a decision on March 20, 2002, 10 annulling the NLRC decision and reinstating that of the Labor Arbiter. The Company and the Union with its officers and members filed separate petitions for review of the CA decision.

On November 8, 1999 the NLRC rendered a decision, 8 affirming that of the Labor Arbiter insofar as the latter declared the strike illegal, ordered the Union officers terminated, and directed them to pay damages to the Company. The NLRC ruled, however, that the Union members involved, who were identified in the proceedings held in the case, should also be terminated for having committed prohibited and illegal acts. The Union filed a petition for certiorari with the CA, questioning the NLRC decision. Finding merit in the petition, the CA rendered a decision on March 20, 2002, 10 annulling the NLRC decision and reinstating that of the Labor Arbiter. The Company and the Union with its officers and members filed separate petitions for review of the CA decision in G.R. 155109 and 155135, respectively.

Issue:

Whether or not the Union staged an illegal strike; Ruling: A strike may be regarded as invalid although the labor union has complied with the strict requirements for staging one as provided in Article 263 of the Labor Code when the same is held contrary to an existing agreement, such as a no strike clause or conclusive arbitration clause. 19 Here, the CBA between the parties contained a "no strike, no lockout" provision that enjoined both the Union and the Company from resorting to the use of economic weapons available to them under the law and to instead take recourse to voluntary arbitration in settling their disputes. No law or public policy prohibits the Union and the Company from mutually waiving the strike and lockout maces available to them to give way to voluntary arbitration. Indeed, no less than the 1987 Constitution recognizes in Section 3, Article XIII, preferential use of voluntary means to settle disputes. Thus – The State shall promote the principle of shared responsibility between workers and employers and the preferential use of voluntary modes in settling disputes, including conciliation, and shall enforce their mutual compliance therewith to foster industrial peace.

132

The Court finds no compelling reason to depart from the findings of the Labor Arbiter, the NLRC, and the CA regarding the illegality of the strike. Social justice is not one-sided. It cannot be used as a badge for not complying with a lawful agreement.

A strike may be regarded as invalid although the labor union has complied with the strict requirements for staging one as provided in Article 263 of the Labor Code when the same is held contrary to an existing agreement, such as a no strike clause or conclusive arbitration clause. 19 Here, the CBA between the parties contained a "no strike, no lockout" provision that enjoined both the Union and the Company from resorting to the use of economic weapons available to them under the law and to instead take recourse to voluntary arbitration in settling their disputes. No law or public policy prohibits the Union and the Company from mutually waiving the strike and lockout maces available to them to give way to voluntary arbitration. Indeed, no less than the 1987 Constitution recognizes in Section 3, Article XIII, preferential use of voluntary means to settle disputes. Thus – The State shall promote the principle of shared responsibility between workers and employers and the preferential use of voluntary modes in settling disputes, including conciliation, and shall enforce their mutual compliance therewith to foster industrial peace. The Court finds no compelling reason to depart from the findings of the Labor Arbiter, the NLRC, and the CA regarding the illegality of the strike. Social justice is not one-sided. It cannot be used as a badge for not complying with a lawful agreement.

Since the Union’s strike has been declared illegal, the Union officers can, in accordance with law be terminated from employment for their actions. This includes the shop stewards. They cannot be shielded from the coverage of Article 264 of the Labor Code since the Union appointed them as such and placed them in positions of leadership and power over the men in their respective work units. As regards the rank and file Union members, Article 264 of the Labor Code provides that termination from employment is not warranted by the mere fact that a union member has taken part in an illegal strike. It must be shown that such a union member, clearly identified, performed an illegal act or acts during the strike.

DIGITAL TELECOMMUNICATIONS PHILS INC. vs. DIGITEL EMPLOYEES UNION ET AL. [G.R. No. 184903-04, October 10, 2012]

Facts: By virtue of a certification election, Digitel Employees Union (Union) became the exclusive bargaining agent of all rank and file employees of Digitel in 1994. The Union and Digitel then commenced collective bargaining negotiations which resulted in a bargaining deadlock. The Union threatened to go on strike, but then Acting Labor Secretary Bienvenido E. Laguesma assumed jurisdiction over the dispute and eventually directed the parties to execute a CBA. However, no CBA was forged between Digitel and the Union. Some Union members abandoned their employment with Digitel. The Union later became dormant. Ten (10) years thereafter or on 28 September 2004, Digitel received from Arceo Rafael A. Esplana (Esplana), who identified himself as President of the Union, a letter containing the list of officers, CBA proposals and ground rules. 3 The officers were respondents Esplana, Alan D. Licando (Vice-President), Felicito C. Romero, Jr. (Secretary), Arnold D. Gonzales (Treasurer), Reynel Francisco B. Garcia (Auditor), Zosimo B. Peralta (PRO), Regino T. Unidad (Sgt. at Arms), and Jim L. Javier (Sgt. at Arms). Digitel was reluctant to negotiate with the Union and demanded that the latter show compliance with the provisions of the Union's Constitution and By-laws on union membership and election of officers. On 4 November 2004, Esplana and his group filed a case for Preventive Mediation before the National Conciliation and Mediation Board based on Digitel's violation of the duty to bargain. On 25 November 2004, Esplana filed a notice of strike. On 10 March 2005, then Labor Secretary Patricia A. Sto. Tomas issued an Order 4 assuming jurisdiction over the labor dispute.

133

During the pendency of the controversy, Digitel Service, Inc. (Digiserv), a non-profit enterprise engaged in call center servicing, filed with the Department of Labor and Employment (DOLE) an Establishment Termination Report stating that it will cease its business operation. The closure affected at least 100 employees, 42 of whom are members of the herein respondent Union. Alleging that the affected employees are its members and in reaction to Digiserv's action, Esplana and his group filed another Notice of Strike for union busting, illegal lock-out, and violation of the assumption order. On 23 May 2005, the Secretary of Labor ordered the second notice of strike subsumed by the previous Assumption Order. Meanwhile, on 14 March 2005, Digitel filed a petition with the Bureau of Labor Relations (BLR) seeking cancellation of the Union's registration on the following grounds: 1) failure to file the required reports from 1994-2004; 2) misrepresentation of its alleged officers; 3) membership of the Union is composed of rank and file, supervisory and managerial employees; and 4) substantial number of union members are not Digitel employees.

Issue: Whether the Secretary of Labor erred in issuing the assumption order despite the pendency of the petition for cancellation of union registration. Ruling: The pendency of a petition for cancellation of union registration does not preclude collective bargaining. The 2005 case of Capitol Medical Center, Inc. v. Hon. Trajano 13 is apropos. The respondent union therein sent a letter to petitioner requesting a negotiation of their CBA. Petitioner refused to bargain and instead filed a petition for cancellation of the union's certificate of registration. Petitioner's refusal to bargain forced the union to file a notice of strike. They eventually staged a strike. The Secretary of Labor assumed jurisdiction over the labor dispute and ordered all striking workers to return to work. Petitioner challenged said order by contending that its petition for cancellation of union's certificate of registration involves a prejudicial question that should first be settled before the Secretary of Labor could order the parties to bargain collectively. When the case eventually reached this Court, we agreed with the Secretary of Labor that the pendency of a petition for cancellation of union registration does not preclude collective bargaining, thus: SDaHEc That there is a pending cancellation proceeding against the respondent Union is not a bar to set in motion the mechanics of collective bargaining. If a certification election may still be ordered despite the pendency of a petition to cancel the union's registration certificate (National Union of Bank Employees vs. Minister of Labor, 110 SCRA 274), more so should the collective bargaining process continue despite its pendency. We must emphasize that the majority status of the respondent Union is not affected by the pendency of the Petition for Cancellation pending against it. Unless its certificate of registration and its status as the certified bargaining agent are revoked, the Hospital is, by express provision of the law, duty bound to collectively bargain with the Union. 14 Trajano was reiterated in Legend International Resorts Limited v. Kilusang Manggagawa ng Legenda (KMLIndependent). 15 Legend International Resorts reiterated the rationale for allowing the continuation of either a CBA process or a certification election even during the pendency of proceedings for the cancellation of the union's certificate of registration. Citing the cases of Association of Court of Appeals Employees v. Ferrer-Calleja 16 and Samahan ng Manggagawa sa Pacific Plastic v. Hon. Laguesma, 17 it was pointed out at the time of the filing of the petition for certification election — or a CBA process as in the instant case — the union still had the personality to file a petition for certification — or to ask for a CBA negotiation — as in the present case.

134

AUTOMOTIVE ENGINE REBUILDERS vs. PROGRESIBONG UNYON NG MGA MANGGAGAWA SA AER [G.R. No. 160138, January 16, 2013 & July 13, 2011 Main Decision]

Facts: On December 22, 1998, Unyon filed a petition for certification election before the Department of Labor and Employment(DOLE) after organizing their employees union within AER. Resenting what they did, AER forced all of its employees to submit their urine samples for drug testing. Those who refused were threatened with dismissal. On January 8, 1999, the results of the drug test came out and some employees were found positive for illegal drugs and was suspended for coming to work under the influence of intoxicating liquor or any drug or drinking any alcoholic beverages on the premises on company time. On January 28, 1999, eighteen (18) employees of AER, acting collectively and in concert, suddenly and without reason staged a walkout and assembled illegally in the company premises. Despite management’s plea for them to go back to work, the concerned employees refused and, instead, walked out of the company premises and proceeded to the office of the AER Performance and Service Center (AER-PSC) located on another street. They threatened and forced the company guards and some company officers and personnel to open the gate of the AER-PSC compound. They also urged the AER-PSC employees to likewise stop working. On February 2, 1999, the affected workers were denied entry into the AER premises by order of management. Because of this, the affected workers staged a picket in front of company premises hoping that management would accept them back to work. When their picket proved futile, they filed a complaint for unfair labor practice, illegal suspension and illegal dismissal. On February 22, 1999, the concerned employees started a wildcat strike, barricaded company premises, and prevented the free ingress and egress of the other employees, officers, clients, and visitors and the transportation of company equipments. They also tried to use force and inflict violence against the other employees. Their wildcat strike stopped after the NLRC issued and served a temporary restraining order (TRO). Issue: WON the CA erred in ruling for the reinstatement of the complaining employees but without grant of backwages. Ruling: The in pari delicto doctrine in labor cases is not novel to us. It has been applied in the case of Philippines Inter-Fashion, Inc. v NLRC, where the Court held: The Solicitor General has correctly stated in his comment that "from these facts are derived the following conclusions which are likewise undisputed: that petitioner engaged in an illegal lockout while the NAFLU engaged in an illegal strike; that the unconditional offer of the 150 striking employees to return to work and to withdraw their complaint of illegal lockout against petitioner constitutes condonation of the illegal lock-out; and that the unqualified acceptance of the offer of the 150 striking employees by petitioner likewise constitutes condonation of the illegal strike insofar as the reinstated employees are concerned." The issues at bar arise, however, from respondent commission's approval of its commissioner's conclusions that (1) petitioner must be deemed to have waived its right to pursue the case of illegal strike against the 114 employees who were not reinstated and who pursued their illegal lockout claim against petitioner; and (2) the said 114 employees are entitled to reinstatement with three months' backwages.

135

The Court approves the stand taken by the Solicitor General that there was no clear and unequivocal waiver on the part of petitioner and on the contrary the record shows that it tenaciously pursued its application for their dismissal, but nevertheless in view of the undisputed findings of illegal strike on the part of the 114 employees and illegal lockout on petitioner's part, both parties are in pari delicto and such situation warrants the restoration of the status quo ante and bringing the parties back to the respective positions before the illegal strike and illegal lockout through the reinstatement of the said 114 employees, as follows: The Bisaya case (102 Phil. 438) is inapplicable to the present case, because in the former, there were only two strikers involved who were both reinstated by their employer upon their request to return to work. However, in the present case, there were more than 200 strikers involved, of which 150 who desired to return to work were reinstated. The rest were not reinstated because they did not signify their intention to return to work. Thus, the ruling cited in the Bisaya case that the employer waives his defense of illegality of the strike upon reinstatement of strikers is applicable only to strikers who signified their intention to return to work and were accepted back ... Truly, it is more logical and reasonable for condonation to apply only to strikers who signified their intention to return and did return to work. The reason is obvious. These strikers took the initiative in normalizing relations with their employer and thus helped promote industrial peace. However, as regards the strikers who decided to pursue with the case, as in the case of the 114 strikers herein, the employer could not be deemed to have condoned their strike, because they had not shown any willingness to normalize relations with it. So, if petitioner really had any intention to pardon the 114 strikers, it would have included them in its motion to withdraw on November 17, 1980. The fact that it did not, but instead continued to pursue the case to the end, simply means that it did not pardon the 114 strikers. xxx xxx xxx The finding of illegal strike was not disputed. Therefore, the 114 strikers employees who participated therein are liable for termination (Liberal Labor Union v. Phil. Can Co., 91 Phil. 72; Insurefco Employees Union v. Insurefco, 95 Phil. 761). On the other hard, the finding of illegal lockout was likewise not disputed. Therefore, the 114 employees affected by the lockout are also subject to reinstatement. Petitioner, however, contends that the application for readmission to work by the 150 strikers constitutes condonation of the lockout which should likewise bind the 114 remaining strikers. Suffice it to say that the 150 strikers acted for themselves, not on behalf of the 114 remaining strikers, and therefore the latter could not be deemed to have condoned petitioner's lockout.

The findings show that both petitioner and the 114 strikers are in pari delicto, a situation which warrants the maintenance of the status quo. This means that the contending parties must be brought back to their respective positions before the controversy; that is, before the strike. Therefore, the order reinstating the 114 employees is proper. With such restoration of the status quo ante it necessarily follows, as likewise submitted by the Solicitor General, that the petition must be granted insofar as it seeks the setting aside of the award of three months' backwages to the 114 employees ordered reinstated on the basis of the general rule that strikers are not entitled to backwages (with some exceptions not herein applicable, such as where the employer is guilty of oppression and union-busting activities and strikers ordered reinstated are denied such reinstatement and therefore are declared entitled to backwages from the date of such denial). More so, is the principle of "no work, no pay" applicable to the case at bar, in view of the undisputed finding of illegality of the strike. Likewise, the in pari delicto doctrine was applied in the case of First City Interlink Transportation Co. Inc. v The Honorable Secretary, thus: 3) Petitioner substantially complied with the Return to Work Order. The medical examination, NBI, Police and Barangay Clearances as well as the driver's and conductor's/conductress licenses and photographs required

136

as conditions for reinstatement were reasonable management prerogatives. However, the other requirements imposed as condition for reinstatement were unreasonable considering that the employees were not being hired for the first time, although the imposition of such requirements did not amount to refusal on the part of the employer to comply with the Return to Work Order or constitute illegal lockout so as to warrant payment of backwages to the strikers. If at all, it is the employees' refusal to return to work that may be deemed a refusal to comply with the Return to Work Order resulting in loss of their employment status. As both the employer and the employees were, in a sense, at fault or in pari delicto, the nonreturning employees, provided they did not participate in illegal acts; should be considered entitled to reinstatement. But since reinstatement is no longer feasible, they should be given separation pay computed up to March 8, 1988 (the date set for the return of the employees) in lieu of reinstatement. [Emphases and underscoring supplied] In the case at bar, since both AER and the union are at fault or in pari delicto, they should be restored to their respective positions prior to the illegal strike and illegal lockout. Nonetheless, if reinstatement is no longer feasible, the concerned employees should be given separation pay up to the date set for the return of the complaining employees in lieu of reinstatement. Unyon filed the subject Motion for Partial Reconsideration questioning the Court's July 13, 2011 Decision insofar as it failed to award backwages to fourteen (14) of its members. Resolution: January 16, 2013 After going over the records again, the Court holds that only nine (9) of the fourteen (14) excluded employees deserve to be reinstated immediately with backwages. Records disclose that thirty-two (32) employees filed a complaint for illegal suspension and unfair labor practice against AER. Out of these 32 workers, only eighteen (18) of them were charged by AER with illegal strike leaving fourteen (14) of them excluded from its complaint. Technically, as no charges for illegal strike were filed against these 14 employees, they cannot be among those found guilty of illegal strike. They cannot be considered in pari delicto. They should be reinstated and given their backwages. Out of these 14 employees, however, five (5) failed to write their names and affix their signatures in the Membership Resolution attached to the petition filed before the CA, authorizing the Union President to represent them. Because of their failure to affix their names and signatures in the Membership Resolution they cannot be granted the relief that Unyon wanted for them in its Motion for Partial Reconsideration. These excluded nine (9) workers, who signed their names in their petition before the CA, deserve to be reinstated immediately and granted backwages. It is basic in jurisprudence that illegally dismissed workers are entitled to reinstatement with backwages plus interest at the legal rate.

HOLY CHILD CATHOLIC SCHOOL vs. HON. STO TOMAS ET AL. [G.R. No. 179146, July 23, 2013]

Facts: A petition for certification election was filed by private respondent Pinag-Isang Tinig at Lakas ng Anakpawis — Holy Child Catholic School Teachers and Employees Labor Union (HCCS-TELU-PIGLAS), alleging that: PIGLAS is a legitimate labor organization duly registered with the Department of Labor and Employment (DOLE) representing HCCS-TELU-PIGLAS; HCCS is a private educational institution duly registered and operating under Philippine laws; HCCS is unorganized, there is no collective bargaining agreement or a duly certified bargaining agent or a labor organization certified as the sole and exclusive bargaining agent of the proposed bargaining unit within one year prior to the filing of the petition.

137

Petitioner averred that of the employees who signed to support the petition, fourteen (14) already resigned and six (6) signed twice. Petitioner raised that members of private respondent do not belong to the same class; it is not only a mixture of managerial, supervisory, and rank-and-file employees — as three (3) are vice-principals, one (1) is a department head/supervisor, and eleven (11) are coordinators — but also a combination of teaching and non-teaching personnel — as twenty-seven (27) are non-teaching personnel. It insisted that, for not being in accord with Article 245 of the Labor Code, private respondent is an illegitimate labor organization lacking in personality to file a petition for certification election, as held in Toyota Motor Philippines Corporation v. Toyota Motor Philippines Corporation Labor Union; and an inappropriate bargaining unit for want of community or mutuality of interest, as ruled in Dunlop Slazenger (Phils.), Inc. v. Secretary of Labor and Employment and De La Salle University Medical Center and College of Medicine v. Laguesma. The med-arbiter denied the petition for certification election on the ground that the unit which private respondent sought to represent was inappropriate, stating that there was no “community or mutuality of interest” since there is a mixture of teaching and non-teaching staff. Private respondent then appealed before the SOLE, who ruled against the dismissal of the petition and directed the conduct of two separate certification elections for the teaching and the non-teaching personnel. The SOLE ruled that the private respondent could continue to exist as a legitimate labor organization with combined teaching and nonteaching personnel in its membership and representing both classes of employees in separate bargaining negotiations and agreements. When the case was brought before the Court of Appeals, the CA affirmed the decision of the SOLE and also ruled on the issue regarding the alleged mixture of supervisory and rank-and-file employees stating that the Toyota case is inapplicable because the vice-principals, department head, and coordinators are neither supervisory nor managerial employees. Issues: 1. Whether or not a petition for certification election is dismissible on the ground that the labor organization’s membership allegedly consists of supervisory and rank-and-file employees. 2. Whether or not the petition for certification election should be dismissed on the ground that respondent is not qualified for its failure to qualify as a legitimate labor organization due to the improper mixture of teaching and non-teaching personnel (absence of mutuality of interest among its members) Ruling: 1. No it does not. In Dunlop, in which the labor organization that filed a petition for certification election was one for supervisory employees, but in which the membership included rank-and-file employees, the Court reiterated that such labor organization had no legal right to file a certification election to represent a bargaining unit composed of supervisors for as long as it counted rank-and-file employees among its members. It should be emphasized that the petitions for certification election involved in Toyota and Dunlop were filed on November 26, 1992 and September 15, 1995, respectively; hence, the 1989 Rules was applied in both cases. But then, on June 21, 1997, the 1989 Amended Omnibus Rules was further amended by Department Order No. 9, series of 1997 (1997 Amended Omnibus Rules). Specifically, the requirement under Sec. 2(c) of the 1989 Amended Omnibus Rules — that the petition for certification election indicate that the bargaining unit of rank-and-file employees has not been mingled with supervisory employees — was removed. Instead, what the 1997 Amended Omnibus Rules requires is a plain description of the bargaining unit, which does not require that, for its creation and registration, a local or chapter submit a list of its members.

138

All said, while the latest issuance is R.A. No. 9481, the 1997 Amended Omnibus Rules, as interpreted by the Court in Tagaytay Highlands, San Miguel and Air Philippines, had already set the tone for it. Toyota and Dunlop no longer hold sway in the present altered state of the law and the rules. In unequivocal terms, we reiterated that the alleged inclusion of supervisory employees in a labor organization seeking to represent the bargaining unit of rank-and-file employees does not divest it of its status as a legitimate labor organization. Indeed, Toyota and Dunlop no longer hold true under the law and rules governing the instant case. The petitions for certification election involved in Toyota and Dunlop were filed on November 26, 1992 and September 15, 1995, respectively; hence, the 1989 Rules and Regulations Implementing R.A. No. 6715 (1989 Amended Omnibus Rules) was applied. In contrast, D.O. No. 9 is applicable in the petition for certification election of private respondent as it was filed on May 31, 2002. Following the doctrine laid down in Kawashima and SMCC-Super, it must be stressed that petitioner cannot collaterally attack the legitimacy of private respondent by praying for the dismissal of the petition for certification election: Except when it is requested to bargain collectively, an employer is a mere bystander to any petition for certification election; such proceeding is non-adversarial and merely investigative, for the purpose thereof is to determine which organization will represent the employees in their collective bargaining with the employer. The choice of their representative is the exclusive concern of the employees; the employer cannot have any partisan interest therein; it cannot interfere with, much less oppose, the process by filing a motion to dismiss or an appeal from it; not even a mere allegation that some employees participating in a petition for certification election are actually managerial employees will lend an employer legal personality to block the certification election. The employer's only right in the proceeding is to be notified or informed thereof.

2. The Court disagrees with Petitioner. The concepts of a union and of a legitimate labor organization are different from, but related to, the concept of a bargaining unit. In case of alleged inclusion of disqualified employees in a union, the proper procedure for an employer like petitioner is to directly file a petition for cancellation of the union's certificate of registration due to misrepresentation, false statement or fraud under the circumstances enumerated in Article 239 of the Labor Code, as amended. To reiterate, private respondent, having been validly issued a certificate of registration, should be considered as having acquired juridical personality which may not be attacked collaterally. In determining the proper collective bargaining unit and what unit would be appropriate to be the collective bargaining agency, the Court, in the seminal case of Democratic Labor Association v. Cebu Stevedoring Company, Inc., mentioned several factors that should be considered, to wit: (1) will of employees (Globe Doctrine); (2) affinity and unity of employees' interest, such as substantial similarity of work and duties, or similarity of compensation and working conditions; (3) prior collective bargaining history; and (4) employment status, such as temporary, seasonal and probationary employees. We stressed, however, that the test of the grouping is community or mutuality of interest, because "the basic test of an asserted bargaining unit's acceptability is whether or not it is fundamentally the combination which will best assure to all employees the exercise of their collective bargaining rights. As the SOLE correctly observed, petitioner failed to comprehend the full import of Our ruling in U.P. It suffices to quote with approval the apt disposition of the SOLE when she denied petitioner's motion for reconsideration: [Petitioner] appears to have confused the concepts of membership in a bargaining unit and membership in a union. In emphasizing the phrase "to the exclusion of academic employees" stated in U.P. v. Ferrer-Calleja, [petitioner] believed that the petitioning union could not admit academic employees of the university to its membership. But such was not the intention of the Supreme Court.

139

A bargaining unit is a group of employees sought to be represented by a petitioning union. Such employees need not be members of a union seeking the conduct of a certification election. A union certified as an exclusive bargaining agent represents not only its members but also other employees who are not union members. As pointed out in our assailed Decision, there were two contending unions in the U.P. case, namely[,] the Organization of Non-Academic Personnel of U.P. (ONAPUP) and the All U.P. Worker's Union composed of both U.P. academic and non-academic personnel. ONAPUP sought the conduct of a certification election among the rank-and-file non-academic personnel only, while the All U.P. Workers Union intended to cover all U.P. rank-and-file employees, involving both academic and non-academic personnel. The Supreme Court ordered the "non-academic rank-and-file employees of U.P. to constitute a bargaining unit to the exclusion of the academic employees of the institution", but did not order them to organize a separate labor organization. In the U.P. case, the Supreme Court did not dismiss the petition and affirmed the order for the conduct of a certification election among the non-academic personnel of U.P., without prejudice to the right of the academic personnel to constitute a separate bargaining unit for themselves and for the All U.P. Workers Union to institute a petition for certification election. In the same manner, the teaching and non-teaching personnel of [petitioner] school must form separate bargaining units. Thus, the order for the conduct of two separate certification elections, one involving teaching personnel and the other involving non-teaching personnel. It should be stressed that in the subject petition, [private respondent] union sought the conduct of a certification election among all the rank-andfile personnel of [petitioner] school. Since the decision of the Supreme Court in the U.P. case prohibits us from commingling teaching and non-teaching personnel in one bargaining unit, they have to be separated into two separate bargaining units with two separate certification elections to determine whether the employees in the respective bargaining units desired to be represented by [private respondent]. In summary, teaching and non-teaching personnel may be commingled in one union but have to be separated for collective bargaining purposes. Thus the need for separate bargaining units, and the need for separate certification elections, one for the teaching personnel and another for the non-teaching personnel.

VISAYAS COMMUNITY MEDICAL CENTER vs. YBALLE, ET AL. [G.R. No. 196156, January 15, 2014]

Facts: Respondents Yballe, Angel, Cortez and Ong were hired as employees of Visayas Community Medical Center, then Metro Cebu Community Hospital Inc. National Federation of Labor was the exclusive bargaining agent of the rank-and-file employees of MCCHI. In the CBA effective January 1994 until December 31, 1995, the signatories include Rev. Iyoy as MCCH administrator and Perla Nava as president of the Nagkahiusang Mamumuno sa MCCH, a NFL chartered union. On December 6, 1995, Nava wrote Rev. Iyoy expressing the union’s desire to renew the CBA. However, the proposal was returned requiring Nava to secure the endorsement of the legal counsel of NFL as the official bargaining representative of MCCHI employees. Nava insisted that the management should directly negotiate with them. MCCHI referred the matter to Atty. Alforque, NFL’S Regional Director, who then advised Nava that their group was no longer recognized by NFL. He sent a letter addressed to Nava that their union membership is suspended for serious violation of the Constitution and By-Laws. The violations occurred on February 27, 1996 when Nava and her group launched a series of mass actions such as wearing of black and red armbands and headbands, marching around the hospital premises and putting up placards. On March 13 and 19, 1996, DOLE Regional Office issued certifications stating that there is nothing in their records which show that NAMA-MCCH-NFL is a registered labor organization. On March 14, 1996, NAMA-MCCG-NFL filed a Notice of Strike but was deemed not filed for want of personality on their part. On March 30 of the same year, MCCHI sent termination letters to union members and other members who participated in the strike and picketing activities.

140

Unfazed, the union members held more mass actions. The means to and egress from the hospital were blocked so that the vehicles carrying patients and employees were barred from entering the premises. Placards were place at the hospital’s entrance. With the intensified violence, MCCHI suffered heavy losses due to low patient admission rates. Issue: Whether or not the union members were illegally dismissed based on their participation in the alleged illegal strike. Ruling: Yes. The Labor Code provides that "any union officer who knowingly participates in an illegal strike and any worker or union officer who knowingly participates in the commission of illegal acts during a strike may be declared to have lost his employment status". In the Decision dated December 7, 2011, we declared as invalid the dismissal of MCCH employees who participated in the illegal strike conducted by NAMA-MCCHNFL which is not a legitimate labor organization. Since there was no showing that the complainants committed any illegal act during the strike, they may not be deemed to have lost their employment status by their mere participation in the illegal strike. On the other hand, the union leaders (Nava group) who conducted the illegal strike despite knowledge that NAMA-MCCH-NFL is not a duly registered labor union were declared to have been validly terminated by petitioner. A worker merely participating in an illegal strike may not be terminated from employment. It is only when he commits illegal acts during a strike that he may be declared to have lost employment status. In contrast, a union officer may be terminated from employment for knowingly participating in an illegal strike or participates in the commission of illegal acts during a strike. The law grants the employer the option of declaring a union officer who participated in an illegal strike as having lost his employment. It possesses the right and prerogative to terminate the union officers from service. In this case, the NLRC affirmed the finding of the Labor Arbiter that respondents supported and took part in the illegal strike and further declared that they were guilty of insubordination. However, the mass termination of complainants was illegal, notwithstanding the illegality of the strike in which they participated. An ordinary striking worker cannot be dismissed for such mere participation in the illegal strike. However, the CA erred in awarding respondents full back wages and ordering their reinstatement despite the prevailing circumstances. With respect to backwages, the principle of "fair day’s wage for a fair day’s labor" remains as the basic factor in determining the award thereof. If there is no work performed by the employee there can be no wage or pay unless, of course, the laborer was able, willing and ready to work but was illegally locked out, suspended or dismissed or otherwise illegally prevented from working. The alternative relief for union members who were dismissed for having participated in an illegal strike is the payment of separation pay in lieu of reinstatement under the following circumstances: (a) when reinstatement can no longer be effected in view of the passage of a long period of time or because of the realities of the situation; (b) reinstatement is inimical to the employer’s interest; (c) reinstatement is no longer feasible; (d) reinstatement does not serve the best interests of the parties involved; (e) the employer is prejudiced by the workers’ continued employment; (f) facts that make execution unjust or inequitable have supervened; or (g) strained relations between the employer and employee. Considering that 15 years had lapsed from the onset of this labor dispute, and in view of strained relations that ensued, in addition to the reality of replacements already hired by the hospital which had apparently recovered from its huge losses, and with many of the petitioners either employed elsewhere, already old and sickly, or otherwise incapacitated, separation pay without back wages is the appropriate relief. Therefore, respondents were illegally dismissed. They are however only entitled to separation pay and not to full back wages.

141

PHILTRANCO SERVICE ENTERPRISES INC. vs. PHILTRANCO WORKERS UNION-ASSOCIATION OF GENUINE LABOR ORGANIZATIONS [G.R. No. 180962, February 26, 2014]

Facts: Petitioner Philtranco Service Enterprises, Inc., a local land transportation company, retrenched 21 of its employees on the ground that it was suffering business losses. Thus the company union, Philtranco Workers Union-Association of Genuine Labor Organizations (PWU-AGLU), filed a Notice of Strike with the Department of Labor and Employment (DOLE), claiming that petitioner engaged in unfair labor practices. The NCMB conducted the preliminary conference but no settlement was arrived at. Hence, the case was referred to the SOLE which ordered the reinstatement of the Union Officers without loss of seniority rights. The order was received on June 14, 2007. Petitioner filed for a Motion for Reconsideration on June 25 (Monday) or the first working day following last day which was Sunday. The Motion was denied by SOLE on August 15, 2007 which was received by Petitioner on Aug 17. It then filed the Petition for Certiorari on August 29, 2007. Issues: 1. WON the remedy of Petition for Certiorari was the appropriate remedy 2. WON the said remedy was timely filed Ruling: 1. Yes. The fact is undeniable that by referring the case to the Secretary of Labor, Conciliator-Mediator Aglibut conceded that the case fell within the coverage of Article 263 of the Labor Code. By assuming jurisdiction over the case, the provisions of Article 263 became applicable. It has long been settled that the remedy of an aggrieved party in a decision or resolution of the Secretary of Labor is to timely file a motion for reconsideration as a precondition for any further or subsequent remedy, and then seasonably file a special civil action for certiorari under Rule 65 of the 1997 Rules on Civil Procedure. 2. Yes. Rule 65 states that where a motion for reconsideration or new trial is timely filed, whether such motion is required or not, the petition shall be filed not later than 60 days counted from the notice of the denial of the motion. The very nature of certiorari – which is an extraordinary remedy resorted to only in the absence of plain, available, speedy and adequate remedies in the course of law – requires that the office issuing the decision or order be given the opportunity to correct itself. Quite evidently, this opportunity for rectification does not arise if no motion for reconsideration has been filed. Clearly, before a petition for certiorari under Rule 65 of the Rules of Court may be availed of, the filing of a motion for reconsideration is a condition sine qua non to afford an opportunity for the correction of the error or mistake complained of. Since Petitioner filed the Petition for Certiorari on August 29, 2007 which was exactly within the 60-day period within which to file, then such petition was timely filed.

WESLEYAN UNIVERSITY-PHILS. vs. WESLEYAN UNIVERSITY-PHILS., FACULTY & STAFF ASSOCIATION [G.R. No. 181806, March 12, 2014]

Facts: Petitioner Wesleyan University-Philippines is a non-stock, non-profit educational institution duly organized and existing under the laws of the Philippines. 6 Respondent Wesleyan University-Philippines Faculty and

142

Staff Association, on the other hand, is a duly registered labor organization 7 acting as the sole and exclusive bargaining agent of all rank-and-file faculty and staff employees of petitioner. In December 2003, the parties signed a 5-year CBA effective June 1, 2003 until May 31, 2008. On 2005, petitioner, through its President, issued a Memorandum providing guidelines on the implementation of vacation and sick leave credits as well as vacation leave commutation. Some of its provisions are: vacation and sick leave credits are not automatic; they have to be earned; only vacation leave is commuted or monetized to cash. Respondent’s President, Cynthia L. De Lara (De Lara) informed the petitioner through a letter that respondent is not amenable to the unilateral changes made by petitioner. Respondent questioned the guidelines for being violative of existing practices and the CBA. A Labor Management Committee (LMC) Meeting was held during which petitioner advised respondent to file a grievance complaint on the implementation of the vacation and sick leave policy. In the same meeting, petitioner announced its plan of implementing a one-retirement policy, which was unacceptable to respondent. Unable to settle their differences at the grievance level, the parties referred the matter to a Voluntary Arbitrator. On November 2, 2006, the Voluntary Arbitrator rendered a Decision declaring the one-retirement policy and the Memorandum dated August 16, 2005 contrary to law. Aggrieved, petitioner appealed the case to the CA via a Petition for Review under Rule 43 of the Rules of Court. On September2007, the CA rendered a Decision finding the rulings of the Voluntary Arbitrator supported by substantial evidence. It also affirmed the nullification of the one-retirement policy and the Memorandum dated August 16, 2005 on the ground that these unilaterally amended the CBA without the consent of respondent Issue: Whether the [CA] committed grave and palpable error in sustaining the Voluntary Arbitrator’s ruling that a university practice of granting its employees two (2) sets of Retirement Benefits had already been established as defined by the law and jurisprudence. Ruling: No. The Non-Diminution Rule found in Article 100 of the Labor Code explicitly prohibits employers from eliminating or reducing the benefits received by their employees. This rule, however, applies only if the benefit is based on an express policy, a written contract, or has ripened into a practice. To be considered a practice, it must be consistently and deliberately made by the employer over a long period of time. An exception to the rule is when "the practice is due to error in the construction or application of a doubtful or difficult question of law." The error, however, must be corrected immediately after its discovery; otherwise, the rule on Non-Diminution of Benefits would still apply. The practice of giving two retirement benefits to petitioner’s employees is supported by substantial evidence. In this case, respondent was able to present substantial evidence in the form of affidavits to support its claim that there are two retirement plans. Based on the affidavits, petitioner has been giving two retirement benefits as early as 1997. Petitioner, on the other hand, failed to present any evidence to refute the veracity of these affidavits. As we see it then, their affidavits, corroborated by the affidavits of incumbent employees, are more than sufficient to show that the granting of two retirement benefits to retiring employees had already ripened into a consistent and deliberate practice.

143

Moreover, petitioner’s assertion that there is only one retirement plan as the CBA Retirement Plan and the PERAA Plan are one and the same is not supported by any evidence. There is nothing in Article XVI of the CBA to indicate or even suggest that the "Plan" referred to in the CBA is the PERAA Plan. Besides, any doubt in the interpretation of the provisions of the CBA should be resolved in favor of respondent. The Memorandum dated August 16, 2005 is contrary to the existing CBA. Neither do we find any reason to disturb the findings of the CA that the Memorandum dated August 16, 2005 is contrary to the existing CBA. Sections 1 and 2 of Article XII of the CBA provide that all covered employees are entitled to 15-day sick leave and 15-day vacation leave with pay every year and that after the second year of service, all unused vacation leave shall be converted to cash and paid to the employee at the end of each school year, not later than August 30 of each year. The Memorandum dated August 16, 2005, however, states that vacation and sick leave credits are not automatic as leave credits would be earned on a month-to-month basis. This, in effect, limits the available leave credits of an employee at the start of the school year. A Collective Bargaining Agreement (CBA) is a contract entered into by an employer and a legitimate labor organization concerning the terms and conditions of employment. 1 Like any other contract, it has the force of law between the parties and, thus, should be complied with in good faith. 2 Unilateral changes or suspensions in the implementation of the provisions of the CBA, therefore, cannot be allowed without the consent of both parties.

TABANGAO SHELL REFINERY EMPLOYEES ASSOCIATION vs. PILIPINAS SHELL PETROLEUM CORP. [G.R. No. 170007, April 7, 2014] Facts: The Collective Bargaining Agreement (CBA) of the Company and the Union expired on 30 April 2004. Thus, as early as 13 April 2004, the Company and the Union already met to discuss the ground rules that would govern their upcoming negotiations. Then, on 15 April 2004, the Union submitted its proposals for the renewal of their CBA. While a total of 41 meetings were held between the parties, several items, including the matter of compensation, remained unresolved. The union proposed an annual across-the-board basic salary increase for the next three years that would be covered by the new CBA. The company made a counter-proposal to grant all covered employees a lump sum amount of yearly for the three-year period of the new CBA. Alleging failure on the part of the company to justify its offer, the union manifested that the company was bargaining in bad faith. The company, in turn, expressed its disagreement with the union’s manifestation. On 2 September 2004, the Union filed a Notice of Strike with the NCMB, Region IV based in Calamba, Laguna anchored on a perceived unfair labor practice consisting of alleged bad faith bargaining on the part of the Company. On September 16, 2004, during the cooling off period, the union conducted the necessary strike vote. The members of the union, who participated in the voting, unanimously voted for the holding of a strike. Upon being aware of this development, the company filed a Petition for Assumption of Jurisdiction with the Secretary of Labor and Employment. The Secretary of Labor and Employment found that the intended strike would likely affect the company’s capacity to provide petroleum products to the company’s various clientele, including the transportation sector, the energy sector, and the manufacturing and industrial sectors. Convinced that such a strike would have adverse consequences on the national economy, the Secretary of Labor and Employment ruled that the labor dispute between the parties would cause or likely to cause a strike in an industry indispensable to the national interest. Thus, the Secretary of Labor and Employment assumed jurisdiction over the dispute of the parties. The union thereafter filed a petition for certiorari in the Court of Appeals. The union alleged in its petition that the Secretary of Labor and Employment acted with grave abuse of discretion in grossly

144

misappreciating the facts and issue of the case. The union’s contended that the Secretary of Labor and Employment cannot resolve the economic issues because the union had not given its consent to the declaration of a deadlock. The Court of Appeals found the position of the union untenable. Neither the union nor the company appealed the Decision dated June 8, 2005 of the Secretary of Labor and Employment. Thus, the said Decision attained finality. Issue: Whether or not the Secretary of Labor committed grave abuse of discretion when is assumed jurisdiction over the dispute between the union and the company Ruling: Findings of fact of the Secretary of Labor and Employment in the Decision dated June 8, 2005 that there already existed a bargaining deadlock when she assumed jurisdiction over the labor dispute between the union and the company, and that there was no bad faith on the part of the company when it was bargaining with the union are both supported by substantial evidence. The contention of the labor union is untenable. The Secretary of Labor has jurisdiction over the labor dispute. The Secretary of the DOLE has been explicitly granted by Article 263(g) of the Labor Code the authority to assume jurisdiction over a labor dispute causing or likely to cause a strike or lockout in an industry indispensable to the national interest, and decide the same accordingly. And, as a matter of necessity, it includes questions incidental to the labor dispute; that is, issues that are necessarily involved in the dispute itself, and not just to that ascribed in the Notice of Strike or otherwise submitted to him for resolution. The labor dispute between the union and the company concerned the unresolved matters between the parties in relation to their negotiations for a new CBA. The power of the Secretary of Labor and Employment to assume jurisdiction over this dispute includes and extends to all questions and controversies arising from the said dispute, such as, but not limited to the union’s allegation of bad faith bargaining. It also includes and extends to the various unresolved provisions of the new CBA such as compensation, particularly the matter of annual wage increase or yearly lump sum payment in lieu of such wage increase, whether or not there was deadlock in the negotiations. In the case at bar, there was already an existing deadlock between the parties. The fact is that the negotiations between the union and the company were stalled by the opposing offers of yearly wage increase by the union, on the one hand, and annual lump sum payment by the company, on the other hand. Each party was adamant in its position. Thus, because of the unresolved issue on wage increase, there was actually a complete stoppage of the ongoing negotiations between the parties and the union filed a Notice of Strike. A mutual declaration would neither add to nor subtract from the reality of the deadlock then existing between the parties. Thus, the absence of the parties’ mutual declaration of deadlock does not mean that there was no deadlock. Furthermore, as the Decision dated June 8, 2005 of the Secretary of Labor and Employment already settled the said issues with finality, the union cannot once again raise those issues in this Court through this petition without violating the principle of res judicata, particularly in the concept of conclusiveness of judgment.

145

TOPIC 14: UNFAIR LABOR PRACTICE

GENERAL MILLING CORP. vs. COURT OF APPEALS [G.R. No. 146728, February 11, 2004]

Facts: In its plants located at Cebu City and Lapu-Lapu City, General Milling Corporation (GMC) employed 190 workers, who were all members of respondent General Milling Corporation Independent Labor Union (union, for brevity), a duly certified bargaining agent. On April 28, 1989, GMC and the union concluded a collective bargaining agreement (CBA) which included the issue of representation effective for a term of three years. The CBA was effective for three years retroactive to December 1, 1988. Hence, it would expire on November 30, 1991. On November 29, 1991, a day before the expiration of the CBA, the union sent GMC a proposed CBA, with a request that a counter-proposal be submitted within ten (10) days. As early as October 1991, GMC received letters from workers who stated that they had withdrawn from their union membership, on grounds of religious affiliation and personal differences. Believing that the union no longer had standing to negotiate a CBA, GMC did not send any counter-proposal. On December 16, 1991, GMC wrote a letter to the union’s officers stating that it felt there was no basis to negotiate with a union which no longer existed. On January 13, 1992, GMC dismissed Marcia Tumbiga, a union member, on the ground of incompetence. The union protested and requested GMC to submit the matter to the grievance procedure provided in the CBA. GMC, however, advised the union to "refer to our letter dated December 16, 1991." Thus, the union filed, on July 2, 1992, a complaint against GMC with the NLRC, Arbitration Division, Cebu City. The complaint alleged unfair labor practice on the part of GMC for: (1) refusal to bargain collectively; (2) interference with the right to self-organization; and (3) discrimination. Issues: 1. Whether or not GMC is guilty of unfair labor practice. 2. Whether or not the Court of Appeals was correct when it imposed upon the GMC the draft CBA proposes by the union for two years to begin from expiration of the original CBA. Ruling: GMC is guilty of unfair labor practice. Failing to comply with the mandatory obligation to submit a reply to the union’s proposals, GMC violated its duty to bargain collectively, making it liable for unfair labor practice.

On the first issue, Article 253-A of the Labor Code, as amended by Rep. Act No. 6715, states: ART. 253-A. Terms of a collective bargaining agreement. – Any Collective Bargaining Agreement that the parties may enter into shall, insofar as the representation aspect is concerned, be for a term of five (5) years. No petition questioning the majority status of the incumbent bargaining agent shall be entertained and no certification election shall be conducted by the Department of Labor and Employment outside of the sixty-day period immediately before the date of expiry of such five year term of the Collective Bargaining Agreement. All other provisions of the Collective Bargaining Agreement shall be renegotiated not later than three (3) years after its execution.

146

The law mandates that the representation provision of a CBA should last for five years. The relation between labor and management should be undisturbed until the last 60 days of the fifth year. Hence, when the union requested for a renegotiation on November 29, 1991, it was still the certified collective bargaining agent of the workers, because it was seeking said renegotiation within 5 years from the date of effectivity of the CBA on December 1, 1988. The union’s proposal was also submitted within the prescribed 3-year period from the date of effectivity of the CBA, just before the last day of said period. It was obvious that GMC had no valid reason to refuse to negotiate in good faith with the union. For refusing to send a counter-proposal to the union and to bargain anew on the economic terms of the CBA, the company committed an unfair labor practice under Article 248 of the Labor Code, which provides that: ART. 248. Unfair labor practices of employers. – It shall be unlawful for an employer to commit any of the following unfair labor practice: (g) To violate the duty to bargain collectively as prescribed by this Code; Article 252 of the Labor Code elucidates the meaning of the phrase "duty to bargain collectively," thus: ART. 252. Meaning of duty to bargain collectively. – The duty to bargain collectively means the performance of a mutual obligation to meet and convene promptly and expeditiously in good faith for the purpose of negotiating an agreement.... We have held that there is no per se test of good faith in bargaining. Good faith or bad faith is an inference to be drawn from the facts.The effect of an employer’s or a union’s actions individually is not the test of good-faith bargaining,but the impact of all such occasions or actions, considered as a whole. Under Article 252, both parties are required to perform their mutual obligation to meet and convene promptly and expeditiously in good faith for the purpose of negotiating an agreement. The union lived up to this obligation when it presented proposals for a new CBA to GMC within three (3) years from the effectivity of the original CBA. But GMC failed in its duty under Article 252. What it did was to devise a flimsy excuse, by questioning the existence of the union and the status of its membership to prevent any negotiation. Procedure in Collective Bargaining It bears stressing that the procedure in collective bargaining prescribed by the Code is mandatory because of the basic interest of the state in ensuring lasting industrial peace. ART. 250. Procedure in collective bargaining. – The following procedures shall be observed in collective bargaining: xxxx The other party shall make a reply thereto not later than ten (10) calendar days from receipt of such notice. GMC’s failure to make a timely reply to the proposals presented by the union is indicative of its utter lack of interest in bargaining with the union. Its excuse that it felt the union no longer represented the workers, was mainly dilatory as it turned out to be utterly baseless. We hold that GMC’s refusal to make a counter-proposal to the union’s proposal for CBA negotiation is an indication of its bad faith. Where the employer did not even bother to submit an answer to the bargaining proposals of the union, there is a clear evasion of the duty to bargain collectively. Failing to comply with the mandatory obligation to submit a reply to the union’s proposals, GMC violated its duty to bargain collectively, making it liable for unfair labor practice. The fact that the resignations of the union members occurred during the pendency of the case before the labor arbiter shows GMC’s desperate attempts to cast doubt on the legitimate status of the union. The ill-

147

timed letters of resignation from the union members indicate that GMC had interfered with the right of its employees to self-organization. CA was correct in imposing the draft proposed by the Union on GMC following the doctrines on Kiok Loy vs. NLRC and Divine Word University of Tacloban vs. Secretary of Labor and Employment. ART. 253 on the duty to bargain collectively when there exists a collective bargaining agreement, mandates the parties to keep the status quo while they are still in the process of working out their respective proposal and counter proposal. The general rule is that when a CBA already exists, its provision shall continue to govern the relationship between the parties, until a new one is agreed upon. The rule necessarily presupposes that all other things are equal. That is, that neither party is guilty of bad faith. However, when one of the parties abuses this grace period by purposely delaying the bargaining process, a departure from the general rule is warranted. In Kiok Loy vs. NLRC,Sweden Ice Cream Plant, who refused to submit any counter proposal proposed by its employees’ certified bargaining agent, had lost its right to bargain the terms and conditions of the CBA. That Sweden’s approach and attitude – stalling the negotiation lead to a conclusion that it is unwilling to negotiate and reach an agreement with the Union. Petitioner has not at any instance, evinced good faith. In Divine Word University of Tacloban vs. Secretary of Labor and Employment, Divine Word University of Tacloban, refused to perform its duty to bargain collectively. Thus, we upheld the unilateral imposition on the university of the CBA proposed by the Divine Word University Employees Union. Applying the principle in the foregoing cases to the instant case, it would be unfair to the union and its members if the terms and conditions contained in the old CBA would continue to be imposed on GMC’s employees for the remaining two (2) years of the CBA’s duration. We are not inclined to gratify GMC with an extended term of the old CBA after it resorted to delaying tactics to prevent negotiations. Since it was GMC which violated the duty to bargain collectively, based on Kiok Loy and Divine Word University of Tacloban, it had lost its statutory right to negotiate or renegotiate the terms and conditions of the draft CBA proposed by the union. However, that as strictly distinguished from the facts of this case, there was no pre-existing CBA between the parties in Kiok Loy and Divine Word University of Tacloban. Nonetheless, we deem it proper to apply in this case the rationale of the doctrine in the said two cases. To rule otherwise would be to allow GMC to have its cake and eat it too. Thus, by imposing on GMC the provisions of the draft CBA proposed by the union, in our view, the interests of equity and fair play were properly served and both parties regained equal footing, which was lost when GMC thwarted the negotiations for new economic terms of the CBA.

STANDARD CHARTERED BANK EMPLOYEES UNION vs. CONFESOR [G.R. No. 114974, June 16, 2004]

Facts: The exclusive bargaining agent of the rank and file employees of the Standard Chartered Bank is the Standard Chartered Bank Employees Union (the Union, for brevity). Prior to the expiration of the 3-year period of their CBA, but within the 60-day freedom period, the Union initiated the negotiations. On February 18, 1993, the Union, through its President, Eddie L. Divinagracia, sent a letter containing its proposals covering political provisions and 34 economic provisions. Included therein was a list of the names of the members of the Union’s negotiating panel.

148

The Union suggested to the Bank’s head of the negotiating panel, Cielito Diokno, that the bank lawyers should be excluded from the negotiating team. The Bank acceded.Meanwhile, Diokno suggested to Divinagracia that Jose P. Umali, Jr., the President of the National Union of Bank Employees (NUBE), the federation to which the Union was affiliated, be excluded from the Union’s negotiating panel. However, Umali was retained as a member thereof. On March 12, 1993, the parties met and set the ground rules for the negotiation. Diokno suggested that the negotiation be kept a "family affair." The proposed non-economic provisions of the CBA were discussed first. Even during the final reading of the non-economic provisions on May 4, 1993, there were still provisions on which the Union and the Bank could not agree. On May 18, 1993, the negotiation for economic provisions commenced. Towards the end of the Bank’s presentation, Umali requested the Bank to validate the Union’s "guestimates," especially the figures for the rank and file staff. In the succeeding meetings, Umali chided the Bank for the insufficiency of its counterproposal on the provisions on salary increase, group hospitalization, death assistance and dental benefits.

In the morning of the June 15, 1993 meeting, the Union suggested that if the Bank would not make the necessary revisions on its counter-proposal, it would be best to seek a 3rd party assistance. After the break, the Bank presented its revised counter-proposal, wherein, except for the provisions on signing bonus and uniforms, the Union and the Bank failed to agree on the remaining economic provisions of the CBA. The Union declared a deadlock and filed a Notice of Strike before the National Conciliation and Mediation Board (NCMB) on June 21, 1993, docketed as NCMB-NCR-NS-06-380-93.26. On the other hand, the Bank filed a complaint for Unfair Labor Practice (ULP) and Damages before the Arbitration Branch of the NLRC in Manila against the Union on June 28, 1993. The Bank alleged that the Union violated its duty to bargain, as it did not bargain in good faith. It contended that the Union demanded "sky high economic demands," indicative of blue-sky bargaining. On July 21, 1993, then SOLE Nieves R. Confesor, pursuant to Article 263(g) of the Labor Code, issued an Order assuming jurisdiction over the labor dispute at the Bank. An order was issued that the parties execute a collective bargaining agreement. Thus, the Bank’s charge for unfair labor practice which it originally filed with the NLRC but which is deemed consolidated herein, is dismissed for lack of merit. On the other hand, the Union’s charge for unfair labor practice is similarly dismissed. The parties then executed a Collective Bargaining Agreement wherein the wage increase was effected and the signing bonuses based on the increased wage were distributed to the employees covered by the CBA. The Union filed this petition for certiorari under Rule 65 of the Rules of Procedure. Issues: 1. Whether or not the company is guilty of unfair labor practice 2. Whether or not the Union is estopped after it executed with the company the CBA 3. Whether or not the Union is engaged in Blue-Sky Bargaining Ruling: The bank is not guilty of ULP. "Interference" under Article248 (a) of the Labor Code to amount to ULP. The petitioner asserts that respondent committed ULP, i.e., interference in the selection of the Union’s negotiating panel, when Cielito Diokno, the Bank’s Human Resource Manager, suggested to the Union’s President Eddie L. Divinagracia that Jose P. Umali, Jr., President of the NUBE, be excluded from the Union’s negotiating panel. In support of its claim, Divinagracia executed an affidavit, stating that prior to the

149

commencement of the negotiation, Diokno approached him and suggested the exclusion of Umali from the Union’s negotiating panel, and that during the first meeting, Diokno stated that the negotiation be kept a "family affair." In U.S. Postal Service and Harley Davidson Motor Co., Inc., AMF, the National Labor Relations Board held that upon the employer’s refusal to engage in negotiations with the Union for collective-bargaining contract when the Union includes a person who is not an employee, or one who is a member or an official of other labor organizations, such employer is engaged in unfair labor practice under Section 8(a)(1) and (5) of the NLRA. In, Insular Life Assurance Co., Ltd. Employees Association – NATU vs. Insular Life Assurance Co. Ltd., the test of whether an employer has interfered with and coerced employees in the exercise of their right to selforganization within the meaning of subsection (a)(1) is whether the employer has engaged in conduct which it may reasonably be said, tends to interfere with the free exercise of employees’ rights under Section 3 of the Act.Further, it is not necessary that there be direct evidence that any employee was in fact intimidated or coerced by statements of threats of the employer if there is a reasonable inference that antiunion conduct of the employer does have an adverse effect on self-organization and collective bargaining. Under the International Labor Organization Convention (ILO) No. 87 FREEDOM OF ASSOCIATION AND PROTECTION OF THE RIGHT TO ORGANIZE to which the Philippines is a signatory, "workers and employers, without distinction whatsoever, shall have the right to establish and, subject only to the rules of the organization concerned, to job organizations of their own choosing without previous authorization.” Workers’ and employers’ organizations shall have the right to draw up their constitutions and rules, to elect their representatives in full freedom to organize their administration and activities and to formulate their programs. Article 2 of ILO Convention No. 98 pertaining to the Right to Organize and Collective Bargaining, provides: Article 2 1. Workers’ and employers’ organizations shall enjoy adequate protection against any acts or interference by each other or each other’s agents or members in their establishment, functioning or administration. 2. In particular, acts which are designed to promote the establishment of workers’ organizations under the domination of employers or employers’ organizations or to support workers’ organizations by financial or other means, with the object of placing such organizations under the control of employers or employers’ organizations within the meaning of this Article. The aforecited ILO Conventions are incorporated in our Labor Code, particularly in Article 243 thereof, which provides: ART. 243. COVERAGE AND EMPLOYEES’ RIGHT TO SELF-ORGANIZATION. – All persons employed in commercial, industrial and agricultural enterprises and in religious, charitable, medical or educational institutions whether operating for profit or not, shall have the right to self-organization and to form, join, or assist labor organizations of their own choosing for purposes of collective bargaining. Ambulant, intermittent and itinerant workers, self-employed people, rural workers and those without any definite employers may form labor organizations for their mutual aid and protection. and Articles 248 and 249 respecting ULP of employers and labor organizations. The 1987 Constitution, aside from making it a policy to "protect the rights of workers and promote their welfare," devotes an entire section, emphasizing its mandate to afford protection to labor, and highlights "the principle of shared responsibility" between workers and employers to promote industrial peace. Article 248(a) of the Labor Code, considers it an unfair labor practice when an employer interferes, restrains or coerces employees in the exercise of their right to self-organization or the right to form association. The right to self-organization necessarily includes the right to collective bargaining.

150

If an employer interferes in the selection of its negotiators or coerces the Union to exclude from its panel of negotiators a representative of the Union, and if it can be inferred that the employer adopted the said act to yield adverse effects on the free exercise to right to self-organization or on the right to collective bargaining of the employees, ULP under Article 248(a) in connection with Article 243 of the Labor Code is committed. Substantial Evidence required to support the claim of ULP under the Labor Code. In order to show that the employer committed ULP under the Labor Code, substantial evidence is required to support the claim. Substantial evidence has been defined as such relevant evidence as a reasonable mind might accept as adequate to support a conclusion. In the case at bar, the Union bases its claim of interference on the alleged suggestions of Diokno to exclude Umali from the Union’s negotiating panel. The circumstances that occurred during the negotiation do not show that the suggestion made by Diokno to Divinagracia is an anti-union conduct from which it can be inferred that the Bank consciously adopted such act to yield adverse effects on the free exercise of the right to self-organization and collective bargaining of the employees, especially considering that such was undertaken previous to the commencement of the negotiation and simultaneously with Divinagracia’s suggestion that the bank lawyers be excluded from its negotiating panel. The records show that after the initiation of the collective bargaining process, with the inclusion of Umali in the Union’s negotiating panel, the negotiations pushed through. The complaint was made only on August 16, 1993 after a deadlock was declared by the Union on June 15, 1993. It is clear that such ULP charge was merely an afterthought. The accusation occurred after the arguments and differences over the economic provisions became heated and the parties had become frustrated. It happened after the parties started to involve personalities. As the public respondent noted, passions may rise, and as a result, suggestions given under less adversarial situations may be colored with unintended meanings.49 Such is what appears to have happened in this case. The Duty to Bargain Collectively If at all, the suggestion made by Diokno to Divinagracia should be construed as part of the normal relations and innocent communications, which are all part of the friendly relations between the Union and Bank. The Union alleges that the Bank violated its duty to bargain; hence, committed ULP under Article 248(g) when it engaged in surface bargaining. It alleged that the Bank just went through the motions of bargaining without any intent of reaching an agreement, as evident in the Bank’s counter-proposals. It explained that of the 34 economic provisions it made, the Bank only made 6 economic counterproposals. Surface Bargaining Surface bargaining is defined as "going through the motions of negotiating" without any legal intent to reach an agreement. The resolution of surface bargaining allegations never presents an easy issue. The determination of whether a party has engaged in unlawful surface bargaining is usually a difficult one because it involves, at bottom, a question of the intent of the party in question, and usually such intent can only be inferred from the totality of the challenged party’s conduct both at and away from the bargaining table.51 It involves the question of whether an employer’s conduct demonstrates an unwillingness to bargain in good faith or is merely hard bargaining.52 The Union has not been able to show that the Bank had done acts, both at and away from the bargaining table, which tend to show that it did not want to reach an agreement with the Union or to settle the differences between it and the Union. Admittedly, the parties were not able to agree and reached a deadlock. However, it is herein emphasized that the duty to bargain "does not compel either party to agree to a proposal or require the making of a concession." 53 Hence, the parties’ failure to agree did not amount to ULP under Article 248(g) for violation of the duty to bargain. In view of the finding of lack of ULP based on Article 248(g), the accusation that the Bank made bad-faith provisions has no leg to stand on.

151

While the refusal to furnish requested information is in itself an unfair labor practice, and also supports the inference of surface bargaining. We, likewise, find that the Union failed to substantiate its claim that the Bank refused to furnish the information it needed. Umali, in a meeting dated May 18, 1993, requested the Bank to validate its guestimates on the data of the rank and file. However, Umali failed to put his request in writing as provided for in Article 242(c) of the Labor Code. The Union, did not, as the Labor Code requires, send a written request for the issuance of a copy of the data about the Bank’s rank and file employees. Moreover, as alleged by the Union, the fact that the Bank made use of the aforesaid guestimates, amounts to a validation of the data it had used in its presentation. Estoppel not ApplicableIn the Case at Bar The respondent Bank argues that the petitioner is estopped from raising the issue of ULP when it signed the new CBA. In the case, however, the approval of the CBA and the release of signing bonus do not necessarily mean that the Union waived its ULP claim against the Bank during the past negotiations. After all, the conclusion of the CBA was included in the order of the SOLE, while the signing bonus was included in the CBA itself. Moreover, the Union twice filed a motion for reconsideration respecting its ULP charges against the Bank before the SOLE. The Union Did Not EngageIn Blue-Sky Bargaining Blue-Sky Bargaining is defined as "unrealistic and unreasonable demands in negotiations by either or both labor and management, where neither concedes anything and demands the impossible." It actually is not collective bargaining at all. (Robert’s Dictionary of Industrial Relations (Revised Edition, 1971, p. 51). We do not agree that the Union is guilty of ULP for engaging in blue-sky bargaining or making exaggerated or unreasonable proposals. The Bank failed to show that the economic demands made by the Union were exaggerated or unreasonable. The minutes of the meeting show that the Union based its economic proposals on data of rank and file employees and the prevailing economic benefits received by bank employees from other foreign banks doing business in the Philippines and other branches of the Bank in the Asian region. While the approval of the CBA and the release of the signing bonus did not estop the Union from pursuing its claims of ULP against the Bank, we find the latter did not engage in ULP. We, likewise, hold that the Union is not guilty of ULP.

PHILIPPINE CARPET EMPLOYEES ASSOCIATION vs. HON. STO. TOMAS [G.R. No. 168719, February 22, 2006]

Facts: Philippine Carpet Manufacturing Corporation is engaged in the business of manufacturing wool and yarn carpets and rugs. The Corporation also had 100% equity investments in the following corporations: Pacific Carpet Mills Corporation (PCMC-USA) which sold carpets and mats on wholesale basis; Pacific Carpet Manufacturing Corporation (PCMC-Clark) which manufactured hand-tufted and machine-tufted carpets and rugs; and the Philippine Woolen Spinning Corporation (PWSC) which manufactured wool yarn. The Corporation also owned 17.95% of the shares of stocks in DI Security and General Services, Inc., and 2.20% of such shares in the Manila Peninsula Hotel, Inc. The Corporation employed 473 employees, 355 of whom were members of the sole bargaining unit of the employees therein, the Philippine Carpet Employees Association (Union for brevity).

152

In a letter dated February 10, 2004 addressed to the Corporation’s Assistant Vice President for Administration, Manuel Ike Diaz, the Union proposed the holding of a conference between representatives of the Union and the Corporation on February 24, 2004, to commence negotiations. Appended to the letter were proposals on revisions of the previous CBA. The Corporation did not respond to the letter. Diaz issued a Memorandum informing all employees that a comprehensive cost reduction program would be implemented by the Corporation on April 15, 2004, "on account of depressed business conditions brought about by the currency crisis in Southeast Asia, the Middle East war and the 9/11 incident in the United States of America. Of the 88 employees who were terminated from employment, 77 were Union members, including Edgardo Villanueva, who was elected Union officer after the personnel reduction program commenced. Frustrated at the Corporation’s reason for retrenchment, the Union filed a notice of strike with the DOLE. Negotiations before the National Conciliation and Mediation Board ensued, but the Corporation stood pat on its stance for a moratorium on increases in wages and benefits. The Union rejected this and accused the Corporation of union busting, as 77 of its members were dismissed. The Union filed a petition with the DOLE for the Secretary of Labor and Employment (SOLE) to assume jurisdiction over the labor dispute involving economic issues on wage increases and certain benefits and non-economic issues such as scope of bargaining unit and on the issue of unfair labor practice. The Union claimed that there was no valid economic reason to retrench employees, and that a "slump" in demand of the Corporation’s products was not a valid ground to dismiss employees. The Union also charged the Corporation of resorting to a sinister scheme of re-channeling its carpet business to its wholly owned subsidiary, PCMC-Clark, while negotiations for a new CBA were ongoing. According to the Union, this was also to justify the dismissal of the 77 Union members and bust the Union in the process. The Union insisted that the Corporation was guilty of unfair labor practice. The Union maintained that in dismissing its employees, the Corporation violated the mandatory 30-day notice rule because such employees received the notice of termination on March 13, 2004 (Saturday), to take effect the following working day, March 15, 2004 (Monday). It stressed that the 30-day mandatory notice could not be substituted by paying the affected employees their respective one month salaries. Corporation alleged that based on the documents submitted to the SOLE, it suffered a sharp decline in business in terms of volume and income derived since 2001, caused by the Asian financial crisis and later aggravated by the 9/11 incident in the U.S. and the ongoing war in the Middle East. The Corporation went on to explain that its income from the domestic market and export operations declined sharply: from its export operations, its income of P28,855,000.00 in 2001 dropped to P23,927,000.00 in 2002; and, thereafter, to P5,796,000.00 in 2003. On June 23, 2004, the SOLE rendered a Decision granting wage increases totaling P8,039,330.00 to the employees for the three years of the CBA. Relative to increased benefits for uniform, Christmas package, rice subsidy, and early retirement plan/separation pay, the SOLE ordered the retention of the status quo. However, the SOLE denied the demand of the Union as to the scope of the bargaining unit. The SOLE likewise affirmed the termination of the 88 employees on the ground that, if not for the personnel reduction program. The Union thereafter filed a petition for certiorari with the CA which rendered judgment dismissing the petition for lack of merit. The appellate court ruled that the Corporation failed to prove that the SOLE committed grave abuse of discretion amounting to excess or lack of jurisdiction in issuing the decision. The appellate court affirmed the finding of the SOLE that there was a slump in the demand of the Corporation’s products, holding that while low volume of work was not listed as a valid ground for dismissal under Articles 282 and 283 of the Labor Code of the Philippines, it nevertheless justified the dismissal on the ground of redundancy. Issue:

153

Whether or not respondent corporation is guilty of unfair labor practice because of dismissing the 77 union members and bust the union in the process. Ruling: The petition is meritorious. 1. Retrenchment Retrenchment is an authorized cause for the termination of employment under Article 283 of the Labor Code. Retrenchment is defined as the termination of employment initiated by the employer through no fault of the employee and without prejudice to the latter, resorted by management during periods of business recession, industrial depression or seasonal fluctuations or during lulls over shortage of materials. It is a reduction in manpower, a measure utilized by an employer to minimize business losses incurred in the operation of its business. In Lopez Sugar Corporation v. Federation of Free Workers, the phrase "to prevent losses" was defined to mean that retrenchment or termination of the services of some employees is authorized to be undertaken by the employer sometime before the losses anticipated are actually sustained or realized. It is not, in other words, the intention of the lawmaker to compel the employer to stay his hand and keep all his employees until sometime after losses shall have, in fact, materialized; if such an intent were expressly written into the law, that law may well be vulnerable to constitutional attack as taking property from one man to give to another. This is simple enough. The prerogative of an employer to retrench its employees must be exercised only as a last resort, considering that it will lead to the loss of the employees’ livelihood. It is justified only when all other less drastic means have been tried and found insufficient or inadequate. 48 Moreover, the employer must prove the requirements for a valid retrenchment by clear and convincing evidence; otherwise, said ground for termination would be susceptible to abuse by scheming employers who might be merely feigning losses or reverses in their business ventures in order to ease out employees. The requirements are: (1) that the retrenchment is reasonably necessary and likely to prevent business losses which, if already incurred, are not merely de minimis, but substantial, serious, actual and real, or if only expected, are reasonably imminent as perceived objectively and in good faith by the employer; (2) that the employer served written notice both to the employees and to the Department of Labor and Employment at least one month prior to the intended date of retrenchment; (3) that the employer pays the retrenched employees separation pay equivalent to one month pay or at least ½ month pay for every year of service, whichever is higher; (4) that the employer exercises its prerogative to retrench employees in good faith for the advancement of its interest and not to defeat or circumvent the employees’ right to security of tenure; and (5) that the employer used fair and reasonable criteria in ascertaining who would be dismissed and who would be retained among the employees, such as status (i.e., whether they are temporary, casual, regular or managerial employees), efficiency, seniority, physical fitness, age, and financial hardship for certain workers. What the law speaks of is serious business losses or financial reverses. Sliding incomes or decreasing gross revenues are not necessarily losses, much less serious business losses within the meaning of the law. The bare fact that an employer may have sustained a net loss, such loss, per se, absent any other evidence on its impact on the business, nor on expected losses that would have been incurred had operations been continued, may not amount to serious business losses mentioned in the law. 50 The employer must also show that its losses increased through a period of time and that the condition of the company will not likely improve in the near future. 2. Redundancy

154

Redundancy, on the other hand, exists when the service capability of the work force is in excess of what is reasonably needed to meet the demands of the enterprise. A redundant position is one rendered superfluous by any number of factors, such as overhiring of workers, decreased volume of business, dropping of a particular product line previously manufactured by the company, or phasing out of a service activity previously undertaken by the business. Under these conditions, the employer has no legal obligation to keep in its payroll more employees than are necessary for the operation of its business. For the implementation of a redundancy program to be valid, the employer must comply with the following requisites: (1) written notice served on both the employees and the Department of Labor and Employment at least one month prior to the intended date of retrenchment; (2) payment of separation pay equivalent to at least one month pay for every year of service, whichever is higher; (3) good faith in abolishing the redundant positions; and (4) fair and reasonable criteria in ascertaining what positions are to be declared redundant and accordingly abolished. 3. Respondent corporation failed to produce clear evidence for valid retrenchment. Respondents failed to adduce clear and convincing evidence to prove the confluence of the essential requisites for a valid retrenchment of its employees. We believe that respondents acted in bad faith in terminating the employment of the members of petitioner Union. Corporation, in fact, amassed substantial earnings from 1999 to 2003. It found no need to appropriate its retained earnings except on March 23, 2001, when it appropriated P60,000,000.00 to increase production capacity. The appropriation of P20,000,000.00 by the respondent Corporation on September 16, 2004 was made barely five months after the 77 Union members were dismissed on the ground that respondent Corporation was suffering from "chronic depression." Cash dividends were likewise declared on March 29, 2004, barely two weeks after it implemented its "retrenchment program." There is likewise no justification for the hiring of more than 100 new employees, more than the number of those who were retrenched, as well as the order authorizing full blast overtime work for six hours daily. All these are inconsistent with the intransigent claim that respondent Corporation was impelled to retrench its employees precisely because of low demand for its products and other external causes. Admittedly, the net income of respondent Corporation of P46,559,917.00 in 2001 decreased to P37,764,303.00 in 2002. However, such decrease ensued because respondent Corporation declared cash dividends for its shareholders amounting to P28,000,000.00. It also appears that respondent Corporation’s personnel costs decreased to P97,971,479.00. There was thus no reason for respondent Corporation to implement its "retrenchment program" and terminate the 88 employees. The net income of the respondent Corporation of P39,553,028.00 in 2002 decreased to P12,729,776.00 in 2003.It bears stressing, however, that the stockholders received cash dividends in the total amount of P12,259,473.00. That respondents acted in bad faith in retrenching the 77 members of petitioner is buttressed by the fact that Diaz issued his Memorandum announcing the cost-reduction program on March 9, 2004, after receipt of the February 10, 2004 letter of the Union president which included the proposal for additional benefits and wage increases to be incorporated in the CBA for the ensuing year. Moreover, respondent Corporation failed to exhaust all other means to avoid further losses without retrenching its employees, such as utilizing the latter’s respective forced vacation leaves. Respondents also failed to use fair and reasonable criteria in implementing the retrenchment program, and instead chose to retrench 77 of the members of petitioner out of the dismissed 88 employees. Worse, respondent Corporation hired new employees and even rehired the others who had been "retrenched."

155

Respondents failed to prove that there was a drastic or severe decrease in the product sales or that it suffered severe business losses within an interval of three (3) months from January 2004 to March 9, 2004 when Diaz issued said Memorandum. 4. Since termination illegal, members are entitled to reinstatement with full backwages. The retrenchment effected by respondent Corporation is invalid due to a substantive defect, noncompliance with the substantial requirements to effect a valid retrenchment; it necessarily follows that the termination of the employment of petitioner Union’s members on such ground is, likewise, illegal. As such, they (petitioner Union’s members) are entitled to reinstatement with full backwages. However, in the case of those employees-members of petitioner Union who had received their respective separation pay, the amounts of such payments shall be deducted from the backwages due them. Where reinstatement is no longer feasible because the positions they previously held no longer exist, respondent Corporation shall pay the employees-members of petitioner Union backwages plus, in lieu of reinstatement, separation pay equivalent to one month pay, or one-half month pay for every year of service, whichever is higher.

ST. JOHN COLLEGES, INC. vs. ST. JOHN ACADEMY FACULTY EMPLOYEES UNION [G.R. No. 167892, October 27, 2006]

Facts: Prior to 1998, petitioner offered a secondary course only. The high school, which it employed, then employed about 80 teaching and non-teaching personnel who were members of the St. John Academy Faculty & Employees Union (Union). The Collective Bargaining Agreement (CBA) between SJCI and the Union was set to expire on May 31, 1997. During the ensuing collective bargaining negotiations, SJCI rejected all the proposals of the Union for an increase in worker’s benefits. This resulted to a bargaining deadlock which led to the holding of a valid strike by the Union on November 10, 1997. In order to end the strike, on November 27, 1997, SJCI and the Union, through the efforts of the National Conciliation and Mediation Board (NCMB), agreed to refer the labor dispute to the Secretary of Labor and Employment (SOLE) for assumption of jurisdiction. After which, the strike ended and classes resumed. Pending resolution of the labor dispute before the SOLE, the Board of Directors of SJCI approved on February 22, 1998 a resolution recommending the closure of the high school which was approved by the stockholders on even date because of the irreconcilable differences between the school management and the Academy’s Union particularly the safety of our students and the financial aspect of the ongoing CBA negotiations. Subsequently, some teaching and non-teaching personnel of the high school agreed to the closure. On April 2, 1998, SJCI informed the DOLE that as of March 31, 1998, 51 employees had received their separation compensation package while 25 employees refused to accept the same. On May 4, 1998, the 25 employees conducted a protest action within the perimeter of the high school. The Union filed a notice of strike with the NCMB only on May 7, 1998. On May 21, 1998, the 25 employees filed a complaint for unfair labor practice (ULP), illegal dismissal and non-payment of monetary benefits against SJCI before the NLRC which was docketed as RAB-IV-5-10039-98L. The Union members alleged that the closure of the high school was done in bad faith in order to get rid of the Union and render useless any decision of the SOLE on the CBA deadlocked issues. Labor Arbiter Antonio dismissed the Union’s complaint for ULP and illegal dismissal while granting SJCI’s petition to declare the strike illegal coupled with a declaration of loss of employment status of the 25 Union members involved in the strike.

156

On June 28, 2002, the NLRC rendered judgment reversing the decision of the Labor Arbiter. It found SJCI guilty of ULP and illegal dismissal and ordered it to reinstate the 25 employees to their former positions without loss of seniority rights and other benefits, and with full backwages. On appeal, the Court of Appeals, affirmed with modification the decision of the NLRC wherein the two month unworked summer vacation should excluded. Issue: Whether or not St. John Colleges is guilty of unfair labor practice through its act of closing the academy. Ruling: The petition lacks merit. 1. Closure. The closure was done to defeat the parties’ agreement to refer the labor dispute to the SOLE; to unilaterally end the bargaining deadlock; to render nugatory any decision of the SOLE; and to circumvent the Union’s right to collective bargaining and its members’ right to security of tenure. Under Article 283 of the Labor Code, the following requisites must concur for a valid closure of the business: (1) serving a written notice on the workers at least one (1) month before the intended date thereof; (2) serving a notice with the DOLE one month before the taking effect of the closure; (3) payment of separation pay equivalent to one (1) month or at least one half (1/2) month pay for every year of service, whichever is higher, with a fraction of at least six (6) months to be considered as a whole year; and (4) cessation of the operation must be bona fide. It is not disputed that the first two requisites were satisfied. The third requisite would have been satisfied were it not for the refusal of the herein private respondents to accept the separation compensation package. The instant case, thus, revolves around the fourth requisite, i.e., whether SJCI closed the high school in good faith. Whether or not the closure of the high school was done in good faith is a question of fact and is not reviewable by this Court in a petition for review on certiorari save for exceptional circumstances. In fine, the finding of the NLRC, which was affirmed by the Court of Appeals, that SJCI closed the high school in bad faith is supported by substantial evidence and is, thus, binding on this Court. Consequently, SJCI is liable for ULP and illegal dismissal. Whether SJCI acted in bad faith depends on the particular facts as established by the evidence on record. Bad faith is, after all, an inference which must be drawn from the peculiar circumstances of a case. The two decisive factors in determining whether SJCI acted in bad faith are (1) the timing of, and reasons for the closure of the high school, and (2) the timing of, and the reasons for the subsequent opening of a college and elementary department, and, ultimately, the reopening of the high school department by SJCI after only one year from its closure. Under these circumstances, it is not difficult to discern that the closure was done to defeat the parties’ agreement to refer the labor dispute to the SOLE; to unilaterally end the bargaining deadlock; to render nugatory any decision of the SOLE; and to circumvent the Union’s right to collective bargaining and its members’ right to security of tenure. By admitting that the closure was due to irreconcilable differences between the Union and school management, specifically, the financial aspect of the ongoing CBA negotiations, SJCI in effect admitted that it wanted to end the bargaining deadlock and eliminate the problem of dealing with the demands of the Union. This is precisely what the Labor Code abhors and punishes as unfair labor practice since the net effect is to defeat the Union’s right to collective bargaining. SJCI contends that these circumstances do not establish its bad faith in closing down the high school. Rather, it claims that it was forced to close down the high school due to alleged difficult labor problems that it encountered while dealing with the Union since 1995, specifically, the Union’s illegal demands in violation of R.A. 6728 or the "Government Assistance to Students and Teachers in Private Education Act." Under R.A. 6728, the income from tuition fee increase is to be used as follows: (a) 70% of the tuition fee shall go to the

157

payment of salaries, wages, allowances, and other benefits of teaching and non-teaching personnel, and (b) 20% of the tuition fee increase shall go to the improvement or modernization of the buildings, equipment, and other facilities as well as payment of the cost of operations. We are not persuaded. These alleged difficult labor problems merely show that SJCI and the Union had disagreements regarding workers’ benefits which are normal in any business establishment. If SJCI found the Union’s demands excessive, its remedy under the law is to refer the matter for voluntary or compulsory dispute resolution. Besides, this incident complained of occurred in 1995, which could hardly establish the good faith of SJCI or justify the closure in 1998. Anent the Union’s claim for the unimplemented 20% tuition fee increase in 1996, suffice it to say that it is erroneous to rule on said issue since the same was submitted before the Voluntary Arbitrator and is not on appeal before this Court. Besides, by referring the labor dispute to the Voluntary Arbitrator, the parties themselves acknowledged that there is a sufficient mechanism to resolve the said dispute. Again, we fail to see how this alleged labor problem in 1996 shows the good faith of SJCI in closing the high school in 1998. At any rate, even assuming that the Union’s demands were illegal or excessive, the important and crucial point is that these alleged illegal or excessive demands did not justify the closure of the high school and do not, in any way, establish SJCI’s good faith. The employer cannot unilaterally close its establishment on the pretext that the demands of its employees are excessive. As already discussed, neither party is obliged to give-in to the other’s excessive or unreasonable demands during collective bargaining, and the remedy in such case is to refer the dispute to the proper tribunal for resolution. This was what SJCI and the Union did when they referred the 1997 CBA bargaining deadlock to the SOLE; however, SJCI preempted the resolution of the dispute by closing the high school. SJCI disregarded the whole dispute resolution mechanism and undermined the Union’s right to collective bargaining when it closed down the high school while the dispute was still pending with the SOLE. The Labor Code does not authorize the employer to close down the establishment on the ground of illegal or excessive demands of the Union. Instead, aside from the remedy of submitting the dispute for voluntary or compulsory arbitration, the employer may file a complaint for ULP against the Union for bargaining in bad faith. If found guilty, this gives rise to civil and criminal liabilities and allows the employer to implement a lock out, but not the closure of the establishment resulting to the permanent loss of employment of the whole workforce. In fine, SJCI undermined the Labor Code’s system of dispute resolution by closing down the high school while the 1997 CBA negotiations deadlock issues were pending resolution before the SOLE. The closure was done in bad faith for the purpose of defeating the Union’s right to collective bargaining. Besides, as found by the NLRC, the alleged illegality and excessiveness of the Union’s demands were not sufficiently proved by SJCI. Even on the assumption that the Union’s demands were illegal or excessive, SJCI’s remedy was to await the resolution by the SOLE and to file a ULP case against the Union. However, SJCI did not have the power to take matters into its own hands by closing down the school in order to get rid of the Union. 2. Circumstances lead to the inescapable conclusion that SJCI merely used the alleged safety and wellbeing of the students as a subterfuge to justify its actions. SJCI next argues that the Union unduly endangered the safety and well-being of the students who joined the valid strike held on November 10, 1997, thus it closed down the high school on March 31, 1998. It claims that the Union coerced the students to join the protest actions to pressure SJCI to give-in to the demands of the Union. The Union categorically denied that it put the students in harm’s way or pressured them to join the protest actions. Given this denial by the Union, it was incumbent upon SJCI to prove that the students were actually harmed or put in harm’s way and that the Union coerced them to join the protest actions. The reason for this is that the employer carries the burden of proof to establish that the closure of the business was done in good faith. In the instant case, SJCI had the burden of proving that, indeed, the closure of the school was necessary to uphold the safety and well-being of the students.

158

There is insufficient evidence to hold that the safety and well-being of the students were endangered and/or compromised, and that the Union was responsible therefor. Even assuming arguendo that the students’ safety and well-being were jeopardized by the said protest actions, the alleged threat to the students’ safety and well-being had long ceased by the time the high school was closed. Moreover, the parents were vehemently opposed to the closure of the school because there was no basis to claim that the students’ safety was at risk. Taken together, these circumstances lead to the inescapable conclusion that SJCI merely used the alleged safety and well-being of the students as a subterfuge to justify its actions. 3. Pieces of evidence regarding the subsequent reopening of the high school after only one year from its closure further show that the high school’s closure was done in bad faith. SJCI next contends that the subsequent reopening of the high school after only one year from its closure did not show that the previous decision to close the high school was tainted with bad faith because the reopening was done due to the clamor of the high school’s former students and their parents. It claims that its former students complained about the cramped classrooms in the schools where they transferred. The contention is untenable. First, the fact that after one year from the time it closed its high school, SJCI opened a college and elementary department, and reopened its high school department showed that it never intended to cease operating as an educational institution. Second, there is evidence on record contesting the alleged reason of SJCI for reopening the Finally, when SJCI reopened its high school, it did not rehire the Union members. Evidently, the closure had achieved its purpose, that is, to get rid of the Union members. Clearly, these pieces of evidence regarding the subsequent reopening of the high school after only one year from its closure further show that the high school’s closure was done in bad faith. Lastly, SJCI asserts that the strike conducted by the 25 employees on May 4, 1998 was illegal for failure to take the necessary strike vote and give a notice of strike. However, protest actions of the Union cannot be considered a strike because, by then, the employer-employee relationship has long ceased to exist because of the previous closure of the high school on March 31, 1998. In sum, the timing of, and the reasons for the closure of the high school and its reopening after only one year from the time it was closed down, show that the closure was done in bad faith for the purpose of circumventing the Union’s right to collective bargaining and its members’ right to security of tenure. Consequently, SJCI is liable for ULP and illegal dismissal.

SAN MIGUEL FOODS, INC. vs. SAN MIGUEL CORPORATION EMPLOYEES UNION-PTGWO [G.R. No. 168569, October 5, 2007]

Facts: On November 9, 1992, some employees of San Miguel Foods, Inc’s Finance Department, through the Union represented by Edgar Moraleda, brought a grievance against Finance Manager Gideon Montesa (Montesa), for "discrimination, favoritism, unfair labor practices, promoting divisiveness, etc, before SMFI Plant Operations Manager George Nava in accordance with Step 1 of the grievance machinery. The Union sought the "1. review, evaluation & upgrading of all Finance staff and 2. Promotion of G.Q. Montesa to other SMC affiliate[s] & subsidiaries." SMFI informed the Union that it planned to address the grievance through a "work management review" to be completed by March 1993. However, it was not completed by March 1993, prompting the Union to, elevate the grievance to Step 2.

159

Almost 9 months after the grievance meeting was held or on October 6, 1993, SMFI rendered a "Decision on Step 1 Grievance" stating that it was still in the process of completing the "work management review," hence, the Union’s requests could not be granted. The Union filed a complaint on October 20, 1993 before the NLRC, Arbitration Branch, against SMFI, its President Amadeo P. Veloso, and its Finance Manager Montesa for "unfair labor practice, [and] unjust discrimination in matters of promotion . . . " It prayed that SMFI et al. be ordered to promote the therein named employees "with the corresponding pay increases, etc.” SMFI et al. filed a motion to dismiss, 8contending that the issues raised in the complaint were grievance issues and, therefore, "should be resolved in the grievance machinery. The Labor Arbiter granted SMFI et al.’s motion to dismiss and ordered the remand of the case to the grievance machinery for completion of the proceedings. On appeal, to the NLRC by "Motion for Reconsideration/Appeal", such was granted and accordingly ordered the Labor Arbiter to continue the proceedings on the Union’s complaint. Hence, SMFI filed a petition for certiorari with SC which they referred the case to the CA pursuant to St. Martin Funeral Homes v. NLRC. Court of Appeals denied SMFI et al.’s petition for certiorari, it holding that the Labor Arbiter has jurisdiction over the complaint of the Union, they having violated the seniority rule under the CBA by appointing and promoting certain employees which amounted to a ULP. Issues: Whether or not the labor arbiter has jurisdiction over the case. Whether or not SMFI is guilty of unfair labor practice. Ruling: Section 1 of Rule 8 of the Rules of Court should thus not be strictly applied to a case filed before a Labor Arbiter. In determining jurisdiction over a case, allegations made in the complaint, as well as those in the position paper, may thus be considered. SMFI argues that the allegations in the Union’s complaint filed before the Labor Arbiter do not establish a cause of action for ULP, the Union having merely contended that SMFI was guilty thereof without specifying the ultimate facts upon which it was based. It cites Section 1 of Rule 8 of the Rules of Court as applying suppletorily to the proceedings before the Labor Arbiter. Thus, SMFI concludes that the Labor Arbiter has no jurisdiction over its complaint. The jurisdiction of Labor Arbiters, enumerated in Article 217 of the Labor Code, includes complaints for ULP. Indeed, the particular acts of ULP alleged to have been committed by SMFI were not specified; neither were the ultimate facts in support thereof. In its Position Paper, however, the Union detailed the particular acts of ULP attributed to SMFI and the ultimate facts in support thereof. Section 7, Rule V of the New Rules of Procedure of the NLRC provides: The proceedings before the Labor Arbiter shall be non-litigious in nature. The technicalities of law and procedure and the rules obtaining in the courts of law shall not strictly apply thereto. The Labor Arbiter may avail himself of all reasonable means to ascertain the facts of the controversy speedily, including ocular inspection and examination of well-informed persons. Section 1 of Rule 8 of the Rules of Court should thus not be strictly applied to a case filed before a Labor Arbiter. In determining jurisdiction over a case, allegations made in the complaint, as well as those in the position paper, may thus be considered. SMFI guilty of ULP but only on the ground of violation of the CBA Agreement.

160

ULP on the ground of discrimination which must allege that that they were done to encourage or discourage membership in a labor organization. Based on Art. 248. Unfair labor practices of employers. – It shall be unlawful for an employer to commit any of the following unfair labor practices: (e) To discriminate in regard to wages, hours of work, and other terms and conditions of employment in order to encourage or discourage membership in any labor organization. On the questioned promotions, the Union did not allege that they were done to encourage or discourage membership in a labor organization. In fact, those promoted were members of the complaining Union. The promotions do not thus amount to ULP under Article 248(e) of the Labor Code.

ULP on the ground of violation of Collective Bargaining Agreement – (1) gross violation of the CBA; AND (2) the violation pertains to the economic provisions of the CBA. As for the alleged ULP committed under Article 248(i), for violation of a CBA, this Article is qualified by Article 261 of the Labor Code, the pertinent portion of which latter Article reads: x x x violations of a Collective Bargaining Agreement, except those which are gross in character, shall no longer be treated as unfair labor practice and shall be resolved as grievances under the Collective Bargaining Agreement. For purposes of this article, gross violations of Collective Bargaining Agreement shall mean flagrant and/or malicious refusal to comply with the economic provisions of such agreement. In Silva v. NLRC, for a ULP case to be cognizable by the Labor Arbiter, and the NLRC to exercise its appellate jurisdiction, the allegations in the complaint should show prima facie theconcurrence of two things, namely: (1) gross violation of the CBA; AND (2) the violation pertains to the economic provisions of the CBA. First, The grievance machinery provision in the CBA is not an economic provision, however, hence, the second requirement for a Labor Arbiter to exercise jurisdiction of a ULP is not present. Second, the Union alleges that violated the Job Security provision in the CBA, specifically the seniority rule, in that SMFI "appointed less senior employees to positions at its Finance Department, consequently intentionally by-passing more senior employees who are deserving of said appointment."

Since the seniority rule in the promotion of employees has a bearing on salary and benefits, it may, following a liberal construction (following the rule on construction in favor of labor) of Article 261 of the Labor Code, be considered an "economic provision" of the CBA. the Union charges SMFI to have promoted less senior employees, thus bypassing others who were more senior and equally or more qualified. It may not be seriously disputed that this charge is a gross or flagrant violation of the seniority rule under the CBA, a ULP over which the Labor Arbiter has jurisdiction. The Court of Appeals having affirmed the NLRC decision finding that the Labor Arbiter has jurisdiction over the Union’s complaint and thus remanding it to the Labor Arbiter for continuation of proceedings thereon, the appellate court’s said finding may be taken to have been made only for the purpose of determining jurisdiction.

161

PUREFOODS CORP. vs. NAGKAKAISANG SAMAHANG MANGGAGAWA NG PUREFOODS RANK AND FILE [G.R. No. 150896, August 28, 2008]

Facts: The respondents in this case, 3 labor organizations and a federation: NAGSAMA-Purefoods), the exclusive bargaining agent of the rank-and-file workers of Purefoods' meat division throughout Luzon; STFWU, in the farm in Sto. Tomas, Batangas; PGFWU, of those in the poultry farm in Sta. Rosa, Laguna. These organizations were affiliates of the federation, Purefoods Unified Labor Organization (PULO). STFWU, NAGSAMA-Purefoods, and PGFWU submitted their respective proposals for CBA renewal, and their general membership resolutions which, affirmed the organizations' affiliation with PULO. Purefoods refused to negotiate with the unions should a PULO representative be in the panel. The parties then agreed to postpone the negotiations indefinitely. On July 24, 1995, however, the petitioner company concluded a new CBA with another union in its farm in Malvar, Batangas. 5 days thereafter, at around 8:00pm, 4 company employees facilitated the transfer of around 23,000 chickens from the poultry farm in Sto. Tomas, Batangas (where STFWU was the exclusive bargaining agent) to that in Malvar. The next day, the regular rank-and-file workers in the Sto. Tomas farm were refused entry in the company premises; and on July 31, 1995, 22 STFWU members were terminated from employment. The farm manager, supervisors and electrical workers of the Sto. Tomas farm, who were members of another union, were nevertheless retained by the company in its employ. Aggrieved by these developments, the respondent labor organizations jointly instituted a complaint for unfair labor practice (ULP), illegal lockout/dismissal and damages, docketed with the Labor Arbitration Branch of the NLRC. In the proceedings before the Labor Arbiter, Purefoods interposed, among others, the defenses that PULO was not a legitimate labor organization or federation for it did not have the required minimum number of member unions; that the closure of the Sto. Tomas farm was not arbitrary but was the result of the financial non-viability of the operations; STFWU, lost its status as bargaining representative when the Sto. Tomas farm was closed. LA rendered a Decision dismissing the complaint. NLRC reversed the ruling of the LA, saying that the company's refusal to recognize the labor organizations' affiliation with PULO was unjustified considering that the latter had been granted the status of a federation by the Bureau of Labor Relations; and that this refusal constituted undue interference in, and restraint on the exercise of the employees' right to self-organization and free collective bargaining. The real motive of the company in the sudden closure and the mass dismissal was union busting, as only the union members were locked out, and the company subsequently resumed operations of the closed farm under a new contract with the landowner. The petitioner corporation filed a Rule 65 petition before the CA who dismissed the petition outright for no proof of authority to act for and on behalf of the corporation was submitted by the corporation's senior vicepresident who signed the non-forum shopping. Petitioner instituted before us the instant petition for review on certiorari under Rule 45. Issue: Whether or not petitioner is guilty of ULP. Whether or not the NLRC gravely abused its discretion. Ruling:

162

The petition is denied. When the petitioner is a corporation, the certification shall be executed by a natural person authorized by the corporation's board of directors. While technical rules of procedure are not designed to frustrate the ends of justice, they are provided to effect the proper and orderly disposition of cases and effectively prevent the clogging of court dockets. Section 1, Rule 65 of the Rules of Court mandates that the petition for certiorari shall be accompanied by a sworn certification of non-forum shopping. When the petitioner is a corporation, the certification shall be executed by a natural person authorized by the corporation's board of directors, and proof of such authority must be attached to the petition. Failure to attach to the certification any proof of the signatory's authority is a sufficient ground for the dismissal of the petition. In the instant case, the senior vice-president of the petitioner corporation signed the certificate of nonforum shopping. No proof of his authority to sign the said certificate was, however, attached to the petition. Thus, applying settled jurisprudence, we find that the CA committed no error when it dismissed the petition. The Court cannot even be liberal in the application of the rules because liberality is warranted only in instances when there is substantial compliance with the technical requirements in pleading and practice, and when there is sufficient explanation that the non-compliance is for a justifiable cause, such that the outright dismissal of the case will defeat the administration of justice. Here, the petitioner corporation did not present a reasonable explanation for its non-compliance with the rules. Further, it cannot be said that petitioner substantially complied therewith, because it did not attach to its motion for reconsideration any proof of the authority of its signatory. It stands to reason, therefore, that this Court now refuses to condone petitioner's procedural transgression. We must reiterate that the rules of procedure are mandatory, except only when, for the most persuasive of reasons, they may be relaxed to relieve a litigant of an injustice. While technical rules of procedure are not designed to frustrate the ends of justice, they are provided to effect the proper and orderly disposition of cases and effectively prevent the clogging of court dockets. 2. Petitioner guilty of ULP. It is crystal clear that the closure of the Sto. Tomas farm was made in bad faith. Badges of bad faith are evident from the following acts of the petitioner: it unjustifiably refused to recognize the STFWU's and the other unions' affiliation with PULO; it concluded a new CBA with another union in another farm during the agreed indefinite suspension of the collective bargaining negotiations; it surreptitiously transferred and continued its business in a less hostile environment; and it suddenly terminated the STFWU members, but retained and brought the non-members to the Malvar farm. Petitioner presented no evidence to support the contention that it was incurring losses or that the subject farm's lease agreement was pre-terminated. Ineluctably, the closure of the Sto. Tomas farm circumvented the labor organization's right to collective bargaining and violated the members' right to security of tenure. 3. Need to prove grave abuse of discretion of the NLRC. It was incumbent for petitioner to prove before the appellate court that the labor commission capriciously and whimsically exercised its judgment tantamount to lack of jurisdiction, or that it exercised its power in an arbitrary or despotic manner by reason of passion or personal hostility, and that its abuse of discretion is so patent and gross as to amount to an evasion of a positive duty enjoined or to act at all in contemplation of law. Here, as aforesaid, no such proof was adduced by petitioner. We, thus, declare that the NLRC ruling is not characterized by grave abuse of discretion. 3. Award of moral and exemplary damages due to existence of ULP. We deem as proper the award of moral and exemplary damages. We hold that the sudden termination of the STFWU members is tainted with ULP because it was done to interfere with, restrain or coerce

163

employees in the exercise of their right to self-organization. Thus, the petitioner company is liable for the payment of the aforesaid damages. Correcting the dispositive portion of the NLRC ruling, payment of P500,000.00 as moral and exemplary damages should be made to the illegally dismissed STFWU members. As to the order of reinstatement, since it is no longer feasible considering the length of time that the employees have been out of petitioner's employ, the company is ordered to pay the illegally dismissed STFWU members separation pay equivalent to one (1) month pay, or one-half (1/2) month pay for every year of service, whichever is higher. The releases and quitclaims, as well as the affidavits of desistance, signed by the employees, who were then necessitous men at the time of execution of the documents, are declared invalid and ineffective.

GENERAL SANTOS COCA-COLA PLANT FREE WORKERS UNION-TUPAS vs. CCBPI(GEN. SANTOS CITY) ET AL. [G.R. No. 178647, Feb. 13, 2009]

Facts: CCBPI experienced a decline in profitability due to the Asian economic crisis, decrease in sales, and tougher competition. It implemented 3 waves of an Early Retirement Program. Meanwhile, there was an inter-office memorandum mandating officers to put on hold “all requests for hiring to fill in vacancies in both regular and temporary positions in the Head Office and in the Plants.” Because several employees availed of the early retirement program, vacancies were created in some departments, including the production department of CCBPI Gen San, where members of petitioner Union worked. Faced with the “freeze hiring” directive, CCBPI engaged the services of JLBP Services Corporation (JLBP), a company in the business of providing labor and manpower services, including janitorial services, messengers, and office workers. The Union petitioner filed with the National Conciliation and Mediation Board (NCMB), a Notice of Strike on the ground of alleged unfair labor practice for contracting-out services regularly performed by union members (“union busting”). The parties failed to come to an amicable settlement. CCBPI filed a Petition for Assumption of Jurisdiction with the Office of the Secretary of Labor and Employment. The Secretary of Labor issued an Order enjoining the threatened strike and certifying the dispute to the NLRC for compulsory arbitration. NLRC ruled that CCBPI was not guilty of unfair labor practice for contracting out jobs to JLBP. The NLRC anchored its ruling on the validity of the “Going-to-the-Market” (GTM) system implemented by the company, which called for restructuring its selling and distribution system, leading to the closure of certain sales offices and the elimination of conventional sales routes. The NLRC held that petitioner failed to prove by substantial evidence that the system was meant to curtail the right to self-organization of petitioner’s members. The CA upheld the NLRC’s finding that CCBPI was not guilty of unfair labor practice. It found that JLBP was an independent contractor and that the decision to contract out jobs was a valid exercise of management prerogative to meet exigent circumstances. Issue: Whether or not CCBPI was guilty of unfair labor practice for contracting out jobs. Ruling: The petition is bereft of merit. Hence, we deny the Petition.

164

Dismissed for issues raised which are questions of facts. First, an examination of the issues raised by petitioner reveals that they are questions of fact. The issues raised, i.e., whether JLBP is an independent contractor, whether CCBPI’s contracting-out of jobs to JLBP amounted to unfair labor practice, and whether such action was a valid exercise of management prerogative, call for a re-examination of evidence, which is not within the ambit of this Court’s jurisdiction. Moreover, factual findings of the NLRC, an administrative agency deemed to have acquired expertise in matters within its jurisdiction, are generally accorded not only respect but finality especially when such factual findings are affirmed by the CA. The company’s action to contract did not constitute unfair labor practice as this was not directed at the members’ right to self-organization. Second, the NLRC found – and the same was sustained by the CA – that the company’s action to contractout the services and functions performed by Union members did not constitute unfair labor practice as this was not directed at the members’ right to self-organization. Article 248 of the Labor Code provides: It shall be unlawful for an employer to commit any of the following unfair labor practices: x x x (c) To contract out services or functions being performed by union members when such will interfere with, restrain or coerce employees in the exercise of their right to self-organization; x x x Unfair labor practice refers to “acts that violate the workers’ right to organize.” The prohibited acts are related to the workers’ right to self-organization and to the observance of a CBA. Without that element, the acts, even if unfair, are not unfair labor practices. Both the NLRC and the CA found that petitioner was unable to prove its charge of unfair labor practice. It was the Union that had the burden of adducing substantial evidence to support its allegations of unfair labor practice, which burden it failed to discharge.

DE LA SALLE UNIVERSITY ET AL. vs. DE LA SALLE UNIVERSITY EMPLOYEES ASSOCIATION [G.R. No. 177283, April 7, 2009]

Facts: In 2001, a group of respondents led by one Belen Aliazas (Aliazas group) filed a petition for conduct of elections with the DOLE, alleging that the then incumbent officers of respondent had failed to call for a regular election since 1985. Disputing the Aliazas group’s allegation, respondent claimed that an election was conducted in 1987 but by virtue of the enactment of Republic Act 6715, which amended the Labor Code, the term of office of its officers was extended to 5 years or until 1992 during which a general assembly was held affirming their hold-over tenure until the termination of collective bargaining negotiations; and that a collective bargaining agreement (CBA) was executed only on March 30, 2000. The DOLE-NCR held that the holdover authority of respondent’s incumbent set of officers had been extinguished by virtue of the execution of the CBA. It accordingly ordered the conduct of elections to be placed under the control and supervision of its Labor Relations Division and subject to pre-election conferences. Notwithstanding the conduct of election imposed, respondent called for a regular election on July 9, 2001, without prior notice to the DOLE and without the conduct of pre-election conference, prompting the Aliazas group to file an Urgent Motion for Intervention with the Bureau of Labor Relations (BLR) of the DOLE. The

165

BLR granted the Aliaza’s group’s motion for intervention directing to cease and desist from holding the general election of DLSUEA officers. The Aliazas group thereupon, via letter of August 7, 2001 to Brother Rolando Dizon, FSC, President of petitioner DLSU, requested the University “to please put on escrow all union dues/agency fees and whatever money considerations deducted from salaries of concerned co-academic personnel until such time that an election of union officials has been scheduled and subsequent elections has been held. Thus, DLSU decided that, the hold-over authority of your incumbent set of officers has been considered extinguished and an election of new union officers, to be conducted and supervised by the DOLE has been directed to be held. It decided the following saying that it does not want itself to be unnecessarily involved in your intra-union dispute. This is the only way that the University can maintain neutrality on this matter of grave concern. 1. Establish a savings account for the Union where all collected union dues and agency fees will be deposited and held in trust; and 2. Discontinue normal relations with any group within the Union including the incumbent set of officers. Consequently, respondent to file a complaint against petitioners for Unfair Labor Practice (ULP complaint), claiming that petitioners unduly interfered with its internal affairs and discriminated against its members. During the pendency of its ULP complaint, respondent filed its First Notice of Strike with the Office of the Secretary of Labor (OSL), charging petitioners for 1) gross violation of the CBA and 2) bargaining in bad faith which was certified for compulsory arbitration to the NLRC (certified case). The certified case, docketed as NLRC-NCR CC000222-02, was raffled to the NLRC Third Division. In the meantime, Labor Arbiter Felipe Pati, by Decision of July 12, 2002, dismissed respondent’s ULP complaint. Respondent appealed to the NLRC, and was lodged at the NLRC Second Division. While the dismissal of its ULP complaint was pending appeal before the NLRC Second Division, respondent, on behalf of some of its members, filed four other cases against petitioners which were lodged at the NLRC Second Division. Respondent thereafter filed in the certified case which was lodged at the NLRC Third Division a motion to have its four other cases and its ULP complaint then pending appeal before the NLRC Second Division to have these cases “subsumed” in the certified case. The NLRC Third Division granted respondent’s motion. The NLRC Second Division, in the meantime, affirmed the dismissal by the Arbiter of respondent’s ULP complaint. The Court of Appeals Tenth Division, REVERSED the said Order of the NLRC Third Divisiom with respect to the “subsuming” of respondent’s ULP complaint, the ULP complaint having been, at the time the NLRC Third Division Order was issued, “already disposed of” (dismissed) by the Arbiter and was in fact pending appeal before the NLRC Second Division. The Court of Appeals First Division SET ASIDE the NLRC Second Division Order affirming the dismissal of respondent’s ULP complaint and ordered NLRC Second Division to transmit the entire records of the ULP complaint to the NLRC Third Division where the ULP complaint had been ordered consolidated. Hence, petitioner’s petition for review on certiorari at bar. Issues: Whether or not the CA’s tenth division’s decision was correct in declaring the act of subsuming wrong. Whether or not petitioner is guilty of unfair labor practice. Ruling:

166

The petition is partly meritorious. 1. To transmit the records of respondent’s ULP complaint to the NLRC Third Division, the same can no longer be effected, the CA’s Tenth Division ruling having become final. The appellate court’s Tenth Division Decision had become final and executory on July 11, 2004. Therefore, with respect to the herein challenged Decision of the appellate court’s First Division ordering the NLRC Second Division to transmit the records of respondent’s ULP complaint to the NLRC Third Division, the same can no longer be effected, the appellate court’s Tenth Division ruling having, it bears repeating, become final. 2. Pending the final resolution of the intra-union dispute, respondent’s officers remained duly authorized to conduct union affairs. On the other matter raised by petitioners – that their acts of withholding union and agency dues and suspension of normal relations with respondent’s incumbent set of officers pending the intra-union dispute did not constitute interference, the Court finds for respondent. Pending the final resolution of the intra-union dispute, respondent’s officers remained duly authorized to conduct union affairs. The clarification letter by BLR Director Hans Leo J. Cacdac enlightens: We take this opportunity to clarify that there is no void in the DLSUEA leadership. The Decision of DOLE-NCR Regional Director should not be construed as an automatic termination of the incumbent officers’ tenure of office. As duly-elected officers of the DLSUEA, their leadership is not deemed terminated by the expiration of their terms of office, for they shall continue their functions and enjoy the rights and privileges pertaining to their respective positions in a hold-over capacity, until their successors shall have been elected and qualified. It bears noting that at the time petitioners’ questioned moves were adopted, a valid and existing CBA had been entered between the parties. It thus behooved petitioners to observe the terms and conditions thereof bearing on union dues and representation. It is axiomatic in labor relations that a CBA entered into by a legitimate labor organization and an employer becomes the law between the parties, compliance with which is mandated by express policy of the law. 3. The grant of nominal damages in order. No basis for exemplary damages. Exemplary or corrective damages are imposed by way of example or correction for the public good in addition to the moral, temperate, liquidated or compensatory damages. While the amount of exemplary damages need not be proved, respondent must show proof of entitlement to moral, temperate or compensatory damages before the Court may consider awarding exemplary damages. No such damages were prayed for, however, hence, the Court finds no basis to grant the prayer for exemplary damages. The grant of nominal damages and attorney’s fees is in order. In so far as the the NLRC Second Division Decision is REVERSED finding petitioners liable for Unfair Labor Practice, and to pay respondent nominal damages in the amount of P250,000 and attorney’s fees in the amount ofP50,000.

TUNAY NA PAGKAKAISA NG MANGGAGAWA SA ASIA BREWERY vs. ASIA BREWERY, INC. [G.R. No. 162025, Aug. 3, 2010]

Facts: Asia Brewery, Inc. (ABI) is engaged in the manufacture, sale and distribution of beer, shandy, bottled water and glass products. ABI entered into a Collective Bargaining Agreement (CBA) with Bisig at Lakas ng mga

167

Manggagawa sa Asia-Independent (BLMA-INDEPENDENT), the exclusive bargaining representative of ABI’s rank-and-file employees. Article I of the CBA defined the scope of the bargaining unit, as follows: Section 1.Recognition. The COMPANY recognizes the UNION as the sole and exclusive bargaining representative of all the regular rank-and-file daily paid employees within the scope of the appropriate bargaining unit with respect to rates of pay, hours of work and other terms and conditions of employment. The UNION shall not represent or accept for membership employees outside the scope of the bargaining unit herein defined. Section 2.Bargaining Unit. The bargaining unit shall be comprised of all regular rank-and-file daily-paid employees of the COMPANY. However, the following jobs/positions as herein defined shall be excluded from the bargaining unit, to wit: 1. Managers 2. Assistant Managers 3. Section Heads 4. Supervisors 5. Superintendents 6. Confidential and Executive Secretaries 7. Personnel, Accounting and Marketing Staff 8. Communications Personnel 9. Probationary Employees 10. Security and Fire Brigade Personnel 11. Monthly Employees 12. Purchasing and Quality Control Staff Subsequently, a dispute arose when ABI’s management stopped deducting union dues from eighty-one (81) employees, believing that their membership in BLMA-INDEPENDENT violated the CBA. Eighteen (18) of these affected employees are QA Sampling Inspectors/Inspectresses and Machine Gauge Technician who formed part of the Quality Control Staff. Twenty (20) checkers are assigned at the Materials Department of the Administration Division, Full Goods Department of the Brewery Division and Packaging Division. The rest are secretaries/clerks directly under their respective division managers. BLMA-INDEPENDENT claimed that ABI’s actions restrained the employees’ right to self-organization and brought the matter to the grievance machinery. As the parties failed to amicably settle the controversy, BLMA-INDEPENDENT lodged a complaint before the National Conciliation and Mediation Board (NCMB). The parties eventually agreed to submit the case for arbitration to resolve the issue of "whether or not there is restraint to employees in the exercise of their right to self-organization." Voluntary Arbitrator sustained the BLMA-INDEPENDENT after finding that the positions of the subject employees qualify under the rank-and-file category because their functions are merely routinary and clerical. On appeal, the CA reversed the Voluntary Arbitrator. BLMA-INDEPENDENT filed a motion for reconsideration. In the meantime, a certification election was held on August 10, 2002 wherein petitioner Tunay na Pagkakaisa ng Manggagawa sa Asia (TPMA) won. As the incumbent bargaining representative of ABI’s rank-and-file employees claiming interest in the outcome of

168

the case, petitioner filed with the CA an omnibus motion for reconsideration of the decision and intervention, with attached petition signed by the union officers.11 Both motions were denied by the CA.12 Issues: Whether or not the 81 employees are excluded from and are not eligible for inclusion in the bargaining unit as defined in the CBA; Whether or not the ABI committed an act in restraining the employees in the exercise of their right to selforganization; Ruling: 1. Although Article 245 of the Labor Code limits the ineligibility to join, form and assist any labor organization to managerial employees, jurisprudence has extended this prohibition to confidential employees or those who by reason of their positions or nature of work are required to assist or act in a fiduciary manner to managerial employees and hence, are likewise privy to sensitive and highly confidential records. Confidential employees are thus excluded from the rank-and-file bargaining unit. The rationale for their separate category and disqualification to join any labor organization is similar to the inhibition for managerial employees because if allowed to be affiliated with a Union, the latter might not be assured of their loyalty in view of evident conflict of interests and the Union can also become company-denominated with the presence of managerial employees in the Union membership. Having access to confidential information, confidential employees may also become the source of undue advantage. Said employees may act as a spy or spies of either party to a collective bargaining agreement. In Philips Industrial Development, Inc. v. NLRC, this Court held that petitioner’s "division secretaries, all Staff of General Management, Personnel and Industrial Relations Department, Secretaries of Audit, EDP and Financial Systems" are confidential employees not included within the rank-and-file bargaining unit. Earlier, in Pier 8 Arrastre & Stevedoring Services, Inc. v. Roldan-Confesor, the court declared that legal secretaries who are tasked with, among others, the typing of legal documents, memoranda and correspondence, the keeping of records and files, the giving of and receiving notices, and such other duties as required by the legal personnel of the corporation, fall under the category of confidential employees and hence excluded from the bargaining unit composed of rank-and-file employees. Also considered having access to "vital labor information" are the executive secretaries of the General Manager and the executive secretaries of the Quality Assurance Manager, Product Development Manager, Finance Director, Management System Manager, Human Resources Manager, Marketing Director, Engineering Manager, Materials Manager and Production Manager. In the present case, the CBA expressly excluded "Confidential and Executive Secretaries" from the rankand-file bargaining unit, for which reason ABI seeks their disaffiliation from the union. Petitioner, however, maintains that except for Daisy Laloon, Evelyn Mabilangan and Lennie Saguan who had been promoted to monthly paid positions, several secretaries/clerks are deemed included among the rank-and-file employees of ABI. It is rather curious that there would be several secretaries/clerks for just one (1) department/division performing tasks which are mostly routine and clerical. ABI insisted they fall under the "Confidential and Executive Secretaries" expressly excluded by the CBA from the rank-and-file bargaining unit. However, perusal of the job descriptions of these secretaries/clerks reveals that their assigned duties and responsibilities involve routine activities of recording and monitoring, and other paper works for their respective departments while secretarial tasks such as receiving telephone calls and filing of office correspondence appear to have been commonly imposed as additional duties. ABI failed to indicate who among these numerous secretaries/clerks have access to confidential data relating to management policies that could give rise to potential conflict of interest with their Union membership. Clearly, the rationale under our previous rulings for the exclusion of executive secretaries or division secretaries would have little or no significance considering the lack of or very limited access to confidential information of these secretaries/clerks.

169

We thus hold that the secretaries/clerks, numbering about forty (40), are rank-and-file employees and not confidential employees. With respect to the Sampling Inspectors/Inspectresses and the Gauge Machine Technician, there seems no dispute that they form part of the Quality Control Staff who, under the express terms of the CBA, fall under a distinct category. But we disagree with ABI’s contention that the twenty (20) checkers are similarly confidential employees being "quality control staff" entrusted with the handling and custody of company properties and sensitive information. Again, the job descriptions of these checkers assigned in the storeroom section of the Materials Department, finishing section of the Packaging Department, and the decorating and glass sections of the Production Department plainly showed that they perform routine and mechanical tasks preparatory to the delivery of the finished products. Consequently, we hold that the twenty (20) checkers may not be considered confidential employees under the category of Quality Control Staff who were expressly excluded from the CBA of the rank-and-file bargaining unit. Confidential employees are defined as those who (1) assist or act in a confidential capacity, (2) to persons who formulate, determine, and effectuate management policies in the field of labor relations. The two (2) criteria are cumulative, and both must be met if an employee is to be considered a confidential employee – that is, the confidential relationship must exist between the employee and his supervisor, and the supervisor must handle the prescribed responsibilities relating to labor relations. The exclusion from bargaining units of employees who, in the normal course of their duties, become aware of management policies relating to labor relations is a principal objective sought to be accomplished by the "confidential employee rule. There is no showing in this case that the secretaries/clerks and checkers assisted or acted in a confidential capacity to managerial employees and obtained confidential information relating to labor relations policies. Not being confidential employees, the secretaries/clerks and checkers are not disqualified from membership in the Union of respondent’s rank-and-file employees.

2. Unfair labor practice refers to "acts that violate the workers’ right to organize." The prohibited acts are related to the workers’ right to self-organization and to the observance of a CBA. For a charge of unfair labor practice to prosper, it must be shown that ABI was motivated by ill will, "bad faith, or fraud, or was oppressive to labor, or done in a manner contrary to morals, good customs, or public policy, and, of course, that social humiliation, wounded feelings or grave anxiety resulted from ABI’s act in discontinuing the union dues deduction from those employees it believed were excluded by the CBA. Considering that the herein dispute arose from a simple disagreement in the interpretation of the CBA provision on excluded employees from the bargaining unit, respondent cannot be said to have committed unfair labor practice that restrained its employees in the exercise of their right to self-organization, nor have thereby demonstrated an antiunion stance.

MANILA MINING CORP. EMPLOYEES ASSOCIATION-FFW vs. MANILA MINING CORP. [G.R. No. 178222-23, Sept. 29, 2010]

Facts: Respondent Manila Mining Corporation (MMC) is a publicly-listed corporation engaged in large-scale mining for gold and copper ore. MMC is required by law to maintain a tailings containment facility to store the waste material generated by its mining operations. Consequently, MMC constructed several tailings dams to treat and store its waste materials. One of these dams it constructed in 1993 and was operated under a permit issued by the DENR, through its Environmental Management Bureau (EMB) in Butuan City, Agusan del Norte.

170

Eleven (11) rank-and-file employees of MMC, who later became complainants before the labor arbiter, attended the organizational meeting of MMC-Makati Employees Association-Federation of Free Workers Chapter (Union). The Union filed with the DOLE all the requirements for its registration, acquired its legitimate registration status and subsequently, submitted letters to MMC relating its intention to bargain collectively. The Union submitted its Collective Bargaining Agreement (CBA) proposal to MMC. Upon expiration of the tailings permit, DENR-EMB did not issue a permanent permit due to the inability of MMC to secure an Environmental Compliance Certificate (ECC). MMC was compelled to temporarily shut down its mining operations, resulting in the temporary lay-off of more than 400 employees in the mine site.

MMC called for the suspension of negotiations on the CBA with the Union until resumption of mining operations. Among the employees laid-off, the 11 complainants, together with the Union filed a complaint before the labor arbiter praying for reinstatement, recognition of the Union as the sole and exclusive representative of its rank-and-file employees, and payment of moral and exemplary damages and attorney’s fees. The labor arbiter ruled in favor of MMC and held that the temporary shutdown of the mining operation, as well as the temporary lay-off of the employees, is valid. The NLRC modified the judgment of the labor arbiter and ordered the payment of separation pay equivalent to one month pay for every year of service. It ratiocinated that the temporary lay-off, which exceeded more than six (6) months, had the effect of severance of the employer-employee relationship. The Court of Appeals maintained the order to pay separation pay but set aside the MMC liability to pay the Union attorney’s fees equivalent to 10% of the award. Issue: Whether or not MMC is guilty of unfair labor practice Ruling: The lay-off is neither illegal nor can it be considered as unfair labor practice. Despite all efforts exerted by MMC, it did not succeed in obtaining the consent of the residents of the community where the tailings pond would operate, one of the conditions imposed by DENR-EMB in granting its application for a permanent permit. It is precisely MMC’s faultless failure to secure a permit which caused the temporary shutdown of its mining operations. As aptly put by the Court of Appeals: The evidence on record indeed clearly shows that MMC’s suspension of its mining operations was bonafide and the reason for such suspension was supported by substantial evidence. MMC cannot conduct mining operations without a tailings disposal system. For this purpose, MMC operates TP No. 7 under a valid permit from the Department of Environment and Natural Resources (DENR) through its Environmental Management Bureau (EMB). In fact, a "Temporary Authority to Construct and Operate" was issued on January 25, 2001 in favor of MMC valid for a period of six (6) months or until July 25, 2001. The NLRC did not dispute MMC’s claim that it had timely filed an application for renewal of its permit to operate TP No. 7 but that the renewal permit was not immediately released by the DENR-EMB, hence, MMC was compelled to temporarily shut down its milling and mining operations. Here, it is once apparent that the suspension of MMC’s mining operations was not due to its fault nor was it necessitated by financial reasons. Such suspension was brought about by the non-issuance of a permit for the continued operation of TP No. 7 without which MMC cannot resume its milling and mining operations. x x x.21 [Emphasis supplied.] Unfair labor practice cannot be imputed to MMC since, as ruled by the Court of Appeals, the call of MMC for a suspension of the CBA negotiations cannot be equated to "refusal to bargain." Article 252 of the Labor Code defines the phrase "duty to bargain collectively," to wit:

171

ARTICLE 252. Meaning of duty to bargain collectively. - The duty to bargain collectively means the performance of a mutual obligation to meet and convene promptly and expeditiously in good faith for the purpose of negotiating an agreement with respect to wages, hours of work and all other terms and conditions of employment including proposals for adjusting any grievances or questions arising under such agreements [and executing a contract incorporating such agreements] if requested by either party but such duty does not compel any party to agree to a proposal or to make any concession. For a charge of unfair labor practice to prosper, it must be shown that the employer was motivated by illwill, bad faith or fraud, or was oppressive to labor. The employer must have acted in a manner contrary to morals, good customs, or public policy causing social humiliation, wounded feelings or grave anxiety. While the law makes it an obligation for the employer and the employees to bargain collectively with each other, such compulsion does not include the commitment to precipitately accept or agree to the proposals of the other. All it contemplates is that both parties should approach the negotiation with an open mind and make reasonable effort to reach a common ground of agreement. The Union based its contention on the letter request by MMC for the suspension of the collective bargaining negotiations until it resumes operations. Verily, it cannot be said that MMC deliberately avoided the negotiation. It merely sought a suspension and in fact, even expressed its willingness to negotiate once the mining operations resume. There was valid reliance on the suspension of mining operations for the suspension, in turn, of the CBA negotiation. The Union failed to prove bad faith in MMC’s actuations.

PRINCE TRANSPORT ET AL. vs. GARCIA ET AL. [G.R. No. 167291, January 12, 2011]

Facts: Prince Transport, Inc. (PTI) is a company engaged in the business of transporting passengers by land. Respondents alleged in their complaints that: that they were hired by petitioner either as drivers, conductors, mechanics or inspectors, except for respondent Diosdado Garcia, who was assigned as Operations Manager.; in addition to their regular monthly income, they also received commissions equivalent to 8 to 10% of their wages.; sometime in October 1997, the said commissions were reduced to 7 to 9%; this led respondents and other employees of PTI to hold a series of meetings to discuss the protection of their interests as employees; these meetings led Renato Claros, who is the president of PTI, to suspect that respondents are about to form a union; he made known to Garcia his objection to the formation of a union; in December 1997, PTI employees requested for a cash advance, but the same was denied by management which resulted in demoralization on the employees' ranks; later, PTI acceded to the request of some, but not all, of the employees; the foregoing circumstances led respondents to form a union for their mutual aid and protection; in order to block the continued formation of the union, PTI caused the transfer of all union members and sympathizers to one of its sub-companies, Lubas Transport (Lubas); despite such transfer, the schedule of drivers and conductors, as well as their company identification cards, were issued by PTI; the daily time records, tickets and reports of the respondents were also filed at the PTI office; and, all claims for salaries were transacted at the same office; later, the business of Lubas deteriorated because of the refusal of PTI to maintain and repair the units being used therein, which resulted in the virtual stoppage of its operations and respondents' loss of employment. Petitioners, on the other hand, denied the material allegations of the complaints contending that herein respondents were no longer their employees, since they all transferred to Lubas at their own request; petitioners have nothing to do with the management and operations of Lubas as well as the control and supervision of the latter's employees; petitioners were not aware of the existence of any union in their company and came to know of the same only in June 1998 when they were served a copy of the summons in the petition for certification election filed by the union; that before the union was registered on April 15, 1998, the complaint subject of the present petition was already filed; that the real motive in the filing of the

172

complaints was because PTI asked respondents to vacate the bunkhouse where they (respondents) and their respective families were staying because PTI wanted to renovate the same. The Labor Arbiter ruled that petitioners are not guilty of unfair labor practice in the absence of evidence to show that they violated respondents' right to self-organization. The Labor Arbiter also held that Lubas is the respondents' employer and that it is an entity which is separate, distinct and independent from PTI. Nonetheless, the Labor Arbiter found that Lubas is guilty of illegally dismissing respondents from their employment. The NLRC partly sustained the Labor Arbiter’s ruling. The CA granted respondents' petition, ruling that petitioners are guilty of unfair labor practice; that Lubas is a mere instrumentality, agent conduit or adjunct of PTI; and that petitioners' act of transferring respondents' employment to Lubas is indicative of their intent to frustrate the efforts of respondents to organize themselves into a union. Issues: (1) Whether or not Lubas is a separate and independent entity from PTI (2) Whether or not PTI is guilty of unfair labor practice Held: (1) The Court agrees with the CA that Lubas is a mere agent, conduit or adjunct of PTI. A settled formulation of the doctrine of piercing the corporate veil is that when two business enterprises are owned, conducted and controlled by the same parties, both law and equity will, when necessary to protect the rights of third parties, disregard the legal fiction that these two entities are distinct and treat them as identical or as one and the same. In the present case, it may be true that Lubas is a single proprietorship and not a corporation. However, petitioners' attempt to isolate themselves from and hide behind the supposed separate and distinct personality of Lubas so as to evade their liabilities is precisely what the classical doctrine of piercing the veil of corporate entity seeks to prevent and remedy. Thus, the Court agrees with the observations of the CA, to wit: As correctly pointed out by petitioners, if Lubas were truly a separate entity, how come that it was Prince Transport who made the decision to transfer its employees to the former? Besides, Prince Transport never regarded Lubas Transport as a separate entity. In the aforesaid letter, it referred to said entity as "Lubas operations." Moreover, in said letter, it did not transfer the employees; it "assigned" them. Lastly, the existing funds and 201 file of the employees were turned over not to a new company but a "new management." The Court also agrees with respondents that if Lubas is indeed an entity separate and independent from PTI why is it that the latter decides which employees shall work in the former? Moreover, petitioners failed to refute the contention of respondents that despite the latter's transfer to Lubas of their daily time records, reports, daily income remittances of conductors, schedule of drivers and conductors were all made, performed, filed and kept at the office of PTI. In fact, respondents' identification cards bear the name of PTI. It may not be amiss to point out at this juncture that in two separate illegal dismissal cases involving different groups of employees transferred by PTI to other companies, the Labor Arbiter handling the cases found that these companies and PTI are one and the same entity; thus, making them solidarily liable for the payment of backwages and other money claims awarded to the complainants therein. Petitioners likewise aver that the CA erred and committed grave abuse of discretion when it ordered petitioners to reinstate respondents to their former positions, considering that the issue of reinstatement was never brought up before it and respondents never questioned the award of separation pay to them.

173

(2) As to whether petitioners are guilty of unfair labor practice, the Court finds no cogent reason to depart from the findings of the CA that respondents' transfer of work assignments to Lubas was designed by petitioners as a subterfuge to foil the former's right to organize themselves into a union. Under Article 248 (a) and (e) of the Labor Code, an employer is guilty of unfair labor practice if it interferes with, restrains or coerces its employees in the exercise of their right to self-organization or if it discriminates in regard to wages, hours of work and other terms and conditions of employment in order to encourage or discourage membership in any labor organization. Indeed, evidence of petitioners' unfair labor practice is shown by the established fact that, after respondents' transfer to Lubas, petitioners left them high and dry insofar as the operations of Lubas was concerned. The Court finds no error in the findings and conclusion of the CA that petitioners "withheld the necessary financial and logistic support such as spare parts, and repair and maintenance of the transferred buses until only two units remained in running condition." This left respondents virtually jobless.

PARK HOTEL ET AL. vs. SORIANO ET AL. [G.R. No. 17118, September 10, 2012]

Facts: Soriano was initially hired by Park Hotel but was transferred to Burgos Corporation. Gonzales and Badilla were employees of Burgos Corporation. Burgos is a sister company of Park Hotel. Harbutt and Percy are the General Manager and owner, respectively, of Park Hotel. Percy, Harbutt and Atty. Roberto Enriquez are also the officers and stockholders of Burgos Corporation. Soriano, Gonzales and Badilla were dismissed from work for allegedly stealing company properties. As a result, respondents filed complaints for illegal dismissal, unfair labor practice, before the Labor Arbiter (LA). In their complaints, respondents alleged that the real reason for their dismissal was that they were organizing a union for the company's employees. Issue: Whether or not corporate officers are solidarily and personally liable in a case for illegal dismissal and unfair labor practice Ruling: A corporation, being a juridical entity, may act only through its directors, officers and employees. Obligations incurred by them, while acting as corporate agents, are not their personal liability but the direct accountability of the corporation they represent. However, corporate officers may be deemed solidarily liable with the corporation for the termination of employees if they acted with malice or bad faith. In the present case, the lower tribunals unanimously found that Percy and Harbutt, in their capacity as corporate officers of Burgos, acted maliciously in terminating the services of respondents without any valid ground and in order to suppress their right to self-organization. Section 31 of the Corporation Code makes a director personally liable for corporate debts if he willfully and knowingly votes for or assents to patently unlawful acts of the corporation. It also makes a director personally liable if he is guilty of gross negligence or bad faith in directing the affairs of the corporation. Thus, Percy and Harbutt, having acted in bad faith in directing the affairs of Burgos, are jointly and severally liable with the latter for respondents' dismissal.

GOYA, INC. vs. GOYA, INC. EMPLOYEES UNION-FFW [G.R. No. 170054, January 21, 2013]

Facts:

174

Sometime in January 2004, petitioner Goya, Inc. (Company), a domestic corporation engaged in the manufacture, importation, and wholesale of top quality food products, hired contractual employees from PESO Resources Development Corporation (PESO) to perform temporary and occasional services in its factory in Parang, Marikina City. This prompted respondent Goya, Inc. Employees Union-FFW (Union) to request for a grievance conference on the ground that the contractual workers do not belong to the categories of employees stipulated in the existing Collective Bargaining Agreement (CBA). When the matter remained unresolved, the grievance was referred to the National Conciliation and Mediation Board (NCMB) for voluntary arbitration. During the hearing on July 1, 2004, the Company and the Union manifested before Voluntary Arbitrator (VA) Bienvenido E. Laguesma that amicable settlement was no longer possible; hence, they agreed to submit for resolution the solitary issue of "[w]hether or not [the Company] is guilty of unfair labor acts in engaging the services of PESO, a third party service provider[,] under the existing CBA, laws[,] and jurisprudence." Both parties thereafter filed their respective pleadings. The Union asserted that the hiring of contractual employees from PESO is not a management prerogative and in gross violation of the CBA tantamount to unfair labor practice (ULP). It noted that the contractual workers engaged have been assigned to work in positions previously handled by regular workers and Union members, in effect violating Section 4, Article I of the CBA, which provides for three categories of employees in the Company, to wit: Section 4.Categories of Employees. — The parties agree on the following categories of employees: (a)Probationary Employee. — One hired to occupy a regular rank-and-file position in the Company and is serving a probationary period. If the probationary employee is hired or comes from outside the Company (non-Goya, Inc. employee), he shall be required to undergo a probationary period of six (6) months, which period, in the sole judgment of management, may be shortened if the employee has already acquired the knowledge or skills required of the job. If the employee is hired from the casual pool and has worked in the same position at any time during the past two (2) years, the probationary period shall be three (3) months. (b)Regular Employee. — An employee who has satisfactorily completed his probationary period and automatically granted regular employment status in the Company. (c)Casual Employee. — One hired by the Company to perform occasional or seasonal work directly connected with the regular operations of the Company, or one hired for specific projects of limited duration not connected directly with the regular operations of the Company. It was averred that the categories of employees had been a part of the CBA since the 1970s and that due to this provision, a pool of casual employees had been maintained by the Company from which it hired workers who then became regular workers when urgently necessary to employ them for more than a year. Likewise, the Company sometimes hired probationary employees who also later became regular workers after passing the probationary period. With the hiring of contractual employees, the Union contended that it would no longer have probationary and casual employees from which it could obtain additional Union members; thus, rendering inutile Section 1, Article III (Union Security) of the CBA, which states:

Section 1.Condition of Employment. — As a condition of continued employment in the Company, all regular rank-and-file employees shall remain members of the Union in good standing and that new employees covered by the appropriate bargaining unit shall automatically become regular employees of the Company and shall remain members of the Union in good standing as a condition of continued employment. The Union moreover advanced that sustaining the Company's position would easily weaken and ultimately destroy the former with the latter's resort to retrenchment and/or retirement of employees and not filling up the vacant regular positions through the hiring of contractual workers from PESO, and that a possible

175

scenario could also be created by the Company wherein it could "import" workers from PESO during an actual strike. In countering the Union's allegations, the Company argued that: (a) the law expressly allows contracting and subcontracting arrangements through Department of Labor and Employment (DOLE) Order No. 18-02; (b) the engagement of contractual employees did not, in any way, prejudice the Union, since not a single employee was terminated and neither did it result in a reduction of working hours nor a reduction or splitting of the bargaining unit; and (c) Section 4, Article I of the CBA merely provides for the definition of the categories of employees and does not put a limitation on the Company's right to engage the services of job contractors or its management prerogative to address temporary/occasional needs in its operation. Issue: Whether or not the voluntary artbitrator committed an error in declaring that the engagement of PESO is not keeping with the intent and spirit of the CBA . Ruling: We confirm that the VA ruled on a matter that is covered by the sole issue submitted for voluntary arbitration. Resultantly, the CA did not commit serious error when it sustained the ruling that the hiring of contractual employees from PESO was not in keeping with the intent and spirit of the CBA. Indeed, the opinion of the VA is germane to, or, in the words of the CA, "interrelated and intertwined with," the sole issue submitted for resolution by the parties. This being said, the Company's invocation of Sections 4 and 5, Rule IV 20 and Section 5, Rule VI 21 of the Revised Procedural Guidelines in the Conduct of Voluntary Arbitration Proceedings dated October 15, 2004 issued by the NCMB is plainly out of order. Likewise, the Company cannot find solace in its cited case of Ludo & Luym Corporation v. Saornido. 22 In Ludo, the company was engaged in the manufacture of coconut oil, corn starch, glucose and related products. In the course of its business operations, it engaged the arrastre services of CLAS for the loading and unloading of its finished products at the wharf. The arrastre workers deployed by CLAS to perform the services needed were subsequently hired, on different dates, as Ludo's regular rank-and-file employees. Thereafter, said employees joined LEU, which acted as the exclusive bargaining agent of the rank-and-file employees. When LEU entered into a CBA with Ludo, providing for certain benefits to the employees (the amount of which vary according to the length of service rendered), it requested to include in its members' period of service the time during which they rendered arrastre services so that they could get higher benefits. Lastly, the Company kept on harping that both the VA and the CA conceded that its engagement of contractual workers from PESO was a valid exercise of management prerogative. It is confused. To emphasize, declaring that a particular act falls within the concept of management prerogative is significantly different from acknowledging that such act is a valid exercise thereof. What the VA and the CA correctly ruled was that the Company's act of contracting out/outsourcing is within the purview of management prerogative. Both did not say, however, that such act is a valid exercise thereof. Obviously, this is due to the recognition that the CBA provisions agreed upon by the Company and the Union delimit the free exercise of management prerogative pertaining to the hiring of contractual employees. Indeed, the VA opined that "the right of the management to outsource parts of its operations is not totally eliminated but is merely limited by the CBA," while the CA held that "[t]his management prerogative of contracting out services, however, is not without limitation. . . . [These] categories of employees particularly with respect to casual employees [serve] as limitation to [the Company's] prerogative to outsource parts of its operations especially when hiring contractual employees." A collective bargaining agreement is the law between the parties: It is familiar and fundamental doctrine in labor law that the CBA is the law between the parties and they are obliged to comply with its provisions. We said so in Honda Phils., Inc. v. Samahan ng Malayang Manggagawa sa Honda:

176

A collective bargaining agreement or CBA refers to the negotiated contract between a legitimate labor organization and the employer concerning wages, hours of work and all other terms and conditions of employment in a bargaining unit. As in all contracts, the parties in a CBA may establish such stipulations, clauses, terms and conditions as they may deem convenient provided these are not contrary to law, morals, good customs, public order or public policy. Thus, where the CBA is clear and unambiguous, it becomes the law between the parties and compliance therewith is mandated by the express policy of the law. Moreover, if the terms of a contract, as in a CBA, are clear and leave no doubt upon the intention of the contracting parties, the literal meaning of their stipulations shall control.

BAPTISTA ET AL. vs. VILLANUEVA ET AL. [G.R. No. 194709, July 31, 2013]

Facts: Petitioners were former union members of Radio Philippines Network Employees Union (RPNEU), a legitimate labor organization and the sole and exclusive bargaining agent of the rank and file employees of Radio Philippines Network (RPN), a government-sequestered corporation involved in commercial radio and television broadcasting affairs, while the respondents were the union’s elected officers and members. On April 26, 2005, on suspicion of union mismanagement, petitioners, together with some other union members, filed a complaint for impeachment of their union president, Reynato Siozon, before the executive board of RPN, which was eventually abandoned. They later re-lodged the impeachment complaint, this time, against all the union officers and members of RPNEU before the Department of Labor and Employment (DOLE). They likewise filed various petitions for audit covering the period from 2000 to 2004. Thereafter, cases were filed against the petitioners and several others for alleged violation of the union’s constitution and by-laws. After an exchange of communications between the grievance committee and the petitioners, petitioners were, through a memorandum, served an expulsion notice from the union. RPNEU’s officers informed their company of the expulsion of petitioners and the 12 others from the union and requested the management to serve them notices of termination from employment in compliance with their CBA’s union security clause. On February 17, 2006, RPN HRD Manager, Lourdes Angeles, informed petitioners and the 12 others of the termination of their employment effective March 20, 2006, enforcing Article II, Section 21 also known as the union security clause of their current CBA. Aggrieved, petitioners filed three (3) separate complaints for ULP against the respondents, which were later consolidated, questioning legality of their expulsion from the union and their subsequent termination from employment. Issue: Whether or not respondents committed unfair labor practices Ruling: The petition is bereft of merit. The primary concept of ULP is embodied in Article 247 of the Labor Code, which provides: Article 247. Concept of unfair labor practice and procedure for prosecution thereof.––Unfair labor practices violate the constitutional right of workers and employees to self-organization, are inimical to the legitimate interests of both labor and management, including their right to bargain collectively and otherwise deal with each other in an atmosphere of freedom and mutual respect, disrupt industrial peace and hinder the promotion of healthy and stable labor-management relations.

177

In essence, ULP relates to the commission of acts that transgress the workers’ right to organize. As specified in Articles 248 and 249 of the Labor Code, the prohibited acts must necessarily relate to the workers' right to self-organization and to the observance of a CBA. Absent the said vital elements, the acts complained, although seemingly unjust, would not constitute ULP Based on RPNEU’s Constitution and By-Laws, the charges against petitioners were not mere internal squabbles, but violations that demand proper investigation because, if proven, would constitute grounds for their expulsion from the union. Besides, any supposed procedural flaw in the proceedings before the Committee was deemed cured when petitioners were given the opportunity to be heard. Due process, as a constitutional precept, is satisfied when a person was notified of the charge against him and was given an opportunity to explain or defend himself. In administrative proceedings, the filing of charges and giving reasonable opportunity for the person so charged to answer the accusations against him constitute the minimum requirements of due process. It is well-settled that workers’ and employers’ organizations shall have the right to draw up their constitutions and rules to elect their representatives in full freedom, to organize their administration and activities and to formulate their programs. In this case, RPNEU’s Constitution and By-Laws expressly mandate that before a party is allowed to seek the intervention of the court, it is a pre-condition that he should have availed of all the internal remedies within the organization. Petitioners were found to have violated the provisions of the union’s Constitution and By-Laws when they filed petitions for impeachment against their union officers and for audit before the DOLE without first exhausting all internal remedies available within their organization. This act is a ground for expulsion from union membership. Thus, petitioners’ expulsion from the union was not a deliberate attempt to curtail or restrict their right to organize, but was triggered by the commission of an act, expressly sanctioned by Section 2.5 of Article IX of the union’s Constitution and By-Laws. For a charge of ULP against a labor organization to prosper, the onus probandi rests upon the party alleging it to prove or substantiate such claims by the requisite quantum of evidence. In labor cases, as in other administrative proceedings, substantial evidence or such relevant evidence as a reasonable mind might accept as sufficient to support a conclusion is required. Moreover, it is indubitable that all the prohibited acts constituting unfair labor practice should materially relate to the workers' right to self-organization. Unfortunately, petitioners failed to discharge the burden required to prove the charge of ULP against the respondents. Aside from their self-serving allegations, petitioners were not able to establish how they were restrained or coerced by their union in a way that curtailed their right to self-organization. The records likewise failed to sufficiently show that the respondents unduly persuaded management into discriminating against petitioners. other than to bring to its attention their expulsion from the union, which in turn, resulted in the implementation of their CBA' s union security clause. TH SHOPFITTERS CORP. ET AL. vs. T&H SHOPFITTERS CORP. UNION [G.R. No. 191714, February 26, 2014]

Facts: On September 7, 2004, the T&H Shopfitters Corporation/ Gin Queen Corporation workers union (THS-GQ Union), its officers and/or members of THS-GQ union, filed their Complaint 7 for Unfair Labor Practice (ULP) by way of union busting, and Illegal Lockout, with moral and exemplary damages and attorney’s fees, against T&H Shopfitters Corporation (T&H Shopfitters) and Gin Queen Corporation (Gin Queen) (collectively referred to as "petitioners"), before the Labor Arbiter (LA). Respondents treated T&H Shopfitters and Gin Queen as a single entity and their sole employer. In their desire to improve their working conditions, respondents and other employees of petitioners held their first formal meeting on November 23, 2003 to discuss the formation of a union. The following day or on November 24, 2003, seventeen (17) employees were barred from entering petitioners’ factory premises located in Castillejos, Zambales, and ordered to transfer to T&H Shopfitters’ warehouse at Subic Bay

178

Freeport Zone (SBFZ) purportedly because of its expansion. Afterwards, the said seventeen (17) employees were repeatedly ordered to go on forced leave due to the unavailability of work. On December 18, 2003, the Department of Labor and Employment (DOLE), Regional Office No. III issued a certificate of registration in favor of THS-GQ Union. On March 24, 2004, THS-GQ Union filed a petition for certification election. On July 12, 2004, an order was issued to hold the certification election in both T&H Shopfitters and Gin Queen. Eventually, the certification election was scheduled on October 11, 2004. Meanwhile, through a memorandum, dated August 17, 2004, petitioner Ben Huang (Huang), Director for Gin Queen, informed its employees of the expiration of the lease contract between Gin Queen and its lessor in Castillejos, Zambales and announced the relocation of its office and workers to Cabangan, Zambales. Some of the respondents, who visited the site in Cabangan, discovered that it was a "talahiban" or grassland. Later, the said union officers and members were made to work as grass cutters in Cabangan, under the supervision of a certain Barangay Captain Greg Pangan. Due to these circumstances, the employees assigned in Cabangan did not report for work. As a consequence, the THS-GQ Union president was made to explain why he should not be terminated for insubordination. The other employees who likewise failed to report in Cabangan were meted out with suspension. On October 10, 2004, petitioners sponsored a field trip to Iba, Zambales, for its employees. The officers and members of the THS-GQ Union were purportedly excluded from the field trip. On the evening of the field trip, a certain Angel Madriaga, a sales officer of petitioners, campaigned against the union in the forthcoming certification election. The following day or on October 11, 2004, the employees were escorted from the field trip to the polling center in Zambales to cast their votes. On October 13, 2004, the remaining employees situated at the SBFZ plant cast their votes as well. Due to the heavy pressure exerted by petitioners, the votes for "no union" prevailed. On October 14, 2004, the THS-GQ Union filed its protest with respect to the certification election proceedings. Issue: Whether or not acts of unfair labor practices were committed by petitioners against respondents Ruling: As to the issue of ULP, petitioners’ argument is utterly without merit. In the case at bench, petitioners are being accused of violations of paragraphs (a), (c), and (e) of Article 257 (formerly Article 248) of the Labor Code, to wit: Article 257. Unfair labor practices of employers.––It shall be unlawful for an employer to commit any of the following unfair labor practices: (a) To interfere with, restrain or coerce employees in the exercise of their right to selforganization; xxxx (c) To contract out services or functions being performed by union members when such will interfere with, restrain, or coerce employees in the exercise of their right to selforganization; xxxx (e) To discriminate in regard to wages, hours of work, and other terms and conditions of employment in order to encourage or discourage membership in any labor organization.

179

xxx The questioned acts of petitioners, namely: 1) sponsoring a field trip to Zambales for its employees, to the exclusion of union members, before the scheduled certification election; 2) the active campaign by the sales officer of petitioners against the union prevailing as a bargaining agent during the field trip; 3) escorting its employees after the field trip to the polling center; 4) the continuous hiring of subcontractors performing respondents’ functions; 5) assigning union members to the Cabangan site to work as grass cutters; and 6) the enforcement of work on a rotational basis for union members, all reek of interference on the part of petitioners. Indubitably, the various acts of petitioners, taken together, reasonably support an inference that, indeed, such were all orchestrated to restrict respondents’ free exercise of their right to self-organization. The Court is of the considered view that petitioners’ undisputed actions prior and immediately before the scheduled certification election, while seemingly innocuous, unduly meddled in the affairs of its employees in selecting their exclusive bargaining representative.

180

TOPIC 15: REVISED GUIDELINES OF THE NCMB FOR THE CONDUCT OF VOLUNTARY ARBITRATION PROCEEDINGS

SANYO PHILIPPINE WORKERS UNION-PPSLU vs. CANIZARES [G.R. No. 101619, July 8, 1992] Facts: PSSLU had an existing CBA with Sanyo Philippines Inc. effective July 1, 1989 to June 30, 1994. The same CBA contained a union security clause which provided: xxx members must retain their membership in good standing in the union as condition of his/her continued employment with the company. The union shall have the right to demand from the company the dismissal of the members of the union by reason of xxx their having formed, organized, joined, affiliated, supported another labor organization xxx PSSLU through its national and local presidents, wrote another letter to Sanyo recommending the dismissal of the following non-union workers: Bernardo Yap, Arnel Salvo, Renato Baybon, Reynaldo Ricohermoso, Salvador Solibel, Benito Valencia, and Allan Misterio, allegedly because: 1) they were engaged and were still engaging in anti-union activities; 2) they willfully violated the pledge of cooperation with PSSLU which they signed and executed on February 14, 1990; and 3) they threatened and were still threatening with bodily harm and even death the officers of the union. Also recommended for dismissal were the following union members who allegedly joined, supported and sympathized with a minority union, KAMAO: Gerardo Lasala, Legardo Tangkay, Alexander Atanacio, and Leonardo Dionisio. As per the attached letter from the local union President SPWU and the federation President, PSSLU, requesting management to put the respondent employees on preventive suspension, effective immediately, and the company having received no information on whether said employees appealed to PSSLU, it considered them dismissed as of March 23, 1991. On May 20, 1991, the dismissed employees filed a complaint with the NLRC for illegal dismissal. Named respondent were PSSLU and Sanyo. On June 20, 1991, PSSLU filed a motion to dismiss the complaint alleging that the Labor Arbiter was without jurisdiction over the case, relying on Article 217 (c) of P.D. 442, as amended by Section 9 of Republic Act No. 6715 which provides that cases arising from the interpretation or implementation of the collective bargaining agreements shall be disposed of by the labor arbiter by referring the same to the grievance machinery and voluntary arbitration. On August 7, 1991, the respondent Labor Arbiter issued the first questioned order. It held that: xxx xxx xxx While there are seemingly contradictory provisions in the aforecited article of the Labor Code, the better interpretation will be to give effect to both, and termination dispute being clearly spelled as falling under the jurisdiction of the Labor Arbiter, the same shall be respected. The jurisdiction of the grievance machinery and voluntary arbitration shall cover other controversies. However, the resolution of the instant issue shall be suspended until both parties have fully presented their respective positions. PSSLU filed another motion to resolve motion to dismiss complaint with a prayer that the Labor Arbiter resolve the issue of jurisdiction. On September 4, 1991, the respondent Labor Arbiter issued the second questioned order which held that it was assuming jurisdiction over the complaint of private respondents, in effect, holding that it had jurisdiction over the case. PSSLU filed this petition alleging that public respondent Labor Arbiter cannot assume jurisdiction over the complaint of public respondents because it had no jurisdiction over the dispute subject of said complaint. It is their submission that under Article 217 (c) of the Labor Code, in relation to Article 261 thereof, as well as Policy Instruction No. 6 of the Secretary of Labor, respondent Arbiter has no jurisdiction and authority to take cognizance of the complaint brought by private respondents which involves the implementation of the union security clause of the CBA. The function of the Labor Arbiter under the same law and rule is to refer this case to the grievance machinery and voluntary arbitration.

181

In its comment, private respondents argue that Article 217(a) 2 and 4 of the Labor Code is explicit, to wit: Art. 217. Jurisdiction of the Labor Arbiters and the Commission. a) Except as otherwise provided under this Code, the Labor Arbiters shall have original and exclusive jurisdiction to hear and decide . . . the following cases involving all workers, . . . : xxx xxx xxx 2) Termination disputes, xxx xxx xxx 4) Claims for actual, moral, exemplary and other forms of damages arising from the employer-employee relations. Issue: Whether or not the resolution between an employee versus the union and the employer need to be referred to the labor arbiter. Ruling: 1. Only disputes involving the union and the company shall be referred to the grievance machinery or voluntary arbitrators. It is clear from Article 217 that termination cases fall under the jurisdiction of the Labor Arbiter. It should be noted however that said article at the outset excepted from the said provision cases otherwise provided for in other provisions of the same Code, thus the phrase "Except as otherwise provided under this Code . . . ." Under paragraph (c) of the same article, it is expressly provided that "cases arising from the interpretation or implementation of collective bargaining agreements and those arising from the interpretation and enforcement of company personnel policies shall be disposed of by the Labor Arbiter by referring the same to the grievance machinery and voluntary arbitration as may be provided in said agreements. It was provided in the CBA executed between PSSLU and Sanyo that a member's voluntary resignation from membership, willful refusal to pay union dues and his/her forming, organizing, joining, supporting, affiliating or aiding directly or indirectly another labor union shall be a cause for it to demand his/her dismissal from the company. The demand for the dismissal and the actual dismissal by the company on any of these grounds is an enforcement of the union security clause in the CBA. This act is authorized by law provided that enforcement should not be characterized by arbitrariness (Manila Mandarin Employee Union v. NLRC, G.R. No. 76989, 29 Sept. 1987, 154 SCRA 368) and always with due process (Tropical Hut Employees Union v. Tropical Food Market, Inc., L-43495-99, Jan. 20, 1990). The reference to a Grievance Machinery and Voluntary Arbitrators for the adjustment or resolution of grievances arising from the interpretation or implementation of their CBA and those arising from the interpretation or enforcement of company personnel policies is mandatory. The law grants to voluntary arbitrators original and exclusive jurisdiction to hear and decide all unresolved grievances arising from the interpretation or implementation of the Collective Bargaining Agreement and those arising from the interpretation or enforcement of company personnel policies (Art. 261, Labor Code). The failure of the parties to the CBA to establish the grievance machinery and its unavailability is not an excuse for the Labor Arbiter to assume jurisdiction over disputes arising from the implementation and enforcement of a provision in the CBA. In the existing CBA between PSSLU and Sanyo, the procedure and mechanics of its establishment had been clearly laid out. All that needs to be done to set the machinery into motion is to call for the convening thereof. If the parties to the CBA had not designated their representatives yet, they should be ordered to do so.The procedure introduced in RA 6715 of referring certain grievances originally and exclusively to the grievance machinery and when not settled at this level, to a panel of voluntary arbitrators outlined in CBA's does not only include grievances arising from the interpretation or implementation of the CBA but applies as well to those arising from the implementation of

182

company personnel policies. No other body shall take cognizance of these cases. The last paragraph of Article 261 enjoins other bodies from assuming jurisdiction thereof: The commission, its Regional Offices and the Regional Directors of the Department of Labor and Employment shall not entertain disputes, grievances or matters under the exclusive and original jurisdiction of the Voluntary Arbitrator or panel of voluntary arbitrators and shall immediately dispose and refer the same to the grievance machinery or voluntary arbitration provided in the Collective Bargaining Agreement.

In the instant case, however, We hold that the Labor Arbiter and not the Grievance Machinery provided for in the CBA has the jurisdiction to hear and decide the complaints of the private respondents. While it appears that the dismissal of the private respondents was made upon the recommendation of PSSLU pursuant to the union security clause provided in the CBA, We are of the opinion that THESE FACTS DO NOT COME WITH IN THE PHRASE "grievances arising from the interpretation or implementation of (their) Collective Bargaining Agreement and those arising from the interpretation or enforcement of company personnel policies," the jurisdiction of which pertains to the Grievance Machinery or thereafter, to a voluntary arbitrator or panel of voluntary arbitrators. Article 260 of the Labor Code on grievance machinery and voluntary arbitrator states that "(t)he parties to a Collective Bargaining Agreement shall include therein provisions that will ensure the mutual observance of its terms and conditions. They shall establish a machinery for the adjustment and resolution of grievances arising from the interpretation or implementation of their Collective Bargaining Agreement and those arising from the interpretation or enforcement of company personnel policies." It is further provided in said article that the parties to a CBA shall name or designate their respective representatives to the grievance machinery and if the grievance is not settled in that level, it shall automatically be referred to voluntary arbitrators (or panel of voluntary arbitrators) designated in advance by the parties. It need not be mentioned that the parties to a CBA are the union and the company. Hence, only disputes involving the union and the company shall be referred to the grievance machinery or voluntary arbitrators. In the instant case, both the union and the company are united or have come to an agreement regarding the dismissal of private respondents. No grievance between them exists which could be brought to a grievance machinery. The problem or dispute in the present case is between the union and the company on the one hand and some union and non-union members who were dismissed, on the other hand . The dispute has to be settled before an impartial body. The grievance machinery with members designated by the union and the company cannot be expected to be impartial against the dismissed employees. Due process demands that the dismissed workers grievances be ventilated before an impartial body. Since there has already been an actual termination, the matter falls within the jurisdiction of the Labor Arbiter.

NAVARRO III vs. DAMASCO [G.R. No. 101875, July 14, 1995] Facts: Petitioner Navarro was employed as typist of private respondent at its plant in Bukidnon. Petitioner went to visit Mercy Baylas, a co-employee, at the ladies' dormitory inside the compound of private respondent. Upon seeing petitioner, Baylas hid behind the divider at the reception room. Rosemarie Basa and Isabel Beleno, co-boarders of Baylas, told petitioner that Baylas was not at the dormitory and advised him to stop courting her because she had no feelings towards him. Afterwards, the two left leaving petitioner alone in the room. When he peeped behind the divider, he saw Baylas, followed her, and after taking hold of her left hand, pulled her towards him. The force caused her to fall on the floor. He then placed himself on top of her. She resisted and futilely struggled to free herself from his grasp. The dormitory housekeeper, responded to Baylas' shouts for help, and saw petitioner embracing and kissing Baylas. She tried to separate petitioner from Baylas but to no avail. So she went outside and asked Basa, Beleno and Subong to help Baylas.

183

According to the medical report issued by Dr. Letecia P. Maraat, Baylas complained of pains on her shoulder and left foot. Petitioner was informed of the complaint against him and was placed under preventive suspension. In the report of Densing who was the one who investigated the incident, he recommended that the maximum penalty be meted out against petitioner. Petitioner was then dismissed from the service for having violated paragraph 3.B (Conduct and Behavior) of the Code of Employee Discipline.

A decision was rendered by the Voluntary Arbitrator dismissing petitioner from his employment and holding that private respondent did not violate the provisions of the grievance procedure under the Collective Bargaining Agreement. Not satisfied with the decision, petitioner filed the instant petition. Issues: 1. Whether or not the Voluntary Arbitrator exceeded in his authority in taking cognizance of the labor case? 2. Whether or not petitioner Navarro was denied due process of law because no hearing was held and he 3. was not given an opportunity to cross-examine the witnesses. Whether or not the quarrel between Baylas and him was a purely private affair. Ruling: 1. No, the Voluntary Arbitrator did not exceed in his authority. The instant case is not a grievance that must be submitted to the grievance machinery. What are subject of the grievance procedure for adjustment and resolution are grievances arising from the interpretation or implementation of the collective bargaining agreement. The acts of petitioner involved a violation of the Code of Employee Discipline, particularly the provision penalizing the immoral conduct of employees. Consequently, there was no justification for petitioner to invoke the grievance machinery provisions of the CBA. The case of petitioner was submitted to voluntary arbitration by agreement of the president of the labor union to which petitioner belongs, and his employer, through its personnel officer. Petitioner himself voluntarily submitted to the jurisdiction of the Voluntary Arbitrator when he, through his counsel, filed his position paper with the Voluntary Arbitrator and even submitted additional documentary evidence. In addition thereto, during the initial conference on March 27, 1991, the parties manifested that they were not questioning the authority of the Voluntary Arbitrator. It is the policy of the State to promote voluntary arbitration as a mode of settling labor. 2. NO, he was not denied of due process. The essence of due process is simply an opportunity to be heard, or as applied to administrative proceedings, an opportunity to explain one's side or an opportunity to seek a reconsideration of the action or ruling complained of. A formal or trial-type hearing is not at all times and in all instances essential. The requirements are satisfied where the parties are fair and reasonable opportunity to explain their side of the controversy at hand. What is frowned upon is the absolute lack of notice and hearing. Concerning the allegation that petitioner was not allowed to cross-examine the witnesses, the record shows that the parties had agreed not to cross-examine their witnesses anymore. 3. NO, the quarrel was not a private affair. The incident happened within the company premises, i.e. the ladies' dormitory which was located inside the plant site, but both of them are employees of private respondent. Management would then be at the mercy of its employees if it cannot enforce discipline within company premises solely because the quarrel is purely

184

personal matter. The harassment of an employee by a co-employee within the company premises even after office hours is a work-related matter considering that the peace of the company is thereby affected. The Code of Employee Discipline is very clear that immoral conduct "within the company premises regardless of whether or not [it is] committed during working time" is punishable. The pretext of petitioner that he was merely helping Baylas is belied by the eyewitnesses. Petitioner admitted that it took Subong to pull him away from Baylas. His alleged act of chivalry is nothing more than a chance to gratify his amorous feelings. WHEREFORE, the Decision of the respondent Voluntary Arbitrator is AFFIRMED.

SAN MIGUEL CORPORATION vs. NLRC [G.R. No. 108001, March 15, 1996] Facts: Private respondents, employed by petitioner San Miguel Corporation (SMC) as mechanics, machinists, and carpenters, were and still are, bona fide officers and members of private respondent Ilaw at Buklod ng Manggagawa. On or about July 31, 1990, private respondents were served a Memorandum from petitioner Angel G. Roa, Vice-President and Manager of SMC’s Business Logistics Division (BLD), to the effect that they had to be seperated from the service effective October 31, 1990 on the ground of “redundancy or excesss personnel.” Respondent union, in behalf of private respondents, opposed the intended dismissal and asked for a dialogue with management. On February 25, 1991, respondents filed a complaint against petitioners for Illegal Dismissal and Unfair Labor Practices, with a prayer for damages and attorney’s fees, with the Arbitration Branch of respondent National Labor Relations Commission. The complaint was assigned to Labor Arbiter Eduardo F. Carpio for hearing and proper disposition. On April 15, 1991, petitioners filed a motion to dismiss the complaint, alleging that respondent Labor Arbiter had no jurisdiction over the subject matter of the complaint, and that respondent Labor Arbiter must defer consideration of the unfair labor practice complaint until after the parties have gone through the grievance procedure provided for in the existing Collective Bargaining Agreement (CBA). Respondent Labor Arbiter denied this motion. The petitioners appealed the denial to respondent Commission and were unimpressed by the grounds therefor, thus, dismissed the appeal. Hence, the instant petition for certiorari. Issues: 1. Whether or not the labor arbiter can exercise jurisdiction over the alleged illegal termination and ULP cases without prior resort to the grievance and arbitration provided under the CBA. 2. Whether or not the strong state policy on the promotion of voluntary modes of settlement of labor disputes crafted in the Constitution and the Labor Code dictates the submission of the CBA dispute to grievance and arbitration. Ruling: FIRST ISSUE: LABOR ARBITER has original and exclusive jurisdiction over termination disputes and unfair labor practices and the exceptions not present in the case at bar. The CBA between the petitioners and respondent union provides, under Section 1, Article V entitled ARBITRATION, that “wages, hours of work, conditions of employment and/or employer-employee relations shall be settled by arbitration.” Petitioners’ thesis is that the dispute as to the termination of the union

185

members and the unfair labor practice should first be settled by arbitration, and not directly by the labor arbiter, following their CBA, which ought to be treated as the law between the parties thereto. The argument is unmeritorious. The law in point is Article 217 (a) of the Labor Code. It is elementary that this law is deemed written into the CBA. In fact, the law speaks in plain and unambiguous terms that termination disputes, together with unfair labor practices, are matters falling under the original and exclusive jurisdiction of the Labor Arbiter. The sole EXCEPTION to the above rule can be found under Article 262 of the same Code, which provides: “Aricle 262. Jurisdiction over other labor disputes - The voluntary arbitrator or panel of voluntary arbitrators, upon agreement of the parties, shall also hear and decide all other labor disputes including unfair labor practices and bargaining deadlocks.” (As added by R.A. 6715) Based on the CBA, Section 1, Article V of the CBA does not provide that petitioners and the respondent union conform to the submission of termination disputes and unfair labor practices to voluntary arbitration. Hence, consistent with the general rule under Article 217 (a) of the Labor Code, the Labor Arbiter properly has jurisdiction over the complaint filed by the respondent union on February 25, 1991 for illegal dismissal and unfair labor practice. Petitioners point however to Section 2, Article III of the CBA, under the heading Job Security, to show that the dispute is a proper subject of the grievance procedure, viz: “x x x The UNION, however, shall have the right to seek reconsideration of any discharge, lay-off or disciplinary action, and such requests for reconsideration shall be considered a dispute or grievance to be dealt with in accordance with the procedure outlined in Article IV hereof [on Grievance Machinery] x x x” Petitioners fail miserably to prove that, indeed, the respondent union requested for a reconsideration or review of the management decision to dismiss the private respondents. The filing of a request for reconsideration by the respondent union, which is the condition sine qua non to categorize the termination dispute and the ULP complaint as a grievable dispute, was absent in the case at bench. The only relevant provision under Article III that may need interpretation or implementation is Section 2 which was cited herein. However, as patiently pointed out by this court, said provision does not come into play considering that the union never exercised its right to seek reconsideration of the discharges effected by the company. It would have been different had the union sought reconsideration. Such recourse under Section 2 would have been treated as a grievance under Article IV (Grievance Machinery) of the CBA, thus calling for the possible interpretation or implementation of the entire provision on Grievance Machinery as agreed upon by the parties. This was not the case however. The union brought the termination dispute directly to the Labor Arbiter rendering Articles III and IV of the CBA inapplicable for the resolution of this case. Hence, the respondent union acted well within their rights in filing their complaint for illegal dismissal and ULP directly with the Labor Arbiter under Article 217 (a) of the Labor Code. A. There is no connection whatsoever between SMC’s management prerogative to effect the discharges and the interpretation or implementation of Articles III and IV of the CBA. Second. Petitioners insist that involved in the controversy is the interpretation and implementation of the CBA which is grievable and arbitrable by law under Article 217(c) of the Labor Code which xxx shall be disposed of by the Labor Arbiter by referring the same to the grievance machinery and voluntary arbitration as may be provided in said agreements.” (As amended by R.A. 6715). Petitioners questions whether SMC had the management right or prerogative to effect the discharges on the ground of redundancy, and this necessarily calls for the interpretation or implementation of Article III (Job Security) in relation to Article IV Grievance Machinery)of the CBA.

186

Petitioner’s theory does not hold water. There is no connection whatsoever between SMC’s management prerogative to effect the discharges and the interpretation or implementation of Articles III and IV of the CBA, except the one stated above if a reconsideration was filed.

B. The questioned discharges due to alleged redundancy can hardly be considered company personnel policies and therefore need not directly be subject to the grievance machinery nor to voluntary arbitration. The discharges, petitioners also contend, call for the interpretation or enforcement of company personnel policies, particularly SMC’s personnel policies on lay-offs arising from redundacy, and so, they may be considered grievable and arbitrable by virtue of Article 2 17(c). Not necessarily so. Company personnel policies are guiding principles stated in broad, long-range terms that express the philosophy or beliefs of an organization’s top authority regarding personnel matters. They deal with matters affecting efficiency and well-being of employees and include, among others, the procedure in the administration of wages, benefits, promotions, transfer and other personnel movements which are usually not spelled out in the collective agreement. The usual source of grievances, however, is the rules and regulations governing disciplinary actions. Judging therefrom, the questioned discharges due to alleged redundancy can hardly be considered company personnel policies and therefore need not directly be subject to the grievance machinery nor to voluntary arbitration. C. We find that based on the circumstances surrounding this case, the complaint filed by the private respondents on alleges facts sufficient to constitute a bona fide case of ULP, and cognizable by the Labor Arbiter. This is consistent with the rule that jurisdiction over the subject matter is determined by the allegations of the complaint. Third. Petitioners would like to persuade us that respondents’ ULP claims are merely conclusory and cannot serve to vest jurisdiction to the Labor Arbiters. Petitioners maintain that respondents’ complaint does not allege a genuine case for ULP. The Court is not convinced. The complaint alleges that: “5. Individual complainants are bona fide officers and members of complainant Ilaw at Buklod ng Manggagawa (IBM). xxx

xxx

xxx

23. The dismissal or lock-out from work of the individual complainants clearly constitutes an act of unfair labor practices in the light of the fact that the work being performed by the individual complainants are being contracted out by the respondent company. xxx

xxx

xxx

25. The acts of the respondent company in economically coercing employees to accept payment of separation and/or retirement benefits, pending final resolution of the labor xxxx there is undue interference, restraint, and coercion of employees in the exercise of their right to self-organization and collective bargaining.” Short of pre-empting the proceedings before the Labor Arbiter, the above complaint, makes Out a genuine case for ULP. In Manila Pencil Co. v. CIR, where business conditions justified a lay-off of employees, unfair labor practices were committed in the form of discriminatory dismissal where only unionists were permanently dismissed. This was despite the valid excuse given by the Manila Pencil Company that the dismissal of the employees was due to the reduction of the company’s dollar allocations for importation and that both union members and non-union members were laid-off.

187

A similar ruling was made by this Court in People’s Bank and Trust Co. v. People’s Bank and Trust Co. Employees Union involving the lay-off by a bank of 65 employees who were active union members allegedly by reason of retrenchment. The Court likewise found the employer in that case to have committed ULP in effecting the discharges. This Court was more emphatic however in Bataan Shipyard and Engineering Co., Inc. v. NLRC, et al.: Under the circumstances, we are inclined to believe that the company had indeed been discriminatory in selecting the employees who were to be retrenched. All of the retrenched employees are officers and members of the NAFLU. As such, the action taken by the firm becomes highly suspect. It leads us to conclude that the firm had been discriminating against membership in the NAFLU, an act which amounts to interference in the employees’ exercise of their right of self-organization. Under Art. 249 (now Art. 248) of the Labor Code of the Philippines, such interference is considered an act of unfair labor practice on the part of the Company x x x. It matters not that the cause of termination in the above cited cases was retrenchment while that in the instant case was redundancy. The important fact is that in all of these cases, including the one at bar, all of the dismissed employees were officers and members of their respective unions, and their employers failed to give a satisfactory explanation as to why this group of employees was singled out. It may be the case that employees other than union members may have been terminated also by petitioner SMC on account of its redundancy program. If that is true, the discharges may really be for a bona fide authorized cause under Article 283 of the Labor Code. On the other hand, it is also possible that such may only be a clever scheme of the petitioner company to camouflage its real intention of discriminating against union members particularly the private respondents. In any case, these matters will be best ventilated in a hearing before the Labor Arbiter. It is for the above reason that we cannot hold the petitioners guilty of the ULP charge. This will be the task of the Labor Arbiter. We however find that based on the circumstances surrounding this case, the complaint filed by the private respondents on February 25, 1991 alleges facts sufficient to constitute a bona fide case of ULP, and therefore properly cognizable by the Labor Arbiter under Article 2 17(a) of the Labor Code. This is consistent with the rule that jurisdiction over the subject matter is determined by the allegations of the complaint. SECOND ISSUE: Finally, petitioners try to impress on this Court the strong State policy on the promotion of voluntary modes of settlement of labor disputes crafted in the Constitution and the Labor Code which dictate the submission of the CBA dispute to grievance and arbitration. In the absence of an express legal conferment thereof, jurisdiction cannot be appropriated by an official or tribunal no matter how well-intentioned it is, even in the pursuit of the clearest substantial right. In the same manner, petitioners cannot arrogate into the powers of voluntary arbitrators the original and exclusive jurisdiction of Labor Arbiters over unfair labor practices, termination disputes, and claims for damages, in the absence of an express agreement between the parties in order for Article 262 of the Labor Law to apply in the case at bar.

PANTRANCO NORTH EXPRESS INC. VS. NLRC [G.R. No. 95940, July 24, 1996]

Facts: Private respondent Suñiga was hired by petitioner as a bus conductor, and joined the Pantranco Employees Association-PTGWO. He continued in petitioner's employ until he retired at the age of fifty-two (52) after

188

having rendered twenty five years' service. The basis of his retirement was the compulsory retirement provision of the CBA between the petitioner and the union. Private respondent received P49,300.00 as retirement pay. Suñiga filed a complaint for illegal dismissal against petitioner with the Sub-Regional Arbitration Branch of the respondent Commission. LA found that the three complainants are illegally and unjustly dismissed and ordered the respondent to reinstate them to their former or substantially equivalent positions without loss of seniority rights with full backwages and other benefits.

The amounts already received by complainants shall be considered as advanced payment of their retirement pay which shall be deducted when they shall actually retire or (be) separated from the service. The order of reinstatement was immediately executory even pending appeal. Petitioner appealed to public respondent, which issued the questioned Resolution affirming the labor arbiter's decision in toto. Issues: 1. Whether or not the Labor Arbiter has jurisdiction 2. Whether or not the CBA stipulation on compulsory retirement after twenty-five years of service is legal and enforceable. Ruling: I. Jurisdiction of Labor Arbiter Pantranco contends that the labor arbiter had no jurisdiction because the dispute concerns a provision of the CBA and its interpretation. It claims that the case falls under the jurisdiction of the voluntary arbitrator or panel of arbitrators under Article 261 of the Labor Code. The Labor Arbiter believed otherwise. First, this is a complaint of illegal dismissal of which original and exclusive jurisdiction under Article 217 has been conferred to the Labor Arbiters. The interpretation of the CBA or enforcement of the company policy is only corollary to the complaint of illegal dismissal. Otherwise, an employee who was on AWOL, or who committed offenses contrary to the personnel polices (sic) can no longer file a case of illegal dismissal because the discharge is premised on the interpretation or enforcement of the company polices. Second. Respondent voluntarily submitted the case to the jurisdiction of this labor tribunal. It adduced arguments to the legality of its act, whether such act may be retirement and/or dismissal, and prayed for reliefs on the merits of the case. A litigant cannot pray for reliefs on the merits and at the same time attacks (sic) the jurisdiction of the tribunal. A person cannot have one's cake and eat it too. x x x." The Court agrees with the public respondent's affirmance of the arbiter's decision in respect of the question of jurisdiction. In Sanyo Philippines Workers Union — PSSLU vs. Cañizares “x x x Hence, only disputes involving the union and the company shall be referred to the grievance machinery or voluntary arbitrators. In the instant case, both the union and the company are united or have come to an agreement regarding the dismissal of private respondents. No grievance between them exists which could be brought to a grievance machinery. The problem or dispute in the present case is between the union and the company on the one hand and some union and non-union members who were dismissed, on the other hand. The dispute has to be settled before an impartial body. The grievance machinery with members designated by the union and the company cannot be expected to be impartial against the dismissed employees. Due process demands that the dismissed workers’ grievances be ventilated before an impartial body. Since there has already been an actual termination, the matter falls within the jurisdiction of the Labor Arbiter." Applying the same rationale to the case at bar, it cannot be said that the "dispute" is between the union and petitioner company because both have previously agreed upon the provision on "compulsory

189

retirement" as embodied in the CBA. Also, it was only private respondent on his own who questioned the compulsory retirement. Thus, the case is properly denominated as a "termination dispute" which comes under the jurisdiction of labor arbiters. Therefore, public respondent did not commit a grave abuse of discretion in upholding the jurisdiction of the labor arbiter over this case. II. Legality and enforceability of CBA stipulation YES, the CBA stipulation on compulsory retirement after twenty-five years of service is legal and enforceable. Article XI, Section 1 (e) (5) of the CBA states: "Section 1. xxx

xxx

The COMPANY shall formulate a retirement plan with the following main features: xxx

(e) The COMPANY agrees to grant the retirement benefits herein provided to regular employees who may be separated from the COMPANY for any of the following reasons: xxx

xxx

xxx

A CBA incorporates the agreement reached after negotiations between employer and bargaining agent with respect to terms and conditions of employment. A CBA is not an ordinary contract. "(A)s a labor contract within the contemplation of Article 1700 of the Civil Code of the Philippines which governs the relations between labor and capital, (it) is not merely contractual in nature but impressed with public interest, thus it must yield to the common good. As such, it must be construed liberally rather than narrowly and technically, and the courts must place a practical and realistic construction upon it, giving due consideration to the context in which it is negotiated and purpose which it is intended to serve.” Being a product of negotiation, the CBA between the petitioner and the union intended the provision on compulsory retirement to be beneficial to the employees-union members, including herein private respondent. When private respondent ratified the CBA with the union, he not only agreed to the CBA but also agreed to conform to and abide by its provisions. Thus, it cannot be said that he was illegally dismissed when the CBA provision on compulsory retirement was applied to his case. Incidentally, we call attention to Republic Act No. 7641, known as "The Retirement Pay Law," which went into effect on January 7, 1993. Although passed many years after the compulsory retirement of herein private respondent, nevertheless, the said statute sheds light on the present discussion when it amended Art. 287 of the Labor Code, to make it read as follows:

"ART. 7. Retirement. — Any employee may be retired upon reaching the retirement age establish in the collective bargaining agreement or other applicable employment contract. xxx

xxx

xxx

In the absence of a retirement plan or agreement providing for retirement benefits of employees in the establishment, an employee upon reaching the age of sixty (60) years or more, but not beyond sixty-five (65) years which is hereby declared the compulsory retirement age, who has served at least five (5) years in the said establishment may retire x x x." The aforequoted provision makes clear the intention and spirit of the law to give employers and employees a free hand to determine and agree upon the terms and conditions of retirement. Providing in a CBA for compulsory retirement of employees after twenty-five (25) years of service is legal and enforceable so long as the parties agree to be governed by such CBA. The law presumes that employees know what they want

190

and what is good for them absent any showing that fraud or intimidation was employed to secure their consent thereto. On this point then, public respondent committed a grave abuse of discretion in affirming the decision of the labor arbiter. The compulsory retirement of private respondent effected in accordance with the CBA is legal and binding. SILVA ET AL vs. NLRC [G.R. No. 110226, June 19, 1997] Facts: Petitioners, then rank-and-file employees and members of Philtread Workers Union (PWU), volunteered for, and availed of, the retrenchment program instituted by Philtread with the understanding that they would have priority in re-employment in the event that the company recovers from its financial crisis, in accordance with their CBA. Philtread, apparently having recovered from its financial reverses, expanded its operations and hired new personnel. Upon discovery of this development, petitioners filed their respective applications for employment with Philtread, which however, merely agreed to consider them for future vacancies. Subsequent demands for re-employment made by petitioners were ignored. Even the request of the incumbent union for Philtread to stop hiring new personnel until petitioners were first hired failed to elicit any favorable response. Thus, petitioners lodged a complaint with the NCR Arbitration Branch of the NLRC for unfair labor practice, damages and attorney’s fees against Philtread. LA dismissed the complaint but directing Philtread to give petitioners priority in hiring, as well as those former employees similarly situated for available positions provided they meet the necessary current qualifications. Petitioners duly appealed the decision of the Labor Arbiter to the NLRC. Philtread opted not to interpose an appeal despite the Labor Arbiter’s failure to rule squarely on the question of jurisdiction. NLRC revered the decision of the LA. It directed Philtread to re-employ petitioners and other employees similarly situated, regardless of age qualifications and other pre-employment conditions, subject only to existing vacancies and a finding of good physical condition.

Atty. Borreta filed with the NLRCVan ex parte manifestation explaining that he was returning the copy of the resolution rendered on April 15, 1992, which, according to him, was erroneously served on him by the process server of the NLRC. NLRC, acting on a motion for reconsideration filed by Atty. Gutierrez, promulgated one of its challenged resolutions dismissing the complaint of petitioners. It ruled that while petitioners had standing to sue, the complaint should have been filed with the voluntary arbitrator, pursuant to Article 261 of the Labor Code, since the primary issue was the implementation and interpretation of the CBA. Petitioners moved for reconsideration. NLRC affirmed its earlier resolution, ruling that even before the amendatory law took effect, matters involving bargaining agreements were already within the exclusive jurisdiction of the voluntary arbitrator, as set forth in Article 262 of the Labor Code. Hence, this petition. Issues: 1. Whether or not NLRC acted with grave abuse of discretion for issuing 2 resolutions which reconsidered a resolution it rendered on April 15, 1992, despite that the latter became final and executory when Philtread failed to seasonably file a motion for reconsideration within the ten-day reglementary period required by Article 223 of the Labor Code. 2. Who has jurisdiction, the voluntary arbitrator or LA and NLRC? Can RA 6715 be applied retroactively? Ruling: 1.

YES, NLRC acted with grave abuse of discretion.

191

The seasonable filing of a motion for reconsideration within the 10-day reglementary period following the receipt by a party of any order, resolution or decision of the NLRC, is a mandatory requirement to forestall the finality of such order, resolution or decision. The statutory bases for this are found in Article 223 of the Labor Code and Section 14, Rule VII of the New Rules of Procedure of the NLRC. Philtread’s counsel filed a motion for reconsideration 31 days after receipt of said resolution. It was thus incumbent upon the NLRC to have dismissed outright Philtread’s late motion for reconsideration. By doing exactly the opposite, its actuation was not only whimsical and capricious but also a demonstration of its utter disregard for its very own rules. Certiorari, therefore, lies. NLRC, as an administrative and quasi-judicial body, is not bound by the rigid application of technical rules of procedure in the conduct of its proceedings. However, the filing of a motion for reconsideration and filing it ON TIME are not mere technicalities of procedure. These are jurisdictional and mandatory requirements which must be strictly complied with. Although there are exceptions to said rule, the case at bar presents no peculiar circumstances warranting a departure therefrom. According to PhilTread, jurisdiction lies instead with the voluntary arbitrator so that when the Labor Arbiter and the NLRC took cognizance of the case, their decisions thereon were null and void and, therefore, incapable of attaining finality. In short, Philtread maintains that the ten-day reglementary period could not have started running and, therefore, its motion could not be considered late. The argument is not tenable. While we agree with the dictum that a void judgment cannot attain finality, said rule, however, is only relevant if the tribunal or body which takes cognizance of a particular subject matter indeed lacks jurisdiction over the same. In this case, the rule adverted to is misapplied for it is actually the Labor Arbiter and the NLRC which possess jurisdiction over petitioners’ complaint and NOT the voluntary arbitrator, as erroneously contended by Philtread. There is confusion in the minds of both Philtread and the NLRC with respect to the proper jurisdiction of the voluntary arbitrator. When the issue concerns an interpretation or implementation of the CBA, one cannot immediately jump to the conclusion that jurisdiction is with the voluntary arbitrator. There is an equally important need to inquire further if the disputants involved are the union and the employer; otherwise, the voluntary arbitrator cannot assume jurisdiction. In Sanyo Philippines Workers Union - PSSLU v. Canizares, we clarified the jurisdiction of the voluntary arbitrator: We hold that the Labor Arbiter and not the Grievance Machinery provided for in the CBA has the jurisdiction to hear and decide the complaints of the private respondents. While it appears that the dismissal of the private respondents was made upon the recommendation of PSSLU pursuant to the union security clause provided in the CBA, We are of the opinion that these facts do not come within the phrase ’grievances arising from the interpretation or implementation of (their) CBA and those arising from the interpretation or enforcement of company personnel policies,’ the jurisdiction of which pertains to the Grievance Machinery or thereafter, to a voluntary arbitrator or panel of voluntary arbitrators. Article 260 of the Labor Code on grievance machinery and voluntary arbitrator states that ‘(t)he parties to a CBA shall include therein provisions that will ensure the mutual observance of its terms and conditions. They shall establish a machinery for the adjustment and resolution of grievances arising from the interpretation or implementation of their CBA and those arising from the interpretation or enforcement of company personnel policies.’ The parties to a CBA shall name or designate their respective representatives to the grievance machinery and if the grievance is not settled in that level, it shall automatically be referred to voluntary arbitrators (or panel of voluntary arbitrators) designated in advance by the parties. It need not be mentioned that the parties to a CBA are the union and the company. Hence, only disputes involving the union and the company shall be referred to the grievance machinery or voluntary arbitrators.” In the case at bar, since the contending parties in the instant case are not the union and Philtread, then pursuant to the Sanyo doctrine, it is not the voluntary arbitrator who can take cognizance of the complaint, notwithstanding Philtread’s claim that the real issue is the interpretation of the CBA provision on reemployment.

192

If the voluntary arbitrator could not have assumed jurisdiction over the case, did the Labor Arbiter and the NLRC validly acquire jurisdiction when both of them entertained the complaint? At the time petitioners filed their complaint for unfair labor practice, damages and attorney’s fees, the governing provision of the Labor Code with respect to the jurisdiction of the Labor Arbiter and the NLRC was Article 217: “ART. 217. Jurisdiction of Labor Arbiters and the Commission. (a) The Labor Arbiters shall have the original and exclusive jurisdiction to hear and decide within thirty (30) working days after submission of the case by the parties for decision, the following cases involving all workers, whether agricultural or non-agricultural: 1. Unfair labor practice cases; 2. Those that workers may file involving wages, hours of work and other terms and conditions of employment; 3. All money claims of workers, including those based on non-payment or underpayment of wages, overtime compensation, separation pay and other benefits provided by law or appropriate agreement, except claims for employees’ compensation, social security, medicare and maternity benefits; 4. Cases involving household services; and 5. Cases arising from any violation of Article 265 of this Code, including questions involving the legality of strikes and lockouts. (b) The Commission shall have exclusive appellate jurisdiction over all cases decided by Labor Arbiters.” Articles 261 and 262, on the other hand, defined the jurisdiction of the voluntary arbitrator, viz.: “ART. 261. Grievance machinery. - Whenever a grievance arises from the interpretation or implementation of a collective agreement, including disciplinary actions imposed on members of the bargaining unit, the employer and the bargaining representative shall meet to adjust the grievance. Where there is no collective agreement and in cases where the grievance procedure as provided herein does not apply, grievances shall be subject to negotiation, conciliation or arbitration as provided elsewhere in this Code. ART. 262. Voluntary arbitration. - All grievances referred to in the immediately preceding Article which are not settled through the grievance procedure provided in the collective agreement shall be referred to voluntary arbitration prescribed in said agreement: Provided, That termination disputes shall be governed by Article 278 of this Code, as amended, unless the parties agree to submit them to voluntary arbitration.” Under the above provisions then prevailing, one can understand why petitioners lodged their complaint for ULP with the Labor Arbiter. To their mind, Philtread’s refusal to re-employ them was tantamount to a violation of the re-employment clause in the 1983 CBA which was also substantially reproduced in the 1986 CBA. At the time, any violation of the CBA was unqualifiedly treated as ULP of the employer falling within the competence of the Labor Arbiter to hear and decide. “ART. 248. Unfair labor practices of employers. - It shall be unlawful for an employer to commit any of the following unfair labor practice: xxx

xxx (i) To violate a collective bargaining agreement.”

On March 21, 1989, however, Republic Act 6715, or the so-called “Herrera-Veloso Amendments,” took effect, amending several provisions of the Labor Code, including the respective jurisdictions of the Labor Arbiter, the NLRC and the voluntary arbitrator. “ART. 217. Jurisdiction of Labor Arbiters and the Commission. - (a) Except as otherwise provided under this Code the Labor Arbiters shall have original and exclusive jurisdiction to hear and decide, within thirty (30)

193

calendar days after the submission of the case by the parties for decision without extension, even in the absence of stenographic notes, the following cases involving all workers, whether agricultural or nonagricultural: 1. Unfair labor practice cases; 2. Termination disputes; 3. If accompanied with a claim for reinstatement, those cases that workers may file involving wages, rates of pay, hours of work and other terms and conditions of employment; 4. Claims for actual, moral, exemplary and other forms of damages arising from the employer-employee relations; 5. Cases arising from any violation of Article 264 of this Code, including questions involving the legality of strikes and lockouts; and 6. Except claims for Employees Compensation, Social Security, Medicare and maternity benefits, all other claims, arising from employer-employee relations, including those of persons in domestic or household service, involving an amount exceeding five thousand pesos (P5,000.00) regardless of whether accompanied with a claim for reinstatement. (b) The Commission shall have exclusive appellate jurisdiction over all cases decided by Labor Arbiters. (c) Cases arising from the interpretation or implementation of collective bargaining agreements and those arising from the interpretation or enforcement of company personnel policies shall be disposed of by the Labor Arbiter by referring the same to the grievance machinery and voluntary arbitration as may be provided in said agreements.” “ART. 261. Jurisdiction of Voluntary Arbitrators or panel of Voluntary Arbitrators. - The Voluntary Arbitrator or panel of Voluntary Arbitrators shall have original and exclusive jurisdiction to hear and decide all unresolved grievances arising from the interpretation or implementation of the Collective Bargaining Agreement and those arising from the interpretation or enforcement of company personnel policies referred to in the immediately preceding article. Accordingly, violations of a Collective Bargaining Agreement, except those which are gross in character, shall no longer be treated as unfair labor practice and shall be resolved as grievances under the Collective Bargaining Agreement. For purposes of this article, gross violations of Collective Bargaining Agreement shall mean flagrant and/or malicious refusal to comply with the economic provisions of such agreement. x x x.” (Underscoring supplied) “ART. 262. Jurisdiction over other labor disputes. - The Voluntary Arbitrator or panel of Voluntary Arbitrators, upon agreement of the parties, shall also hear and decide all other labor disputes including unfair labor practices and bargaining deadlocks.” With the amendments introduced by RA 6715, it can be gleaned that the Labor Arbiter still retains jurisdiction over ULP cases. There is, however, a significant change: The unqualified jurisdiction conferred upon the Labor Arbiter prior to the amendment by RA 6715 has been narrowed down so that “violations of a Collective Bargaining Agreement, except those which are gross in character, shall no longer be treated as unfair labor practice but as grievances under the Collective Bargaining Agreement. It is further stated that “gross violations of Collective Bargaining Agreement shall mean flagrant and/or malicious refusal to comply with the economic provisions of such agreement.” For a ULP case to be cognizable by the Labor Arbiter, and the NLRC to exercise its appellate jurisdiction, the allegations in the complaint should show prima facie the concurrence of two things, namely: (1) gross violation of the CBA; AND (2) the violation pertains to the economic provisions of the CBA.

194

RA 6715 is in the nature of a curative statute. Thus, it can be applied retroactively to pending cases. Briad Agro Development Corporation v. Dela Cerna - Republic Act No. 6715 has retroactive application. Thus, when this new law divested Regional Directors of the power to hear money claims, the divestment affected pending litigations. (Note that under par. 6, where the claim does not exceed P5,000.00, regional directors have jurisdiction) Garcia v. Martinez - The lack of jurisdiction was cured by the issuance of the amendatory decree which is in the nature of a curative statute with retrospective application to a pending proceeding, like Civil Case No. 9657. With the Briad ruling in place, the implication is that the qualified jurisdiction of the Labor Arbiter and the NLRC should have been applied when the ULP complaint was still pending. This means that petitioners should have been required to show in their complaint the gross nature of the CBA violation, as well as the economic provision violated, without which the complaint would be dismissible. Thus, given the foregoing considerations, may the Briad doctrine be applied to the instant case and cause its dismissal for want of jurisdiction of the Labor Arbiter and the NLRC? NO, it cannot be applied. We adopt instead the more recent case of Erectors, Inc. v. National Labor Relations Commission, where we refused to give retroactive application to Executive Order No. 797 which created the Philippine Overseas Employment Administration (POEA). Under said law, POEA was vested with “original and exclusive jurisdiction over all cases, including money claims, involving employer-employee relations arising out of or by virtue of any law or contract involving Filipino workers for overseas employment,” which jurisdiction was originally conferred upon the Labor Arbiter. As in the instant case, the Labor Arbiter’s assumption of jurisdiction therein was likewise questioned in view of the subsequent enactment of E.O. 797. “The rule is that jurisdiction over the subject matter is determined by the law in force at the time of the commencement of the action. On March 31, 1982, at the time private respondent filed his complaint against the petitioner, the prevailing laws were Presidential Decree No. 1691 and Presidential Decree No. 1391 which vested the Regional Offices of the Ministry of Labor and the Labor Arbiters with ‘original and exclusive jurisdiction over all cases involving employer-employee relations including money claims arising out of any law or contracts involving Filipino workers for overseas employment.’ At the time of the filing of the complaint, the Labor Arbiter had clear jurisdiction over the same. E.O. No. 797 did not divest the Labor Arbiter’s authority to hear and decide the case filed by private respondent prior to its effectivity. Laws should only be applied prospectively unless the legislative intent to give them retroactive effect is expressly declared or is necessarily implied from the language used. We fail to perceive in the language of E.O. No. 797 an intention to give it retroactive effect. E.O. No. 111, amended Article 217 of the Labor Code to widen the worker’s access to the government for redress of grievances by giving the Regional Directors and Labor Arbiters concurrent jurisdiction over cases involving money claims. This amendment, however, created a situation where the jurisdiction of the Regional Directors and the Labor Arbiters overlapped. As a remedy, R.A. 6715 further amended Article 217 by delineating their respective jurisdictions. Under R.A. 6715, the Regional Director has exclusive original jurisdiction over cases involving money claims provided: (1) the claim is presented by an employer or person employed in domestic or household service, or househelper under the Code; (2) the claimant, no longer being employed, does not seek reinstatement; and (3) the aggregate money claim of the employee or househelper does not exceed P5,000.00. All other cases within the exclusive and original jurisdiction of the Labor Arbiter. E.O. No. 111 and R.A. 6715 are therefore curative statutes. A curative statute is enacted to cure defects in a prior law or to validate legal proceedings, instruments or acts of public authorities which would otherwise be void for want of conformity with certain existing legal requirements. The law at bar, E.O. No. 797, is not a curative statute. x x x.” Hence, RA 6715 cannot be applied retroactively.

195

In Briad, the underlying reason for applying RA 6715 retroactively was the fact that prior to its amendment, Article 217 of the Labor Code, as amended by then E.O. 111, created a scenario where the Labor Arbiters and the Regional Directors of the (DOLE) had overlapping jurisdiction over money claims. This situation was viewed as a defect in the law so that when RA No. 6715 was passed and delineated the jurisdiction of the Labor Arbiters and Regional Directors, the Court deemed it a rectification of such defect; hence, the conclusion that it was curative in nature and, therefore, must be applied retroactively. The same thing cannot be said of the case at bar. Like in Erectors, the instant case presents no defect in the law requiring a remedy insofar as the jurisdiction of the Labor Arbiter and the Voluntary Arbitrator is concerned. There is here no overlapping of jurisdiction to speak of because matters involving interpretation and implementation of CBA provisions, as well as interpretation and enforcement of company personnel policies, have always been determined by the Voluntary Arbitrator even prior to RA 6715. Similarly, all ULP cases were exclusively within the jurisdiction of the Labor Arbiter. What RA 6715 merely did was to reapportion the jurisdiction over ULP cases by conferring exclusive jurisdiction over such ULP cases that do not involve gross violation of a CBA’s economic provision upon the voluntary arbitrator. We do not see anything in the act of re-apportioning jurisdiction curative of any defect in the law as it stood prior to the enactment of RA 6715. The Court view it as merely a matter of change in policy of the lawmakers, especially since the 1987 Constitution adheres to the preferential use of voluntary modes of dispute settlement. This, instead of the inherent defect in the law, must be the rationale that prompted the amendment. Hence, we uphold the jurisdiction of the Labor Arbiter which attached to this case at the time of its filing on December 5, 1988. WHEREFORE, the instant petition is hereby GRANTED. UNION OF NESTLE WORKERS CAGAYAN DE ORO FACTORY vs. NESTLE PHILIPPINES INC. [G.R. No. 148303, October 17, 2002] Facts: On August 1, 1999, Nestle Philippines, Inc. adopted Policy No. HRM 1.8, otherwise known as the "Drug Abuse Policy." Pursuant to this policy, the management shall conduct simultaneous drug tests on all employees from different factories and plants. However, there was resistance to the policy in the Nestle Cagayan de Oro factory. Out of 496 employees, only 141 or 28.43% submitted themselves to drug testing. On August 20, 1999, the Union of Nestle Workers Cagayan de Oro Factory and its officers, petitioners, wrote Nestle challenging the implementation of the policy and branding it as a mere subterfuge to defeat the employees’ constitutional rights. Nestle claimed that the policy is in keeping with the government’s thrust to eradicate the proliferation of drug abuse, explaining that the company has the right: (a) to ensure that its employees are of sound physical and mental health and (b) to terminate the services of an employee who refuses to undergo the drug test. On August 23, 1999, petitioners filed with the RTC, Cagayan de Oro City, a complaint for injunction with prayer for the issuance of a temporary restraining order against Nestle. On August 24, 1999, the RTC issued a temporary restraining order enjoining respondents. Nestle filed a motion to dismiss the complaint on the ground that the RTC has no jurisdiction over the case as it involves a labor dispute or enforcement of a company personnel policy cognizable by the Voluntary Arbitrator or Panel of Voluntary Arbitrators. Petitioners filed their opposition, contending that the RTC has jurisdiction since the complaint raises purely constitutional and legal issues. RTC dismissed the complaint for lack of jurisdiction since the constitutional issue is closely related or intertwined with the labor issue, so much so that this Court is inclined to believe that it has no jurisdiction but the NLRC. Petitioners filed with this Court a petition for certiorari under Rule 65 of the 1997 Rules of Civil Procedure but was referred to the Court of Appeals.

196

The Appellate Court rendered its Decision dismissing the petition for petitioners xxx submitted themselves to the drug test required by management and was confirmed free from illegal drug abuse. In view thereof, the instant petition, which prays for an injunction had become moot and academic. The remedy of injunction could no longer be entertained because the act sought to be prevented had been consummated. Hence this petition for review on certiorari. Issues: 1. Whether or not the Regional Trial Court has jurisdiction over petitioners’ suit for injunction 2. Whether or not petitioners’ resort to certiorari under Rule 65 is in order. Ruling: I. Considering that the Drug Abuse Policy is a company personnel policy, it is the Voluntary Arbitrators or Panel of Voluntary Arbitrators, not the RTC, which exercises jurisdiction over this case. On the first issue, we hold that petitioners’ insistence that the RTC has jurisdiction over their complaint since it raises constitutional and legal issues is sorely misplaced. The fact that the complaint was denominated as one for injunction does not necessarily mean that the RTC has jurisdiction. Well-settled is the rule that jurisdiction is determined by the allegations in the complaint. It is indubitable from the allegations in the complaint that petitioners are not per se questioning "whether or not the person will undergo the drug test" or the constitutionality or legality of the Drug Abuse Policy. They are assailing the manner by which respondents are implementing the policy. According to them, it is "arbitrary in character" because: (1) the employees were not consulted prior to its implementation; (2) the policy is punitive inasmuch as an employee who refuses to abide with the policy may be dismissed from the service; and (3) such implementation is subject to limitations provided by law which were disregarded by the management. Is the complaint, on the basis of its allegations, cognizable by the RTC? Respondent Nestle’s Drug Abuse Policy is a guiding principle adopted by Nestle to safeguard its employees’ welfare and ensure their efficiency and well-being. To our minds, this is a company personnel policy. In San Miguel Corp. vs. NLRC, this Court held: "Company personnel policies are guiding principles stated in broad, long-range terms that express the philosophy or beliefs of an organization’s top authority regarding personnel matters. They deal with matter affecting efficiency and well-being of employees and include, among others, the procedure in the administration of wages, benefits, promotions, transfer and other personnel movements which are usually not spelled out in the collective agreement." Considering that the Drug Abuse Policy is a company personnel policy, it is the Voluntary Arbitrators or Panel of Voluntary Arbitrators, not the RTC, which exercises jurisdiction over this case. Article 261 of the Labor Code, as amended, pertinently provides: Art. 261. Jurisdiction of Voluntary Arbitrators or Panel of Voluntary Arbitrators. – The Voluntary Arbitrator or panel of Voluntary Arbitrators shall have original and exclusive jurisdiction to hear and decide all unresolved grievances arising from the interpretation or implementation of the Collective Bargaining Agreement and those arising from the interpretation or enforcement of company personnel policies x x x." II. What should have interposed is an appeal to the Court of Appeals, not a petition for certiorari since the assailed RTC order is final. Certiorari is not a substitute for an appeal. With respect to the second issue raised by petitioners, what they should have interposed is an appeal to the Court of Appeals, not a petition for certiorari which they initially filed with this Court, since the assailed RTC order is final. Certiorari is not a substitute for an appeal. For certiorari to prosper, it is not enough that the trial court committed grave abuse of discretion amounting to lack or excess of jurisdiction, as alleged by petitioners. The requirement that there is no appeal, nor any plain, speedy and adequate remedy in the

197

ordinary course of law must likewise be satisfied. 10 We must stress that the remedy of appeal was then available to petitioners, but they did not resort to it. And while this Court in exceptional instances allowed a party’s availment of certiorari instead of appeal, we find that no such exception exists here.

TABIGUE ET AL. vs. INTERNATIONAL COPRA EXPORT CORPORATION [G.R. No. 183335, December 23, 2009] Facts: Juanito Tabigue and his 19 co-petitioners, employees of International Copra Export Corp-oration (INTERCO), filed a Notice of Preventive Mediation with the Department of Labor and Employment – National Conciliation and Mediation Board (NCMB), Regional Branch No. XI, Davao City against respondent, for violation of Collective Bargaining Agreement (CBA) and failure to sit on the grievance conference/meeting. As the parties failed to reach a settlement before the NCMB, petitioners requested to elevate the case to voluntary arbitration. The NCMB thus set a date for the parties to agree on a Voluntary Arbitrator. Before the parties could finally meet, respondent presented before the NCMB a letter stating that petitioners “are not duly authorized by [the] board or the officers to represent the union, [hence] . . . all actions, representations or agreements made by these people with the management will not be honored or recognized by the union.” Respondent thus moved to dismiss petitioners’ complaint for lack of jurisdiction. The parties having failed to arrive at a settlement, NCMB Director Teodorico O. Yosores wrote petitioner Alex Bibat and respondent’s plant manager Tangente of the lack of willingness of both parties to submit to voluntary arbitration, which willingness is a pre-requisite to submit the case thereto; and that under the CBA forged by the parties, the union is an indispensable party to a voluntary arbitration but that since Tan informed respondent that the union had not authorized petitioners to represent it, it would be absurd to bring the case to voluntary arbitration. The NCMB Director thus concluded that “the demand of [petitioners] to submit the issues . . . to voluntary arbitration CAN NOT BE GRANTED.” He thus advised petitioners to avail of the compulsory arbitration process to enforce their rights. On petitioners’ Motion for Reconsideration,the NCMB Director, by letter of April 11, 2007 to petitioners’ counsel, stated that the NCMB “has no rule-making power to decide on issues [as it] only facilitates settlement among the parties to . . . labor disputes.” Via Petition for Review before the Court of Appeals, it dismissed said petition, x x x x Considering that NCMB is not a quasi-judicial agency exercising quasi-judicial functions but merely a conciliatory body for the purpose of facilitating settlement of disputes between parties, its decisions or that of its authorized officer cannot be appealed either through a petition for review under Rule 43 or under Rule 65 of the Revised Rules of Court. Petitioners filed the present Petition for Review on Certiorari. Issues: 1. Whether or not the NCMB, when exercising adjudicative powers acts as a quasi-judicial agency. 2. Whether or not final judgments of RTCs and quasi-judicial bodies like the NCMB are appealable by petition for review to the CA. Ruling: The petition fails.

198

Section 7 of Rule 43 of the Rules of Court provides that: The failure of the petitioner to comply with any of the foregoing requirements regarding the payment of the docket and other lawful fees, the deposit for costs, proof of service of the petition, and the contents of and the documents which should accompany the petition shall be sufficient ground for the dismissal thereof. (underscoring and emphasis supplied) Petitioners claim that they had completed the payment of the appellate docket fee and other legal fees when they filed their motion for reconsideration before the Court of Appeals. While the Court has, in the interest of justice, given due course to appeals despite the belated payment of those fees, petitioners have not proffered any reason to call for a relaxation of the above-quoted rule. On this score alone, the dismissal by the appellate court of petitioners’ petition is in order. But even if the above-quoted rule were relaxed, the appellate court’s dismissal would just the same be sustained. Under Section 9 (3) of the Judiciary Reorganization Act of 1980,the Court of Appeals exercises exclusive appellate jurisdiction over all final judgments, decisions, resolutions, orders or awards of Regional Trial Courts and quasi-judicial agencies, instrumentalities, boards or commissions. Rule 43 of the Rules of Court under applies to awards, judgments, final orders or resolutions of or authorized by any quasi-judicial agency in the exercise of its quasi-judicial functions. A[n agency] is said to be exercising judicial function where [it] has the power to determine what the law is and what the legal rights of the parties are, and then undertakes to determine these questions and adjudicate upon the rights of the parties. Quasi-judicial function is a term which applies to the action, discretion, etc. of public administrative officers or bodies, who are required to investigate facts or ascertain the existence of facts, hold hearings, and draw conclusions from them as a basis for their official action and to exercise discretion of a judicial nature. (underscoring supplied) Given NCMB’s following functions, as enumerated in Section 22 of Executive Order No. 126 (the Reorganization Act of the Ministry of Labor and Employment), viz: (a) Formulate policies, programs, standards, procedures, manuals of operation and guidelines pertaining to effective mediation and conciliation of labor disputes; (b)

Perform preventive mediation and conciliation functions;

(c) Coordinate and maintain linkages with other sectors or institutions, and other government authorities concerned with matters relative to the prevention and settlement of labor disputes; (d) Formulate policies, plans, programs, standards, procedures, manuals of operation and guidelines pertaining to the promotion of cooperative and non-adversarial schemes, grievance handling, voluntary arbitration and other voluntary modes of dispute settlement; (e) Administer the voluntary arbitration program; maintain/update a list of voluntary arbitrations; compile arbitration awards and decisions; (f) Provide counseling and preventive mediation assistance particularly in the administration of collective agreements; (g) Monitor and exercise technical supervision over the Board programs being implemented in the regional offices; and (h)

Perform such other functions as may be provided by law or assigned by the Minister;

It cannot be considered a quasi-judicial agency. Petitioners have not, been duly authorized to represent the union. Apropos is this Court’s pronouncement in Atlas Farms, Inc. v. National Labor Relations Commission, viz: x x x Pursuant to Article 260 of the Labor Code, the parties to a CBA shall name or designate their respective representatives to the grievance machinery and if the grievance is unsettled in that level, it shall automatically be referred to the voluntary

199

arbitrators designated in advance by parties to a CBA. Consequently only disputes involving the union and the company shall be referred to the grievance machinery or voluntary arbitrators. Clutching at straws, petitioners invoke the first paragraph of Article 255 of the Labor Code which states: Art. 255. The labor organization designated or selected by the majority of the employees in an appropriate collective bargaining unit shall be the exclusive representative of the employees in such unit for the purpose of collective bargaining. However, an individual employee or group of employees shall have the right at any time to present grievances to their employer. To petitioners, the immediately quoted provision “is meant to be an exception to the exclusiveness of the representative role of the labor organization/union.”

SAINT LOUIS UNIVERSITY vs. COBARRUBIAS [G.R. No. 187104, August 3, 2010] Facts: Evangeline C. Cobarrubias is an associate professor of ST. Louis University, College of Human Sciences. She is an active member of the Union of Faculty and Employees of Saint Louis University (UFESLU). The 2001-2006 and 2006-2011 Collective Bargaining Agreements (CBAs) between SLU and UFESLU contain the following common provision on forced leave: Section 7.7. For teaching employees in college who fail the yearly evaluation, the following provisions shall apply: (a) Teaching employees who are retained for three (3) cumulative years in five (5) years shall be on forced leave for one (1) regular semester during which period all benefits due them shall be suspended. SLU placed Cobarrubias on forced leave for the first semester of School Year (SY) 2007-2008 when she failed the evaluation for SY 2002-2003, SY 2005-2006, and SY 2006-2007, with the rating of 85, 77, and 72.9 points, respectively, below the required rating of 87 points. To reverse the imposed forced leave, Cobarrubias sought recourse from the CBA’s grievance machinery. Despite the conferences held, the parties still failed to settle their dispute, prompting Cobarrubias to file a case for illegal forced leave or illegal suspension with the NCMB. When circulation and mediation again failed, the parties submitted the issues between them for voluntary arbitration before Voluntary Arbitrator (VA) Daniel T. Fariñas. The voluntary arbitrator dismissed the case. He noted that the CBA clearly authorized SLU to place its teaching employees on forced leave when they fail in the evaluation for three (3) years within a five-year period, without a distinction on whether the three years fall within one or two CBA periods. Cobarrubias filed with the CA a petition for review under Rule 43 of the Rules of Court, but failed to pay the required filing fees and to attach to the petition copies of the material portions of the record.14 SLU insisted that the VA decision had already attained finality for Cobarrubias’ failure to pay the docket fees on time. The CA brushed aside SLU’s insistence on the finality of the VA decision and annulled it, declaring that the "three (3) cumulative years in five (5) years" phrase in Section 7.7(a) of the 2006-2011 CBA means within the five-year effectivity of the CBA. Thus, the CA ordered SLU to pay all the benefits due Cobarrubias for the first semester of SY 2007-2008, when she was placed on forced leave.

Issue:

200

Whether the CA erred in reinstating Cobarrubias’ petition despite her failure to pay the appeal fee within the reglementary period, and in reversing the VA decision. To state the obvious, the appeal fee is a threshold issue that renders all other issues unnecessary if SLU’s position on this issue is correct. Ruling: Payment of Appellate Court Docket Fees Appeal is not a natural right but a mere statutory privilege, thus, appeal must be made strictly in accordance with the provision set by law.25 Rule 43 of the Rules of Court provides that appeals from the judgment of the VA shall be taken to the CA, by filing a petition for review within fifteen (15) days from the receipt of the notice of judgment.26 Furthermore, upon the filing of the petition, the petitioner shall pay to the CA clerk of court the docketing and other lawful fees non-compliance with the procedural requirements shall be a sufficient ground for the petition’s dismissal. Thus, payment in full of docket fees within the prescribed period is not only mandatory, but also jurisdictional. It is an essential requirement, without which, the decision appealed from would become final and executory as if no appeal has been filed. As early as the 1932 case of Lazaro v. Endencia and Andres we stressed that the payment of the full amount of the docket fee is an indispensable step for the perfection of an appeal. In Lee v. Republic, we decided that even though half of the appellate court docket fee was deposited, no appeal was deemed perfected where the other half was tendered after the period within which payment should have been made. In Aranas v. Endona, we reiterated that the appeal is not perfected if only a part of the docket fee is deposited within the reglementary period and the remainder is tendered after the expiration of the period. In the present case, Cobarrubias filed her petition for review on December 5, 2007, fifteen (15) days from receipt of the VA decision on November 20, 2007, but paid her docket fees in full only after seventy-two (72) days, when she filed her motion for reconsideration on February 15, 2008 and attached the postal money orders for P4,230.00. Undeniably, the docket fees were paid late, and without payment of the full docket fees, Cobarrubias’ appeal was not perfected within the reglementary period. Exceptions to the Rule on Payment of Appellate Court Docket Fees not applicable Procedural rules do not exist for the convenience of the litigants; the rules were established primarily to provide order to and enhance the efficiency of our judicial system. While procedural rules are liberally construed, the provisions on reglementary periods are strictly applied, indispensable as they are to the prevention of needless delays, and are necessary to the orderly and speedy discharge of judicial business. Viewed in this light, procedural rules are not to be belittled or dismissed simply because their nonobservance may have prejudiced a party's substantive rights; like all rules, they are required to be followed. However, there are recognized exceptions to their strict observance, such as: (1) most persuasive and weighty reasons; (2) to relieve a litigant from an injustice not commensurate with his failure to comply with the prescribed procedure; (3) good faith of the defaulting party by immediately paying within a reasonable time from the time of the default; (4) the existence of special or compelling circumstances; (5) the merits of the case; (6) a cause not entirely attributable to the fault or negligence of the party favored by the suspension of the rules; (7) a lack of any showing that the review sought is merely frivolous and dilatory; (8) the other party will not be unjustly prejudiced thereby; (9) fraud, accident, mistake or excusable negligence without the appellant's fault; (10) peculiar, legal and equitable circumstances attendant to each case; (11) in the name of substantial justice and fair play; (12) importance of the issues involved; and (13) exercise of sound discretion by the judge, guided by all the attendant circumstances.52 Thus, there should be an effort, on the part of the party invoking liberality, to advance a reasonable or meritorious explanation for his/her failure to comply with the rules. In Cobarrubias' case, no such explanation has been advanced. Other than insisting that the ends of justice and fair play are better served if the case is decided on its merits, Cobarrubias offered no excuse for her failure to pay the docket fees in full when she filed her petition for review. To us, Cobarrubias’ omission is fatal to her cause.

201

We, thus, find that the CA erred in reinstating Cobarrubias’ petition for review despite the nonpayment of the requisite docket fees within the reglementary period. The VA decision had lapsed to finality when the docket fees were paid; hence, the CA had no jurisdiction to entertain the appeal except to order its dismissal.

TENG vs. PAHAGAC ET AL. [G.R. No. 169704, November 17, 2010]

Facts: Albert Teng Fish Trading is engaged in deep sea fishing and, for this purpose, owns boats (basnig), equipment, and other fishing paraphernalia. As owner of the business, Teng claims that he customarily enters into joint venture agreements with master fishermen (maestros) who are skilled and are experts in deep sea fishing; they take charge of the management of each fishing venture, including the hiring of the members of its complement. He avers that the maestros hired the respondent workers as checkers to determine the volume of the fish caught in every fishing voyage. On February 20, 2003, the respondent workers filed a complaint for illegal dismissal against Albert Teng Fish Trading, Teng, and Chua before the NCMB, Region Branch No. IX, Zamboanga City. The VA rendered a decision in Teng’s favor and declared that no employer-employee relationship existed between Teng and the respondent workers. The respondent workers received the VA’s decision on June 12, 2003. They filed a motion for reconsideration, which was denied in an order dated June 27, 2003 and which they received on July 8, 2003. The VA reasoned out that Section 6, Rule VII of the 1989 Procedural Guidelines in the Conduct of Voluntary Arbitration Proceedings (1989 Procedural Guidelines) does not provide the remedy of a motion for reconsideration to the party adversely affected by the VA’s order or decision. Respondent-workers elevated the case to the CA, which reversed the decision of the VA. Teng moved to reconsider the CA’s decision, but the CA denied the motion in its resolution of September 1, 2005. He, thereafter, filed the present Petition for Review on Certiorari under Rule 45 of the Rules of Court, claiming that: a. the VA’s decision is not subject to a motion for reconsideration; and b. no employer-employee relationship existed between Teng and the respondent workers. Teng contends that the VA’s decision is not subject to a motion for reconsideration in the absence of any specific provision allowing this recourse under Article 262-A of the Labor Code. He cites the 1989 Procedural Guidelines, which, as the VA declared, does not provide the remedy of a motion for reconsideration. He claims that after the lapse of 10 days from its receipt, the VA’s decision becomes final and executory unless an appeal is taken. He argues that when the respondent workers received the VA’s decision on June 12, 2003, they had 10 days, or until June 22, 2003, to file an appeal. As the respondent workers opted instead to move for reconsideration, the 10-day period to appeal continued to run; thus, the VA’s decision had already become final and executory by the time they assailed it before the CA on July 21, 2003. Issue: Whether or not the VA’s decision is subject to a motion for reconsideration. Ruling: Article 262-A of the Labor Code does not prohibit the filing of a motion for reconsideration.

202

These rulings fully establish that the absence of a categorical language in Article 262-A does not preclude the filing of a motion for reconsideration of the VA’s decision within the 10-day period. Teng’s allegation that the VA’s decision had become final and executory by the time the respondent workers filed an appeal with the CA thus fails. We consequently rule that the respondent workers seasonably filed a motion for reconsideration of the VA’s judgment, and the VA erred in denying the motion because no motion for reconsideration is allowed. By allowing a 10-day period, the obvious intent of Congress in amending Article 263 to Article 262-A is to provide an opportunity for the party adversely affected by the VA’s decision to seek recourse via a motion for reconsideration or a petition for review under Rule 43 of the Rules of Court filed with the CA. Indeed, a motion for reconsideration is the more appropriate remedy in line with the doctrine of exhaustion of administrative remedies. For this reason, an appeal from administrative agencies to the CA via Rule 43 of the Rules of Court requires exhaustion of available remedies as a condition precedent to a petition under that Rule. The requirement that administrative remedies be exhausted is based on the doctrine that in providing for a remedy before an administrative agency, every opportunity must be given to the agency to resolve the matter and to exhaust all opportunities for a resolution under the given remedy before bringing an action in, or resorting to, the courts of justice. Where Congress has not clearly required exhaustion, sound judicial discretion governs,[38] guided by congressional intent. By disallowing reconsideration of the VA’s decision, Section 7, Rule XIX of DO 40-03 and Section 7 of the 2005 Procedural Guidelines went directly against the legislative intent behind Article 262-A of the Labor Code. These rules deny the VA the chance to correct himself[40] and compel the courts of justice to prematurely intervene with the action of an administrative agency entrusted with the adjudication of controversies coming under its special knowledge, training and specific field of expertise. In this era of clogged court dockets, the need for specialized administrative agencies with the special knowledge, experience and capability to hear and determine promptly disputes on technical matters or intricate questions of facts, subject to judicial review, is indispensable.[41] In Industrial Enterprises, Inc. v. Court of Appeals,[42] we ruled that relief must first be obtained in an administrative proceeding before a remedy will be supplied by the courts even though the matter is within the proper jurisdiction of a court.

CAONG, JR. vs. BEGUALOS [G.R. No. 179428, January 26, 2011] Facts: Petitioners Primo E. Caong, Jr. (Caong), Alexander J. Tresquio (Tresquio), and Loriano D. Daluyon (Daluyon) were employed by respondent Avelino Regualos under a boundary agreement, as drivers of his jeepneys. In November 2001, they filed separate complaints for illegal dismissal against respondent who barred them from driving the vehicles due to deficiencies in their boundary payments. Labor Arbiter decided in favor of respondent. Petitioners appealed to the NLRC which agreed with the decision of the LA and dismissed the appeal. The petitioners’ motion for reconsideration was also denied. Petitioners filed a petition for certiorari with the CA. The CA found no grave abuse of discretion on the part of the NLRC. They filed a motion for reconsideration but was denied. Petitioners thus elevated the case to the Supreme Court arguing that they were illegally dismissed and were not afforded due process of law. Issues: 1. Whether or not the respondents were illegally dismissed.

203

2. Whether or not the policy of suspending drivers pending payment of arrears in their boundary obligations reasonable. Ruling: We have no reason to deviate from such findings. Indeed, petitioners’ suspension cannot be categorized as dismissal, considering that there was no intent on the part of respondent to sever the employer-employee relationship between him and petitioners. In fact, it was made clear that petitioners could put an end to the suspension if they only pay their recent arrears. As it was, the suspension dragged on for years because of petitioners’ stubborn refusal to pay. It would have been different if petitioners complied with the condition and respondent still refused to readmit them to work. Then there would have been a clear act of dismissal. But such was not the case. Instead of paying, petitioners even filed a complaint for illegal dismissal against respondent. Respondent’s policy of suspending drivers who fail to remit the full amount of the boundary was fair and reasonable under the circumstances. Respondent explained that he noticed that his drivers were getting lax in remitting their boundary payments and, in fact, herein petitioners had already incurred a considerable amount of arrears. He had to put a stop to it as he also relied on these boundary payments to raise the full amount of his monthly amortizations on the jeepneys. Demonstrating their obstinacy, petitioners, on the days immediately following the implementation of the policy, incurred deficiencies in their boundary remittances. It is acknowledged that an employer has free rein and enjoys a wide latitude of discretion to regulate all aspects of employment, including the prerogative to instill discipline on his employees and to impose penalties, including dismissal, if warranted, upon erring employees. This is a management prerogative. Indeed, the manner in which management conducts its own affairs to achieve its purpose is within the management’s discretion. The only limitation on the exercise of management prerogative is that the policies, rules, and regulations on work-related activities of the employees must always be fair and reasonable, and the corresponding penalties, when prescribed, commensurate to the offense involved and to the degree of the infraction A company policy must be implemented in such manner as will accord social justice and compassion to the employee. In case of noncompliance with the company policy, the employer must consider the surrounding circumstances and the reasons why the employee failed to comply. When the circumstances merit the relaxation of the application of the policy, then its noncompliance must be excused.

ESTATE OF DULAY vs. ABOITIZ JEBSEN MARITIME INC. ET AL. [G.R. No. 172642, June 13, 2012] Facts: Nelson R. Dulay (Nelson, for brevity) was employed by [herein respondent] General Charterers, Inc. (GCI), a subsidiary of co-petitioner [herein co-respondent] Aboitiz Jebsen Maritime, Inc. since 1986. He initially worked as an ordinary seaman and later as bosun on a contractual basis. From September 3, 1999 up to July 19, 2000, Nelson was detailed in petitioners' vessel, the MV Kickapoo Belle. On August 13, 2000, or 25 days after the completion of his employment contract, Nelson died due to acute renal failure secondary to septicemia. At the time of his death, Nelson was a bona fide member of the Associated Marine Officers and Seaman's Union of the Philippines (AMOSUP), GCI's collective bargaining agent. Nelson's widow, Merridy Jane, thereafter claimed for death benefits through the grievance procedure of the Collective Bargaining Agreement (CBA) between AMOSUP and GCI. However, on January 29, 2001, the grievance procedure was "declared deadlocked" as petitioners refused to grant the benefits sought by the widow. On March 5, 2001, Merridy Jane filed a complaint with the NLRC Sub-Regional Arbitration Board in General Santos City against GCI for death and medical benefits and damages.

204

On March 8, 2001, Joven Mar, Nelson's brother, received P20,000.00 from [respondents] pursuant to article 20(A)2 of the CBA and signed a "Certification" acknowledging receipt of the amount and releasing AMOSUP from further liability. Merridy Jane contended that she is entitled to the aggregate sum of Ninety Thousand Dollars ($90,000.00) pursuant to [A]rticle 20 (A)1 of the CBA . . . Merridy Jane averred that the P20,000.00 already received by Joven Mar should be considered advance payment of the total claim of US$90,000.[00]. [Herein respondents], on the other hand, asserted that the NLRC had no jurisdiction over the action on account of the absence of employer-employee relationship between GCI and Nelson at the time of the latter's death. Nelson also had no claims against petitioners for sick leave allowance/medical benefit by reason of the completion of his contract with GCI. They further alleged that private respondent is not entitled to death benefits because petitioners are only liable for such "in case of death of the seafarer during the term of his contract pursuant to the POEA contract" and the cause of his death is not workrelated. Petitioners admitted liability only with respect to article 20(A)2 [of the CBA]. . . . However, as petitioners stressed, the same was already discharged. The Labor Arbiter ruled in favor of private respondent. It took cognizance of the case by virtue of Article 217 (a), paragraph 6 of the Labor Code and the existence of a reasonable causal connection between the employer-employee relationship and the claim asserted. It ordered the petitioner to pay P4,621,300.00, the equivalent of US$90,000.00 less P20,000.00, at the time of judgment . . . The Labor Arbiter also ruled that the proximate cause of Nelson's death was not work-related. On appeal, [the NLRC] affirmed the Labor Arbiter's decision as to the grant of death benefits under the CBA but reversed the latter's ruling as to the proximate cause of Nelson's death. Issue: Whether or not the CA committed error in ruling that the Labor Arbiter has no jurisdiction over the case. Ruling: Petitioner contends that Section 10 of Republic Act (R.A.) 8042, otherwise known as the Migrant Workers and Overseas Filipinos Act of 1995, vests jurisdiction on the appropriate branches of the NLRC to entertain disputes regarding the interpretation of a collective bargaining agreement involving migrant or overseas Filipino workers. Petitioner argues that the abovementioned Section amended Article 217 (c) of the Labor Code which, in turn, confers jurisdiction upon voluntary arbitrators over interpretation or implementation of collective bargaining agreements and interpretation or enforcement of company personnel policies. The pertinent provisions of Section 10 of R.A. 8042 provide as follows: SEC. 10.Money Claims. — Notwithstanding any provision of law to the contrary, the Labor Arbiters of the National Labor Relations Commission (NLRC) shall have the original and exclusive jurisdiction to hear and decide, within ninety (90) calendar days after filing of the complaint, the claims arising out of an employeremployee relationship or by virtue of any law or contract involving Filipino workers for overseas deployment including claims for actual, moral, exemplary and other forms of damages. Article 217 (c) of the Labor Code, on the other hand, states that: xxx xxx xxx (c)Cases arising from the interpretation or implementation of collective bargaining agreements and those arising from the interpretation or enforcement of company personnel policies shall be disposed by the Labor Arbiter by referring the same to the grievance machinery and voluntary arbitration as may be provided in said agreements. cCSHET

205

On their part, respondents insist that in the present case, Article 217, paragraph (c) as well as Article 261 of the Labor Code remain to be the governing provisions of law with respect to unresolved grievances arising from the interpretation and implementation of collective bargaining agreements. Under these provisions of law, jurisdiction remains with voluntary arbitrators. Article 261 of the Labor Code reads, thus: ARTICLE 261.Jurisdiction of Voluntary Arbitrators or panel of Voluntary Arbitrators. — The Voluntary Arbitrator or panel of Voluntary Arbitrators shall have original and exclusive jurisdiction to hear and decide all unresolved grievances arising from the interpretation or implementation of the Collective Bargaining Agreement and those arising from the interpretation or enforcement of company personnel policies referred to in the immediately preceding article. Accordingly, violations of a Collective Bargaining Agreement, except those which are gross in character, shall no longer be treated as unfair labor practice and shall be resolved as grievances under the Collective Bargaining Agreement. For purposes of this article, gross violations of Collective Bargaining Agreement shall mean flagrant and/or malicious refusal to comply with the economic provisions of such agreement. The Commission, its Regional Offices and the Regional Directors of the Department of Labor and Employment shall not entertain disputes, grievances or matters under the exclusive and original jurisdiction of the Voluntary Arbitrator or panel of Voluntary Arbitrators and shall immediately dispose and refer the same to the Grievance Machinery or Voluntary Arbitration provided in the Collective Bargaining Agreement. The petition is without merit. It is true that R.A. 8042 is a special law governing overseas Filipino workers. However, a careful reading of this special law would readily show that there is no specific provision thereunder which provides for jurisdiction over disputes or unresolved grievances regarding the interpretation or implementation of a CBA. Section 10 of R.A. 8042, which is cited by petitioner, simply speaks, in general, of "claims arising out of an employer-employee relationship or by virtue of any law or contract involving Filipino workers for overseas deployment including claims for actual, moral, exemplary and other forms of damages." On the other hand, Articles 217 (c) and 261 of the Labor Code are very specific in stating that voluntary arbitrators have jurisdiction over cases arising from the interpretation or implementation of collective bargaining agreements. Stated differently, the instant case involves a situation where the special statute (R.A. 8042) refers to a subject in general, which the general statute (Labor Code) treats in particular. 5 In the present case, the basic issue raised by Merridy Jane in her complaint filed with the NLRC is: which provision of the subject CBA applies insofar as death benefits due to the heirs of Nelson are concerned. The Court agrees with the CA in holding that this issue clearly involves the interpretation or implementation of the said CBA. Thus, the specific or special provisions of the Labor Code govern. In any case, the Court agrees with petitioner's contention that the CBA is the law or contract between the parties. Article 13.1 of the CBA entered into by and between respondent GCI and AMOSUP, the union to which petitioner belongs, provides as follows: The Company and the Union agree that in case of dispute or conflict in the interpretation or application of any of the provisions of this Agreement, or enforcement of Company policies, the same shall be settled through negotiation, conciliation or voluntary arbitration. The Company and the Union further agree that they will use their best endeavor to ensure that any dispute will be discussed, resolved and settled amicably by the parties hereof within ninety (90) days from the date of filing of the dispute or conflict and in case of failure to settle thereof any of the parties retain their freedom to take appropriate action. From the foregoing, it is clear that the parties, in the first place, really intended to bring to conciliation or voluntary arbitration any dispute or conflict in the interpretation or application of the provisions of their CBA. It is settled that when the parties have validly agreed on a procedure for resolving grievances and to submit a dispute to voluntary arbitration then that procedure should be strictly observed. It may not be amiss to point out that the abovequoted provisions of the CBA are in consonance with Rule VII, Section 7 of the present Omnibus Rules and Regulations Implementing the Migrant Workers and Overseas Filipinos Act of 1995, as amended by Republic Act No. 10022, which states that "[f]or OFWs with

206

collective bargaining agreements, the case shall be submitted for voluntary arbitration in accordance with Articles 261 and 262 of the Labor Code." The Court notes that the said Omnibus Rules and Regulations were promulgated by the Department of Labor and Employment (DOLE) and the Department of Foreign Affairs (DFA) and that these departments were mandated to consult with the Senate Committee on Labor and Employment and the House of Representatives Committee on Overseas Workers Affairs. In the same manner, Section 29 of the prevailing Standard Terms and Conditions Governing the Employment of Filipino Seafarers on Board Ocean Going Vessels, promulgated by the Philippine Overseas Employment Administration (POEA), provides as follows: Section 29.Dispute Settlement Procedures. — In cases of claims and disputes arising from this employment, the parties covered by a collective bargaining agreement shall submit the claim or dispute to the original and exclusive jurisdiction of the voluntary arbitrator or panel of arbitrators. If the parties are not covered by a collective bargaining agreement, the parties may at their option submit the claim or dispute to either the original and exclusive jurisdiction of the National Labor Relations Commission (NLRC), pursuant to Republic Act (RA) 8042, otherwise known as the Migrant Workers and Overseas Filipinos Act of 1995 or to the original and exclusive jurisdiction of the voluntary arbitrator or panel of arbitrators. If there is no provision as to the voluntary arbitrators to be appointed by the parties, the same shall be appointed from the accredited voluntary arbitrators of the National Conciliation and Mediation Board of the Department of Labor and Employment. DHSEcI The Philippine Overseas Employment Administration (POEA) shall exercise original and exclusive jurisdiction to hear and decide disciplinary action on cases, which are administrative in character, involving or arising out of violations of recruitment laws, rules and regulations involving employers, principals, contracting partners and Filipino seafarers. (Emphasis supplied) It is clear from the above that the interpretation of the DOLE, in consultation with their counterparts in the respective committees of the Senate and the House of Representatives, as well as the DFA and the POEA is that with respect to disputes involving claims of Filipino seafarers wherein the parties are covered by a collective bargaining agreement, the dispute or claim should be submitted to the jurisdiction of a voluntary arbitrator or panel of arbitrators. It is only in the absence of a collective bargaining agreement that parties may opt to submit the dispute to either the NLRC or to voluntary arbitration. It is elementary that rules and regulations issued by administrative bodies to interpret the law which they are entrusted to enforce, have the force of law, and are entitled to great respect. 8 Such rules and regulations partake of the nature of a statute and are just as binding as if they have been written in the statute itself. 9 In the instant case, the Court finds no cogent reason to depart from this rule. The above interpretation of the DOLE, DFA and POEA is also in consonance with the policy of the state to promote voluntary arbitration as a mode of settling labor disputes. No less than the Philippine Constitution provides, under the third paragraph, Section 3, Article XIII, thereof that "[t]he State shall promote the principle of shared responsibility between workers and employers and the preferential use of voluntary modes in settling disputes, including conciliation, and shall enforce their mutual compliance therewith to foster industrial peace." Consistent with this constitutional provision, Article 211 of the Labor Code provides the declared policy of the State "[t]o promote and emphasize the primacy of free collective bargaining and negotiations, including voluntary arbitration, mediation and conciliation, as modes of settling labor or industrial disputes." On the basis of the foregoing, the Court finds no error in the ruling of the CA that the voluntary arbitrator has jurisdiction over the instant case.

LEPANTO CONSOLIDATED MINING COMPANY vs. THE LEPANTO CAPATAZ UNION [G.R. No. 157086, February 18, 2013] Facts:

207

As a domestic corporation authorized to engage in large-scale mining, Lepanto operated several mining claims in Mankayan, Benguet. On May 27, 1998, respondent Lepanto Capataz Union (Union), a labor organization duly registered with DOLE, filed a petition for consent election with the Industrial Relations Division of the Cordillera Regional Office (CAR) of DOLE, thereby proposing to represent 139 capatazes of Lepanto. In due course, Lepanto opposed the petition, contending that the Union was in reality seeking a certification election, not a consent election, and would be thereby competing with the Lepanto Employees Union (LEU), the current collective bargaining agent. Lepanto pointed out that the capatazes were already members of LEU, the exclusive representative of all rank-and-file employees of its Mine Division. On May 2, 2000, Med-Arbiter Michaela A. Lontoc of DOLE-CAR issued a ruling to the effect that the capatazes could form a separate bargaining unit due to their not being rank-and-file employees. On the day of the certification election, however, Lepanto presented an opposition/protest. Hence, on February 8, 2001, a hearing was held on Lepanto's opposition/protest. Although the parties were required in that hearing to submit their respective position papers, Lepanto later opted not to submit its position paper, and contended that the issues identified during the hearing did not pose any legal issue to be addressed in a position paper. On April 26, 2001, Med-Arbiter Florence Marie A. Gacad-Ulep of DOLE-CAR rendered a decision certifying the Union as the sole and exclusive bargaining agent of all capatazes of Lepanto. On May 18, 2001, Lepanto appealed the decision of Med-Arbiter Gacad-Ulep to the DOLE Secretary. By her Resolution dated September 17, 2002, 15 DOLE Secretary Patricia A. Sto. Tomas affirmed the decision dated April 26, 2001, holding and disposing thus: Appellant accused Med-Arbiter Ulep of grave abuse of discretion amounting to lack of jurisdiction based on her failure to resolve appellant's motion to modify order to submit position papers and on rendering judgment on the basis only of appellee's position paper. We deny. Section 5, Rule XXV of Department Order No. 9, otherwise known as the New Rules Implementing Book V of the Labor Code, states that "in all proceedings at all levels, incidental motions shall not be given due course, but shall remain as part of the records for whatever they may be worth when the case is decided on the merits". Further, the motion to modify order to submit position papers filed by appellant is without merit. Appellant claimed that the issues over which Med-Arbiter Ulep directed the submission of position papers were: (1) failure to challenge properly; (2) failure (especially of LEU) to participate actively in the proceedings before the decision calling for the conduct of certification election; and (3) validity of earlier arguments. According to appellant, the first issue was for appellee LCU to reply to in its position paper, the second issue was for the LEU and the third issue for appellant company to explain in their respective position paper. It was the position of appellant company that unless the parties filed their position paper on each of their respective issues, the other parties cannot discuss the issues they did not raise in the same position papers and have to await receipt of the others' position paper for their appropriate reply. Section 9, Rule XI of Department Order No. 9, which is applied with equal force in the disposition of protests on the conduct of election, states that "the Med-Arbiter shall in the same hearing direct all concerned parties, including the employer, to simultaneously submit their respective position papers within a nonextendible period of ten days". The issues as recorded in the minutes of 28 February 2001 hearing before the Med-Arbiter are clear. The parties, including appellant company were required to submit their respective positions on whether there was proper challenge of the voters, whether LEU failed to participate in the proceedings, if so, whether it should be allowed to participate at this belated stage and whether the arguments raised during the pre-election conferences and in the protests are valid. The parties, including appellant company were apprised of these issues and they agreed thereto. The minutes of the hearing even

208

contained the statement that "no order will issue" and that "the parties are informed accordingly". If there is any matter that had to be clarified, appellant should have clarified the same during the said hearing and refused to file its position paper simultaneously with LCU and LEU. It appears that appellant did not do so and acquiesced to the filing of its position paper within fifteen days from the date of said hearing. Issue: Whether or not the secretary of labor acted without or in excess of jurisdiction, [o]r with grave abuse of discretion amounting to lack or excess of jurisdiction in issuing the decision dated september 17, 2002, when she deliberately ignored the facts and ruled in favor of the respondent union, despite her own finding that there had been a premature canvass of votes. Ruling: The filing of the motion for reconsideration is a pre-requisite to the filing of a petition for certiorari to assail the decision of the DOLE Secretary. We hold to be untenable and not well taken Lepanto's submissions that: (1) a motion for reconsideration was not an available remedy from the decision of the DOLE Secretary because of Section 15, Rule XI, Book V of the Omnibus Rules Implementing the Labor Code, as amended; and (2) the ruling in National Federation of Labor v. Laguesma 21 (recognizing the remedy of certiorari against the decision of the DOLE Secretary to be filed initially in the CA) actually affirms its position that an immediate recourse to the CA on certiorari is proper even without the prior filing of a motion for reconsideration. To start with, the requirement of the timely filing of a motion for reconsideration as a precondition to the filing of a petition for certiorari accords with the principle of exhausting administrative remedies as a means to afford every opportunity to the respondent agency to resolve the matter and correct itself if need be. 22 SIEHcA And, secondly, the ruling in National Federation of Labor v. Laguesma reiterates St. Martin's Funeral Home v. National Labor Relations Commission, where the Court has pronounced that the special civil action of certiorari is the appropriate remedy from the decision of the National Labor Relations Commission (NLRC) in view of the lack of any appellate remedy provided by the Labor Code to a party aggrieved by the decision of the NLRC. Accordingly, any decision, resolution or ruling of the DOLE Secretary from which the Labor Code affords no remedy to the aggrieved party may be reviewed through a petition for certiorari initiated only in the CA in deference to the principle of the hierarchy of courts. Agreeing with Med-Arbiter Lontoc's findings, then DOLE Undersecretary Baldoz, acting by authority of the DOLE Secretary, observed in the resolution dated July 12, 2000, thus: The bargaining unit sought to be represented by the appellee are the capataz employees of the appellant. There is no other labor organization of capatazes within the employer unit except herein appellant. Thus, appellant is an unorganized establishment in so far as the bargaining unit of capatazes is concerned. In accordance with the last paragraph of Section 11, Rule XI, Department Order No. 9 which provides that "in a petition filed by a legitimate labor organization involving an unorganized establishment, the Med-Arbiter shall, pursuant to Article 257 of the Code, automatically order the conduct of certification election after determining that the petition has complied with all requirements under Sections 1, 2 and 4 of the same rules and that none of the grounds for dismissal thereof exists", the order for the conduct of a certification election is proper. We cannot undo the affirmance by the DOLE Secretary of the correct findings of her subordinates in the DOLE, an office that was undeniably possessed of the requisite expertise on the matter in issue. In dealing with the matter, her subordinates in the DOLE fairly and objectively resolved whether the Union could lawfully seek to be the exclusive representative of the bargaining unit of capatazes in the company. Their factual findings, being supported by substantial evidence, are hereby accorded great respect and finality. Such findings cannot be made the subject of our judicial review by petition under Rule 45 of the Rules of Court, because:

209

. . . [T]he office of a petition for review on certiorari under Rule 45 of the Rules of Court requires that it shall raise only questions of law. The factual findings by quasi-judicial agencies, such as the Department of Labor and Employment, when supported by substantial evidence, are entitled to great respect in view of their expertise in their respective field. Judicial review of labor cases does not go far as to evaluate the sufficiency of evidence on which the labor official's findings rest. It is not our function to assess and evaluate all over again the evidence, testimonial and documentary, adduced by the parties to an appeal, particularly where the findings of both the trial court (here, the DOLE Secretary) and the appellate court on the matter coincide, as in this case at bar. The Rule limits that function of the Court to review or revision of errors of law and not to a second analysis of the evidence. Here, petitioners would have us re-calibrate all over again the factual basis and the probative value of the pieces of evidence submitted by the Company to the DOLE, contrary to the provisions of Rule 45. Thus, absent any showing of whimsical or capricious exercise of judgment, and unless lack of any basis for the conclusions made by the appellate court may be amply demonstrated, we may not disturb such factual findings. In any event, we affirm that capatazes or foremen are not rank-and-file employees because they are an extension of the management, and as such they may influence the rank-and-file workers under them to engage in slowdowns or similar activities detrimental to the policies, interests or business objectives of the employers.

7K CORP. vs. ALBARICO [G.R. No. 182295, June 26, 2013] Facts: Petitioner 7K Corporation terminated Albarico’s employment allegedly for his poor sales performance. Respondent submitted his money claims against petitioner for arbitration before the NCMB. While the NCMB arbitration case was pending, Albarico filed a complaint for illegal dismissal with money claims against petitioner before the Regional Arbitration Branch of the NLRC. Petitioner filed its position paper in the NCMB arbitration case denying that respondent was terminated from work, much less illegally dismissed. It claimed that Albarico was guilty of abandonment of employment. The NCMB voluntary arbitrator rendered a decision finding petitioner corporation liable for illegal dismissal. In lieu of reinstatement, however, the voluntary arbitrator ordered the corporation to pay Albarico his separation pay, backwages, and attorney’s fee. Issue: Whether the voluntary arbitrator properly assumed jurisdiction to decide the issue of the legality of the dismissal of respondent even if the same was not expressly claimed in the Submission Agreement of the parties. Ruling: The voluntary arbitrator rightly assumed jurisdiction to decide the said issue. Voluntary arbitrators may, by agreement of the parties, assume jurisdiction over a termination dispute such asthe present case, contrary to the assertion of petitioner that they may not. Under the Labor Code, separation pay may be given not only when there is illegal dismissal. In fact, it is also given to employees who are terminated for authorized causes, such as redundancy, retrenchment or installation of labor-saving devices under Article 283 of the Labor Code. Additionally, jurisprudence holds that separation pay may also be awarded for considerations of social justice, even if an employee has been terminated for a just cause other than serious misconduct or an act reflecting on moral character. The Court has also ruled that separation pay may be awarded if it has become an established practice of the company to pay the said benefit to voluntarily resigning employees or to those validly dismissed for non-membership in a union as required in a closed-shop agreement. Having established that the issue of the legality of dismissal of Albarico was in fact necessarily albeit not explicitly included in the Submission Agreement signed by the parties, this Court rules that the voluntary arbitrator rightly assumed jurisdiction to decide the said

210

issue. Consequently, we also rule that the voluntary arbitrator may award backwages upon a finding of illegal dismissal, even though the issue of entitlement thereto is not explicitly claimed in the Submission Agreement. Backwages, in general, are awarded on the ground of equity as a form of relief that restores the income lost by the terminated employee by reason of his illegal dismissal.

211

View more...

Comments

Copyright ©2017 KUPDF Inc.
SUPPORT KUPDF